CRT Test Banks

You might also like

Download as pdf or txt
Download as pdf or txt
You are on page 1of 257

CHAPTER 3

Name: __________________________ Date: _____________

1. Which of the following specialized imaging tests would be most useful in diagnosing a
pulmonary emboli?
A) chest X-ray
B) pulmonary function test (PFT)
C) ventilation-perfusion scan (V/Q scan)
D) arterial blood gas (ABG)
Ans: C
Response:
A chest X-ray and an ABG might be useful in detecting an abnormality, but not specifically a
pulmonary emboli. Pulmonary function testing (PFTs) may reveal abnormal flows and
volumes/capacities but no perfusion problems inherent in a pulmonary emboli. A ventilation-
perfusion scan would show a lack of blood flow due to emboli (a clot) in the blood pulmonary
circulation. Ventilation to lung regions would be shown as well.

2. While examining a patient in the ICU, you note that they appear somewhat edematous and the
nurse has indicated that the patient's urine output is “minimal.” In what section of the medical
record would you check to determine the patient's fluid balance (intake vs. output)?
A) physican orders
B) consent
C) lab results
D) nurses notes and flowsheet
Ans: D
Response:
A patient's fluid balance is the relationship between fluid intake and outputs primarily from
urination. It is normally noted in the nurses notes and flowsheet section of the medical record.
The normal fluid intake and output for adults is 1 to 2 liters per day, or 25 to 50 ml per hour.
A positive fluid balance results from excessive intake and/or decreased output and may
contribute to pulmonary or peripheral edema and hypertension. A negative fluid balance is
generally due to insufficient hydration and/or excessive urination from medications such as
diuretics or theophylline and may lead to hypotension and low cardiac output.

3. Which of the following physical findings would you expect to see in an alert but anxious
asthmatic who has just been admitted to the ED?
A) respiratory acidosis
B) respiratory alkalosis
C) clubbing
D) cor pulmonale
Ans: B
Response:
Asthmatics typically present with respiratory alkalosis. Shortness of breath and accompanying
hypoxemia cause the patient to increase his rate of breathing and alkalosis occurs. It is
important to note here that once hypoxemia is relieved by the administration of supplemental
oxygen, patient's PaCO2 and pH will normalize.

Page 1
4. In the lab results section of a patient's medical record, the overall WBC is shown as of 22,000,
for a febrile patient who appear acutely ill and in moderate respiratory distress. Which of the
following is this patient's most likely diagnosis?
A) bacterial pneumonia
B) emphysema
C) pulmonary embolus
D) pulmonary fibrosis
Ans: A
Response:
The elevated WBC suggests a bacterial infection. The respiratory distress further points to a
respiratory infection such as bacterial pneumonia. In addition, the other choices are not
infectious processes and therefore you would not likely see an elevated WBC.

5. A PET scan would be most useful in the diagnosis of which of the following conditions?
A) bronchogenic carcinoma
B) chronic bronchitis
C) pulmonary fibrosis
D) smoke inhalation
Ans: A
Response:
A PET scan is a nuclear imaging technique used in the diagnosis/staging/management of
tumors and cancer. The answer choice bronchogenic carcinoma describes lung cancer.

6. Negative inspiratory pressure (NIF; MIP) is useful in the determination of which of the
following?
A) airway resistance
B) functional residual capacity
C) respiratory muscle strength
D) sustained maximal inspiration
Ans: C
Response:
Negative inspiratory force (NIF) or Maximal Inspiratory Pressure (MIP) is used for the
bedside assessment of respiratory muscle strength. You probably know that it is a
measurement of pressure only. Since airway resistance is a measurement of pressure divided
by flow, and since Functional Residual Capacity and Sustained Maximal Inspiration are
measurements of volume, the process of elimination leaves muscle strength as the only correct
answer.

7. Assessment of a 28 y/o trauma patient reveals diminished breath sounds, asymmetrical chest
expansion, severe chest pain and an SpO2 of 90%, despite receiving oxygen via cannula at 5
L/min. These findings are most consistent with what diagnosis?
A) complete airway obstruction
B) pneumothorax
C) viral pneumonia
D) pleural effusion
Ans: B
Response:
Both viral pneumonia and pleural effusion can be ruled out as correct answers as they are not
related to trauma. Breath sounds, though diminished, can be heard and chest expansion,
though asymmetrical, is present; both of which are not consistent with a complete airway
obstruction. The physical assessment is consistent with the correct answer, pneumothorax.

Page 2
8. A 23 y/o firefighter is admitted with suspected smoke inhalation. You place him on a non-
rebreathing mask. What is the most appropriate method of monitoring his oxygenation?
A) arterial blood gas analysis
B) co-oximetry
C) pulse oximetry
D) calculation of P(A-a)O2
Ans: B
Response:
In the case of smoke inhalation, carbon monoxide (CO) binds to the hemoglobin molecule in
place of oxygen. Neither ABGs, pulse oximetry nor calculation of the A-a gradient will
provide indication of how much oxygen is bound to hemoglobin. Any victim of suspected
smoke inhalation must be monitored with CO-oximetry.

9. The ratio of lecithin to sphingomyelin, or L/S ratio, is a test to determine fetal lung maturity.
Such a test may be done in the later stages of pregnancy and the values will initially be noted
in the lab results section of the mother's medical record. Approximately what ratio is
associated with the onset of mature surfactant production?
A) 2:1
B) 20:1
C) 1:2
D) 1:20
Ans: A
Response:
The lecithin to sphingomyelin ratio, or L/S ratio, which is used to determine fetal lung
maturity. Generally, this ratio reaches 2:1 (twice as much lethicin then sphingomyelin) near
week 35 of gestation, which corresponds to the onset of mature surfactant production. The L/S
ratio will generally be noted in the lab results section of the mother's medical record.

10. Sputum culture and sensitivity would be indicated in the evaluation of which of the following
clinical conditions?
A) pulmonary edema
B) bacterial pneumonia
C) bronchiectasis
D) empyema
Ans: B
Response:
Sputum culture & sensitivity is used to identify microorganisms and their most appropriate
drug therapy. Bacterial pneumonia is the obvious infectious process in the choice of answers.

11. When interviewing a patient, which of the following might be relevant to the patient's
pulmonary status?

I. occupational history
II. medical history
III. smoking history
IV. family history
A) I and IV
B) II and III
C) I, III and IV
D) I, II, III and IV
Ans: D
Response:
In addition to a patient's chief complaint, a variety of history relating to the patient's
occupation, family, smoking and past medical conditions, should be gathered to help in your
assessment.

Page 3
12. You come upon a patient who is unresponsive and apneic. You had heard from another
therapist that this patient had discussed a Do Not Resuscitate (DNR) order with her physician.
You should
A) check the patient's chart for a DNR order
B) immediately call a code and begin resuscitation efforts
C) ask the RN what to do
D) call a “slow code”
Ans: B
Response:
If there is any doubt regarding a DNR order, resuscitation should begin.

13. A patient has acute respiratory acidosis. You would expect the base excess (BE) to be
approximately
A) + 6 mEq/L
B) –6 mEq/L
C) + or -2 mEq/L
D) + or -8 mEq/L
Ans: C
Response:
In acute respiratory acidosis, the base excess/deficit will be in normal range because retention
of HCO3 does not occur for 48-72 hours.

14. Prior to giving an aerosol treatment, you find a note in the chart that states your patient had
pink frothy secretions on admission to the ED. This is most indicative of
A) cor pulmonale
B) left ventricular failure
C) an electrolyte imbalance
D) ARDS
Ans: B
Response:
Frothy pink-tinged secretions are a hallmark sign of cardiogenic pulmonary edema, which is
the result of left ventricular failure or CHF.

15. In reviewing the medical record of a 60 y/o trauma patient, you note that a ventilation-
perfusion scan was ordered. What condition is being ruled out?
A) ARDS
B) coronary artery blockage
C) pulmonary edema
D) pulmonary embolism
Ans: D
Response:
Ventilation-perfusion scan is the diagnostic tool for the rule-out of pulmonary embolus. In the
presence of a pulmonary embolism, perfusion would be decreased in the affected lung region.

Page 4
16. Which of the following would provide the best bedside assessment of the need for mechanical
ventilation in a patient with Guillian-Barre syndrome?
A) FRC
B) VC
C) airway resistance
D) TLC
Ans: B
Response:
Guillian-Barre syndrome is a restrictive neuromuscular disorder that results in
hypoventilation. Neither FRC, TLC, nor airway resistance would indicate a restrictive
ventilatory impairment. Furthermore, vital capacity (VC) is the only parameter of those listed
which could be measured at bedside.

17. Which of the following would be the likely cause of increased plateau pressure of a patient
who is being mechanically ventilated?
A) a loss of compliance
B) water accumulation in the ventilator circuit
C) an obstructed endotracheal tube
D) a decrease in sensitivity
Ans: A
Response:
Lung and/or chest wall compliance are the only factors which affect plateau pressure. Water
accumulation and/or an obstructed ET tube would cause increased airway resistance and as
such, would increase dynamic or peak pressure only. Sensitivity has no bearing on ventilator
inspiratory pressure readings.

18. Laboratory findings in allergic asthma usually include which of the following?

I. increased eosinophils
II. increased serum levels of IgE
III. plugs and spirals in the sputum
IV. arterial hypoxemia
A) I, II, III, IV
B) II, III, IV
C) II and IV
D) I, III and IV
Ans: A
Response:
All of the above are indicators of an allergic asthmatic response.

Page 5
19. A 14 y/o patient with no known history of respiratory problems is comatose in the ED. The
Arterial blood gases on room air are

pH 7.13
PaCO2 14 mmHg
HCO3 5 mEq/L
BE -18
PaO2 120 torr

You can concluded that the patient


A) must be breathing supplemental oxygen
B) is hyperventilating and is hypoxemic
C) is hyperventilating due to an acid-base disturbance
D) has compensated for the hyperventilation
Ans: C
Response:
The paCO2 of 14 clearly indicates hyperventilation. Since the patient is not hypoxemic, the
hyperventilation is due to an acid-base disturbance as suggested by the BE of 18. In this case,
the best answer is that the hyperventilation is coming from a severe metabolic acidosis.

20. A patient is receiving ventilatory support following thoracic surgery. You measure the
patient's maximal inspiratory pressure (MIP) as -34cmH2O. Based on this value, the patient
A) needs continued ventilatory support
B) should be placed on non-invasive positive pressure ventilation
C) has a leak in the cuff of the endotracheal tube
D) has an adequate inspiratory pressure to consider weaning
Ans: D
Response:
An MIP of -34 cmH2O is acceptable and suggests that the patient has sufficient respiratory
muscle strength to be considered for weaning. Of course, other parameters, such as
oxygenation and ventilation, must be considered as well.

21. An otherwise healthy 25 year old male patient who took an overdose of sedatives is being
supported on a ventilator. Which of the following measures of total static compliance (lungs +
thorax) would you expect in this patient?
A) 100 mL/cm H2O
B) 10 mL/cm H2O
C) 1 mL/cm H2O
D) 0.1 mL/cm H2O
Ans: A
Response:
As measured on a ventilator, normal static compliance of the lungs and thorax combined is
about 100 mL/cm H2O or 0.1 L/cm H2O.

Page 6
22. A patient's advanced directive:
A) is usually obtained at the time of admission
B) can be found in the doctor's progress notes
C) represents a guideline, not a legal requirement
D) cannot be altered after it is written and signed
Ans: A
Response:
An advance directive allows patients to specify the health care decisions they want made
should they become unable to make informed decisions. An advanced directive may take the
form of a Living Will or Durable Power of Attorney. Normally, both types of advance
directives are obtained when a patient is admitted to the hospital and can be found in the
patient's chart.

23. In reviewing the PFT results of a 67 year old smoker with an admitting diagnosis of
emphysema and chronic bronchitis you would expect which of the following general findings?
A) increased airway resistance and decreased lung compliance
B) increased airway resistance and increased lung compliance
C) decreased airway resistance and decreased lung compliance
D) decreased airway resistance and increased lung compliance
Ans: B
Response:
Emphysema and chronic bronchitis are components of chronic obstructive pulmonary diseases
(COPD). COPD patients characteristically have increased expiratory airway resistance. If
emphysema is present, destruction of elastic tissue in the lung can also INCREASE overall
compliance. In fact, emphysema is one of the only pulmonary disorders that is associated with
greater than normal lung compliance.

24. A patient has a pH of 7.22 and a PaCO2 of 60 torr. Based on these data, what is the primary
acid-base disturbance?
A) respiratory alkalosis
B) metabolic acidosis
C) respiratory acidosis
D) metabolic alkalosis
Ans: C
Response:
To determine the primary acid-base disturbance, first determine whether the pH is low (< 7.35
= academia) or high (> 7.45 = alkalemia). After judging the pH, you then assess the PaCO2. If
the pH is low, the primary disturbance must be either respiratory acidosis (PaCO2 > 45 torr) or
metabolic acidosis (PaCO2  45 torr). With a high pH, the primary disturbance must be either
respiratory alkalosis (PaCO2 < 35 torr) or metabolic alkalosis (PaCO2  35 torr).

25. A patient has a pH of 7.58 and a PaCO2 of 50 torr. Based on these data, what is the primary
acid-base disturbance?
A) respiratory alkalosis
B) metabolic acidosis
C) respiratory acidosis
D) metabolic alkalosis
Ans: D
Response:
To determine the primary acid-base disturbance, first determine whether the pH is low (< 7.35
= academia) or high (> 7.45 = alkalemia). After judging the pH, you then assess the PaCO2. If
the pH is low, the primary disturbance must be either respiratory acidosis (PaCO2 > 45 torr) or
metabolic acidosis (PaCO2  45 torr). With a high pH, the primary disturbance must be either
respiratory alkalosis (PaCO2 < 35 torr) or metabolic alkalosis (PaCO2  35 torr).

Page 7
26. A patient has a pH of 7.58 and a PaCO2 of 25 torr. Based on these data, what is the primary
acid-base disturbance?
A) respiratory alkalosis
B) metabolic acidosis
C) respiratory acidosis
D) metabolic alkalosis
Ans: A
Response:
To determine the primary acid-base disturbance, first determine whether the pH is low (< 7.35
= academia) or high (> 7.45 = alkalemia). After judging the pH, you then assess the PaCO2. If
the pH is low, the primary disturbance must be either respiratory acidosis (PaCO2 > 45 torr) or
metabolic acidosis (PaCO2  45 torr). With a high pH, the primary disturbance must be either
respiratory alkalosis (PaCO2 < 35 torr) or metabolic alkalosis (PaCO2  35 torr).

27. A patient has a pH of 7.22 and a PaCO2 of 38 torr. Based on these data, what is the primary
acid-base disturbance?
A) respiratory alkalosis
B) metabolic acidosis
C) respiratory acidosis
D) metabolic alkalosis
Ans: B
Response:
To determine the primary acid-base disturbance, first determine whether the pH is low (< 7.35
= academia) or high (> 7.45 = alkalemia). After judging the pH, you then assess the PaCO2. If
the pH is low, the primary disturbance must be either respiratory acidosis (PaCO2 > 45 torr) or
metabolic acidosis (PaCO2  45 torr). With a high pH, the primary disturbance must be either
respiratory alkalosis (PaCO2 < 35 torr) or metabolic alkalosis (PaCO2  35 torr).

28. The normal body temperature varies daily by about:


A) l° - 2°F
B) l° - 2°C
C) 2° - 4°F
D) 3° - 5°F
Ans: A
Response:
The normal body temperature for most individuals is about 98.6°F (37°C) with a range from
97°F to 99.5°F, and with daily variations of l° to 2°F. The body temperature usually is lowest
in the early morning and highest in the late afternoon. Metabolic functions occur optimally
when the temperature is normal.

Page 8
CHAPTER 4

Name: __________________________ Date: _____________

1. Which of the conditions is associated with jugular venous distension?


A) cor pulmonale
B) pneumonia
C) simple pneumothorax
D) septic shock
Ans: A
Response:
Cor pulmonale is right heart failure due to chronic lung disease. Right heart failure occurs
when chronic hypoxemia elevates the pulmonary vascular resistance and puts a strain on the
right ventricle to pump blood through the constricted pulmonary capillaries. Right heart
failure causes venous blood to back up into the neck veins.

2. You palpate your patient's neck and notice that the trachea is shifted to the patient's left.
Which of the following conditions could explain this finding?
A) left ventricular enlargement
B) left upper lobe collapse
C) right lower lobe collapse
D) pleural effusion on the left
Ans: B
Response:
Volume loss in the upper lobe will pull the trachea towards the collapsed lung. In this case the
trachea has shifted to the left indicating that either volume loss has occurred on the left or a
tumor on the right is pushing the trachea to the left.

3. On reviewing the results of the attending physician's physical examination of a patient's chest,
you note "a dull percussion note and bronchial breath sounds - LLL." All of the following are
potential problems EXCEPT:
A) infiltrates
B) atelectasis
C) consolidation
D) pneumothorax
Ans: D
Response:
A patient with a dull percussion note and bronchial breath sounds on chest examination most
likely has either pulmonary infiltrates, atelectasis or consolidation of the affected area. A
pneumothorax normally results in a hyperresonant percussion note.

4. You hear bronchial breath sound over the patient's right middle lobe. What condition is
probably present?
A) emphysema
B) asthma
C) pneumonia
D) pleural effusion
Ans: C
Response:
Bronchial breath sounds normally are heard only over the trachea. When heard over the lung
periphery, consolidation (due to pneumonia) is present. Consolidation of the lung allows the
turbulent flow sounds of the larger airways to pass directly through the lung as attenuation is
reduced

Page 1
SAN PEDRO COLLEGE COMPETENCY APPRAISAL - OXYGENATION

5. Upon exam of an acutely dyspneic and hypotensive patient, you note the following – all on
the left side of the chest: reduced chest expansion, hyperresonance to percussion, absent of
breath sounds and tactile fremitus, and a tracheal shift to the right. These findings suggest:
A) left-sided pneumothorax
B) left-sided consolidation
C) left lobar obstruction/atelectasis
D) left-sided pleural effusion
Ans: A
Response:
An acutely ill patient with dyspnea, hypotension, unilateral findings of reduced chest
expansion, a hyperresonant percussion note, absent of breath sounds and tactile fremitus, and
a tracheal shift to the right has most likely suffered a large pneumothorax on the affected side.
If the pneumothorax is severe enough to disrupt cardiac function, blood pressure will also fall.

6. During an interview with your patient you determine that she is disoriented to time, place, and
person. What may explain this finding?
A) respiratory alkalosis
B) severe hypoxemia
C) metabolic acidosis
D) hyperthermia
Ans: B
Response:
An abnormal sensorium is often caused by a lack of adequate oxygenation to the brain. Severe
hypoxemia reduced the oxygen available to the brain and leads to confusion. An abnormal
sensorium in any patient should first be assumed to be caused by hypoxia until proven
otherwise.

7. To assess the level of pain that a 2 year old child is experiencing during a procedure, you
would consider all of the following EXCEPT:
A) look for facial grimacing
B) ask for the mother's judgment
C) use a numeric pain scale
D) observe for crying
Ans: C
Response:
A numeric pain scale is not useful for young children or patients who cannot express
themselves. The best way to assess the pain level of a young child is to get the input of a
family member. Without such information, you may have to rely on observing patient
behaviors that indicate severe pain, such as moaning, crying, or grimacing.

8. During an interview with your patient you discover that he gets short of breath at night when
he lays down so he often sleeps with several pillow propping his head up. What symptom is
present?
A) apnea
B) orthopnea
C) platypnea
D) orthodeoxia
Ans: B
Response:
Orthopnea is present when the patient has difficult breathing upon laying down. In most cases
this is due to pulmonary edema filling the lungs when the patient assumes a horizontal
position. Keeping the patient's head up during sleep with the use of pillows reduces the
collection of fluid in the lungs.

Page 2
SAN PEDRO COLLEGE COMPETENCY APPRAISAL - OXYGENATION

9. A patient complains that she has a chronic cough usually accompanied by sputum production.
This information indicates that the patient probably has which of the following conditions?
A) acute asthma
B) chronic bronchitis
C) pulmonary emphysema
D) bacterial pneumonia
Ans: B
Response:
Chronic bronchitis is defined primarily by its symptoms: chronic cough and sputum
production.

10. All of the following are associated with a patient being at risk for malnutrition EXCEPT:
A) if the patient has had a recent weight gain
B) if the patient is significantly underweight
C) if the patient has poor dietary habits
D) if the patient is unable to prepare his/her own food
Ans: A
Response:
Any patient who (1) is significantly underweight, (2) has recently lost a significant amount of
weight; (3) has poor dietary habits or inadequate food intake; or (4) is impoverished, isolated,
or unable to prepare own food is at high risk for malnutrition. Weight gain is not normally
associated with malnutrition.

11. Which of the following aspects of a patient's social history is most important in the diagnosis
of lung disease?
A) marital status
B) cultural background
C) education
D) occupational history
Ans: D
Response:
Of the items list, occupational history is most important in the diagnosis of lung disease.
Many lung diseases are associated with inhalation of dusts or toxic chemicals in the work
setting.

12. You come upon an elderly patient who is unresponsive and is not breathing. You had heard
from her nurse that she had discussed with her physician whether or not to have a do not
resuscitate (DNR) order in her chart. You should
A) Check the patient's chart for a DNR order/advanced directive
B) Immediately call a code and begin resuscitation efforts
C) Contact the nurses' station and ask how best to proceed
D) Call a "slow code," i.e., apply basic CPR, but not ACLS
Ans: B
Response:
If there is any doubt regarding a do not resuscitate (DNR) order or when written orders are not
present, resuscitation efforts should be begun. Formal or informal directives such as "slow
code" or "code gray" bypass the patient's rights and are generally inappropriate.

Page 3
SAN PEDRO COLLEGE COMPETENCY APPRAISAL - OXYGENATION

13. The best way to determine whether or not a patient has learned the information needed to
understand how her disease impacts on lung function would be to:
A) have her take a multiple choice quiz
B) discuss the information with patient's family
C) have her "teach" the information back to you
D) have her perform a return demonstration
Ans: C
Response:
Teach-back is the best way to determine whether or a patient has learned critical information
related to their care. A return demonstration is appropriate for procedural skills, whereas
discussion is the best method for assessing attitudinal change.

14. You note on inspection of an AP chest radiograph that the left hemidiaphragm is elevated
above normal. Which of the following is the most likely cause of this abnormality?
A) right pleural effusion
B) right tension pneumothorax
C) right phrenic nerve paralysis
D) right lower lobe pneumonia
Ans: C
Response:
An elevated hemidiaphragm indicates phrenic nerve paralysis on the affected or hepatomegaly
(right side). Pleural effusions blunt the costophrenic angles, whereas hyperinflation tends to
flatten the hemidiaphragms, as does tension pneumothorax (on affected side).

15. An AP X-ray of a 3 year-old child with wheezing and stridor shows are area of prominent
subglottic edema, but the lateral neck X-ray appears normal. What s the most likely problem?
A) croup
B) foreign body
C) epiglottitis
D) cystic fibrosis
Ans: A
Response:
Croup is the most likely problem. On an AP X-ray, croup is characterized by the "steeple
sign," i.e., narrowed and tapering airway below larynx due to subglottic edema. Typically, the
lateral neck X-ray in these patients is normal (little or no evidence of supraglottic
involvement).

16. On reviewing an ECG print-out you note widen QRS complexes. Which of the following is
the most likely cause of this problem?
A) atrial fibrillation
B) 1st degree heart block
C) sinus arrhythmia
D) bundle branch block
Ans: D
Response:
QRS complexes appear wider than normal (> 0.12 sec) in PVCs, bundle branch block,
ventricular fibrillation, and hyperkalemia.

Page 4
SAN PEDRO COLLEGE COMPETENCY APPRAISAL - OXYGENATION

17. A 150 pound patient is breathing at a frequency of 20/minute, with a tidal volume of 550 mL.
What is his estimated alveolar ventilation per minute?
A) 11.00 L/min
B) 8.00 L/min
C) 3.00 L/min
D) 14.00 L/min
Ans: B
Response:
The formula for alveolar minute ventilation is VE = f x (VT - VDS). In this case the physiologic
deadspace is estimated at 1 mL/lb ideal body weight, or 150 mL. Substituting the patient's
values for f, VT and estimated VDS, we compute an alveolar minute ventilation of 20 x (550-
150) = 8,000 mL/ min, or 8.00 L/min.

18. You obtain a bedside vital capacity (VC) of 400 mL on a cooperative 50 kg female patient
receiving ventilatory support in the CMV mode. Which of the following conclusions can you
draw from this finding?
A) the patient's VC is normal for her size and weight
B) the patient cannot sustain prolonged spontaneous ventilation
C) the patient is in acute hypoxemic respiratory failure
D) the patient has a generalized obstructive disease process
Ans: B
Response:
This patient has a VC of about 8 mL/kg (400 mL/50 kg). A normal VC is about 65-75 mL/kg.
Values below 65-75 mL/kg indicate a restrictive disorder, which may be due to neuromuscular
weakness, acutely decreased lung volumes, or interstitial lung disease. When the VC drops
below 10-15 mL/kg it is unlikely that a patient can support prolonged spontaneous ventilation.

19. When performing bedside spirometry on a 46-year-old man who is 6 feet tall, you obtain a
peak flow measurement of 3.3 L/sec. Which of the following are possible explanations for this
finding?

I. the patient's peak flow is within normal limits


II. the patient is not exerting full effort
III. the patient has expiratory flow obstruction
IV. the patient has poor pulmonary gas distribution
A) I only
B) II and III
C) II, III and IV
D) III only
Ans: B
Response:
Normal adult male peak flows range between 8-12 L/sec, while adult female normal peak
flows range between 6-9 L/sec. This patient's peak flow is considerably below normal which
generally indicates an expiratory flow obstruction. However, the peak flow test is highly
effort-dependent. For this reasons, poor effort should always be considered when peak flow
results are below predicted norms.

Page 5
SAN PEDRO COLLEGE COMPETENCY APPRAISAL - OXYGENATION

20. A patient's bedside spirometry results (as compared to normal) are as follows:

FVC decreased
FEV1 normal
FEV1% increased

What is the most likely problem?


A) an obstructive disorder
B) poor patient effort
C) a restrictive disorder
D) within normal limits
Ans: C
Response:
A patient with a decreased FVC, normal FEV1 and increased FEV1% is exhibiting the classic
pattern of a restrictive pulmonary disorder, i.e. decreased volumes and normal (or increased)
flows. .

21. A patient has a vital capacity of 3200 mL, a functional residual capacity of 4500 mL and
expiratory reserve volume of 1200 mL. What is her residual volume (RV)?
A) 8900 mL
B) 2000 mL
C) 3300 mL
D) 5700 mL
Ans: C
Response:
The functional residual capacity or FRC equals the sum of the residual volume and the
expiratory reserve volume (FRC = RV + ERV). Rearranging this equation (to solve for RV)
yields RV = FRC - ERV. In this case, RV = 4500 - 1200 or 3300 mL.

22. After performing comprehensive pulmonary function testing on a patient, you note the
following results:

Test % Predicted
FEV1% 50%
RV 150%
TLC 135%
DLco 60%

Which of the following interpretations is most consistent with these findings?


A) The patient has pulmonary fibrosis
B) The patient has pulmonary emphysema
C) The patient has chronic bronchitis
D) The patient is not exerting maximum effort
Ans: B
Response:
The low FEV1% and higher than normal TLC and RV confirm an obstructive disorder with
hyperinflation (emphysema, chronic bronchitis, acute asthma, etc). Among these obstructive
disorders, only patients with emphysema exhibit a low DLco, due to the destruction of the
alveolar capillary membrane.

Page 6
SAN PEDRO COLLEGE COMPETENCY APPRAISAL - OXYGENATION

23. An apnea monitor on a premature infant indicates an abnormal decrease in respiratory rate
and an abnormal increased in heart rate. What is the most likely cause of this problem?
A) hypoxemia
B) apnea of prematurity
C) periodic breathing
D) motion/activity artifact
Ans: A
Response:
A decreased respiratory rate in combination with an increased heart rate in a neonate most
likely indicates hypoxemia, which should be confirmed by pulse oximetry or an ABG.

24. In analyzing overnight oximetry data, a desaturation event represents a decrease in SpO2 of
what amount?
A) 2% or more
B) 3% or more
C) 4% or more
D) 5% or more
Ans: C
Response:
Most sleep disorder specialists agree that a desaturation event represents a decrease in
saturation of 4% or more. The total number of these desaturation events per hour is the
oxygen desaturation index (ODI).

25. You conduct a 6-minute walk test on four patients before and after participation in a
pulmonary rehabilitation program. Based on the 6-minute walking distance (6MWD) data
provided below, for which of these patients has the program been effective in improving their
functional capacity?

Patient Pre-Program 6MWD Post-Program 6MWD


A. 200 m 210 m
B. 150 m 200 m
C. 250 m 270 m
D. 400 m 430 m
A) Patient A
B) Patient B
C) Patient C
D) Patient D
Ans: B
Response:
When using the 6MWT to assess medical or surgical interventions, one should expect at least
a 10-20% improvement in the 6MWD to consider the treatment effective. Only patient B has
more than a 10% improvement in this measure of functional capacity [(200 – 150)/150 = 33%
improvement].

26. As measured on the Borg scale, which of the following exertion levels is appropriate for
titrating a COPD patient's O2 flows to support exercise?
A) Weak/light exertion (rating of 2)
B) Somewhat strong exertion (rating of 4)
C) Very strong exertion (rating of 7)
D) Maximal exertion (rating of 10)
Ans: B
Response:
For patients continuing on, you have them initiate walking and slowly increase their activity
until it replicates the highest intensity they will likely perform in the home environment,
usually equivalent to a 3 to 4 on the Borg rating of perceived exertion.

Page 7
SAN PEDRO COLLEGE COMPETENCY APPRAISAL - OXYGENATION

27. To assess gas exchange at the tissues you would sample blood from which of the following?
A) systemic artery
B) central vein
C) pulmonary artery
D) peripheral vein
Ans: C
Response:
To assess gas exchange at the tissues we need to assess blood after it leaves the capillaries.
For the body as a whole, we need to wait until after all the blood from all the capillary beds
mixes together, which is complete only in the pulmonary artery.

28. A patient has a pulmonary capillary wedge pressure (PCWP) of 18 mm Hg. All of the
following are potential causes for this finding EXCEPT:
A) hypovolemia
B) mitral valve stenosis
C) positive end-expiratory pressure
D) left ventricular failure
Ans: A
Response:
A pulmonary capillary wedge pressure (PCWP) of 18 mm Hg is higher than the normal of 4-
12 mm Hg. All of the causes listed except hypovolemia can increase PCWP. Hypovolemia
tends to lower ALL vascular pressures.

29. Based on the results of cardiopulmonary exercise testing, which of the following patients most
likely has a ventilatory limitation to exercise?

Patient VO2max Anaerobic Threshold Breathing Reserve


A. Decreased Decreased Normal
B. Decreased Normal Normal
C. Normal Increased Increased
D. Decreased Normal Decreased
A) Patient A
B) Patient B
C) Patient C
D) Patient D
Ans: D
Response:
All patient with poor exercise capacity have a reduced VO2max. In addition, patients with a
pulmonary limitation to exercise tend to have a normal anaerobic threshold (if it can be
reached), but a decreased breathing reserve. Patients with a cardiovascular limitation to
exercise typically have a reduced anaerobic threshold, but normal breathing reserve. In the
presence of a low VO2max, poor effort is revealed by both a normal anaerobic threshold and
breathing reserve.

Page 8
SAN PEDRO COLLEGE COMPETENCY APPRAISAL - OXYGENATION

30. A patient undergoing CPAP titration during sleep exhibits three obstructive apnea events
during a 5-minute observation interval at a pressure of 8 cm H2O. What would be the
appropriate action at this time?
A) switch the patient to BiPAP with EPAP = 8 cm H2O and IPAP = 12 cm H2O
B) increase the CPAP to 10 cm H2O for 5 minutes and continuing observation
C) discontinue the titration trial and place the patient on nasal O2 at 2 L/min
D) decrease the CPAP to 6 cm H2O for 5 minutes and continuing observation
Ans: B
Response:
If any of the following events occur during a CPAP titration observation interval (up to the
maximum protocol pressure, typically 20 cm H2O), you should increase the CPAP level by at
least 1 cm H2O:  2 obstructive apneas,  3 hypopneas,  5 RERAs,  3 min of loud snoring.

31. While assessing a patient's brachial pulse, you note that some beats are stronger than others
and that the rhythm is irregular. What would most likely cause this?
A) hypertension
B) hypotension
C) cardiac arrhythmia
D) Cheyne-Stokes breathing
Ans: C
Response:
An irregular pulse with alternating strong and weak beats (pulsus alternans) indicates a
cardiac arrhythmia (e.g., PACs, atrial fibrillation, PVCs, heart bock) and possible congestive
heart failure

32. Which of the following are common abnormalities identified during inspection of the face in
patients with respiratory disorders?

I. nasal flaring
II. pursed-lip breathing
III. cyanosis
A) I and II only
B) II and III only
C) I and III only
D) I, II and III
Ans: D
Response:
Abnormalities identified during inspection of the face produced by respiratory disease include
nasal flaring, cyanosis, and pursed-lip breathing. Nasal flaring is identified by observing the
external nares flare outward during inhalation. This occurs especially in neonates with
respiratory distress and indicates an increase in the work of breathing.

Page 9
SAN PEDRO COLLEGE COMPETENCY APPRAISAL - OXYGENATION

33. On assessment of an acutely ill patient, you note all the following in the region of the left
lower lobe: decreased expansion, a dull percussion note, and the absent of breath
sounds/tactile fremitus. You also observe a shift in the trachea toward the left, more prominent
during inspiration. These findings suggest:
A) left-sided obstruction/atelectasis
B) left-sided pneumothorax
C) left-sided consolidation
D) left-sided pleural effusion
Ans: A
Response:
A unilateral decrease in lung expansion, combined with a dull percussion note and the absent
of breath sounds & tactile fremitus signifies either local lobar obstruction with atelectasis or a
pleural effusion on the affected side. In general, the trachea shifts away from large effusions
but toward areas of atelectasis.

34. Vesicular breath sounds indicate which of the following?


A) pneumothorax
B) normal lungs
C) lung consolidation
D) airways obstruction
Ans: B
Response:
Vesicular sounds are described as low-pitched and soft and are heard primarily during
inhalation. Vesicular breath sounds are considered normal.

35. Cause of decreased or diminished breath sounds include all of the following EXCEPT:
A) hyperinflation
B) severe obesity
C) deep breathing
D) pleural effusion
Ans: C
Response:
Breath sounds are decreased or diminished when the patient's breathing is shallow or slow. A
decrease in breath sounds intensity also occurs when airways are obstructed or the lung tissue
is hyperinflated, as in asthma or COPD. Air or fluid in the pleural space and obesity also can
reduce breath sounds.

36. To determine the movement of the diaphragm by inspection, you should look for
A) abdominal movement
B) use of accessory muscles
C) intercostal retractions
D) supraclavicular retractions
Ans: A
Response:
As the diaphragm drops during inspiration, it displaces the abdominal contents and the
abdomen moves outward (in men more than women). If this does not occur, or it occurs out of
phase with inspiration, an abnormality exists

Page 10
SAN PEDRO COLLEGE COMPETENCY APPRAISAL - OXYGENATION

37. All of the following findings are associated with potential difficulty orally intubating a patient
EXCEPT:
A) macroglossia
B) soft palate not visible
C) deviated septum
D) short/thick neck
Ans: C
Response:
A short/thick neck, poor range of neck motion, macroglossia (enlarged tongue), and
Mallampati Class 4 assessment of pharygeal anatomy (soft palate, uvula and tonsils not
visible; only hard palate visible) are all associated with the potential fore difficult oral
intubation. A deviated septum would only be a potential problem for nasal intubation

38. When asking a patient who he is, where he is, and the time of day, you are trying to assess the
patient's:
A) level of consciousness
B) reaction to chronic illness
C) understanding of the language
D) ability to cooperate
Ans: A
Response:
Asking patients the time of day, where they are, and who they are is a quick way to assess
their level of consciousness or "sensorium." Alert patients are well-oriented as to time, place,
and person, or "oriented ×3." The most common reasons for a patient not being well-oriented
to time, place, and person are neurologic injury, pharmacologic sedation and severe
hypoxemia or hypercapnia. In general, only alert patients can be expected to cooperate and
fully participate in their own care.

39. You are about to start a patient on a new regimen of therapy. The patient is irritable, has
difficulty focusing, and exhibits mild trembling of his hands. You should:
A) request that the physician discontinue the therapy
B) postpone therapy until the patient's anxiety is resolved
C) request that the nurse be present during the therapy
D) proceed as quickly as possible with the therapy
Ans: B
Response:
Patients who appear depressed or overly anxious, as evident by irritability, difficulty focusing,
breathlessness, dizziness, trembling, palpitations and/or chest pain are in an in an abnormal
emotional state. In general, patients in an abnormal emotional state will be difficult to manage
until their anxiety can be resolved.

40. When you make a return visit to a postoperative patient to assess her progress with incentive
spirometry, she indicates that compared to yesterday her pain is preventing her from carry out
the treatment. On a 10-point scale, she rates the effect of her pain on her activities as an '8.'
You should
A) coach her to go ahead with the incentive spirometry anyway
B) tell the nurse to up the dosage of the patient's pain medication
C) switch the patient to intermittent positive pressure breathing therapy
D) report this finding to the patient's attending physician
Ans: D
Response:
Good pain assessment includes determining how much it interferes with the patient's
activities. A 10-point scale can be used to make this assessment, with 0 signifying "no
interference" and 10 signifies "unable to carry on usual activities." Whenever a patient reports
an interference level above 4, you report this finding to the patient's attending physician

Page 11
SAN PEDRO COLLEGE COMPETENCY APPRAISAL - OXYGENATION

41. How would you characterize the degree of dyspnea of a patient who walks slower than people
of the same age because of breathlessness?
A) slight
B) moderate
C) severe
D) very severe
Ans: B
Response:
You can assess a patient's exercise tolerance via interview using the American Thoracic
Society Breathlessness Scale. By inquiring as to when breathlessness is first noticed by the
patient, you can assign a rating to the symptom, with a descriptive term for each level. In this
case, a patient who walks slower than people of the same age on level ground because of
breathlessness or has to stop for breath when walking at own pace on level ground would be
characterized as having moderate dyspnea.

42. When interviewing a patient to obtain information about sputum production, he states that
after coughing he typically produces about a "tablespoon full of spit'. You would record this in
your assessment as equivalent to about what volume of sputum production?
A) 2 mL
B) 5 mL
C) 15 mL
D) 30 mL
Ans: C
Response:
When interviewing patients about sputum production, you can help them estimate the amount
or volume of sputum they produce by using familiar measures such as a teaspoon (about 5
mL), tablespoon (about 15 mL) or shot-glass full (about 1 oz. or 30 mL). To obtain more
precise quantification of sputum volume, you would need a calibrated collection cup (one with
mL marking on the side).

43. The Apgar score for a normal newborn infant ranges between
A) 1-4
B) 4-7
C) 7-10
D) 10-13
Ans: C
Response:
At birth, an Apgar score of 7-10 is considered normal. Scores of 4-6 are intermediate and
usually dictate the need for more intensive support. Infants with Apgar scores of 0-3 usually
undergo aggressive resuscitation. Needed interventions should never be delayed in order to
obtain the Apgar score, nor should these scores dictate resuscitation procedures.

44. The Dubowitz and Ballard scales are used to assess:


A) gestational age after birth
B) responsiveness at birth
C) gestational age before birth
D) fetal distress in utero
Ans: A
Response:
After birth, determination of gestational age involves careful assessment of selected
neuromuscular and physical characteristics using methods developed by Dubowitz and
Ballard. This assessment is normally conducted by a neonatologist or nurse practitioner within
30-40 hours after birth. Although respiratory therapists do not normally conduct this
assessment, you should be familiar with its components.

Page 12
SAN PEDRO COLLEGE COMPETENCY APPRAISAL - OXYGENATION

45. All of the following can be used to assess gestational age or fetal maturity before birth
EXCEPT:
A) ultrasonography
B) Dubowitz/Ballard scale
C) analysis of amniotic fluid
D) time since last menses
Ans: B
Response:
The 3 most common methods used to assess gestational age or fetal maturity are (1) time since
last menses (traditional but unreliable), (2) ultrasonography (accurate and reliable), and (3)
biochemical analysis of amniotic fluid phospholipid levels (most helpful in determining
likelihood of respiratory distress due to surfactant deficiency)

46. A premature infant receiving positive pressure ventilation exhibits acute respiratory distress,
asymmetrical chest motion and hypotension. Which of the following procedures would you
initially recommend?
A) chest transillumination
B) arterial blood gas
C) A-P chest radiograph
D) capillary heal stick
Ans: A
Response:
Transillumination can quickly detect the presence of a pneumothorax in small infants. You
should recommend transillumination for any high-risk infant (especially those receiving
positive pressure ventilation) with clinical signs of unilateral pneumothorax, i.e., retractions,
tachypnea, cyanosis, hypotension, and asymmetrical chest motion. When transillumination
results are positive in infants with these signs (the affected side "lights up"), immediate chest
tube insertion is indicated. On the other hand, if transillumination is negative but the infant
still exhibits these signs, you should recommend an immediately chest X-ray.

47. A patient is cachexic, exhibits generalized edema and dry skin, and appears listless. The most
likely problem is:
A) heart failure
B) Addison's disease
C) renal failure
D) malnutrition
Ans: D
Response:
A weak or emaciated appearance (cachexia); generalized edema (anasarca); cracked lips
(cheilosis); dry, scaly skin; and listlessness are all physical signs associated with severe
malnutrition

Page 13
SAN PEDRO COLLEGE COMPETENCY APPRAISAL - OXYGENATION

48. Which of the following elements of a patient's social history would be relevant in planning a
home-based disease management program?

I. marital status and family relationships


II. living situation and social support
III. diet, exercise and other habits
A) I and II only
B) II and III only
C) I and III only
D) I, II and III
Ans: D
Response:
Key information about a patient's social history include marital status and family
relationships; cultural and religious influences; living situation and social support; education,
employment, finances; diet, exercise and other habits; social activities, hobbies and recreation;
tobacco, alcohol or drug use; and satisfaction/stress with life situation All of this information
would be relevant in planning a home-based disease management program.

49. The wife of a comatose patient asks you to cancel a previously approved DNR order. You
should
A) record the request in the patient record
B) notify the attending physician of the request
C) explain that only her husband can cancel the order
D) notify the charge nurse of the request
Ans: B
Response:
Should a patient or their surrogate ask you to change or revoke an advance directive or DNR
order, you must immediately notify the attending physician, who either cancel the DNR order
or have the new request formalized to replace any prior written documentation.

50. After showing a patient how to perform incentive spirometry, the best way to confirm his
ability to perform it correctly would be to:
A) have him take a multiple choice quiz
B) discuss the information with patient's family
C) have him perform a return demonstration
D) have him "teach" the information back to you
Ans: C
Response:
A return demonstration is the best way to determine whether or not a patient can perform a
procedural skills, Teach-back is appropriate for assessing if the patient has learned critical
information related to their care. Discussion is the best method for assessing attitudinal
change.

51. You note in the medical record that a 50 year old patient for whom you are planning an
instructional activity has a 4th-grade education. Based on this knowledge, you should
A) emphasize oral (vs. written) instruction
B) recommend canceling the activity
C) use "fun and games" approach
D) keep the teaching activity short
Ans: A
Response:
A low educational level is a common barrier to patient learning. To overcome this barrier, you
should (1) emphasize oral (vs. written) instruction; (2) adjust language level as appropriate;
and (3) provide written materials at the appropriate grade level

Page 14
SAN PEDRO COLLEGE COMPETENCY APPRAISAL - OXYGENATION

52. On an AP chest X-ray, a patient exhibits a blunted left costophrenic angle. Which of the
following is the most likely problem?
A) hepatomegaly
B) pleural effusion
C) hyperinflation
D) pneumothorax
Ans: B
Response:
A blunted costophrenic angle is seen in lower lobe pneumonias and pleural effusions.
Hyperinflation or pneumothorax would tent to cause flattening of the diaphragm on the
affected side, while hepatomegaly tends to elevate the right hemidiaphragm.

53. When checking for proper adult placement of an endotracheal or tracheostomy tube on chest
X-ray, the distal tip of the tube should be positioned where?
A) 1 to 3 cm above the carina
B) 1 to 3 cm below the larynx
C) 4 to 6 cm above the carina
D) 23 cm from the base of the tongue
Ans: C
Response:
An AP chest X-ray is the most common method used to confirm proper placement of an
endotracheal or tracheostomy tube. Ideally, the tube tip should be positioned about 4 to 6 cm
above the carina. This normally corresponds to a location between thoracic vertebrae T2 and
T4, or about the same level as the superior border of the aortic knob.

54. A small child is admitted to the Emergency Department with fever, difficulty swallowing,
drooling, and stridor. An AP X-ray of the neck area is negative, but a lateral neck film
indicates supraglottic swelling. Which of the following is the most likely diagnosis?
A) asthma
B) croup
C) foreign body obstruction
D) epiglottitis
Ans: D
Response:
Based on the clinical signs and symptoms and the radiographs, the most likely diagnosis is
epiglottitis. In this condition, the AP X-ray may appear normal, but the lateral neck film often
reveals a prominent shadow caused by the swollen epiglottis, called the "Thumb sign."

55. All of the following abnormalities tend to create areas of increased radiodensity on a chest X-
ray EXCEPT:
A) pneumopericardium
B) atelectasis/consolidation
C) interstitial lung disease
D) pulmonary infiltrates
Ans: A
Response:
The following conditions create areas of increased radiodensity on a chest X-ray: atelectasis,
consolidation, interstitial lung disease, pulmonary infiltrates/edema, pleural effusion,
lung/mediastinal tumors, and calcification. A pneumopericardium would appear as an area of
increased radiolucency (decreased density) around the heart.

Page 15
SAN PEDRO COLLEGE COMPETENCY APPRAISAL - OXYGENATION

56. Ideally, where should a properly positioned pulmonary artery catheter appear on an AP chest
X-ray?
A) in the lower lobe, posteriorly
B) in the superior vena cava or right atrium
C) in the lower lobe, anteriorly
D) in the upper lobe, posteriorly
Ans: A
Response:
A pulmonary artery catheter is used for hemodynamic monitoring and administering fluids
and medications. Accurate wedge pressure measurement requires the catheter tip be visualized
in the lower lobe, ideally posteriorly. Improper placement can result is false readings, e.g.,
pressure in the alveoli pressure instead of the pulmonary veins/left atrium

57. On inspection of a 12-lead ECG, you note the absence of P waves and a variable R-R interval
(> 0.12 sec). Which of the following is the most likely problem?
A) atrial hypertrophy
B) first-degree heart block
C) atrial fibrillation
D) sinus arrhythmia
Ans: C
Response:
A variable R-R interval (> 0.12 sec or > 10% variation) indicates either sinus arrhythmia or
atrial fibrillation. The absence of P waves rules out sinus arrhythmia, making atrial fibrillation
the most likely problem.

58. On inspection of an adult patient's 12-lead ECG, you note a regular R-R interval of 0.40 sec,
with no other apparent abnormalities. Which of the following is the most likely problem?
A) ventricular tachycardia
B) sinus bradycardia
C) atrial fibrillation
D) sinus tachycardia
Ans: D
Response:
In an adult patient, the presence of a regular R-R interval of 0.40 sec (rate = 60/0.40 =
150/min) indicates a sinus tachycardia

59. A 62 year old male patient admitted with chest pain and shortness of breath exhibits a
depressed ST segment on his ECG chest leads. Which of the following is the most likely
problem?
A) myocardial ischemia
B) pneumopericardium
C) hyperkalemia
D) respiratory acidosis
Ans: A
Response:
Causes of a depressed ST segment on the ECG chest leads include: myocardial ischemia;
acute MI, digoxin, ventricular hypertrophy, pulmonary embolism, conduction disturbances,
hyperventilation, and hypokalemia.

Page 16
SAN PEDRO COLLEGE COMPETENCY APPRAISAL - OXYGENATION

60. A doctor requests that you assess an unconscious patient's readiness for weaning from
mechanical ventilation. You could measure all of the following on this patient EXCEPT:
A) spontaneous respiratory rate
B) maximum expiratory pressure
C) spontaneous tidal volume
D) maximum inspiratory pressure
Ans: B
Response:
Assuming there is spontaneous breathing present you could measure an unconscious patient's
spontaneous respiratory rate and tidal volume (and minute volume), as well as the maximum
inspiratory pressure. Patient cooperation is needed to measure a patient's vital capacity and
maximum expiratory pressure. For this reason, these measures cannot be obtained on
unconscious patients

61. Which of the following information is needed to compute a patient's minute volume?

I. PaCO2
II. tidal volume
III. respiratory rate
IV. dead space
A) I and IV only
B) III and IV only
C) II and III only
D) I, II, and IV only
Ans: C
Response:
Minute volume is defined as the product of tidal volume x respiratory rate. These measures
are the only ones needed to calculate a patient's minute volume.

62. Bedside spirometry performed on a patient reveals the following:

Respiratory rate = 22
Tidal volume = 360 mL
Dead space = 150 mL
Inspiratory capacity = 1.0 L

Based on these data, what is the patient's minute ventilation?


A) 3.3 L/min
B) 4.6 L/min
C) 7.9 L/min
D) 22.0 L/min
Ans: C
Response:
Minute volume = f x VT. In this case minute volume = 22 x 360 = 7920 mL/min, which
rounds to 7.9 L/min.

Page 17
SAN PEDRO COLLEGE COMPETENCY APPRAISAL - OXYGENATION

63. Which of the following best represents the normal range for adult patients' minute volume?
A) 10- 15 L/min
B) 12 - 20 L/min
C) 8 - 12 L/min
D) 5 - 10 L/min
Ans: D
Response:
Minute volume is the total volume of air exhaled per minute and equals the patient's rate of
breathing times the average tidal volume (f x VT). The normal range for adults is about 5 - 10
L/min and depends on both body size and metabolic rate.

64. When measuring a patient's peak expiratory flow rate (PEFR) using a peak flow meter, all of
the following are required in order to obtain accurate data EXCEPT:
A) the patient must exhale fully to residual volume
B) the meter must be properly positioned
C) the patient must inhale fully to total lung capacity
D) patient must exhale in a strong, sharp burst
Ans: A
Response:
When measuring a patient's PEFR using a peak flow meter, all of the following are required in
order to obtain accurate data: (1) the metered should be 'zeroed' before the test; (2) the meter
should be properly positioned (some devices are only accurate when held level); (3) the
patient should stand up straight (if possible); (4) the patient must inhale fully to TLC; (5) the
mouthpiece should be inserted between the teeth, above the tongue; (6) the patient should
form a tight seal around the mouthpiece; and (7) the patient should exhale in a strong, sharp
burst with maximum force (active coaching may be needed). Full exhalation to residual
volume is not needed unless the FVC is being measured at the same time.

65. Before administration of a prescribed bronchodilator, an asthmatic patient with moderate


wheezing has a peak expiratory flow rate (PEFR) of 3.7 L/sec. Thirty minutes after aerosol
administration of the drug, the patient's PEFR is 4.1 L/sec. Based on this information you
would:
A) wait another 30 minutes and repeat the test
B) recommend increasing the bronchodilator dosage
C) recommend discontinuing bronchodilator therapy
D) recommend decreasing the bronchodilator dosage
Ans: B
Response:
In this patient's case, the change in PEFR is less than 10% [(4.1 – 3.7)/4.1 = 9.7%], indicates a
lack of significant improvement in airway caliber. In order to relieve the patient's symptoms,
you should recommend increasing the dose of the prescribed bronchodilator.

Page 18
SAN PEDRO COLLEGE COMPETENCY APPRAISAL - OXYGENATION

66. A patient's bedside spirometry results (as compared to normal) are as follows:

FVC normal
FEV1 decreased
FEV1% decreased

What is the most likely problem?


A) an obstructive disorder
B) poor patient effort
C) a restrictive disorder
D) a mixed (obstructive + restrictive) disorder
Ans: A
Response:
A patient with a decreased FEV1 and decreased FEV1% (even with a normal FVC is
exhibiting the classic pattern of an obstructive pulmonary disorder, i.e. decreased expiratory
flows.

67. A patient has a vital capacity of 3200 mL, a functional residual capacity of 4500 mL and
expiratory reserve volume of 1200 mL. What is his total lung capacity (TLC)?
A) 8900 mL
B) 2000 mL
C) 3300 mL
D) 6500 mL
Ans: D
Response:
First, RV = FRC - ERV. In this case, RV = 4500 - 1200 = 3300 mL. Second, TLC = VC +
RV. In this case, TLC = 3200 mL + 3300 mL = 6500 mL

68. A patient has an FRC of 2900 mL measured by helium dilution but a thorax gas volume of
4100 mL as measured by body plethysmography. Which of the following is the most likely
diagnosis?
A) bullous emphysema
B) pulmonary atelectasis
C) interstitial lung disease
D) kyphoscoliosis
Ans: A
Response:
Helium dilution and nitrogen washout methods both measure the actual FRC (lung volume
communicating with the airways). Body plethysmography measures the total thorax gas
volume (TGV). Normally FRC = TGV. When TGV > FRC (as in this case), "trapped' gas (air
not in communication with the airways) is present. This is a common finding in bullous
emphysema.

Page 19
SAN PEDRO COLLEGE COMPETENCY APPRAISAL - OXYGENATION

69. After performing comprehensive pulmonary function testing on a patient, you note the
following results:

Test % Predicted
FEV1% 110%
VC 58%
TLC 65%
DLco 102%

Which of the following interpretations is most consistent with these findings?


A) The patient has interstitial lung disease
B) The patient has a pulmonary vascular disorder
C) The patient has a neuromuscular disorder
D) The patient is not exerting maximum effort
Ans: C
Response:
The low lung volumes (VC, TLC) and higher than normal FEV1% confirm a restrictive
disorder. The normal DLco rules out both interstitial lung disease, and (along with the reduced
volumes) a pulmonary vascular problem. With reduced volumes, normal or high flows and a
normal DLco, the most likely cause of the restriction is either a neuromuscular or chest wall
disorder.

70. The proper starting point for FRC measurement via helium dilution or nitrogen washout is:
A) end of a maximum exhalation
B) end of a normal resting inspiration
C) end of a maximum inhalation
D) end of a normal resting exhalation
Ans: D
Response:
The validity of FRC measurement via either helium dilution or nitrogen depends on proper
starting point, i.e., the end of a normal resting expiration. In addition, it is critical that the
spirometer and breathing circuit be leak free and that the gas analyzers be properly calibrated.

71. As measured by the single breath DLco method, the diffusing capacity of the lungs would be
decreased in all of the following EXCEPT:
A) pulmonary embolism
B) pulmonary emphysema
C) severe anemia
D) secondary polycythemia
Ans: D
Response:
The DLco is low in conditions that actually impair membrane diffusion (as in pulmonary
fibrosis) or decrease surface area (as in emphysema). The DLco can also be less than normal
with reduced Hb (as in anemia), decreased pulmonary capillary blood flow, or decreased
alveolar volume are. Increases in DLco occur with increased Hb (as in secondary
polycythemia), increased pulmonary blood flow, increased alveolar volume, and during
exercise.

Page 20
SAN PEDRO COLLEGE COMPETENCY APPRAISAL - OXYGENATION

72. All of the following patient categories should be considered for apnea monitoring EXCEPT:
A) infants at risk of recurrent apnea, bradycardia and hypoxemia
B) infants with airway abnormalities that increase the risk of obstruction
C) infants or children being assessed for obstructive sleep apnea
D) infants receiving drug therapy for a history of apnea and bradycardia
Ans: C
Response:
Apnea monitoring is indicated in infants at risk of recurrent apnea, bradycardia and
hypoxemia. This procedure may also be considered for infants: receiving drug therapy for
apnea; with bronchopulmonary dysplasia; with symptomatic gastroesophageal reflux; born to
substance abusing mothers (if clinically symptomatic); with a trach or an airway abnormality
that increases the risk of obstruction; and with neurologic or metabolic disorders affecting
respiratory control. Because apnea monitors only measure chest wall movement and not air
flow, they cannot detect airway obstruction and thus should not be used to assess patients for
obstructive sleep apnea-hypopnoea syndrome.

73. Which of the following alarm limits would be appropriate for setting on an apnea monitor
being used on a neonate being discharge to the home setting?
A) an apnea alarm limit of 20 seconds and a low/high heart rate range of 40-120/min
B) an apnea alarm limit of 60 seconds and a low/high heart rate range of 80-220/min
C) an apnea alarm limit of 20 seconds and a low/high heart rate range of 80-220/min
D) an apnea alarm limit of 40 seconds and a low/high heart rate range of 80-120/min
Ans: C
Response:
Most modern apnea monitors designed for home use has a setup program by which you
provide patient and time/date information and set the alarm and event limits. The most
common settings for neonates are: (1) an apnea alarm limit of 20 seconds and (2) a low/high
heart rate range of 80-220/min.

74. Which of the following overnight oximetry results would tend to confirm a diagnosis of sleep
apnea-hypopnoea syndrome?
A) > 15 desaturation events/hour
B) 10 – 15 desaturation events/hour
C) 5 – 10 desaturation events/hour
D) 1 – 5 desaturation events/hour
Ans: A
Response:
In general, an oxygen desaturation index of 15 or more desaturation events/hour (SpO2 falling
by at least 4%) confirms the presence of sleep apnea-hypopnoea syndrome. Patients who
exhibit less than 15 desaturation events/hour should undergo full laboratory polysomnography
to diagnose SAHS and help determine its cause.

75. Based on overnight oximetry results, a COPD patient receiving continuous long-term oxygen
therapy at 1 L/min exhibits a drop in SpO2 from 90% to 84% at night. Which of the following
would you recommend for this patient?
A) have the patient undergo polysomnography to determine the cause of the desaturation
B) titrate the patient's nocturnal O2 flow upward until the SpO2 remains above 88%
C) change the continuous oxygen therapy prescribed flow from 1 to 2 L/min
D) titrate the patient's nocturnal O2 flow upward until the SpO2 remains above 92%
Ans: B
Response:
For COPD patients already certified for continuous long-term oxygen therapy who also
exhibit nocturnal desaturation, the liter flow can be titrated upward in 1 L/min increments
until the nighttime SpO2 consistently exceeds 88% and no further desaturation events occur.

Page 21
SAN PEDRO COLLEGE COMPETENCY APPRAISAL - OXYGENATION

76. A patient undergoing CPAP titration for sleep apnea exhibits prolonged periods of central
sleep apnea during the procedure, even at a baseline pressure of 15 cm H2O. Which of the
following would you recommend for this patient?
A) titrate the CPAP pressure up to 20 cm H2O and reassess
B) discontinue the titration and recommend a trial of a xanthine
C) consider a trial of bi-level positive airway pressure (BiPAP)
D) switch to an auto-CPAP unit and continue the titration
Ans: C
Response:
If during CPAP titration (1) the patient cannot tolerant high CPAP pressures, (2) there are
continued obstructive respiratory events at higher levels of CPAP (> 15 cm H2O), or (3) the
patient exhibits periods of central sleep apnea during titration, you should consider a trial of
BiPAP, starting at EPAP = 4 cm H2O and IPAP = 8 cm H2O.

77. Which of the following tests would you recommend in order to identify the cause of dyspnea
and factors limiting a patient's exercise tolerance?
A) 6-minute walking distance
B) overnight oximetry assessment
C) peak expiratory flow rate
D) comprehensive exercise test
Ans: D
Response:
To identify the cause of dyspnea and factors limiting a patient's exercise tolerance, you would
need to conduct a comprehensive cardiopulmonary exercise test. The 6-minute walk test only
evaluates how well the body as a whole responds to exertion. Its use therefore is limited to
determining overall functional capacity or changes in capacity due therapy

78. All of the following should be measured before and after the 6-minute walk test EXCEPT:
A) ECG
B) level of dyspnea
C) SpO2
D) level of exertion
Ans: A
Response:
Before beginning a 6-minute walk test, you should gather the needed demographic data and
measure/record the patient's vital signs, including the baseline SpO2. Then have the patient
stand and rate their baseline dyspnea and exertion levels using the Borg scale. At the end of
the test, repeat the Borg scale assessment of dyspnea and exertion levels, being sure to remind
the patient of their prior ratings, In addition, record the end-of-walk SpO2 and pulse rate from
the oximeter. Normally, if the patient has had a recent resting ECG, it is reviewed by a
physician before testing.

Page 22
SAN PEDRO COLLEGE COMPETENCY APPRAISAL - OXYGENATION

79. During the middle of a 6-minute walk test, a patient complains of chest pain, exhibits
diaphoresis and appears very pale. Which of the following actions should you immediately
take?

I. administer O2 as appropriate
II. re-take the vital signs
III. sit the patient down
A) I and II only
B) II and III only
C) I and III only
D) I, II and III
Ans: D
Response:
You should immediately stop a 6MWT if the patient complains develops chest pain,
intolerable dyspnea, leg cramps, staggering, diaphoresis, or a pale or ashen appearance. In
these cases, sit the patient in the chair, re-take the vital signs, administer O2 as appropriate and
arrange for a physician assessment. Once you are sure the patient is stable, record the time
stopped, distance walked and the reason the patient could not continue.

80. Based on the results of cardiopulmonary exercise testing, which of the following patients most
likely has a cardiovascular limitation to exercise?

Patient VO2max Anaerobic Threshold Breathing Reserve


A. Decreased Decreased Normal
B. Decreased Normal Normal
C. Normal Increased Increased
D. Decreased Normal Decreased
A) Patient A
B) Patient B
C) Patient C
D) Patient D
Ans: A
Response:
All patient with poor exercise capacity have a reduced VO2max. In addition, patients with a
cardiovascular limitation to exercise will exhibit a decreased anaerobic threshold, but may
have a normal breathing reserve. Patients with a pulmonary limitation to exercise typically
have a normal anaerobic threshold (if it can be reached), but a reduced breathing reserve. In
the presence of a low VO2max, poor effort is revealed by both a normal anaerobic threshold
and breathing reserve.

81. To compute a patient's breathing reserve you should:


A) multiply the forced expiratory volume in one second by 40
B) subtract the maximum minute volume at peak exercise capacity from the MVV
C) divide the MVV by the maximum minute volume at peak exercise
D) subtract the MVV from the maximum minute volume at peak exercise capacity
Ans: B
Response:
Breathing reserve is defined as the difference between a patient's maximum voluntary
ventilation (MVV) and the maximum ventilation he or she can achieve at peak exercise
capacity. Based on this definition, to compute the breathing reserve you simply subtract the
maximum minute volume at peak exercise capacity from the MVV. If the patient's MVV is
not available, you can estimate it as being equal to the FEV1 x 40.

Page 23
SAN PEDRO COLLEGE COMPETENCY APPRAISAL - OXYGENATION

82. Which of the following equipment can be used to control a patient's exercise level during an
oxygen titration test?

I. 6-minute walk test


II. cycle ergometer
III. treadmill
A) I and II only
B) II and III only
C) I and III only
D) I, II and III
Ans: C
Response:
Ideally, a treadmill should be used to control a patient's exercise level during an oxygen
titration test. If a treadmill is not available, a step test or the 6-minute walk test can be
substituted. A cycle ergometer is not recommended for O2 titration. This is because patients'
O2 needs during exercise must be established while carrying the portable system they use or
that is planned for use.

83. After three minutes of peak activity during an oxygen titration test, a patient's SpO2 drops
from 90% to 87%. Which of the following actions is indicated?
A) continue the test for another three minutes at the same liter flow
B) increment the O2 flow by 1 liter per minute and continue the test
C) immediately terminate the test and assess the patient's vital signs
D) increment the O2 flow by 1 liter per minute and terminate the test
Ans: B
Response:
During an oxygen titration test, you have the patient maintain their peak activity level for at
least 3 minutes, then measure the SaO2. If the patient's saturation at the peak activity level is 
88%, the patient does not need any additional oxygen and the test can be terminated. If on the
other hand, the patient's SaO2 drops by 2% or more or if the SaO2 < 88% (PaO2  55 torr),
increment their O2 flow by 1 L/min (up to 6 L/min). After stabilization on the new O2 flow for
3 minutes, re-assess the SaO2 while the patient continues to exercise. Repeat this procedure
until the SaO2 is at least 88% or the PaO2 is above 55 torr. Note that for safety purposes, the
AARC recommends a target SaO2 of 93%.

84. The normal range for the pulmonary capillary wedge pressure (PCWP) as measured via the
distal port of a Swan-Ganz catheter (with the balloon inflated) is:
A) 20 - 30 mm Hg
B) 4 - 12 mm Hg
C) 10 - 20 mm Hg
D) 0 - 4 mm Hg
Ans: B
Response:
The pulmonary capillary wedge pressure (PCWP) normally ranges between 4-12 mm Hg.
PCWP reflects downstream pressure in the pulmonary circulation, i.e., pulmonary venous
pressure. PCWP normally rises during positive pressure breaths. Abnormal increases in
PCWP can indicate 1)LV failure/cardiogenic shock; 2) hypervolemia; 3) cardiac
tamponade/constrictive pericarditis; 4) mitral stenosis; 5) positive pressure ventilation/peep;
and 6) pneumothorax. PCWP normally falls during spontaneous inspiration. Abnormal
decreases in PCWP indicate either noncardiogenic shock or hypovolemia.

Page 24
SAN PEDRO COLLEGE COMPETENCY APPRAISAL - OXYGENATION

85. If the rate of breathing increases without any change in total minute ventilation (VE constant):
A) the alveolar ventilation per minute will increase
B) the alveolar ventilation per minute will decrease
C) the deadspace ventilation per minute will decrease
D) the alveolar ventilation per minute will remain constant
Ans: B
Response:
Due to the influence of deadspace, high rates and/or low tidal volumes result in a high percent
of wasted ventilation per minute. Thus, if the rate of breathing increases without any change
in minute ventilation (VE constant), alveolar ventilation per minute will decrease.

86. Which of the following formula is used to calculate cardiac output?


A) rate (f) x stroke volume (SV)
B) stroke volume (SV)/rate (f)
C) blood pressure (BP) x stroke volume (SV)
D) blood pressure (BP)/stroke volume (SV)
Ans: A
Response:
The total amount of blood pumped by the heart per minute, or cardiac output, is simply the
product of the heart rate times the volume ejected by the left ventricle on each contraction, or
stroke volume.

87. The normal C(a-v)O2 in a healthy adult at rest is about:


A) 5 ml/dL
B) 15 ml/dL
C) 20 ml/dL
D) 250 ml/dL
Ans: A
Response:
The difference between the arterial and venous oxygen contents is normally about 5 ml/dL.
This arterial-venous oxygen contents difference, or C(a-v)O2, represents the amount of
oxygen given up to the tissues by every 100 ml of blood on each pass through the systemic
capillaries. Obviously, this value reflects the mean of the body as a whole, with different
organ systems extracting more or less oxygen according to need.

88. According to Fick principle, if the oxygen consumption remains constant, a DECREASE in
cardiac output will manifest itself as:
A) an increase in the CaO2
B) a decrease in the C(a-v)O2
C) an increase in the C(a-v)O2
D) an increase in the CvO2
Ans: C
Response:
According to the Fick equation, the C(a-v)O2 (indicating oxygen extraction in proportion to
blood flow), together with the total body oxygen consumption (VO2) may be used to calculate
cardiac output: CO = VO2/[C(a-v)O2 x 10]. Based on this formula, if the O2 consumption
remains constant, a decrease in cardiac output will manifest itself by an increase in the C(a-
v)O2.

Page 25
SAN PEDRO COLLEGE COMPETENCY APPRAISAL - OXYGENATION

89. Normally, an individual can maintain about what percent of their maximum voluntary
ventilation (MVV) on maximum exercise?
A) 60-70%
B) 70-80%
C) 80-90%
D) 90-100%
Ans: A
Response:
At peak exercise, normal individuals use only about two-thirds or less of their MVV to meet
their metabolic demands, meaning that they still have a have a breathing reserve of at least
30%, Patients with pulmonary disorders that limit exercise typically have a breathing reserve
< 30%. In addition, these patients also may exhibit a reduced SaO2 during exercise.

90. A patient in shock exhibits the following cardiovascular responses: an INCREASED


pulmonary artery pressure, an INCREASED systemic vascular resistance, and a
DECREASED cardiac output. Given these data, the most likely type of shock is:
A) hypovolemic shock
B) neurogenic shock
C) septic shock
D) cardiogenic shock
Ans: D
Response:
In combination, an INCREASE in pulmonary artery pressure, an INCREASE in systemic
vascular resistance, and a DECREASE in cardiac output are all consistent with cardiogenic
shock. In this case, the key distinguishing feature is the increase in pulmonary artery pressure,
which occurs only in cardiogenic shock.

91. A patient in shock exhibits the following cardiovascular measures: a DECREASE in


pulmonary artery pressure, an INCREASE in systemic vascular resistance, and a DECREASE
in cardiac output. Given these data, the most likely type of shock is:
A) hypovolemic shock
B) neurogenic shock
C) septic shock
D) cardiogenic shock
Ans: A
Response:
In combination, a DECREASE in pulmonary artery pressure, an INCREASE in systemic
vascular resistance, and a DECREASE in cardiac output are all consistent with hypovolemic
shock. These findings are also consistent with hypodynamic septic shock.

92. While assisting a physician who is inserting a pulmonary artery catheter, you note a
changeover on the monitor from pulsatile pressures of about 25/5 mm Hg to pulsatile
pressures of 25/15 mm Hg. Which of the following has occurred?
A) the catheter has advanced from right atrium to right ventricle
B) the catheter has moved from right ventricle to pulmonary artery
C) the catheter has advanced into the pulmonary wedge position
D) the catheter has moved from the vena cava into the right atrium
Ans: B
Response:
During insertion of a pulmonary artery catheter, pressure waveforms indicate its position. In
the vena cava/right atrium (RA), pressures are < 10 mm Hg and barely pulsatile. As the
catheter moves into the right ventricle, a changeover to pulsatile pressures of about 25/5 mm
Hg occurs. As the catheter passes into the pulmonary artery (PA), pulsatile pressures
continue, but with a raised diastolic baseline (about 25/12 mm Hg). In the "wedge" position
(PCWP), strong pulsations are lost, and pressures drops to 4-12 mm Hg.

Page 26
SAN PEDRO COLLEGE COMPETENCY APPRAISAL - OXYGENATION

93. At the end of each graded step of a cardiopulmonary exercise test, you normally would
measure all of the following EXCEPT:
A) blood pressure
B) heart rate
C) percent shunt
D) Borg exertion rating
Ans: C
Response:
Toward the end of each graded interval of an exercise test, you typically measure the patient's
blood pressure, heart rate, SpO2, Borg exertion rating, and symptoms (if any). You sometimes
may need to draw an ABG specimen (for blood gas values and lactate levels) at each step in
the protocol.

94. The systemic arterial pressure provides information on all of the following hemodynamic
parameters EXCEPT:
A) left ventricular afterload
B) vascular tone
C) left ventricular preload
D) blood volume
Ans: C
Response:
The systemic arterial pressure provides information valuable in assessing left ventricular
afterload, vascular tone, and blood volume. To assess left ventricular preload (filling pressure)
you would need to measure the pulmonary capillary wedge pressure (PCWP).

95. A balloon-tipped pulmonary artery catheter can provide information on all of the following
hemodynamic parameters EXCEPT:
A) left ventricular afterload
B) pulmonary vascular tone
C) left ventricular preload
D) blood volume/cardiac output
Ans: A
Response:
A balloon-tipped pulmonary artery catheter can provide information valuable in assessing
pulmonary vascular tone, blood volume/cardiac output, and left ventricular preload (filling
pressure). To assess left ventricular afterload you would need to measure the systemic arterial
pressure.

Page 27
SAN PEDRO COLLEGE COMPETENCY APPRAISAL - OXYGENATION

96. A patient receiving mechanical ventilation with 8 cm H2O PEEP has a balloon-tipped
pulmonary artery catheter in place. To obtain an accurate measurement of her pulmonary
capillary wedge pressure (PCWP) you would:

I. level the transducer to 4th intercostal space/midaxillary line


II. remove the patient from the ventilator and PEEP
III. make the PCWP measurement at end-expiration
A) I and II only
B) II and III only
C) I and III only
D) I, II and III
Ans: C
Response:
For accurate central venous and pulmonary artery pressure measurements, you need to level
the transducer at the patient's plebostatic axis, i.e., the intersection of 4th intercostal space
with midaxillary line. To minimize the affect of changes in intrathoracic pressure during
breathing on PCWP, you should make your measurements at end-expiration Do not remove
patients from PEEP/CPAP to measure PCWP; if PEEP  10 cm H2O simply obtain the end-
expiratory reading; if PEEP > 10 cm H2O apply the following correction formula: corrected
value = measured value – [.5 x (PEEP/1.36)]

97. All of the following can cause a damped waveform when measuring pulmonary artery
pressures with a transducer system EXCEPT:
A) catheter tip against vessel wall
B) transducer not positioned at plebostatic axis
C) partial occlusion of catheter tip by clot
D) air bubbles in system
Ans: B
Response:
Causes of damped pressure waveforms when measuring pulmonary artery pressures with a
transducer system include: catheter tip against vessel wall; partial occlusion of catheter tip by
clot, presence of a clot in stopcock or transducer; and air bubbles in transducer or connector
tubing. A change in the transducer reference level would tend to cause unexpectedly high or
low pressure readings.

98. You note on the vascular pressure monitor of a conscious patient in no apparent distress that
the arterial pressure waveform is absent (pressure reading = 0 mm Hg), and the alarm is
sounding. You would consider all of the following actions appropriate EXCEPT:
A) calling a code
B) checking the stopcock position
C) aspirating line/flush with heparin
D) confirming monitor not set to zero/cal
Ans: A
Response:
An absent pressure waveform with low pressure could indicate cardiac arrest, but not in a
conscious patient in no apparent distress. Instead, you need to troubleshoot this apparent
equipment problem. An absent pressure waveform may indicate an occluded catheter (aspirate
the line and flush with heparin); catheter positioned out of the vessel (notify doctor and
prepare to replace line); stopcock off to patient (position stopcock correctly); loose vascular
line or electrical connections (tighten loose connections); monitor set to zero, cal, or off (make
sure monitor set to proper function/display); or incorrect scale selection (select appropriate
scale, e.g., arterial = high/venous = low).

Page 28
SAN PEDRO COLLEGE COMPETENCY APPRAISAL - OXYGENATION

99. A patient has a systolic arterial pressure of 180 mm Hg and a diastolic value of 90 mm Hg.
What is his approximate mean arterial pressure?
A) 100 mm Hg
B) 110 mm Hg
C) 120 mm Hg
D) 130 mm Hg
Ans: C
Response:
You estimate mean vascular pressures using the following formula: mean pressure = diastolic
+ 1/3 x (systolic – diastolic). In this case, the estimated mean arterial pressure = 90 + 1/3 x
(180 - 90) = 90 + 30 = 120 mm Hg.

100. A patient has a lower than normal mixed venous O2 content. All of the following could cause
this condition EXCEPT:
A) cardiogenic shock
B) hyperthermia
C) cyanide poisoning
D) hypovolemia
Ans: C
Response:
A lower than normal mixed venous O2 content (CvO2) indicates impaired tissue oxygenation.
A low CvO2 can be due to reduced oxygen delivery (decreased Hb, PO2, or cardiac output), or
increased oxygen demand. Cardiogenic shock and hypovolemia both decrease cardiac output,
while hyperthermia increases oxygen demand. In cyanide poisoning, the CvO2 can be higher
than normal, even though tissue hypoxia may be present.

101. A patient breathing 100% O2 has a P(A-a)O2 of 400 torr. What is her approximate % shunt?
A) 5%
B) 10%
C) 15%
D) 20%
Ans: D
Response:
In estimating the percent shunt, with the FIO2 = 1.0 (100% O2), every 100 torr P(A-a)O2
equals about a 5% shunt. In this case, you would estimate the %shunt as 400/100 = 4 x 5 =
20%

102. A patient has a body surface area (BSA) of 2.0 m2 and a cardiac output (CO) of 3.0 L/min.
What conclusions can you draw regarding the patient's cardiac index?
A) the patient's cardiac index is below normal
B) the patient's cardiac index is normal
C) the patient's cardiac index is above normal
D) insufficient data to compute the cardiac index
Ans: A
Response:
Cardiac index (CI) = CO/BSA. In this case CI = 3.0/2.0 = 1.5 L/min/m2. Because a normal
range for cardiac index is 2.5-5 L/min/m2, this patient's cardiac index is below normal.

Page 29
SAN PEDRO COLLEGE COMPETENCY APPRAISAL - OXYGENATION

103. In observing a patient, you note a cycle of gradual increases and decreases in the depth of
breathing, followed by a period of apnea. Which of the following terms would you use to
chart this observation?
A) Biot's breathing
B) Cheyne-Stokes breathing
C) Kussmaul's breathing
D) paradoxic breathing
Ans: B
Response:
Cheyne-Stokes breathing is an irregular pattern characterized by gradual increases and
decreases in the depth of breathing, usually followed by a period of apnea. Cheyne-Stokes
breathing may be caused by CNS disorders or congestive heart failure.

104. In observing a patient, you note that her breathing is extremely deep and fast. Which of the
following terms would you use in charting this observation?
A) Kussmaul's breathing
B) Biot's breathing
C) Cheyne-Stokes breathing
D) apneustic breathing
Ans: A
Response:
Kussmaul's breathing is an abnormal pattern characterized by deep and fast respirations.
Kussmaul's breathing is usually associated with the body's attempt to compensate for a
metabolic acidosis, as in diabetic ketoacidosis.

Page 30
CHAPTER 5

Name: __________________________ Date: _____________

1. A patient admitted to the Emergency Department is suspected of having suffered airway


injury due to inhalation of toxic fumes. To determine the location and extent of potential
injury you would recommend which of the following procedures?
A) V/Q scan
B) chest X-ray
C) blood gas analysis
D) bronchoscopy
Ans: D
Response:
Injury from toxic inhalation or aspiration most immediately affects the airways. In these
patients the location and extent of injury is best determined initially using fiberoptic
bronchoscopy.

2. Your patient in ICU is suspected of having developed a bacterial ventilator-associated


pneumonia. Which of the following procedures would you recommend as best able to
diagnose the cause of this problem?
A) sputum culture and sensitivity
B) chest X-ray
C) bronchoalveolar lavage
D) CT scan
Ans: C
Response:
A sputum culture and sensitivity could help diagnose this problem. However, bronchoalveolar
lavage (BAL) is one of the principal best tools available to diagnose bacterial ventilator-
associated pneumonia (VAP), and is thus the better choice for this patient. According to the
American Thoracic Society, bronchoalveolar lavage (BAL) is indicated in patients with non-
resolving pneumonia, unexplained lung infiltrates (interstitial and/or alveolar), and suspected
alveolar hemorrhage.

3. To estimate the metabolic rate of a patient receiving mechanical ventilation, you would
recommend:
A) hemoximetry
B) ABG analysis
C) capnography
D) maximum voluntary ventilation
Ans: C
Response:
Capnography (expired CO2 analysis) is used primarily to noninvasively monitor the
effectiveness (PetCO2) and efficiency (PaCO2-PetCO2) of ventilation, usually during
mechanical ventilation. Most capnographs also can compute the accumulated volume of CO2
over time if the expired minute ventilation is known or measured. CO2 production per minute
is one measure of metabolic activity, the other being O2 consumption.

Page 1
4. To evaluate and follow the course of a patient with interstitial lung diseases, which of the
following pulmonary function testing procedures would you recommend?
A) diffusing capacity (DLco)
B) He dilution FRC and TLC
C) forced expiratory volumes/flows
D) methacholine challenge test
Ans: A
Response:
The primary indication for the carbon monoxide diffusing capacity (DLco) test is to evaluate
and follow the course of parenchymal and interstitial lung diseases such as pulmonary
fibrosis, pneumoconiosis and sarcoidosis, In addition the DLco test can be used to
differentiate among the various patterns of airway obstruction (emphysema patients typically
have a low DLco) and is helpful in following the course of emphysema and cystic fibrosis.
The DLco test also can help predict arterial desaturation during exercise in patients with lung
disease.

5. Which of the following tests of lung mechanics would you recommend to detect the presence
of auto-PEEP on a patient receiving ventilatory support?
A) pressure-volume loop
B) flow-volume loop
C) static compliance (inspiratory hold)
D) airway resistance (inspiratory hold)
Ans: B
Response:
Of the tests listed, only the flow-volume loop would help detect the presence of auto-PEEP on
a patient receiving ventilatory support. When viewing the flow-volume loop of a patient with
auto-PEEP, you will note that the expiratory flow does not return baseline before start of next
breath.

6. The wife of a patient receiving post-operative incentive spirometry asks if this therapy will
help get rid of his snoring, daytime sleepiness, and morning headaches. In communicating this
information to the patient's surgeon, you would recommend which of the following diagnostic
procedures?
A) lateral neck X-ray
B) arterial blood gas
C) polysomnography
D) diffusing capacity
Ans: C
Response:
You should recommend polysomnography for patients who complain of or exhibit signs or
symptoms associated with sleep-disordered breathing, e.g., daytime somnolence and fatigue;
morning headaches; pulmonary hypertension, and polycythemia.

7. To continuously monitor the adequacy of ventilation of a patient in ICU being supported by


mask BiPAP™, you would recommend which of the following?
A) transcutaneous PCO2
B) pulse oximetry
C) ABG analysis
D) capnography
Ans: A
Response:
Although traditionally used only with infants and children, recent research indicates that the
transcutaneous PCO2 is an accurate measure of ventilation in hemodynamically stable adults,
making it a good choice for continuously monitoring of ventilation when capnography is
unavailable or impractical, e.g., during noninvasive ventilation.

Page 2
8. As you are fitting him with a nonrebreathing mask, a 62 YO patient in the Emergency
Department complains of severe chest pain. Which of the following tests would you first
recommend for this patient?
A) arterial blood gas
B) electrocardiogram (ECG)
C) V/Q scan
D) bedside spirometry
Ans: B
Response:
You should recommend obtaining an electrocardiogram to screen for heart disease, rule out
heart disease in surgical patients, evaluate patients with chest pain, follow the progression of
patients with CAD and evaluate heart rhythm disorders

9. To assess tissue oxygenation in a patient with ARDS, you would recommend which of the
following?
A) a CVP line
B) a pulmonary artery catheter
C) an arterial line
D) pulse oximetry
Ans: B
Response:
To assess how well the tissues are getting oxygenated, we need to know how much oxygen is
left over after the blood leaves the capillaries. This measure, called the mixed venous oxygen
content (CvO2) can only be obtained from the distal port of a pulmonary artery catheter.

10. You would consider recommending all of the following procedures to help diagnose a
pulmonary embolism EXCEPT:
A) helical CT scan
B) angiography
C) V/Q scan
D) chest X-ray
Ans: D
Response:
Traditionally, V/Q scans have been used as the screening test for pulmonary embolism, with
pulmonary angiography applied as the 'gold standard' to confirm the diagnosis. Helical CT
angiography is replacing both of these tools in diagnosing pulmonary embolism. A standard
chest X-ray does not reliably detect pulmonary embolism.

11. A two year-old child is admitted to the Emergency Department with stridor, nasal flaring,
tachypnea and inspiratory retractions. Which of the following procedures would you
recommend to help diagnose this patient's problem?
A) ventilation-perfusion scan
B) AP and lateral neck X-rays
C) arterial blood gas analysis
D) bedside spirometry
Ans: B
Response:
Stridor and respiratory distress in children indicates upper airway obstruction, usually due to
either infection (croup or epiglottitis) or aspirated foreign bodies. In combination, AP and
lateral neck X-rays can help differentiate among these problems.

Page 3
12. Which of the following would you recommend to help guide a physician in locating the
appropriate needle insertion site for thoracentesis performed at the bedside?
A) ultrasound
B) CT scan
C) thoracic MRI
D) bronchoscopy
Ans: A
Response:
Thoracic ultrasound is indicated to guide thoracentesis and percutaneous needle biopsies. In
addition, this toll can be used to detect (1) free fluid in the thorax (pleural or pericardial
effusion, hemithorax, etc.); (2) pneumothorax; (3) mediastinal masses; and (4) pulmonary
atelectasis or consolidation. Thoracic ultrasound can also be used to assess the pleural surfaces
for pleuritis or granulomatous processes and identify thoracic wall lesions and rib masses.
Last, ultrasound exams can help diagnose trauma to the diaphragm, heart and large thoracic
blood vessels, as well as fractures of the ribs and sternum.

13. You normally should recommend AGAINST performing a diagnostic bronchoscopy on which
of the following patients?

I. a patient with severe refractory hypoxemia


II. a patient who is hemodynamically unstable
III. a patient with a an uncorrected bleeding disorder
A) I and II only
B) II and III only
C) I and III only
D) I, II and III
Ans: D
Response:
According to the AARC, you should recommend against performing diagnostic bronchoscopy
in patients who (1) cannot be adequately oxygenated during the procedure (severe refractory
hypoxemia); (2) have a bleeding disorder that cannot be corrected; (3) have severe obstructive
airways disease; and (4) are hemodynamically unstable.

14. An ambulatory care patient has a white blood cell count (WBC) of 20,000, along with
increased sputum production. Which of the following tests would you recommend for this
patient?
A) sputum Gram stain, culture & sensitivity
B) arterial blood gas analysis
C) lab spirometry (FVC volumes/ flows)
D) bronchoalveolar lavage
Ans: A
Response:
The patient's WBC is elevated above normal (10,000), suggesting an acute bacterial infection.
You should recommend a sputum Gram stain and culture and sensitivity (C&S) on any patient
suspected of having a respiratory track infection. By identifying whether the organisms are
primarily Gram+ or Gram-, the Gram stain can be used to as a general guide to initial
antibiotic therapy. By determining which specific antibiotics that the organisms are
susceptible to, the subsequent C&S study can help the doctor decide on the most appropriate
drug or drugs to prescribe for the patient

Page 4
15. Due to her patient's minimal response to the standard prescription for an aerosolized
bronchodilator, a doctor asks your advice on how best to adjust the dosage. You would
recommend:
A) peak expiratory flow rate monitoring
B) methacholine challenge (provocation) test
C) carbon monoxide diffusing capacity (DLco)
D) pre/post bronchodilator spirometry
Ans: D
Response:
At this stage in the patient's management, the best way to determine if a change in dose,
frequency, or medication is needed for this patient would be pre/post bronchodilator
spirometry. Peak expiratory flow rate monitoring is used primarily to assess asthma patients'
airway tone over time, whereas the methacholine challenge test is used mainly to assess the
severity of airway hyperresponsiveness or evaluate occupational asthma

16. A doctor asks your advice on the best way for his home care asthma patient to assess changes
in her airway tone over time. You would recommend:
A) peak expiratory flow rate monitoring
B) methacholine challenge (provocation) test
C) carbon monoxide diffusing capacity (DLco)
D) pre/post bronchodilator spirometry
Ans: A
Response:
Peak expiratory flow rate monitoring is the primary means by which asthma patients can
assess their airway tone over time, as well as changes in tone in response to bronchodilator
therapy. Pre/post bronchodilator spirometry is used primarily to determine the effectiveness of
bronchodilator therapy or the need for a change in the drug dose or frequency of
administration. Methacholine challenge testing is used mainly to assess the severity of airway
hyperresponsiveness or evaluate occupational asthma

17. A doctor suspects that a patient's asthma-like symptoms are due to airway hyperreactivity. She
asks your advice on the best way to confirm this diagnosis. You would recommend:
A) peak expiratory flow rate monitoring
B) bronchial provocation testing
C) carbon monoxide diffusing capacity
D) pre/post bronchodilator spirometry
Ans: B
Response:
Bronchial provocation testing (aka methacholine challenge testing) is used to confirm or
exclude a diagnosis of airway hyperreactivity. It also may be used to determine the relative
risk of developing asthma, evaluate patients for occupational asthma, and assess the response
to therapeutic interventions.

18. Which of the following tests of lung mechanics would you recommend to detect suspected
overinflation on a patient receiving ventilatory support?
A) pressure-volume curve
B) flow-volume curve
C) static compliance (inspiratory hold)
D) airway resistance (inspiratory hold)
Ans: A
Response:
On patients receiving ventilatory support, a pressure-volume curve can detect changes in
compliance (slope of curve) and resistance (width of curve), as well as suspected overinflation
("beaking" appearance). A pressure-volume curve can also help determine the optimum PEEP
level (just above lower inflection point).

Page 5
19. Which of the following would you recommend to determine actual blood O2 saturation of a
patient being treated for methemoglobinemia?

I. blood gas analysis


II. hemoximetry (CO-oximetry)
III. pulse oximetry
A) I or II
B) II or III
C) II only
D) I, II or III
Ans: C
Response:
Methemoglobin (metHb) is an abnormal hemoglobin most commonly cause by environmental
exposure to oxidizing drugs and their metabolites (such as benzocaine, dapsone and nitrates).
Only hemoximetry (CO-oximetry) can accurately metHb and other common abnormal
hemoglobins (HbCO, sulfhemoglobin) and determine actual blood O2 saturation (as opposed
to that computed with a simple blood gas analyzer)

20. In which of the following circumstances would you recommend using pulse oximetry?
A) to assess changes in oxygenation during procedures that can cause hypoxemia
B) to monitor oxygenation in patients with poor peripheral perfusion
C) to obtain precise/accurate assessment of a patient's blood oxygenation
D) to monitor for or warning of hyperoxemia in infants
Ans: A
Response:
Pulse oximetry is best used for basic monitoring of a patient's arterial O2 saturation, including
the response to therapeutic intervention or to diagnostic procedure (e.g., bronchoscopy).
However, pulse oximetry should never be substituted for ABG analysis or hemoximetry when
the clinical situation demands accurate assessment blood oxygenation. You should also
recommend against reliance on pulse oximetry data for patients with poor peripheral perfusion
and when there is a need to monitor for or warn of hyperoxemia, as when protecting
premature infant against retrolental fibroplasia.

21. Which of the following would you recommend to provide graphic data useful in evaluating
the ventilator-patient interface?
A) capnography
B) pulse oximetry
C) hemoximetry
D) electrocardiography
Ans: A
Response:
Capnography (expired CO2 analysis) provides a noninvasive method for assessing both the
effectiveness (PETCO2) and efficiency (PaCO2-PETCO2) of a patient's ventilation. It also
provides graphic data useful in evaluating the ventilator-patient interface. Analysis of the
shape of the capnogram at high sweep speed can be helpful in identifying conditions such as
circuit rebreathing, esophageal intubation, and maldistribution of ventilation. In addition,
capnography can be used to monitor levels of therapeutically administered CO2 gas and
measure CO2 production.

Page 6
22. You detect an irregular pulse and pulse deficit in a patient by palpation and auscultation, and
suspect atrial fibrillation as the cause. Which of the following tests would you recommend to
confirm if atrial fibrillation is the problem?
A) cardiac catheterization
B) electrocardiogram
C) coronary angiogram
D) echocardiogram
Ans: B
Response:
You should recommend obtaining an electrocardiogram to screen for heart disease, rule out
heart disease in surgical patients, evaluate patients with chest pain, follow the progression of
patients with CAD and evaluate heart rhythm disorders, such as atrial fibrillation.

23. All of the following are contraindications against systemic arterial monitoring via arterial line
EXCEPT:
A) severe coagulopathy
B) presence of a dialysis shunt
C) cardiovascular instability
D) inadequate collateral circulation
Ans: C
Response:
Contraindications against systemic arterial monitoring via arterial line include: (1) inadequate
collateral arterial circulation (negative Allen Test); (2) evidence of infection or peripheral
vascular disease in the selected limb; (3) severe coagulopathy, e.g., platelet count <50,000 or
PTT > 37 sec; and (4) presence of a surgical/dialysis shunt in the selected arm (consider
contralateral limb). The need to continuously monitor arterial pressure in unstable/hypotensive
patients is an indication for systemic arterial monitoring.

24. For which of the following reasons would you recommend overnight oximetry?

I. to determine if COPD patients desaturate during sleep


II. to assess sleep apnea patients' response to CPAP
III. to identify patients with obstructive sleep apnea
A) I and II only
B) II and III only
C) I and III only
D) I, II and III
Ans: D
Response:
You should recommend overnight oximetry to: (1) help identify patients with obstructive
sleep apnea-hypopnoea syndrome (SAHS); (2) help assess SAHS patients' response to
therapy, such as CPAP; and (3) identify whether serious desaturation occurs in COPD patients
during sleep. In terms of diagnosing SAHS, the gold standard is lab polysomnography.
However, polysomnography is expensive and not readily available to all patients. In
comparison, overnight oximetry is readily available, inexpensive, and can be performed in the
patient's home.

Page 7
25. A physician requests transcutaneous blood gas monitoring on a premature infant in the NICU.
Which of the following conditions would cause you to recommend against using this device to
monitor this patient?
A) hemodynamic instability
B) congenital heart disease
C) respiratory distress syndrome
D) meconium aspiration
Ans: A
Response:
You should avoid using a transcutaneous monitor on patients with poor skin integrity or those
with an adhesive allergy. Since accurate PtcO2 and PtcCO2 values generally require that the
patient be hemodynamically stable, you should not use these devices on patients in shock or
with poor peripheral circulation. Lengthy set-up and stabilization time (10-20 minutes) also
makes the transcutaneous monitor a poor choice for assessing gas exchange in emergency
situations.

Page 8
CHAPTER 6
Name: __________________________ Date: _____________

1. You measure an oxygen concentration of 55% being delivered by an air-entrainment mask set
to deliver 31% oxygen. Which of the following actions is most appropriate?
A) add an aerosol collar to the mask
B) decrease the oxygen input flow
C) check the entrainment ports
D) increase the oxygen input flow
Ans: C
Response:
Because air-entrainment masks mix air and oxygen at a constant ratio, an alteration in the
delivered oxygen concentration could result only from an alteration in the mixing ratio. Since
jet size (for a given adapter) is fixed, the problem must be with the entrainment ports.
Obstruction of the entrainment ports will decrease air entrainment and raise the delivered
oxygen concentration

2. Which of the following is the approximate total output flow delivered from a 35% air-
entrainment mask operating at 8 L/min?
A) 12 L/min
B) 48 L/min
C) 52 L/min
D) 72 L/min
Ans: B
Response:
To compute the total output flow delivered by an air-entrainment device, multiply the sum of
its ratio parts by the input flow. A 35% air-entrainment mask mixes air an oxygen at a fixed
5:1 ratio; thus (5+1) x 8 = 48 L/min.

3. You observe that the reservoir bag on a patient receiving O2 at 10 L/min does not deflate at all
when the patient inspires. What should you do first?
A) tell the patient to breathe deeper
B) decrease the O2 flow to 6 L/min
C) check the mask for a snug fit
D) remove mask valve flaps
Ans: C
Response:
If a nonrebreathing mask reservoir bag does not deflate at all when the patient inspires, either
the flow is higher than needed, there are large inspiratory air leaks (around the mask or
through the exhalation valves), or the inspiratory valve is jammed. The most common cause is
large air leakage, which can easily be corrected by making sure that the mask is fit
comfortably tight to the patient's face.

4. Which of the following is the most probable cause of insufficient mist in a croup tent?
A) decreased temperature within the canopy
B) insufficient ice in the cooling chamber
C) inadequate size of the tent
D) a clogged capillary tube in the nebulizer
Ans: D
Response:
Since most croup tents use large-volume jet nebulizers to generate mist, insufficient mist
indicates malfunction of the nebulizer. The most common cause of nebulizer malfunction is
clogging of the capillary tube that feeds liquid water to the jet.

Page 1
SAN PEDRO COLLEGE COMPETENCY APPRAISAL - OXYGENATION

5. A physician specifies the following objective in her respiratory orders for a patient with an
artificial airway: "to overcome the patient's humidity deficit." Which of the following aerosol
generators should you select for this patient?
A) small-volume sidestream jet nebulizer
B) large-reservoir heated jet nebulizer
C) small-particle aerosol generator (SPAG)
D) metered-dose inhaler (MDI)
Ans: B
Response:
The large reservoir air entrainment jet nebulizer is the primary gas-powered aerosol generator
used to provide humidification to the respiratory tract.

6. During CPR, the oxygen reservoir bag of a bag-valve manual resuscitator collapses during the
refill phase. You can correct this problem by
A) shortening the bag refill time
B) increasing O2 flow to the bag
C) increasing the rate of compression
D) increasing the volume delivery
Ans: B
Response:
Think of the reservoir bag of a bag-valve manual resuscitator as equivalent to the bag on a
reservoir mask. If the reservoir bag collapses during the resuscitator's refill phase (equivalent
to patient inspiration), the flow is inadequate and should be increased until the bag does not
empty.

7. While a patient is being ventilated with a bag-valve resuscitator, the bag fills rapidly and
collapses on minimal pressure, although little chest movement by the patient is noted. The
cause of the problem may be which of the following?
A) absence of the inlet valve
B) excessive oxygen flow
C) plugged endotracheal (ET) tube
D) plugged inlet valve
Ans: A
Response:
If a bag-valve resuscitator fills rapidly but collapses on minimal pressure and delivers little
volume, the likely problem is a missing, torn or malpositioned inlet valve. In this case, when
you squeeze the bag, gas follows the path of least resistance and escapes out the inlet port
(instead of going to the patient). This problem could have been avoided by performing an
operational check on the bag before applying it to the patient. If discovered after application,
you should quickly secure a working replacement bag.

8. An intubated adult patient with severe expiratory airway obstruction requires ventilatory
support. Which of the following factors is most important in selecting a ventilator for this
patient?
A) ability to compensate for airway interface leaks
B) variable flow control and adjustable I:E ratios
C) ability to run on 12-volt DC (battery) power
D) certification for use during MRI procedures
Ans: B
Response:
Of the functions listed, the most important capability when selecting a ventilator for an
intubated adult patient with severe expiratory airway obstruction would be variable flow
control and adjustable I:E ratios. This will allow clinicians to make sure the expiratory time is
sufficiently long to prevent air –trapping/auto-PEEP.

Page 2
SAN PEDRO COLLEGE COMPETENCY APPRAISAL - OXYGENATION

9. Which of the following types of breathing circuits should you select when assembling a home-
care ventilator?
A) dual-limb “Y” circuit
B) single-limb circuit with expiratory balloon valve
C) single-limb circuit with leakage-type exhaust valve
D) single-limb circuit without any valve
Ans: B
Response:
Most home-care ventilators use a single-limb circuit with expiratory balloon valve. Single-
limb circuits have a separate pneumatic line running from the ventilator to the expiratory
valve that pressurizes the expiratory valve during inspiration in order to block gas outflow
during the application of positive pressure. An expiratory valve also can be used to control or
retard expiratory flow or provide CPAP/PEEP. The mechanical deadspace in these circuits is
that between the built-in expiratory valve and patient airway.

10. To provide a low to moderate concentration of oxygen to a patient receiving nasal BiPAP
via a device that uses a turbine or blower to generated pressure, you should:
A) connect the BiPAP device to a 50 psi O2 outlet
B) apply a nasal cannula to the patient under the mask
C) bleed supplemental O2 from a flowmeter into the circuit
D) add a reservoir bag to the BiPAP breathing circuit
Ans: C
Response:
Most NPPV ventilators and BiPAP devices use a simple air blower to generated pressure. In
order to provide supplemental oxygen with these devices, you place a small bore tubing
adapter at either the patient interface or machine outlet. After connecting this adapter to a
flowmeter via O2 delivery tubing, you bleed O2 into the circuit until the desired level is
confirmed by O2 analysis. Note that high supplemental O2 flows can interfere with the proper
triggering or cycling of some NPPV ventilators. For this reason, you should always follow the
manufacturer's recommendations when considering how best to increase the FIO2 of these
devices.

11. Which of the following alarm conditions indicates a potential system leak when delivering
volume-controlled ventilation?
A) high volume + low pressure
B) low volume + high pressure
C) high volume + high pressure
D) low volume + low pressure
Ans: D
Response:
When delivering volume-oriented ventilatory support, potential system leaks are indicated by
a low volume and low pressure alarm condition. In such cases you should quickly check for
and correct any loose circuit connections.

Page 3
SAN PEDRO COLLEGE COMPETENCY APPRAISAL - OXYGENATION

12. A patient with a chronic neuromuscular condition requires nocturnal positive pressure
ventilation over the long-term. Which of the following airways should you recommend for
this patient?
A) oral endotracheal tube
B) fenestrated tracheostomy tube
C) laryngeal mask airway
D) standard tracheostomy tube
Ans: B
Response:
A fenestrated tracheostomy tube is the best choice to support patients needing intermittent
(e.g., nocturnal) ventilatory support. For positive pressure ventilation, the inner cannula is
inserted to close the fenestration and the cuff inflated to provide a seal. When the patient is not
on the ventilator, the inner cannula is removed (to open the fenestration), the cuff is deflated,
and the tube is plugged. This allows normal use of the upper airway.

13. Which of the following would indicate an obstruction in the endotracheal tube of a patient
who is being mechanically ventilated by a volume-cycled ventilator?

I. high pressure is required to deliver the volume


II. the low pressure alarm is sounding
III. a suction catheter cannot be passed
A) I and III only
B) I and II only
C) I, II, and III
D) II and III only
Ans: A
Response:
An obstruction in the ET tube of a patient who is being mechanically ventilated increases flow
resistance and thus the peak pressure needed by a volume-cycled ventilator to deliver its
preset volume. In addition, you could detect the obstruction by trying to pass a suction
catheter through the ET tube. Failure to pass a suction catheter through a tracheal airway
indicates tube obstruction.

14. Prior to intubation in an emergency, injection of air into the pilot line fails to inflate the cuff
of the endotracheal tube. You should:
A) check the cuff for leaks
B) check the valve on the pilot line
C) replace the endotracheal tube
D) inspect the pilot line for patency
Ans: C
Response:
A cuff that fails to inflate when injected with air has a large leak. The faulty ET tube should
be replaced and the new tube tested in the same manner.

Page 4
SAN PEDRO COLLEGE COMPETENCY APPRAISAL - OXYGENATION

15. Resistance is encountered while suctioning through a size 6 mm ID endotracheal tube with a
14 Fr catheter. Which of the following is the most appropriate action for you to take?
A) lubricate the catheter
B) use a 10 Fr catheter
C) turn the patient's head
D) instill normal saline solution
Ans: B
Response:
The catheter is too large for the ET tube. In general, the outside diameter of a suction catheter
should not be more than about 1/2 as large as the inside diameter of the airway through which
it passes. To quickly estimate the correct catheter size in French units (Fr), simply multiply
the internal diameter (ID) of the tracheal tube (in mm) by 2, and select the next smallest
catheter size. In this example, to suction a patient with an 6.0 mm tube, 2 x 6 = 12. Next
smallest catheter size = 10 Fr. Thus a 10 Fr catheter is the best size to use on this patient.

16. All of the following could cause suctioning to stop suddenly during tracheobronchial
aspiration EXCEPT:
A) disconnected tubing
B) clearance of secretions
C) a full suction reservoir
D) a mucous plug in the catheter
Ans: B
Response:
Conditions that could cause suctioning to stop suddenly during tracheobronchial aspiration
include 1) disconnected tubing (leak; loss of vacuum), 2) a full suction reservoir (ball-valve
shut-off) and 3) plugging of the catheter (system obstruction). Normal clearance of secretions
would not cause loss of suction pressure.

17. About how many hours will an H-cylinder of oxygen last if it has 1100 psig and is emptying
at a flow of 8 L/min?
A) 0.6 hours
B) 8.2 hours
C) 7.2 hours
D) 0.7 hours
Ans: C
Response:
Duration of flow = (cylinder factor x psig) ÷ flow. The factor for an H cylinder is 3.14. Thus
duration of flow = (3.14 x 1100) ÷ 8 = 432 minutes = 7.2 hours.

18. After attaching a yoke connector to an E size cylinder and opening the cylinder valve, you
notice a leak at the gas outlet. Which of the following are possible causes for this leak?

I. the gas outlet bushing is missing or damaged


II. missing Pin-Indexed Safety System pins
III. the yoke hand screw is not tight enough
A) I and II only
B) II and III only
C) I and III only
D) I, II and III
Ans: C
Response:
Yoke connectors for cylinders A-E use a hand screw to hold the yoke on the valve stem. If not
tightened properly, a leak could result. In addition, the small receiving nipple on the yoke is
normally sealed to the gas outlet with a nylon bushing. A missing or damaged bushing could
also cause a leak. Missing DISS pins would not in themselves cause a leak.

Page 5
SAN PEDRO COLLEGE COMPETENCY APPRAISAL - OXYGENATION

19. Which obtaining an arterial sample for analysis using a point-of-care analyzer, you should do
all of the following EXCEPT:
A) analyze the sample within 3 minutes
B) place the sample in an ice slush
C) thoroughly mix the sample
D) prevent sample exposure to air
Ans: B
Response:
When obtaining an arterial sample for analysis using a point-of-care analyzer, the equipment
used and procedure performed are essentially the same as when preparing the sample for
central lab analysis. The exceptions are (1) the sample should be analyzed within 3 minutes;
and (2) the sample should NOT be placed in ice.

20. An alert 55 year-old patient who two days earlier had abdominal surgery has moderate basilar
crackles on the left side, a respiratory rate of 13 breaths/min, and an SpO2 of 96% on nasal
cannula at 2 L/min. The surgeon is concerned that the patient may be developing atelectasis.
Which of the following approaches should you select to manage this patient?
A) intermittent positive pressure breathing
B) incentive breathing therapy
C) postural drainage and percussion
D) aerosolized bronchodilator therapy
Ans: B
Response:
Incentive spirometry is the first choice for treating suspected or confirmed atelectasis in an
alert patient who does not have evidence of bronchospasm or secretion clearance problems.

21. To apply vibrations to a patient when using an electrically powered percussor during postural
drainage, you should:
A) use the highest available frequency and apply during exhalation only
B) use the lowest available frequency and apply during exhalation only
C) use the highest available frequency and apply during inhalation only
D) use the lowest available frequency and apply during inhalation only
Ans: A
Response:
To provide vibration with a mechanical percussor, you should use the highest available
frequencies (20-30 Hz) and apply during exhalation only.

22. After setting up a flow resistor expiratory airway pressure (PEP) device for a patient with
postoperative atelectasis, you note that the system does not generate any pressure during
active patient expiration. Your first action in this case should be to:
A) dispose of the device and get a new one
B) recheck and tighten all connections
C) select and connect a larger orifice
D) have the patient exert less expiratory effort
Ans: B
Response:
If a PEP device fails to generate pressure during patient expiration, the most likely problem is
a leak, either in the device itself or in the connecting tubing (including any connection to a
pressure manometer. In these cases, you should recheck and tighten all connections.

Page 6
SAN PEDRO COLLEGE COMPETENCY APPRAISAL - OXYGENATION

23. You can use a fluid column pressure manometer for all of the following EXCEPT:
A) to measure atmospheric pressure
B) to measure static pressures
C) to calibrate other manometers
D) to measure rapid pressure changes
Ans: D
Response:
Fluid column pressure manometers are to used to measure (1) atmospheric pressure
(barometer); (2) static or slowing changing pressures, e.g., CVP; (3) systolic/diastolic blood
pressures (occlusion method); and also to calibrate other pressure measuring devices. They are
not suited for measuring rapidly changing pressures, their accuracy depends on position, and
they can be messy or hazardous (if mercury is used).

24. You need to measure the forced vital capacity of an adult patient at the bedside. Which of the
following devices would you select to make this measurement?
A) a computerized electronic spirometer with flow sensor
B) strain-gauge pressure transducer and amplifier
C) mechanical turbine-type volumeter/Wright respirometer
D) water-sealed bell spirometer with high speed kymograph
Ans: A
Response:
If you need to measure a patient's forced vital capacity or related measures (e.g., peak flow,
FEVt, FEF25-75) at the bedside, you should choose a portable electronic spirometer that
incorporates a flow sensor (pneumotachometer) and computer analysis module. Mechanical
turbine-type volumeters like the Wright respirometer are used to measure tidal volume,
minute volume, inspiratory capacity, and slow vital capacity. Because the Wright respirometer
can be damaged by flows greater than 60 L/min, it should not be used to measure forced
inspiratory or expiratory volumes.

25. Upon inspection of a portable spirometer's FVC curve obtained on an adult outpatient, you
determine that the back extrapolated volume is excessive. Prior to repeating the maneuver,
which of the following instructions should you provide to the patent?
A) "Don't hesitate"
B) "Blast out faster"
C) "Blow out longer"
D) "Breathe deeper"
Ans: A
Response:
In adults, a back extrapolated volume greater than 150 mL indicates patient hesitation at the
beginning of the breath, which will invalidate the results. In these cases, you need to make
sure the patient does not hesitate when beginning the forced exhalation.

Page 7
SAN PEDRO COLLEGE COMPETENCY APPRAISAL - OXYGENATION

26. After setting up a 12-lead ECG on a patient, you note a noisy and unstable signal. All of the
following would help to resolve this problem EXCEPT:
A) verifying that the leads are connected properly
B) checking the ECG main lead cable for damage
C) turning off filtering of extraneous electrical activity
D) confirming that the patient is staying motionless
Ans: C
Response:
The two most common problems in obtaining a good 12-lead ECG recording are absent or
'noisy' signals. In either case you should check and confirm that: (1) the ECG snaps and
connectors are clean and corrosion-free; (2) the lead electrodes are connected properly to the
patient; (3); the electrode gel is not dry (replace any suspect electrodes); (4) the main lead
cable is undamaged; (5) the patient is motionless; and (6) the device's filter settings (if
available) are properly set to eliminate extraneous electrical activity.

27. To spot check a patient's oxygen saturation at the bedside you should select which of the
following?
A) oxygen analyzer
B) hemoximeter
C) transcutaneous monitor
D) pulse oximeter
Ans: D
Response:
If you need to spot check, monitor or obtain trend data on a patient's oxygen saturation
(SpO2), you should select a pulse oximeter. You would select a laboratory hemoximeter if you
needed precise measures of both normal and abnormal hemoglobin saturations. A
transcutaneous monitor would be your best choice to continuously and noninvasively monitor
arterial blood gases (PO2 and PCO2) in infants.

28. A neonatal intensive care unit (NICU) nurse calls you to check an infant on a transcutaneous
monitor PO2/PCO2 monitor due to a rapid rise in PtcO2 and concurrent fall in PtcCO2 to below
10 torr. The most likely cause of this problem is:
A) the presence of peripheral vasoconstriction
B) a defective sensor or sensor membrane
C) interference due to bright ambient lighting
D) air leakage around the sensor's adhesive ring
Ans: D
Response:
The most common problem with transcutaneous monitoring is air leaks around the adhesive
ring. Air leaks always cause a dramatic fall in PtcCO2. If the leak is large leaks the PtcO2 and
PtcCO2 values will mimic those in room air (PO2 ~ 150 torr/ PCO2 ~ 0 torr). In these cases,
you should reapply the sensor using a new adhesive ring.

Page 8
SAN PEDRO COLLEGE COMPETENCY APPRAISAL - OXYGENATION

29. A 2 year old child is admitted to the emergency department with severe asthmatic symptoms.
The attending physician orders a bronchodilator that is available in both solution and MDI
preparations. Which of the following would be the best delivery system for this drug to this
patient?
A) breath-actuated MDI with mask
B) small volume nebulizer (SVN) with mouthpiece
C) MDI with holding chamber and a mask
D) SVN using the “blow by” technique
Ans: C
Response:
Most infants and small children should receive aerosolized drugs via an MDI with a valved
holding chamber and a mask. If tolerated by the patient, an SVN with a mask could be
considered as an alternative. Avoid using the “blow by” technique (i.e., holding the mask or
open tube near the infant’s nose and mouth) with small volume nebulizers.

30. Which of the following is the preferred delivery method for cromolyn sodium to young
children?
A) MDI with mask
B) small volume nebulizer
C) MDI with holding chamber
D) dry powder inhaler
Ans: B
Response:
A small volume nebulizer (SVN) is the method of choice for administering cromolyn sodium
to young children. You should use a tightly fitting face mask for any child unable to use a
mouthpiece.

31. An air-entrainment mask will deliver more oxygen than intended if:
A) the flow is set too high
B) an aerosol collar is being used
C) the holes in the mask are too large
D) the air-entrainment ports are blocked
Ans: D
Response:
An air-entrainment mask will deliver more oxygen than intended if either (1) the air-
entrainment ports are blocked, or (2) there is downstream obstruction to flow. In both cases,
total output flow will decrease.

Page 9
SAN PEDRO COLLEGE COMPETENCY APPRAISAL - OXYGENATION

32. A patient receiving oxygen at 2 L/min via cannula complains that she cannot 'feel' any gas
flow at her nose. Which of the following actions would help determine the nature of this
problem?

I. occlude the O2 tubing and see if the pop-off functions


II. turn the nursing unit's oxygen zone valve on and off
III. place the cannula tips in a container of clean water
A) I and II only
B) II and III only
C) I and III only
D) I, II and III
Ans: C
Response:
Although an O2 flow of 2 L/min can be hard to 'feel,' the patient may be correct. First, check
the wall outlet and flowmeter. If these are OK, the problem must be a leak in the system. You
confirm a leak by occluding the O2 tubing -- if the humidifier pop-off fails to function, there is
a leak. To simply prove there is flow, you can immerse the cannula tips in a container of clean
water.

33. A patient is receiving O2 via a nonrebreathing mask set at 8 L/min. You notice that the mask's
reservoir bag collapses completely before the end of each inspiration. Which of the following
actions is appropriate in this case?
A) change to a partial-rebreather
B) decrease the O2 liter flow
C) increase the O2 liter flow
D) loosen the mask straps
Ans: C
Response:
The oxygen inflow on both partial rebreathing and nonrebreathing oxygen masks should
always be adjusted and maintained so that the reservoir bag does not collapse during
inhalation.

34. A 52 year-old male is admitted to the hospital emergency room with a chief complaint of
severe radiating chest pain and signs of central cyanosis. The attending physician orders
STAT O2 therapy. Which of the following would you recommend?
A) simple oxygen mask at 8 L/min
B) air entrainment mask at 40% O2
C) nonrebreathing mask at 10-12 L/min
D) nasal cannula at 5 L/min
Ans: C
Response:
Oxygen therapy is indicated whenever there a patient exhibits signs of hypoxemia such as
cyanosis or is suspected of having an acute myocardial infarction. For such patients, you
initially should try to provide as high an FIO2 as possible, usually via either a partial or
nonrebreathing mask set to a high O2 input flow.

Page 10
SAN PEDRO COLLEGE COMPETENCY APPRAISAL - OXYGENATION

35. If you increase the oxygen input flow to a 28% air-entrainment mask from 5 to 7 L/min, what
will be the oxygen concentration?
A) 24%
B) 28%
C) 35%
D) 40%
Ans: B
Response:
For a given jet and port size, air-entrainment devices operate at relatively fixed ratios. This
means that the air to O2 ratios remains about the same, regardless of input flow. Thus
increasing the oxygen input flow to a 28% air-entrainment mask from 5 to 8 L/min will have
little or no effect on the %O2 delivered (remains at 28%). It will, however, increase the total
flow from about 55 L/min (11 x 5) to about 77 L/min (11 x 7).

36. Which of the following humidifiers would be most effective in overcoming a humidity
deficit?
A) bubble humidifier
B) heated wick humidifier
C) cascade humidifier
D) heat and moisture exchanger
Ans: B
Response:
The single most important factor determining a humidifier's overall performance is
temperature. In all humidifiers, heat is lost due to evaporative cooling. This cooling lowers a
humidifier's humidity output to somewhat less than of saturated air at ambient temperature.
The simple solution is to heat the humidifier. In fact, heating is the only way to ensure high
humidity at the high flows required with some O2 delivery systems and mechanical
ventilators.

37. You hear a high-pitched sound coming from a patient's bubble-type humidifier. Gas can be
felt coming from the top of the device. Which of the following could cause this problem?
A) the flowmeter is leaking
B) the O2 flow is too high
C) the water reservoir is empty
D) the O2 tubing has popped off
Ans: B
Response:
The high-pitched sound on a bubble humidifier is the pressure relief valve opening. Sounding
of the pressure relief valve indicates excessive pressure in the humidifier, usually caused by
either an obstruction distal to the humidifier or excessive flow.

38. Which of the following devices would you select if the goal were to deliver the maximum
water content by inhalation to a patient with dried secretions?
A) small volume jet nebulizer
B) heated wick humidifier
C) large volume jet nebulizer
D) ultrasonic nebulizer
Ans: D
Response:
To deliver the maximum water content by inhalation to a patient with dried secretions, you
would select a nebulizer (humidifiers provide only water vapor at or near BTPS). Among
nebulizers in general, ultrasonics have the highest aerosol output, as high as 0.5 g/L of carrier
gas (10 x BTPS water vapor content).

Page 11
SAN PEDRO COLLEGE COMPETENCY APPRAISAL - OXYGENATION

39. Which of the following devices is LEAST appropriate for a patient with an artificial airway?
A) a heated jet nebulizer
B) an ultrasonic nebulizer
C) a heated wick humidifier
D) a bubble humidifier
Ans: D
Response:
Patients with artificial tracheal airways (ET and trach tubes) should be provided with gas at
100% relative humidity at or near body temperature (32-35 degrees Celsius). This can only be
accomplished with a heated humidifier or nebulizer.

40. Which of the following is indicated by a sound of popping and whistling when the water
bottle of a humidifier is tested by obstructing the gas outlet with the finger?
A) no leakage
B) a worn gasket
C) broken tubing
D) a cracked bottle
Ans: A
Response:
The relief valve of a humidifier sounds ('popping and whistling') when the pressure in the
reservoir container exceeds the valve's threshold pressure (usually between 1-2 psig). The
most common reasons for this to occur are 1) an excessive oxygen input flow and 2)
downstream obstruction to outflow. Of course, if one purposely obstruct flow, the relief valve
also should sound. If a relief valve fails to sound when you obstruct a humidifier outlet, there
must be a leak in the system.

41. In order to deliver as high a concentration of oxygen as possible with a self-inflating bag-
valve unit (manual resuscitator), you would:

I. use the highest recommended O2 input flow


II. use the shortest possible refill time
III. connect an oxygen reservoir to the bag
A) I and II only
B) II and III only
C) I and III only
D) I, II and III
Ans: C
Response:
The actual FIO2 provided by manual resuscitators depends on: 1) the oxygen input flow, 2)
reservoir volume, 3) delivered stroke volume and rate, and 4) bag refill time. In order deliver
the highest FIO2 possible, one should always use an O2 reservoir, use the highest acceptable
O2 input flow, and use the longest refill time that the conditions allow.

42. Which of the following methods would you select to deliver the highest possible FIO2 to a
CPR victim?
A) mouth-to-mouth breathing with the rescuer breathing 100%O2
B) a mouth-to-valve mask resuscitator with 8 L/min of O2
C) a nonrebreathing mask with 8 L/min of O2
D) a manual resuscitator with O2 reservoir and 12 L/min of O2
Ans: D
Response:
The best way to deliver 100% O2 during CPR is to use a manual resuscitator (bag-valve-mask
unit) equipped with an oxygen reservoir and with an O2 input flow set to the maximum
recommended by the manufacturer (usually in the 10 to 15 L/min range). Purposefully
slowing the bag's refill time will also help achieve the highest possible FIO2.

Page 12
SAN PEDRO COLLEGE COMPETENCY APPRAISAL - OXYGENATION

43. You are attempting to ventilate a patient during CPR using a mouth-to-valve emergency
resuscitation mask. When you try to exhale into the mouthpiece, you meet resistance and the
patient's chest does not rise. Repositioning the patient's head/neck does not improve the
situation. Which of the following would you do next?
A) intubate the patient
B) check the position of the valve
C) change to a smaller mask size
D) switch to mouth-to-mouth ventilation
Ans: B
Response:
The high resistance felt on expiration and the fact that the patient's chest does not rise indicate
an obstruction in the mouth-to-valve mask system. The most likely cause of obstruction is a
malpositioned (reversed) or malfunctioning one-way valve. If the valve is reversed and not
allowing expired air ventilation, you can quickly reposition it and restart ventilation.

44. While attempting to ventilate an apneic patient with a mask and resuscitator bag, the
respiratory therapist realizes the bag is inoperative. The therapist should do which of the
following?
A) continue to try to ventilate while attempting to repair the bag
B) leave the patient and obtain another bag
C) administer mouth-to-mask resuscitation and obtain another bag
D) insert an oropharyngeal airway
Ans: C
Response:
As a general rule, if a piece of equipment fails during an emergency, discard it and obtain a
replacement. If a replacement is not immediately available, use alternative methods to provide
essential support. In this case, administer mouth-to-mask resuscitation while obtaining another
bag.

45. While trying to ventilate an adult patient in cardiac arrest with a manual resuscitator, you
encounter extreme difficulty in maintaining an effective seal with the mask. The most
appropriate action at this time would be to:
A) call for a new manual resuscitator and mask
B) turn the patient's head to the side and retry
C) apply mouth-to-mouth or mouth-to-mask ventilation
D) further hyperextend the patient's neck and retry
Ans: C
Response:
Effective use of a manual resuscitator with a mask is difficult, and some clinicians may not be
able to provide adequate ventilation with an appropriate seal on all patients. In such cases, the
victim should be ventilated via the mouth-to-mouth or mouth-to-mask route.

Page 13
SAN PEDRO COLLEGE COMPETENCY APPRAISAL - OXYGENATION

46. Which of the following types of breathing circuits would you select when assembling a
noninvasive positive pressure/BiPAP™ ventilator?
A) dual-limb “Y” circuit
B) single-limb circuit with expiratory balloon valve
C) single-limb circuit with leakage-type exhaust valve
D) single-limb circuit without any valve
Ans: C
Response:
Most noninvasive positive pressure (NPPV) ventilators/BiPAP devices use single-limb circuit
with a leakage-type exhaust valve. These circuits consist of a single section of large-bore
tubing and an open exhaust port, usually either a small orifice or a set of slotted vent holes.
The continuous flow that noninvasive positive pressure ventilators provide through the circuit
forces expired gas out this exhaust port during exhalation.

47. Which of the following alarm conditions indicates a circuit obstruction when delivering
volume-oriented ventilatory support?
A) high volume + low pressure
B) low volume + high pressure
C) high volume + high pressure
D) low volume + low pressure
Ans: B
Response:
When delivering volume-oriented ventilatory support, a circuit obstruction is indicated by a
low volume and high pressure alarm condition. In such cases you should quickly check for
and correct any loose circuit connections. In such cases, you should quickly find and correct
any factors causing obstruction, e.g., circuit condensate, a clogged HME or blockage of the
expiratory port.

48. When using a transport ventilator with a single-limb breathing circuit, the low volume and
low PEEP/CPAP alarms sound simultaneously. The most likely cause of this problem is:
A) condensate buildup in the circuit
B) a disconnected expiratory valve line
C) a kink in the main tubing circuit
D) blockage of the expiratory port
Ans: B
Response:
Simultaneous sounding of the low volume and low PEEP/CPAP alarms on a ventilator that
uses a single-limb breathing circuit usually indicates a loss of pressurization of the expiratory
valve, the most common cause of which is a disconnected expiratory valve line.

49. A patient has been supported by a mechanical ventilator using a heat and moisture exchanger
for the last 3 days. Suctioning reveals an increase in the amount and tenacity of secretions.
Which of the following actions are indicated?
A) switch the patient to a large volume heated humidifier
B) switch the patient to continuous ultrasonic nebulization
C) reassess the patient's secretions over the next 24-48 hours
D) replace the heat and moisture exchanger with a new one
Ans: A
Response:
Heat and moisture exchangers are designed for short-term use (24-48 hours) to warm and
humidify the inspired gas in ventilator circuits when the patient is 1) normothermic, 2)
adequately hydrated, and 3) does not require therapeutic humidity for retained secretions. If
these conditions are not met, a conventional large volume heated humidifier should be
employed.

Page 14
SAN PEDRO COLLEGE COMPETENCY APPRAISAL - OXYGENATION

50. An ICU nurse calls to the bedside of a patient receiving volume-oriented SIMV via a
tracheostomy tube. The patient is exhibiting signs of respiratory distress and several alarms
are sounding together. You first action should be to:
A) call a code and begin cardiopulmonary resuscitation
B) remove the patient from the ventilator and bag with 100% O2
C) check for and correct any loose ventilator circuit connections
D) reset the alarms and call for a replacement ventilator
Ans: B
Response:
Because safety considerations demand that you always attend to patient needs first, when any
major problem occurs during ventilatory support, you should immediately remove the patient
from the ventilator and provide appropriate support using a manual resuscitator connected to
an oxygen source.

51. Which of the following actions can decrease the likelihood of condensate occluding the tubing
of a large volume heated humidifier system?

I. placing water traps at low points in the delivery system


II. checking the tubing for condensate and draining periodically
III. preventing "drooping" (low points) in the delivery tubing
IV. heating the delivery tubing to temperatures above ambient
A) II and IV only
B) I, II and III only
C) III and IV only
D) I, II, III and IV
Ans: D
Response:
In order to prevent condensate from occluding large-bore delivery tubing, one must check and
drain the system periodically. One should also avoid "drooping" the tubing, or place water
traps at low points in the delivery system. Last, heating the delivery tubing allows the
humidifier to operate at lower temperatures and can prevent or minimize condensation in the
tubing circuit.

52. During a ventilator test, you determine that the delivered volume is substantially less than the
set volume. To determine if the ventilator volume setting is out of calibration, you would:
A) measure the volume delivered at the ventilator outlet using a calibrated volumeter
B) perform a manual circuit leak test using a low volume and inspiratory pause
C) compare a variety of inspiratory time settings against a digital stop watch
D) measure the volume delivered at the patient connector using a calibrated volumeter
Ans: A
Response:
Circuit leaks are among the most common problem causing loss of ventilator volume and
pressure. However, ventilator malfunction can have the same effect. To distinguish a circuit
leak from a ventilator malfunction, run a circuit leak test. If the circuit leak test is negative,
then the ventilator may not be delivering the preset volume. To determine whether a ventilator
is delivering the preset volume, you should compare the ventilator setting to that measured at
the ventilator outlet using a calibrated volumeter.

Page 15
SAN PEDRO COLLEGE COMPETENCY APPRAISAL - OXYGENATION

53. If you are performing endotracheal intubation on an average-sized adult male, you would most
likely require an endotracheal tube in which of the following size ranges?
A) 4.5 – 6.0 mm
B) 5.5 - 7.0 mm
C) 7.0 – 8.5 mm
D) 8.5 – 10.0 mm
Ans: C
Response:
For an average-sized adult male patient requiring endotracheal intubation, an ET tube with an
ID between 7.0 and 8.5 mm is satisfactory.

54. Soon after insertion of a 90 mm/#4 oropharyngeal airway in an adult male, the patient gags
and retches. You would consider all of the following EXCEPT:
A) insert a larger oropharyngeal airway
B) remove the oropharyngeal airway
C) use a head-tilt/chin-lift to maintain the airway
D) using a nasopharyngeal airway instead
Ans: A
Response:
If a patient gags or retches with an oropharyngeal airway, you should remove the device,
maintain the airway by positioning the head/neck, and consider using a nasopharyngeal
airway. A 90 mm/#4 oropharyngeal airway is the appropriate size for an adult male; inserting
a larger one might only worsen the problem.

55. You are called to the ICU to check an orally intubated, mechanically ventilated patient who
was just shaved and repositioned in bed. The high pressure alarm is sounding on each breath.
Breath sounds are absent on the left and the trachea is midline. You should:
A) suction the patient
B) increase the high pressure limit
C) increase the FIO2 to 1.0
D) withdraw the tube 1-2 cm
Ans: D
Response:
In combination with the physical findings, the fact that the patient was just shaved and
repositioned in bed suggests that the likely problem is ET tube displacement into the right
mainstem bronchus. In this case, the therapist should carefully withdraw the tube 1-2 cm and
reassess for restoration of breath sounds to the left.

56. You are performing tracheobronchial suctioning on an adult patient with an 8.0 mm ET tube
using a 14 Fr catheter with the vacuum pressure set to -60 mm Hg. Despite the presence of
copious secretions, you are having difficulty aspirating them. To better remove the patient's
secretions, you should:
A) reposition the endotracheal tube 1-2 cm higher
B) lavage the patient first with 0.1 L normal saline
C) replace the catheter with a larger one (18 Fr)
D) reset the vacuum pressure to -120 mm Hg
Ans: D
Response:
The likely problem here is that the vacuum setting is too low. Normal wall regulator vacuum
settings for adults range between -100 and -120 mm Hg.

Page 16
SAN PEDRO COLLEGE COMPETENCY APPRAISAL - OXYGENATION

57. You are setting up transport of an intubated patient who requires frequent endobronchial
suctioning to a procedure room that does not have a piped vacuum source. To manage this
patient during transport you will need which of the following items:
A) a Yankauer suction tip
B) a Lukens sputum collection trap
C) a battery- or hand-powered suction pump
D) a closed suction system
Ans: C
Response:
Suctioning requires a negative pressure (vacuum) source. If a central piped DISS vacuum
source is not available, for transport a patient needing frequent endobronchial suctioning, you
will need a battery- or hand-powered suction pump.

58. Which of the following equipment would you select in order to suction the oral cavity or
pharynx of an infant?
A) a DeLee mucous trap
B) a Yankauer tip
C) a Coude catheter
D) a simple bulb syringe
Ans: D
Response:
Oral and pharyngeal suctioning in infants is best accomplished with a bulb syringe. Either a
DeLee trap or a mechanical vacuum source with attached catheter may be used for
nasopharyngeal and nasotracheal suctioning of the neonate.

59. A newborn is receiving oxygen therapy via hood. Which of the following should you select to
check the accuracy of the air-oxygen blender?
A) line pressure manometer
B) galvanic cell analyzer
C) precision flowmeter
D) pulse oximeter
Ans: B
Response:
To check the accuracy of an air-oxygen blender, you need an oxygen analyzer (usually a Clark
electrode or galvanic fuel type). Normal operational check of a blender involves confirming
both the air and oxygen low pressure alarms (via disconnecting each gas source) and checking
the FIO2 at 21%, 100% and the prescribed concentration.

60. You are preparing to conduct a complex transport of a patient receiving oxygen, and expect to
have to alter O2 flows during the transport. Which of the following devices would best meet
your needs?
A) flow restrictor
B) uncompensated Thorpe tube
C) Bourdon-type gauge
D) compensated Thorpe tube
Ans: C
Response:
When patients must be transported with a portable O2 source, upright positioning of the
oxygen supply (usually small cylinders) is seldom possible, and movement of both the O2
supply and patient is common. In these cases, either a flow restrictor or Bourdon gauge may
be used. However, only the Bourdon gauge provides a continuous range of flows.

Page 17
SAN PEDRO COLLEGE COMPETENCY APPRAISAL - OXYGENATION

61. If an oxygen blender pressure alarm sounds, which of the following should be your FIRST
action?
A) verify that the O2 and air lines are providing the required pressure
B) check for loose connection or leaks between the gas source and blender
C) separately disconnect the O2 and air lines to see if the alarm stops
D) verify the 100% and 21% O2 settings with a calibrated oxygen analyzer
Ans: A
Response:
If an oxygen blender pressure alarm sounds when both gas sources are attached, you should
first verify that both gas sources are at the required inlet pressures (usually 35-50 psig). If this
is does not correct the problem, you should next check for leaks between the gas sources and
blender. If these check out, you should replace the blender.

62. When running an arterial blood gas on a point-of-care analyzer, you receive a calibration error
message. You should:
A) repeat the analysis using a fresh sample and the same cartridge
B) repeat analysis using a fresh sample and new cartridge
C) send the sample to the central laboratory for analysis
D) repeat the analysis using the same sample and same cartridge
Ans: B
Response:
A calibration error on a point-of-care analyzer usually indicates a problem with the sample,
calibrating solutions, sensors, or the function of the device's electrical or mechanical
components. Normally this requires that you repeat the analysis using a fresh sample and new
cartridge. If a message accompanies the error warning, you also should take the action
recommended and report the error code to the point-of-care testing manager.

63. When running an arterial blood gas on a point-of-care analyzer, the device 'flags' the PCO2
results. You should:
A) repeat the analysis using a fresh sample and the same cartridge
B) repeat analysis using a fresh sample and new cartridge
C) send the sample to the central laboratory for analysis
D) repeat the analysis using the same sample and same cartridge
Ans: C
Response:
A flagged result on a point-of-care analyzer usually indicates that the value measured is
outside the analyzer's reportable ranges. In these cases, you should send the sample to the
central laboratory for analysis.

64. When checking on a postoperative patient's progress with incentive spirometry, you note that
neither the flow or volume indicators rise during her inspiratory efforts. Which of the
following are potential causes of this problem?

I. the one-way valve is not functioning properly


II. there is a loose connections between components
III. the patient's inspiratory flow is too high
A) I and II only
B) II and III only
C) I and III only
D) I, II and III
Ans: A
Response:
If an incentive spirometer doesn't record any inspired flow or volume, the likely problem is a
loose connection between the mouthpiece, connecting tubing and spirometer inlet or an
improperly functioning one-way valve.

Page 18
SAN PEDRO COLLEGE COMPETENCY APPRAISAL - OXYGENATION

65. A surgeon requests accurate tracking of his postoperative patient's inspiratory capacity (IC)
over first 48 hours of incentive spirometry therapy. To provide this information you would:
A) send the patient to the pulmonary function laboratory for a full work-up
B) read the IC measurement directly from the device's volume accumulator
C) use a one-way valve and calibrated respirometer to measure the IC
D) report the patient's normal IC using the appropriate prediction nomogram
Ans: C
Response:
If you need to obtain an accurate measures of a patient's inspired volume during incentive
spirometry, you should attach a one-way breathing valve and calibrated respirometer to the
device and take at least 3 measures to insure repeatability.

66. A hospitalized adolescent with cystic fibrosis requires both regular aerosol drug therapy
(dornase alpha and tobramycin) and help with clearance of retained secretions. In addition to
postural drainage and directed coughing, which of the following devices would your
recommend using with this patient?
A) a pneumatically powered percussor/vibrator
B) a high frequency chest wall oscillation vest
C) an electrically powered percussor/vibrator
D) intrapulmonary percussive ventilation (IPV)
Ans: D
Response:
For hospitalized adults or large children requiring percussion or vibration in combination with
aerosol drug therapy, you should consider an intrapulmonary percussive ventilation (IPV)
device, either pneumatically or electrically (compressor) powered.

67. An adult patient with cystic fibrosis cares for himself at home. The patient's doctor has
ordered postural drainage, percussion and vibration. Which of the following would your
recommend to assist this patient in clearing his retained secretions during postural drainage?
A) a pneumatically powered percussor/vibrator
B) a high frequency chest wall oscillation vest
C) an electrically powered percussor/vibrator
D) properly sized percussion cups or wand
Ans: B
Response:
For care of adults requiring postural drainage, percussion and vibration in the home who do
not have caregiver support, you could recommend either a a high frequency chest wall
oscillation (HFCWO) vest system or an electrically powered intrapulmonary percussive
ventilation (IPV) device.

68. Which of the following types of positive expiratory pressure (PEP) devices would you
recommend for a patient if the goal is to help mobilize retained secretions?
A) a simple spring-loaded threshold PEP device
B) a PEP device that provides high frequency vibrations
C) a simple adjustable orifice flow resistor PEP device
D) a simple PEP device with a small volume nebulizer
Ans: B
Response:
If the goal is to help mobilize retained secretions, then a vibratory PEP device like the Flutter
valve® or Acapella® is the best choice. If concurrent bronchodilator aerosol therapy is
indicated, all devices except the Flutter valve® and Threshold™ PEP device provide adaptors
for attaching a small volume nebulizer.

Page 19
SAN PEDRO COLLEGE COMPETENCY APPRAISAL - OXYGENATION

69. To increase the PEP level when using a flutter-type valve you would:
A) connect an aneroid manometer to the device
B) have the patient exhale more slowly
C) increase the spring tension on the device
D) raise the angle of the valve above horizontal
Ans: D
Response:
During PEP therapy, you normally start at a low PEP level and raise the pressure during
therapy to between 10-20 cm H2O as tolerated. To do so with a flutter-type valve, you have
the patient slowly raise the angle of the valve above horizontal. On flow resistors, this would
be achieved by applying progressively smaller resistors. On threshold devices that use a
spring-loaded valve to generate PEP, you would slowly increase the spring tension.

70. When open to the atmosphere, a manometer calibrated in cm H2O units should read:
A) 0 cm H2O
B) 1 atmosphere
C) 760 cm H2O
D) 1034 cm H2O
Ans: A
Response:
When open to the atmosphere, any pressure measuring device that measures pressure relative
to atmospheric should read '0' in its measurement units. Indeed, this a basic first step in
confirming the accuracy of such devices.

71. To measure a patient's inspiratory muscle strength at the bedside, which of the following
devices would you select?
A) ergometer
B) respirometer
C) flowmeter
D) manometer
Ans: D
Response:
Beside measurement of a patient's respiratory muscle strength is most often performed using a
simple aneroid pressure manometer, usually in combination with a disposable valve system.
Both inspiratory (MIP) and expiratory (MEP) muscle strength can be measured. MIP is often
used to assess the need for mechanical ventilation, while MEP is helpful in assessing the
ability to cough and clear secretions (for extubation).

72. When using a mechanical respirometer to measure the expired minute volume of multiple ICU
patients, which of the following is the best way to protect them against cross-contamination?
A) sterilize the respirometer with ethylene oxide gas between each patient use
B) use a short segment of flex tubing to separate the patients from the respirometer
C) use a disposable one-way breathing valve and bacterial filter on each patient
D) disinfect the respirometer surface with isopropyl alcohol between each patient use
Ans: C
Response:
Using a separate disposable one-way valve and HEPA bacterial filter on each patient
generally will prevent contamination of a Wright respirometer and allow a single device to be
applied to multiple patients. Only if a Wright respirometer actually becomes contaminated
should it be gas sterilized. However, the required aerated period makes this approach
unrealistic for preventing cross-contamination associated with frequent application to multiple
patients.

Page 20
SAN PEDRO COLLEGE COMPETENCY APPRAISAL - OXYGENATION

73. Upon inspection of a portable spirometer's FVC curve obtained on an adult outpatient, you
observe that the breath last only 4 seconds and that the exhaled volume is still changing
substantially during the last half second of the breath. Prior to repeating the maneuver, which
of the following instructions would you provide to the patent?
A) "Don't hesitate"
B) "Blast out faster"
C) "Blow out longer"
D) "Deeper breath"
Ans: C
Response:
In adults, if the forced expiratory time < 6.0 sec and if the change in exhaled volume during
the last 0.5 sec of the maneuver exceeds 100 mL, then the patient is prematurely ending the
breath, which will invalidate the results. In these cases, you need to make sure that the patient
continues the effort to complete the breath, i.e., blow out longer."

74. When measuring a patient's forced vital capacity at the bedside using a portable electronic
spirometer, you note that the device starts to read flow and volume before patient actually
exhales. The most likely cause of this problem is which of the following?
A) the flow sensor tubing not connected to computer module
B) the patient is moving the flow sensor before starting the test
C) the flow sensor is obstructed with packaging material
D) the patient's lips or tongue is obstructing mouthpiece
Ans: B
Response:
If a portable electronic spirometer accumulates volume or reads flow before the patient
exhales, the most likely problem is that the sensor and/or tubing is not stationary at the start of
test, i.e., is registering flow when it is moved through the air. To avoid this problem, have the
patient hold the sensor assembly steady until prompted to exhale.

75. To evaluate a patient with chest pain, you would recommend which of the following?
A) MRI
B) 12-lead ECG
C) bedside spirometry
D) capnography
Ans: B
Response:
To evaluate a patient with chest pain, you would recommend a 12-lead ECG. Physicians and
other health related personnel use the 12-lead ECG data to assess rhythm disturbances,
determine the heart's electrical axis, and identify the site and extend of myocardial ischemia or
damage. In addition to obtaining a 12-lead ECG to evaluate patients with chest pain, you
should recommend this procedure to (1) screen for heart disease; (2) rule out heart disease in
surgical patients; (3) follow the progression of coronary heart disease; (4) and evaluate heart
rhythm disorders.

76. Which of the following is proper placement of the V2 chest lead for obtaining a 12-lead ECG?
A) 5th intercostal space, midclavicular line
B) 4th intercostal space, right sternal border
C) 5th intercostal space, anterior axillary line
D) 4th intercostal space, left sternal border
Ans: D
Response:
You place the 6 chest leads needed to obtain a 12-lead ECG as follows: V1 - 4th intercostal
space, right sternal border; V2 - 4th intercostal space, left sternal border; V3 - between V2 and
V4; V4 - 5th intercostal space, midclavicular line; V5 - 5th intercostal space, anterior axillary
line; V6 - 5th intercostal space, midaxillary line.

Page 21
SAN PEDRO COLLEGE COMPETENCY APPRAISAL - OXYGENATION

77. After applying a pulse oximeter's disposable probe to a patient's finger, you note on the
display an unstable and poor quality pulse pressure waveform. All of the following actions
could help improve the signal quality EXCEPT:
A) reposition the probe
B) remove any nail polish
C) apply the probe more tightly
D) replace the probe
Ans: C
Response:
The most common problem with pulse oximeters is an unstable or poor quality signal. In these
cases you should: (1) recheck the site and clean it and the probe (if multi-use) with alcohol;
(2) reposition the probe; (3) remove any fingernail polish; try a different site; or (4) replace
the probe. Tightly applying either disposable or multi-patient probes may result in inaccurate
readings or cause skin damage.

78. For continuous monitoring of a patient's oxygen saturation with a pulse oximeter, which of the
following LOW alarm limits would be appropriate?
A) 79-81%
B) 85-87%
C) 92-94%
D) 97-99%
Ans: C
Response:
For continuous monitoring of a patient's oxygen saturation with a pulse oximeter, you should
set the low alarm limit according to your institution's protocol, but generally no lower that
92%. A higher setting will likely results in many false alarms, while a lower setting can result
in the patient becoming hypoxemic before the alarms sounds (a SpO2 below 92% represents a
PaO2 of about 65 torr).

79. A patient complains to you that her MDI 'does not mist when squeezed.' After confirming that
the canister is new and full, you hand warm it, fit it tightly in its boot, detach the protective
cap and actuate it, but observe that no aerosol plume is produced at the mouthpiece. Which of
the following would you do to correct this problem?
A) check and correct the patient's technique
B) switch to a universal MDI boot adapter
C) clean the boot and canister outlet
D) recommend the patient use a spacer
Ans: C
Response:
The most likely problem in this case is that the aerosol outflow is being obstructed by dirt or
debris. You should remove or clean out any dirt or foreign material in the boot and clean the
canister's outlet in hot water. Using a universal adapter may only worsen the problem since
different MDI formulations operate at different pressures, and thus may have different sized
boot nozzles.

Page 22
SAN PEDRO COLLEGE COMPETENCY APPRAISAL - OXYGENATION

80. All of the following are correct dry powder inhaler (DPI) technique EXCEPT:
A) hold device horizontally after loading
B) rapid (1-2 sec) and deep inhalation
C) lips tightly sealed around mouthpiece
D) exhalation back into the device
Ans: D
Response:
The following general guidelines apply to effective use of a DPI: (1) never use a spacer or
VHC with a DPI; (2) lips must be tightly sealed around the mouthpiece; (3) after loading,
most DPIs must be held horizontally (to avoid loss of drug); (4) patient should inhale rapidly
(60 L/min or 1-2 sec) and deeply; and (5) patient must exhale to room (not back into the
device).

81. A physician has ordered administration of a steroid available only in a metered dose (MDI)
preparation. In training the patient in its use, you cannot get her to coordinate MDI discharge
with her breathing. Which of the following would your recommend to the ordering physician?
A) discontinue the steroid treatments altogether
B) use a spacer or valved holding chamber with the MDI
C) substitute an oral steroid for the MDI preparation
D) substitute a bronchodilator via jet nebulization
Ans: B
Response:
Two techniques can be used to aid proper application of MDI aerosols: extension devices
(spacers and holding chambers) and breath-actuated systems. Extension devices minimize
aerosol loss and maximize propellant evaporation, thereby increasing stability and penetration.
Breath-actuated synchronize MDI discharge with the patient's inspiration, thereby requiring
less patient coordination.

82. When checking a ventilator, you discover that the set PEEP level cannot be maintained.
Which of the following might be causing this problem?

I. leak in the tubing


II. faulty exhalation valve
III. leak around the airway cuff
IV. loose humidifier connection
A) I and II only
B) I and III only
C) II and IV only
D) I, II, III and IV
Ans: D
Response:
A basic rule of thumb is that leaks prevent pressure build-up and obstructions cause pressure
build-up. According to this rule of thumb, this is definitely a leak scenario. And since any
mechanical connection or the patient's airway can be the source of a leak, ALL of the cited
problems could be the cause.

Page 23
CHAPTER 7
Name: __________________________ Date: _____________

1. The first step in processing reusable equipment is:


A) pasteurization
B) cleaning
C) disinfection
D) sterilization
Ans: B
Response:
The first step in equipment processing is cleaning. Equipment is cleaned by removing dirt and
organic material from its surfaces, usually by washing. If equipment is improperly cleaned,
subsequent processing efforts may be ineffective.

2. The label of a disinfectant indicates that it does NOT inactivate or kill either Mycobacterium
tuberculosis or bacterial spores. What class of disinfectant is this?
A) surface active
B) low-level
C) intermediate-level
D) high-level
Ans: B
Response:
A low-level disinfectant inactivates most bacteria, some viruses, and fungi but cannot destroy
resistant microorganisms such as Mycobacterium tuberculosis or bacterial spores.

3. A patient with pneumonia has her noninvasive ventilatory support discontinued. Which of the
following should be used to disinfect the device's nondisposable breathing circuit before it is
placed back into service?
A) pasteurization
B) isopropyl alcohol
C) acetic acid
D) hydrogen peroxide
Ans: A
Response:
Reusable breathing circuits are semicritical items. According to CDC recommendations,
reusable semicritical equipment should be sterilized or undergo high-level disinfection. Of the
available options, only pasteurization meets this standard.

4. For patients receiving bronchodilator therapy via small-volume nebulizer (SVN), which of the
following precautions would be beneficial in preventing nosocomial infection?

I. use a different SVN for each patient


II. change the nebulizer and tubing every 24 hours
III. perform thorough hand hygiene before each patient contact
A) I only
B) II only
C) I and III only
D) I, II, and III
Ans: D
Response:
To minimize the likelihood of infection in patients receiving bronchodilator therapy via SVN,
you should (1) use a different SVN for each patient, (2) change the SVN and tubing every 24
hours, and (3) perform thorough handwashing prior to each therapy session. It is also
recommended that the nebulizer not be rinsed with tap water but rather rinsed with sterile
water and blown dry between uses.

Page 1
SAN PEDRO COLLEGE COMPETENCY APPRAISAL - OXYGENATION

5. A patient who was hospitalized with ARDS of unknown etiology recently traveled to an area
with a documented outbreak of severe acute respiratory syndrome (SARS). Which of the
following infection control precautions should you recommend for this patient?

I. airborne precautions
II. contact precautions
III. droplet precautions
A) I and II only
B) I, II, and III
C) I and III only
D) II and III only
Ans: B
Response:
If either the suspicion of SARS is strong or laboratory evidence confirms a SARS-CoV
infection, the CDC recommends a combination of standard, contact, and strict airborne
precautions to manage these patients.

6. ALL of the following help minimize the risk of cross-contamination during suctioning except:
A) using a fresh sterile single-use catheter on each patient
B) using only sterile fluid to remove secretions from the catheter
C) instilling 10 mL sterile saline before the suctioning attempt
D) performing proper handwashing and gloving before suctioning
Ans: C
Response:
To decrease the risk of patient contamination during suctioning, you should use a fresh sterile
single-use catheter on each patient and use only sterile water or saline to clear the catheter. In
addition, both the suction collection tubing and collection canister should be changed between
patients, except in short-term care units (where only the collection tubing should be changed).

7. If sterilization is not feasible, which of the following are acceptable alternatives for processing
a specialized reusable plastic airway?

I. exposure to a high-level chemical disinfectant


II. surface disinfection with 70% ethyl alcohol
III. pasteurization at 63° C for 30 minutes
A) I and II only
B) II and III only
C) I and III only
D) I, II, and III
Ans: C
Response:
Because it directly comes in contact with mucous membranes, a reusable airway is
categorized as semicritical equipment. If sterilization of a semicritical item is not feasible, the
alternatives are high-level disinfection or pasteurization.

Page 2
SAN PEDRO COLLEGE COMPETENCY APPRAISAL - OXYGENATION

8. What is the primary reason for using chemical indicators in equipment sterilization
processing?
A) to warn the user about the presence of toxic residues
B) to show that a package has been through a sterilizing process
C) to indicate the remaining safe shelf-life of the equipment
D) to verify that the sterilization process has worked
Ans: B
Response:
Chemical indicators are usually impregnated on packaging tape. These indicators change color
when exposed to specific conditions, telling the user that the package has been processed via
an appropriate sterilizer cycle. Chemical indicators cannot ensure that the contents are actually
sterile. Only biological indicators can provide that information.

9. Cultures taken from a respirometer that has been used in the surgical intensive care unit to
monitor several patients indicate that it is contaminated. The most practical way to prevent
cross-contamination is to:
A) provide a new respirometer for each patient
B) sterilize the respirometer after each use
C) replace the respirometer with a water-sealed spirometer
D) use a disposable HEPA filter and one-way valve for each patient
Ans: D
Response:
The best way to prevent the contamination of a Wright respirometer used on several different
patients is to use a one-way valving system, preferably with an in-line HEPA filter. Such an
approach can ensure that patients only breathe out through the device, thus preventing cross-
contamination.

10. ALL of the following will reduce the incidence of ventilator-acquired pneumonia (VAP) on
your patients except:
A) elevating the head of the bed at least 30°
B) implementing daily spontaneous breathing trials
C) continuously aspirating subglottic secretions
D) changing ventilator circuits every 48 hours
Ans: D
Response:
The CDC VAP 'bundle' includes elevating the head of the bed at least 30°, implementing a
daily “sedation vacation” and spontaneous breathing trial, and providing peptic ulcer and deep
venous thrombosis disease prophylaxis. Airway management techniques that can help
decrease the incidence of VAP include: (1) avoiding intubation when possible (using NPPV
instead); (2) intubating orally (as opposed to nasal) route; (3) maintaining proper ET cuff
pressures; (4) aspirating subglottic secretions; and (5) using only sterile water or saline to
flush suction catheters. Ventilator circuits should be changed only when visibly soiled or
malfunctioning.

Page 3
SAN PEDRO COLLEGE COMPETENCY APPRAISAL - OXYGENATION

11. The label of a disinfectant indicates that it destroys vegetative bacteria, mycobacteria, fungi
and viruses, and can inactivate bacterial spores if the contact time is adequate. What class of
disinfectant is this?
A) surface active
B) low-level
C) intermediate-level
D) high-level
Ans: D
Response:
A high-level disinfectant kills vegetative bacteria, mycobacteria, fungi and viruses, but not
necessarily bacterial spores. Some high-level disinfectant chemicals ("chemical sterilants") are
be capable of sterilization with adequate contact time.

12. After a bronchoscopic biopsy, the physician asks that your properly process the metal forceps
used to obtain the tissue sample. Which of the following processing methods should you use?
A) steam autoclaving
B) immersion in acetic acid
C) pasteurization at 63 °C
D) washing in a detergent
Ans: A
Response:
Because they are introduced into the body, bronchoscope forceps/brushes are considered
critical items that must be sterilized before reuse. Because this equipment is heat tolerant, the
best processing method to ensure sterilization would be steam autoclaving.

13. During a procedure, a glass suction collection bottle fills with fluid, interrupting the vacuum
pressure. Which of the following would be the proper way to dispose of the contents and
restore vacuum?
A) pour the bottle contents into a biohazard bag
B) seal the bottle with a stopper and replace it
C) pour the bottle contents down a utility sink or toilet
D) send the full bottle to central supply for sterilization
Ans: C
Response:
The collection bottle needs to be emptied in order to restore vacuum. Most liquid wastes (e.g.,
blood, suction fluids, etc.) can be carefully poured down a utility sink drain or toilet. If
transported, liquid wastes should be placed in capped or tightly-stoppered bottles or flasks.
Large quantities of liquid wastes may be placed in containment tanks.

14. During a procedure in ICU, blood spills onto the outer metal casing of an in-use ventilator.
After cleaning the surface using gloves and disposable gauze pads, how would to
decontaminate it?
A) spray the area with a concentrated isopropyl alcohol solution
B) swab the area with a tuberculocidal disinfectant or bleach solution
C) take the ventilator out of service and have it sterilized
D) wash the surface with a detergent and rinse with sterile water
Ans: B
Response:
Should a spill of blood or body fluids occur, you should first use don gloves and other
protective equipment and clean area with disposable absorbent material, which should be
discarded as infectious waste. You should then swab the area with a cloth or paper towels
wetted with an EPA-registered hospital disinfectants labeled tuberculocidal solution.
Alternatively, you can use either a registered germicide that is active against HIV or hepatitis
B or a 1:100 bleach solution. More concentrated bleach solutions (1:10) should be used for
spills involves large amounts of blood or body fluids.

Page 4
SAN PEDRO COLLEGE COMPETENCY APPRAISAL - OXYGENATION

15. Hand hygiene should be performed:

I. after touching blood/body fluids


II. immediately after removing gloves
III. between patient contacts
A) I and II only
B) II and III only
C) I and III only
D) I, II and III
Ans: D
Response:
Hand hygiene should be performed (1) after touching blood, body fluids, secretions,
excretions, contaminated items; (2) immediately after removing gloves; and (3) between all
patient contacts.

16. In preparing for a bronchoscopy on a patient suspected of having severe acute respiratory
syndrome (SARS), which of the following equipment would provide inadequate protection?
A) eye shield
B) disposable gloves
C) disposable gown
D) surgical mask
Ans: D
Response:
During aerosol-generating procedures on patients with suspected or proven infections
transmitted by respiratory aerosols (e.g., SARS), in addition to gloves, gown, and face/eye
protection, you should wear a fit-tested N95 respirator. A standard surgical mask provides
inadequate protection.

17. While assessing a post-operative patient for atelectasis, you note that his visitors are sneezing
and coughing. You should instruct them to:

I. wash their hands after soiling with respiratory secretions


II. cover their mouth and nose when sneezing/coughing
III. use tissues and dispose of them in a no-touch receptacle
A) I and II only
B) II and III only
C) I and III only
D) I, II and III
Ans: D
Response:
The CDC recommends respiratory hygiene/cough etiquette to help prevent the spread of
respiratory infections. Under this protocol, patients and visitors who are sneezing or coughing
should be instructed to: (1) cover the mouth/nose when sneezing/coughing; (2) use tissues and
dispose in no-touch receptacle; and (3) observe hand hygiene after soiling of hands with
respiratory secretions. In addition, in common waiting those who are who are sneezing or
coughing should wear surgical masks. If masks are not tolerated, efforts should be made to
separate individuals from each other by at least 3 feet..

Page 5
SAN PEDRO COLLEGE COMPETENCY APPRAISAL - OXYGENATION

18. In addition to standard precautions, which of the following transmission-based precautions


would you recommend for an elderly patient admitted to a medical unit with severe viral
influenza?
A) contact precautions
B) droplet precautions
C) airborne precautions
D) universal precautions
Ans: B
Response:
Influenza is one of several common infections spread by the transmission of large droplets
(greater than 5µm) in the air. Droplet precautions are designed to thwart this route of
transmission and include (1) placing the patient in a private room; (2) wearing a surgical mask
when within 3 feet of the patient; (3) using eye/face protection if aerosol-generating procedure
performed or contact with respiratory secretions anticipated; and (4) having the patient wear a
mask and follow respiratory hygiene/cough etiquette when being transported outside their
rooms.

19. Airway management techniques can help decrease the incidence of ventilator-acquired
pneumonia (VAP) except:
A) using the nasal route for intubation
B) aspirating subglottic secretions
C) maintaining proper ET tube cuff pressures
D) only sterile fluids to flush suction catheters
Ans: A
Response:
Airway management techniques that can help decrease the incidence of VAP include: (1)
avoiding intubation when possible (using noninvasive ventilation instead); (2) intubating via
the oral (as opposed to nasal) route; (3) maintaining proper ET tube cuff pressures; (4)
aspirating subglottic secretions; and (5) using only sterile water or saline to flush suction
catheters. Ventilator circuits should be changed only when visibly soiled or malfunctioning.

20. In which of the following clinical circumstances should you wear a gown?

I. when one's clothing can be soiled with infective material


II. when in the room of a patient with a highly contagious disease
III. whenever assisting in a noninvasive procedure
IV. whenever assigned to emergency care settings
A) I and II only
B) I and III only
C) I, II and IV only
D) I, II, III and IV
Ans: A
Response:
Gowns protect clothing from contamination that can occur in patient care activities. Because
such soiling is uncommon, gowns are not necessary for most activities. Gowns are indicated
when there is likely contact with blood or body fluids or clothes are likely to be soiled
infective secretions or excretions of a patient in isolation. Gowns also are indicated for
persons entering the room of patients who have highly contagious disorders such as varicella
(chickenpox) or disseminated zoster.

Page 6
SAN PEDRO COLLEGE COMPETENCY APPRAISAL - OXYGENATION

21. Which of the following conform to the CDC standards for standard precautions?

I. extreme care with sharp instruments


II. gowns for touching blood/body fluids
III. gloves for touching blood/body fluids
IV. masks/protective eyewear for invasive procedures
V. immediate skin/handwashing if contamination occurs
A) I and III only
B) II, IV and V
C) II, III, IV and V
D) I, II, III, IV and V
Ans: D
Response:
CDC standards for standard precautions include: 1) gloves and gowns for touching
blood/body fluids: 2) masks and protective eyewear for procedures resulting in splashing of
blood/body fluids; 3) discarding or bagging/labeling contaminated articles before removal; 4)
taking extreme care with sharp instruments; and 5) immediate skin/handwashing if
contamination occurs.

22. You are gathering a sputum specimen from a patient in isolation. Which of the following
procedures should be followed in gathering, removing and transporting this specimen?

I. disinfect outside of specimen container if contaminated


II. placed specimen in a sturdy container with a secure lid
III. place specimen container in an impervious, labeled bag
A) III only
B) I, II and III
C) I and III only
D) I and II only
Ans: B
Response:
When gathering a laboratory specimen, great care should be taken to prevent external
contamination of the container. If the outside of the container gets contaminated, it must be
disinfected or placed in an bag. To minimize leakage during transport, specimens should
always be placed in a sturdy container with a secure lid. When a specimen comes from a
patient in isolation, the container should be placed in an impervious bag and labeled before
removal from the room.

23. You are assisting a physician who is inserting a catheter into the radial artery of an HIV
positive patient. Which of the following barrier precautions should you employ?

I. don gloves and a surgical mask


II. wear protective eyewear
III. wear a gown or apron
A) I and II only
B) I, II and III
C) I and III only
D) I only
Ans: B
Response:
Gloves and masks are required for all invasive procedures. Protective eyewear and gowns or
aprons are recommended for any procedure that could result in the generation of droplets or
the splashing of blood or other body fluids. If hands or other skin areas become contaminated
during an invasive procedure, the gloves should be removed, and the affected area thoroughly
washed.

Page 7
SAN PEDRO COLLEGE COMPETENCY APPRAISAL - OXYGENATION

24. Which of the following measures can reduce the incidence of infection associated with large
volume heated humidifier systems?

I. using prepackaged sterile disposables or wick humidifiers


II. using closed-feed sterile water reservoir systems
III. carefully draining condensate back into the humidifier
IV. changing the humidifier/circuitry every 24 hours
A) I and IV only
B) I, II, III, and IV
C) III and IV only
D) I and II only
Ans: D
Response:
The incidence of infection due to humidifier contamination can be reduced by: 1) using
prepackaged sterile disposables or wick humidifiers, 2) using closed feed reservoir systems
with sterile water, and 3) carefully draining condensate AWAY from the humidifier and
patient. Changing components every 24 hrs has not been shown to reduce the incidence of
nosocomial infection associated with heated water humidifiers.

25. The proper infection control procedures to be used when drawing an arterial blood gas are:
A) hand washing and gloves only
B) apron and protective eyewear
C) mask and protective eyewear
D) all CDC standard precautions
Ans: D
Response:
Standard blood and body fluid precautions should be implemented in the care of all patients.
In particular, blood samples from all patients must be treated with full CDC standard
precautions, as if they were known to be contaminated. This requires diligent hand washing,
gloves, masks and protective eyewear and aprons or gowns if blood is likely to be splashed.

26. An increase in the number of staphylococcal infections is noted among hospitalized patients
receiving respiratory therapy. The most likely source of the infections is
A) the ventilators
B) the medications
C) the personnel
D) distilled water
Ans: C
Response:
Staphylococcal infections are most commonly transmitted via direct contact. Consequently,
health care personnel who fail to adequately wash their hands before caring for each patient
are often implicated in the spread of such infections.

27. Airborne precautions are indicated for which of the following?


A) tuberculosis patient
B) hepatitis patient
C) patient with an open wound
D) cancer patient
Ans: A
Response:
Airborne precautions help reduce airborne transmission of infectious agents. Common
examples of diseases transmitted via this route include legionellosis, tuberculosis, varicella,
measles and histoplasmosis. Organisms transmitted via this route can be dispersed over long
distances by air currents. For this reason, special air handling and ventilation are required to
prevent airborne transmission.

Page 8
SAN PEDRO COLLEGE COMPETENCY APPRAISAL - OXYGENATION

28. You have only one mainstream capnometer available in ICU, but the attending doctor has
asked that you make end-tidal CO2 (PETCO2) measurements on several different patients
before morning rounds. In order to obtain these measurement while minimizing the risk of
cross infection to you patients, which of the following actions would be most appropriate?
A) gas sterilize the sensor between patient
B) provide an individual airway adapter for each patient
C) sterilize the capnometer between patient uses
D) substitute arterial blood gas analysis for infected patients
Ans: B
Response:
Whenever a nondisposable device (respirometer, capnometer, pressure manometer) is used to
monitor ventilatory parameters on multiple patients, the best way to minimize the risk of cross
infection is to use a single-use disposable adapter for each patient.

29. The most frequent route for transmission of nosocomial infections is:
A) contact transmission
B) vehicle transmission
C) airborne transmission
D) vectorborne transmission
Ans: A
Response:
Contact transmission is the most important and most frequent route for transmission of
nosocomial infections.

30. A patient has been receiving nebulized albuterol (Proventil) via SVN for the last 24 hours.
Which of the following would you recommend to minimize the risk of her getting a
nosocomial infection?

I. add an antibiotic to the SVN for every other treatment


II. rinse the SVN with sterile water and dry between treatments
III. use only sterile fluids for nebulization
IV. perform handwashing before giving her a treatment
A) I, II only
B) II, III, IV only
C) III, IV only
D) I, II, III, IV only
Ans: B
Response:
Measures that can be taken to minimize the likelihood of getting a hospital-acquired infection
from a small volume nebulizer (SVN) include: proper handwashing before and after handling
such equipment, rinsing out the SVN with sterile water and air-drying it between treatments
and storing, handling and dispensing multidose medications according to manufacturers'
instructions.

Page 9
CHAPTER 8
Name: __________________________ Date: _____________

1. To avoid the preanalytical errors that are associated with air contamination of a blood gas
sample, all of the following are appropriate EXCEPT:
A) removing all air bubbles
B) mixing only after all air is removed
C) capping the syringe quickly
D) using the minimum amount of heparin
Ans: D
Response:
To avoid pre-analytical errors associated with air contamination of a blood gas sample, you
should fully remove any air bubbles, cap the syringe quickly, and mix the sample only after all
air has been removed. Using the minimum amount of heparin is important, but is not
associated with air contamination.

2. The best way to avoid arterial blood gas analysis errors associated with blood metabolism is
to:
A) analyze the sample immediately
B) place the sample in an ice slush
C) keep the sample at body temperature
D) use dry (lithium) heparin
Ans: A
Response:
ABG errors caused by blood metabolism are time and temperature dependent. It is important
to analyze all ABG samples immediately on receipt in the lab after transport with minimal
delay. If this is not possible, the sample should be kept in an ice/water bath and analyzed as
soon as possible.

3. How often should a transcutaneous PO2 electrode be relocated in a neonate?


A) every 2 hours
B) every 8 hours
C) every 12 hours
D) every 24 hours
Ans: A
Response:
Both transcutaneous PCO2 and PO2 electrodes are heated to 44 °C to “arterialize” the capillary
blood. As a result of the increased temperature at the site the electrodes must be moved
frequently, every 2 hours for neonates and every 2–4 hours for adults.

4. Under ideal conditions, electrochemical oxygen analyzers have an accuracy of:


A) ± 1%
B) ± 2%
C) ± 5%
D) ± 10%
Ans: B
Response:
There are two common types of electrochemical oxygen analyzers: the polarographic (Clark)
electrode and the galvanic fuel cell. Under ideal conditions of temperature, pressure, and
relative humidity, both types are accurate to within 2% of the actual concentration.

Page 1
SAN PEDRO COLLEGE COMPETENCY APPRAISAL - OXYGENATION

5. Which of the following hemoximeter measurements are affected by air contamination?


A) HbCO levels
B) metHb levels
C) HbO2 levels
D) total Hb levels
Ans: C
Response:
Levels of HbO2 values for samples contaminated with air bubbles should be questioned.
Levels of HbCO, metHb, SHb, and total Hb are unaffected by air contamination. Additionally,
inadequate mixing of either a syringe or capillary sample just prior to analysis will result in
erroneous total hemoglobin measurements.

6. Which of the following devices would you select to assess the output accuracy of a Bourdon
gauge regulator used for O2 transport?
A) mercury manometer
B) Clark electrode
C) paramagnetic oxygen analyzer
D) precision flow meter
Ans: D
Response:
To assess the output or flow accuracy of a Bourdon gauge regulator used for O2 transport, you
would use a precision (calibrated) flow meter.

7. To periodically confirm the validity of blood gas analyzer results, you would perform:
A) external statistical quality control
B) instrument performance validation
C) regular preventive maintenance
D) control media calibration verification
Ans: D
Response:
To periodically confirm the validity of a blood gas analyzer, you would perform a calibration
verification using control media. Calibration verification requires analysis of at least three
materials with known values (controls) spanning the entire range of results expected for
clinical samples. At least one control should be analyzed every 8-hour shift. All three levels of
the control media are analyzed at least once every 24 hours.

8. According to ATS recommendations, diagnostic spirometers should be calibrated to within:


A) ± 1% or 10 mL, whichever is greater
B) ± 3% or 50 mL, whichever is greater
C) ± 5% or 100 mL, whichever is greater
D) ± 10% or 500 mL, whichever is greater
Ans: B
Response:
American Thoracic Society (ATS) recommendations for diagnostic spirometers are that
calibration checks should be within ± 3% or 50 mL, whichever is larger.

Page 2
SAN PEDRO COLLEGE COMPETENCY APPRAISAL - OXYGENATION

9. How often should ventilators be tested to verify their performance?


A) daily
B) between patient uses
C) once a week
D) once a year
Ans: B
Response:
All ventilators should be tested after each use to verify performance before the unit is used on
another patient. A complete operational verification procedure, based on the manufacturer's
recommendations, should be performed each time. These procedures should be detailed in the
health care facility's policies and procedures manual.

10. A fuel cell oxygen analyzer is reading 18% when exposed to ambient air. The initial
corrective action should be to:
A) calibrate the sensor
B) check the batteries
C) replace the fuel cell
D) replace the display
Ans: A
Response:
If a galvanic cell oxygen analyzer reads 18% when exposed to ambient air, it should first be
recalibrated to 20.9%. Replace the sensor only if an analyzer fails to calibrate at 21% and
100% oxygen.

11. The process by which the response of an instrument is compared and adjusted to a known
standard is:
A) proficiency testing
B) calibration
C) statistical quality control
D) tonometry
Ans: B
Response:
Calibration is the process whereby the response of an instrument is compared and adjusted to
a known standard. During calibration the analyzer is adjusted to ensure that its response is
accurate and linear, i.e., that the measured value (response) equals the known value.

12. Laboratories receive “unknown” samples for analysis on a regular basis which must be
analyzed and reported back to the agency. This process is known as:
A) statistical quality control
B) calibration verification
C) proficiency testing
D) instrumental comparison
Ans: C
Response:
Under proficiency testing requirement, laboratories receive unknown samples from an agency
on a regular schedule. The samples are analyzed and the results reported back to the agency.
The agency compares the results against its own and those of other laboratories using similar
instrumentation. Discrepancies must be addresses and remediated in order for any laboratory
to maintain CLIA certification, which is a prerequisite for Medicare/Medicaid reimbursement.

Page 3
SAN PEDRO COLLEGE COMPETENCY APPRAISAL - OXYGENATION

13. When performing FVC maneuvers, reproducibility of results can be demonstrated by


confirming that
A) the two largest FVC measures are within150 mL of each other
B) the two largest FVC measures are within 250 mL of each other
C) three consecutive FVC measures are within 150 mL of the predicted value
D) the largest FVC is within 150 mL of the patient's predicted value
Ans: A
Response:
After obtaining the acceptable maneuvers, you need to assure the reproducibility of the
patient's efforts. You assure reproducibility by confirming that the two largest values for both
the FVC and FEV1 are within 150 mL of each other.

14. When checking for leaks in a ventilator circuit, you occlude the patient outlet after setting the
ventilator pressure limit and tidal volume to maximum and peak flow to minimum. After
delivering a manual breath, you judge the test as 'passed' (no leaks) if
A) the high volume alarm sounds
B) the disconnect alarm activates
C) the high pressure limit is reached
D) the I:E ratio warning activates
Ans: C
Response:
Occlude the patient connection, set the pressure limit and tidal volume to their maximum
levels and the peak flow and rate to their minimum levels, and initiate a breath. The pressure
limit should reach the maximum set level, and the high pressure alarm should activate.

15. When calibrating a galvanic fuel cell oxygen analyzer, a reading of 90% is observed and
cannot be adjusted to 100% when analyzing a gas with a known concentration of 100%. You
should:
A) replace the battery
B) replace the fuel cell
C) change the drying crystals
D) decrease the flow of oxygen
Ans: B
Response:
If a galvanic fuel cell oxygen analyzer cannot be adjusted to 100% when analyzing a sample
of 100% O2, the fuel cell is expended and should be replaced.

16. You note a sudden increase in pO2 measurement and decrease in the PCO2 measurement on a
transcutaneous monitor. The most likely cause is:
A) a decrease in blood flow in the area of the probe
B) electrode temperature is too low
C) electrode temperature is too high
D) an air leak at the transducer site
Ans: D
Response:
If the electrode is not prepared and positioned correctly, or detaches during use, air may leak
into the area around the electrode and the pO2 and pCO2 of the air will effect the
measurement.

Page 4
SAN PEDRO COLLEGE COMPETENCY APPRAISAL - OXYGENATION

17. All of the following are true statements regarding the use of a transcutaneous PO2 monitor
EXCEPT
A) these monitors do not require calibration
B) the transducer site must be changed on a regular basis (3-4 hours)
C) the temperature at the probe site must be set above 37o C. but less than 42 o C.
D) the PtcO2 value should be correlated with arterial blood gas samples periodically
Ans: A
Response:
As with blood gas analyzers and capnographs, obtaining accurate data from these monitor
requires proper set-up and calibration.

18. When preparing to use a microprocessor-based mechanical ventilator between patient


applications, you should:
A) change the ventilator circuit and perform both a power-on and extended self test
B) change the ventilator circuit and perform a power-on-self-test only
C) change the circuit and perform a leak test only
D) change the circuit, perform a leak test and verify the function of alarms only
Ans: A
Response:
You always should confirm a successful POST before applying a ventilator to a patient and
whenever you change the circuit. You should also perform an EST between each application
of a ventilator on different patients.

19. You are analyzing quality control samples on a blood gas analyzer as part of a routine quality
control program. Multiple but not successive PCO2 values fall above and below the two
standard deviation limit. You should:
A) record the results as an acceptable
B) record the results as an acceptable after correcting for the difference in measurements
C) record the results as an acceptable if they are within +/- 2 SD of the mean
D) perform a two point calibration and reanalyze the control sample
Ans: D
Response:
Frequent random errors like this indicate a lack of precision, i.e., poor repeatability of
measurement. Any instrument that demonstrates poor repeatability over time is deemed "out-
of-control." In such cases, you would have to identify the problem, take appropriate corrective
action and re-confirm that the analyzer is back in-control prior to the reporting results for any
patient sample.

Page 5
SAN PEDRO COLLEGE COMPETENCY APPRAISAL - OXYGENATION

20. The following data are obtained on a patient using the same arterial sample:

Blood Gas Analyzer CO-Oximeter


pH 7.35 Oxyhemoglobin 97%
PaCO2 28 torr Carboxyhemoglobin 1%
HCO3 14 mEq/L Methemoglobin 1%
BE -10 mEq/L Hemoglobin 13.8 g/dL
PaO2 40 torr
SaO2 73%

You should do which of the following?


A) report the SaO2 value as 73%
B) report the SaO2 value as 97%
C) recommend administration of bicarbonate
D) recalibrate the instruments and repeat the analyses
Ans: D
Response:
The blood gas analyzer (SaO2 = 73%) and CO-oximeter saturation (SaO2 = 97%) are
inconsistent with each other. Because you do not know which instrument is providing the
false reading, the only good decision would be to recalibrate both instruments and repeat the
analyses.

21. After a patient performs a forced vital capacity (FVC) maneuver, you observe the following
volume vs. time plot on the screen of a portable spirometer:

What validity error does this plot indicate?


A) too slow a start to forced exhalation
B) coughing during the maneuver
C) breathing during the maneuver
D) premature end to expiration
Ans: A
Response:
The 'S' shaped pattern of this FVC plot reveals too slow a start to forced exhalation. The slope
(equivalent to flow) starts out low, then increases markedly toward the middle of the
maneuver, finally plateauing toward the end of the breath. A normal FVC trace would have
the steepest slope early in the breath.

Page 6
SAN PEDRO COLLEGE COMPETENCY APPRAISAL - OXYGENATION

22. After a patient performs a forced vital capacity (FVC) maneuver, you observe the following
volume vs. time plot on the screen of a portable spirometer:

What validity error does this plot indicate?


A) too slow a start to forced exhalation
B) coughing during the maneuver
C) breathing during the maneuver
D) premature end to expiration
Ans: B
Response:
This FVC graph indicates coughing during the maneuver. An abrupt pause occurs about a
third of the way through the effort, followed by a short inspiratory effort preceding the cough
and then an irregular pattern of exhalation. A normal FVC trace would be smooth throughout.

23. After a patient performs a forced vital capacity (FVC) maneuver, you observe the following
volume vs. time plot on the screen of a portable spirometer:

What validity error does this plot indicate?


A) too slow a start to forced exhalation
B) coughing during the maneuver
C) breathing during the maneuver
D) premature end to expiration
Ans: C
Response:
This FVC graph shows an extra breath as a short plateau occurring about a third of the way
through the effort, which could result in falsely overestimating the actual FVC. A normal FVC
curve consists of a single breath with a smooth and uninterrupted trace throughout.

Page 7
SAN PEDRO COLLEGE COMPETENCY APPRAISAL - OXYGENATION

24. After a patient performs a forced vital capacity (FVC) maneuver, you observe the following
volume vs. time plot on the screen of a portable spirometer:

What validity error does this plot indicate?


A) too slow a start to forced exhalation
B) coughing during the maneuver
C) breathing during the maneuver
D) premature end to expiration
Ans: D
Response:
This FVC graph reveals a normal, smooth high slope (high flow) at the beginning of the
effort; however, the breath ends prematurely, as evident in the flat plateau. In a normal FVC
curve, the slope progressively decreases after the initial blast, eventually transitioning to a
plateau, but only at the true end-of-test volume.

25. A blood gas laboratory uses +2 standard deviations as its quality control limits. Which of the
following is indicated by the following chart for a PCO2 control value of 40 torr?

A) single random error


B) positive bias error
C) multiple random errors
D) negative bias error
Ans: A
Response:
This Levy-Jennings chart for a PCO2 control value of 40 torr shows a single random error (6th
measurement). Single aberrant values like these are relatively common (occurring on average
once in every 20 analyses), and are due to random errors of measurement that occur with any
laboratory instrument. As long as subsequent measures are in-control, no remedial action is
needed.

Page 8
SAN PEDRO COLLEGE COMPETENCY APPRAISAL - OXYGENATION

26. A blood gas laboratory uses +2 standard deviations as its quality control limits. Which of the
following is indicated by the following chart for a PCO2 control value of 40 torr?

A) single random error


B) positive bias error
C) multiple random errors
D) negative bias error
Ans: B
Response:
This Levy-Jennings chart for a PCO2 control value of 40 torr shows systematic error or bias.
Note that beginning with observation #13 and continuing through #20 there is an upward trend
in the reported values for the 40 torr PCO2 control. Over time, this particular trend is shifting
the mean above the control value, causing a positive bias in measurement. Bias errors like
these are serious, indicating either incorrect procedure or instrument component failure.

Page 9
CHAPTER 9
Name: __________________________ Date: _____________

1. On checking a ventilator patient's progress notes, you see that the attending physician's
treatment plan includes starting spontaneous breathing trials. Your most appropriate action is
to do which of the following?
A) begin a spontaneous breathing trial immediately
B) wait until after the patient's sedation is discontinued
C) check to verify that a valid physician's order is present
D) review the plan changes with the patient's nurse
Ans: C
Response:
All respiratory care is normally provided by order of the patient's personal doctor or attending
physician. A progress note or plan is not the same as an order. Before initiating any
therapeutic or diagnostic procedure, you need to check to see that a valid physician order is
present.

2. After completing a ventilator check, you note that you incorrectly computed the patient's
compliance in a prior entry. Which of the following is the most appropriate course of action?
A) inform the medical director of the error
B) erase the error and write over it with the correction
C) copy the entire ventilator sheet over to correct the error
D) line out the error, write the word “error,” and correct and initial it
Ans: D
Response:
The ventilator flow sheet is part of the patient record and represents a legal document.
Consequently, whenever errors are detected, the error should be lined out, the word “error”
should be noted, the corrected information should be given, and the therapist correcting the
error should write his or her initials.

3. When must you contact the ordering physician when implementing a respiratory care
treatment protocol?
A) after your initial assessment of the patient
B) whenever a change in therapy is needed
C) prior to providing the initial therapy
D) whenever a limit or boundary rule takes effect
Ans: D
Response:
When implementing a protocol, you must know the limits (also called boundaries) within
which you are permitted to make independent adjustments, as well as know what conditions
require physician notification. In general, if your assessment indicates that the patient's
response strays outside any of the protocol's defined boundaries, you cannot proceed until you
notify the ordering physician and determine a new course of action.

Page 1
SAN PEDRO COLLEGE COMPETENCY APPRAISAL - OXYGENATION

4. A prescription for an aerosolized drug for a patient under your care is complete except for the
actual drug dosage. Which of the following is the appropriate action to take in this case?
A) use the standard dosage listed in the package insert
B) ask your medical director to rewrite the prescription
C) contact the ordering physician for clarification
D) postpone the therapy until the following day
Ans: C
Response:
The minimum requirements for a proper prescription for respiratory care-related drugs include
the following: (1) the drug name, (2) the drug dosage/concentration, (3) the frequency of
administration, (4) the route of administration, and (5) the signature of the prescribing
physician. Always seek clarification from the patient's physician if the order does not specify
this necessary information.

5. The best way to routinely communicate a patient's clinical status to the appropriate members
of the health care team is by:
A) reporting information to the respiratory therapy supervisor
B) reporting information to the next shift of respiratory therapy staff
C) recording information in the respiratory therapy department records
D) recording information in the patient's chart
Ans: D
Response:
The patient's chart is an official record of their ongoing progress and course of treatment.
Consequently, the best way to routinely communicate a patient's clinical status is by
accurately recording all essential information in the patient's medical record.

6. If you are giving routine therapy and note adverse changes in the patient's condition, you
should do which of the following?

I. notify the nurse who is responsible for the patient


II. contact the physician if a change in therapy seems warranted
III. record the patient’s reactions in the chart
A) II only
B) I and III only
C) II and III only
D) I, II, and III
Ans: D
Response:
When noting an adverse change in a patient's condition, the therapist should notify the nurse,
contact the physician (if a therapy change is warranted), and record the reaction in the patient's
chart. If the patient becomes unstable due to the adverse change, the therapist should also stay
with the patient until he or she is stable or help arrives. Recording this information in the
medical record is a very important part of making sure that vital patient information is
communicated among all health care professionals providing care to the patient.

Page 2
SAN PEDRO COLLEGE COMPETENCY APPRAISAL - OXYGENATION

7. Midway through an aerosol drug treatment via IPPB, a patient complains of dizziness and
tingling in her fingers. After stopping the therapy and adjusting the equipment to correct the
problem and completing the treatment, you should record which of the following in the chart?

I. medication used during the treatment


II. the patient’s pulse and blood pressure before and after the treatment
III. the nature of the problem and the way in which it was corrected
A) I and II only
B) I and III only
C) II and III only
D) I, II, and III
Ans: D
Response:
If any adverse reaction is noted to a patient receiving therapy, the treatment should initially be
stopped, the patient assessed, the problem corrected. Once the patient is stable, the therapy can
be completed. All of these points should be recorded in the patient's medical record.

8. When assessing a patient after a treatment, you note a significant deterioration in vital signs.
The most appropriate action in this case is to:
A) call for the institution's rapid response team
B) report the findings to the next shift of respiratory therapy staff
C) chart the findings as an unexpected response to therapy
D) orally communicate the findings to the patient's physician
Ans: A
Response:
Any unexpected response to therapy or adverse effects noted when charting a patient
encounter should also be communicated orally to the patient's physician and nurse. In general,
the more serious the problem, the sooner these key people should be informed. However, if it
is clear that the patient's vital signs are deteriorating, do not wait to inform the doctor. Instead,
call for your institution's rapid response team.

9. When a patient refuses to take a treatment, you should do which of the following?
A) notify the nurse in charge and chart the patient's refusal
B) give the treatment anyway and record that it was done
C) ask the head nurse to convince the patient to take the treatment
D) leave the patient's room and discontinue the therapy
Ans: A
Response:
When a patient is intent on refusing treatment, the nurse should be notified immediately and
the refusal must be noted in the patient's medical record.

10. Which of the following represents a charting entry that would be placed in the “S” portion of a
problem-oriented (SOAP) record entry?
A) color of patient's sputum
B) the patient is not tolerating weaning
C) patient states, “I'm short of breath”
D) perform another ABG after next attempt
Ans: C
Response:
Color of sputum can be verified by others and should appear in the Objective portion of the
SOAP “Not tolerating weaning” is a qualitative judgment and therefore belongs in the
Analysis portion of a SOAP “Perform another ABG after next attempt” is a statement that fits
best in the Plan portion of the SOAP.

Page 3
SAN PEDRO COLLEGE COMPETENCY APPRAISAL - OXYGENATION

11. Which of the following would be the most effective method to identify noncompliance with a
smoking cessation program?
A) Measure the SpO2
B) Obtain blood for co-oximetry
C) Interview the patient
D) Obtain an arterial blood gas
Ans: C
Response:
The best way to identify whether a patient is complying with a smoking cessation program is
usually to interview him or her. If there is doubt about the patient's truthfulness, measurement
of urinary nicotine can be helpful, but may yield false positive due to environmental exposure.
Of the choices given, C is the BEST choice.

12. In reviewing the ventilator flow sheet for a patient who is being mechanically ventilated, you
note the following data: Dynamic compliance = 38 mL/cm H2O; Static compliance = 33
mL/cm H2O. You should conclude which of the following?
A) the calculations are erroneous
B) airway resistance is 5 cm H2O/L/sec
C) a bronchopleural fistula has developed
D) the patient has significant airway obstruction
Ans: A
Response:
You compute static compliance based on the difference between the plateau and PEEP
pressures. Dynamic compliance includes airway resistance and is therefore based on the
difference between the peak and PEEP pressures. Since this pressure difference is always
greater than that for static compliance, dynamic compliance must always be less than static
compliance. In this example dynamic compliance is greater than static compliance, which
indicates an error in the calculations.

13. The first step in implementing a respiratory care treatment protocol is to:
A) Formulate a treatment plan
B) Initiate patient contact
C) Review the relevant medical literature
D) Check the physician order for the protocol
Ans: D
Response:
The first area addressed in implementing a respiratory care treatment protocol is to perform an
initial patient evaluation. This includes: (1) checking the physician order for the protocol, (2)
reviewing the medical record, (3) initiating patient contact (using universal precautions) and
(4) assessing the patient (e.g., vital signs; chest inspection, palpation, percussion, auscultation;
bedside spirometry; pulse oximetry; and other diagnostic evaluations as needed, such as
arterial blood gases. Remember that you cannot implement any respiratory care protocol
unless the physician has activated it by order in the medical record.

Page 4
SAN PEDRO COLLEGE COMPETENCY APPRAISAL - OXYGENATION

14. The SOAP format for charting includes all of the following EXCEPT:
A) assessment
B) objective findings
C) patient prognosis
D) subjective findings
Ans: C
Response:
SOAP stands for Subjective (patient complaints/concerns), Objective (clinical signs or
measures), Assessment (problem identification or patient response to therapy) and Plan
(treatment or therapy recommendations). When charting using any format at your health care
setting, you must know the format and its application to charting situations. SOAPing is a well
known format especially for the problem-oriented medical record. After that, in this question,
the main thing to pay attention to is that word EXCEPT, it changes the answer you are looking
for and can easily be missed, that is why it is always uppercase on NBRC exams.

15. According to the Joint Commission which of the following charting abbreviations or methods
is allowed or encouraged?
A) Q.D.
B) leading zeros, e.g. 0.5 mg
C) IU (international unit)
D) trailing zeros, e.g. 5.0 mg
Ans: B
Response:
The Joint Commission's list of 'do not use' abbreviations includes the following: U (for unit),
IU (for international unit), Q.D. and Q.O.D., trailing zeros, and MS/MSO4/MgSO4. Use of
leading zeros (e.g., 0.5 mg) is encouraged. See section in Chapter 8 and the CD for more on
acceptable and banned abbreviations.

16. What is incorrect about the following medical record notation?

.5 mL albuterol .5% in 3 mL normal saline administered via SVN


A) SVN not a common or recommended abbreviation
B) the 3 mL should be written as 3.0 mL
C) the .5 mL and .5% should have leading zeros
D) the concentration of normal saline is not provided
Ans: C
Response:
Whether in prescriptions or when charting treatments, failure to provide leading zeros to the
left of a decimal point (.X mL) is one of the JCAHO's five mandatory 'do not use'
abbreviations. Following zeros (X.0 mL) are also 'banned'. This is because the decimal point
can easily be missed, resulting in 10X the dosage. Always use a zero before a decimal point
(0.X mL. This questions tests your knowledge of JCAHO's regulations and your attention to
detail.

Page 5
SAN PEDRO COLLEGE COMPETENCY APPRAISAL - OXYGENATION

17. You start O2 therapy via nasal cannula on a patient. In addition to your signature and
credentials, which of the following is the best example of appropriate documentation?
A) nasal cannula placed on patient, HR=96, SpO2=92%, RR=22
B) 12/6/95, 2 L/min oxygen placed on patient at 0830
C) 0830, nasal cannula at 2 L/min, temp=101, HR=96, RR=22, BP=150/80
D) 12/6/95, 0830, nasal cannula started at 2 L/min, SpO2=92%, RR=22, HR=96
Ans: D
Response:
When charting the initiation of oxygen therapy, in addition to the therapist's signature and
credentials, the date, time, type of delivery device and O2 liter flow as well as patient
assessment parameters such as SpO2, RR and HR should be noted. This questions is testing
your knowledge of, and attention to, the detail of a correct medical record entry for the
administration of Oxygen to a patient via nasal cannula.

18. Which of the following statements would be most appropriate to use in explaining to a patient
why it is necessary to cough after an aerosol treatment?
A) "If you don't cough, you will get pneumonia"
B) "Coughing will help you keep your airways clear"
C) "I'll have to suction you if you don't cough"
D) "You might as well not be in the hospital if you don't cooperate"
Ans: B
Response:
When instructing a patient on any aspect of their therapy, it is important to do so in a
supportive, plain language and non-threatening manner. Of the choices given, only "Coughing
will help you keep your airways clear" fulfills these criteria. This is a perfect example of a
question testing your ability to explain therapy to a patient properly. Remember also, this
may not be how you would say it, but on this kind of test your job is to select the best choice
of those offered, not look for the best choice in the world.

19. You are called to the ED to provide a bronchodilator treatment for a patient having a severe
asthma attack. When quickly confirming the written order you find it contains some
prohibited notations and thus could be read as either '.5 U albuterol by SVN' or 5 c.c albuterol
by SVN." The prescribing physician is busy overseeing a code. You should:
A) wait until the physician is done with the code so you can clarify the improper notation
and the correct order
B) administer the treatment using the standard dosage (0.5 mL) and clarify the order as
soon as possible thereafter
C) cross out the prohibited notations, provide the correct abbreviations and initial and date
the changes
D) have the nurse review the order and correct the improper notation
Ans: B
Response:
This order contains at least two improper notations, i.e., c.c. (use mL), and lack of leading
zeros before a decimal point (that may be 'lost' on the order line). Normally, if an order
contains a prohibited notation, the respiratory therapist must confirm the intent of the order
before proceeding. The exception is when order confirmation might delay essential or
emergency patient treatment (as here). In these cases, if, in the judgment of the caregiver the
order is clear and complete and the delay to obtain confirmation from the physician would
place the patient at greater risk, then the order should be carried out and the confirmation
obtained as soon as possible thereafter.

Page 6
SAN PEDRO COLLEGE COMPETENCY APPRAISAL - OXYGENATION

20. Which of the following are important considerations to remember when a patient is being
scheduled for respiratory treatments?

I. make certain postural drainage does not follow meals


II. make sure therapy does not conflict with other procedures
III. make changes in scheduling note in medical record
A) II only
B) I and III only
C) II and III only
D) I, II and III
Ans: D
Response:
When a patient is being scheduled for respiratory treatments under a patient-focused, team
approach, it is important to remember that the therapy should not conflict with other
procedures and meals. In addition, scheduling changes should be conveyed to other clinicians
on the patient-care team. The best way to communicate that information to other health care
team members is the medical record. Knowing this information and selecting the response
that includes all the correct answers is the key to answering this type of question.

21. A portable spirometer requires that you enter the patient's height in cm in order to derive
normal values. The patient tells you that she is 5 feet 6 inches tall. What value would you
enter into the device?
A) 26 cm
B) 66 cm
C) 168 cm
D) 186 cm
Ans: C
Response:
The conversion factor for inches to cm is 2.54 cm = 1 inch. The patient is 66 inches tall [(5 x
12) + 6]. 66 x 2.54 = 167.6 in.

22. The results of an arterial blood gas analysis for a patient who is breathing 100% oxygen are
below:

pH 7.24
PaCO2 38 torr
HCO3 23 mEq/L
BE -1
PaO2 610 torr
SaO2 100%

Which of the following is the likely problem?


A) respiratory acidosis
B) large physiologic shunt
C) metabolic acidosis
D) laboratory error
Ans: D
Response:
In terms of oxygenation, a PaO2 of 610 torr on 100% O2 is not only possible, but near normal
(based on the alveolar air equation). On the other hand, the acid-base values are not consistent
with the underlying relationship that determines pH (the Henderson-Hasselbach equation). In
this case, both the PaCO2 and HCO3 are normal. With both these values being within the
normal range, the pH also would have to be close to normal, which it clearly is not (pH =
7.24). The only possibility here is a laboratory error.

Page 7
SAN PEDRO COLLEGE COMPETENCY APPRAISAL - OXYGENATION

23. A nurse tells you that her patient is scheduled to start chest physiotherapy QID this morning
and that she would like you to get started before he goes to radiology for a CT scan. Which of
the following should you do FIRST?
A) auscultate and percuss the patient's chest.
B) initiate therapy after reviewing the X-ray
C) interview the patient.to obtain a history
D) confirm the doctor's order in the chart
Ans: D
Response:
Your state licensure regulations and institutional policies will dictate if you can accept orders
from other health care professionals, and may vary according to whether the order is written
or verbal. If you can accept orders, they must come from an authorized health care provider
with prescribing privileges. You cannot accept orders transmitted to you via unauthorized
third parties, such as registered nurses. If an order is transmitted to you via a third party, you
must verify the order in the patient's chart before proceeding.

24. You start nasal O2 therapy on a post-op patient. All of the following are objective responses to
the therapy except:
A) SpO2 = 91% @ 4 L/min
B) resp rate = 19/min
C) patient complains of general discomfort
D) no evidence of central cyanosis
Ans: C
Response:
Objective responses are signs observed by or measurements made by you, the therapist. All of
the available choices fall into this category except the complaint of discomfort, which is a
subjective patient symptom.

25. A patient complains of shortness of breath and dizziness after lung expansion therapy. You
should:
A) check the medical history and notify the nurse
B) instruct the patient to take rapid and deep breaths
C) reassure the patient and document the response
D) observe subsequent therapy for similar responses
Ans: C
Response:
Shortness of breath and dizziness are possible side effects of lung expansion therapy,
particularly if the patient is breathing too rapidly. In this instance, the therapy should be
stopped, the patient reassured and the response documented.

26. All spirometric values obtained under ambient conditions should be converted to:
A) ambient temperature and pressure, saturated (ATPS)
B) body temperature, ambient pressure, saturated (BTPS)
C) standard temperature and pressure, dry (STPD)
D) ambient temperature and pressure, dry (ATPD)
Ans: B
Response:
All volumes (and flows) measured on PFT apparatus under ambient conditions must be
corrected to those existing in the lungs (body temperature, ambient pressure, saturated, or
BTPS).

Page 8
SAN PEDRO COLLEGE COMPETENCY APPRAISAL - OXYGENATION

27. In explaining a therapeutic goal of IPPB therapy to postoperative patients, it would be most
appropriate for you to say which of the following?
A) "This will prevent pneumonitis."
B) "This will help you take deep breaths."
C) "This will prevent atelectasis."
D) "This will increase your intrathoracic pressure."
Ans: B
Response:
When instructing a patient on any aspect of their therapy, it is important to do so in a
supportive, plain language and non-threatening manner. Of the choices given, only "This will
help you take deep breaths" fulfills these criteria.

Page 9
CHAPTER 10
Name: __________________________ Date: _____________

1. A patient suddenly loses consciousness. Which of the following is the first procedure you
should perform to maintain an open airway in this patient?
A) inserting a laryngeal mask airway
B) applying the “jaw thrust” maneuver
C) inserting an oropharyngeal airway
D) applying the “head-tilt/chin-lift” maneuver
Ans: D
Response:
The initial procedure used to maintain an open airway in an unconscious patient is the “head-
tilt/chin-lift” maneuver. This maneuver helps displace the tongue away from the posterior
pharyngeal wall.

2. A patient in the intensive care unit exhibits signs of acute upper airway obstruction and is
concurrently having severe seizures that make it impossible to open the mouth. In this case,
what is the adjunct airway of choice?
A) oral endotracheal tube
B) nasopharyngeal airway
C) tracheostomy tube
D) oropharyngeal airway
Ans: B
Response:
When the mouth is unavailable as a route for airway access, you should try the nose. In this
case, a nasopharyngeal airway would help overcome upper airway obstruction and can be
easily inserted.

3. An oropharyngeal airway is least appropriate for a patient who:


A) is having seizures
B) requires manual ventilation
C) is conscious and alert
D) is heavily sedated
Ans: C
Response:
Oropharyngeal airways can provoke a gag reflex and possible vomiting and should therefore
generally not be used in conscious patients.

4. When ventilating a patient with a bag-valve resuscitator through a laryngeal mask airway
(LMA), you note significant air leakage. Which of the following should be your first approach
to eliminating this leakage?
A) bag slowly to reduce peak pressure
B) add more air to LMA the cuff
C) pull the tube out 2–3 cm
D) lower the cuff pressure
Ans: A
Response:
To avoid leaks during positive pressure ventilation through an LMA, you should use slow
inflation, keep peak inspiratory pressures less than 20–30 cm H2O, and limit delivered tidal
volume to no more than 8 mL/kg. If a leak persists in spite of these efforts to limit
pressure/volume, you should readjust the tube's position by pressing it downward and
resecuring it. Do NOT add more air to the LMA cuff, since this can worsen the leak by
pushing the cuff away from the larynx.

Page 1
SAN PEDRO COLLEGE COMPETENCY APPRAISAL - OXYGENATION

5. Which of the following should be prescribed in order to provide adequate humidification to an


intubated patient?
A) inspired gas with 100% relative humidity
B) inspired gas with an absolute humidity greater than 30 mg/L
C) inspired gas through an unheated bubble humidifier
D) tracheobronchial suctioning
Ans: B
Response:
To provide adequate humidity to an intubated patient, strive to provide a gas with an absolute
humidity of at least 30 mg/L. Inspired gas with 50% relative humidity or that provided
through a cold bubble humidifier provides an absolute humidity of about 15–20 mg/L. If
administered to a patient with a bypassed upper airway, this level of humidity can cause
damage to the tracheal mucosa and impair mucociliary clearance.

6. When checking for proper placement of an endotracheal tube in an adult patient on chest X-
ray, it is noted that the distal tip of the tube is 3 cm above the carina. Which of the following
actions is appropriate?
A) none, since the tube is properly positioned in the trachea
B) withdraw the tube by 4–5 cm (using tube markings as a guide)
C) withdraw the tube by 1–2 cm (using tube markings as a guide)
D) advance the tube by 1–2 cm (using tube markings as a guide)
Ans: C
Response:
The tip of an endotracheal or tracheostomy tube should be positioned 4–6 cm above the
carina. If the tube is malpositioned, the therapist should gain physician permission to remove
the old tape and reposition the tube using the markings on the tube as a guide. This often
requires two people to prevent extubation. The new position should again be confirmed by X-
ray or laryngoscopy.

7. After insertion of a esophageal-tracheal Combitube®, you begin ventilation through the #1


pharyngeal airway connection. Your partner reports an absence of breath sounds and the
presence of gurgling over the epigastrium. In order to provide effective ventilation, you
should:
A) deflate the large (#1) cuff
B) withdraw the Combitube® 3–4 cm
C) ventilate through the other (#2) tube
D) deflate the small (#2) cuff
Ans: C
Response:
After insertion of a esophageal-tracheal Combitube®, you normally begin ventilation through
the #1 pharyngeal airway connection. If you detect no chest motion or breath sounds via this
route, or if there are stomach sounds present, the tube is in the trachea and you should switch
to ventilating the patient via the shorter esophageal/tracheal tube (labeled as #2).

8. Significant overinflation of an endotracheal tube cuff may cause which of the following?
A) laryngospasm
B) tissue damage
C) tachycardia
D) stridor
Ans: B
Response:
Tracheal wall tissue damage can occur as a result of overinflating the cuff of an endotracheal
tube, because pressures significantly above 20–25 cm H2O can decrease or occlude capillary
blood flow.

Page 2
SAN PEDRO COLLEGE COMPETENCY APPRAISAL - OXYGENATION

9. Which of the following devices are contraindicated for a patient whose upper airway has been
bypassed?
A) a heat and moisture exchanger (HME)
B) a heated large-volume jet nebulizer
C) a simple bubble humidifier
D) a heated cascade humidifier
Ans: C
Response:
When the upper airway has been bypassed, the only way to prevent a large humidity deficit is
to provide inspired gases at or near BTPS conditions. This is not possible with a simple
(unheated) bubble humidifier. Instead, either a heated wick humidifier, heat and moisture
exchanger, or heated large-volume jet nebulizer must be used. These systems are capable of
controlling temperature and humidity levels and providing saturated gases at or near the
normal body temperature.

10. The methylene blue test is used to confirm:


A) “leakage” type aspiration
B) tracheal granuloma
C) infection
D) artificial airway obstruction
Ans: A
Response:
The methylene blue test can help determine if “leakage” aspiration is occurring. Methylene
blue may be added to the patient's feedings or swallowed by the patient in a small amount of
water. Once the dye is introduced, the patient's trachea is suctioned through the artificial
airway. If blue-tinged secretions are retrieved while suctioning, aspiration is occurring.

11. In which of the following unilateral lung disorders would you recommend placing the “good”
lung down?

I. unilateral interstitial emphysema


II. unilateral lung contusion/abscess
III. unilateral pneumonia
A) I and II only
B) II and III only
C) III only
D) I, II and III
Ans: C
Response:
In patients with unilateral lung disease, placing the good lung in the dependent or "down"
position tends to improve oxygenation. Oxygenation improves because gravity directs more
blood flow to the well-ventilated dependent alveoli. In conditions such as lung contusion or
abscess, however, the good lung normally is kept in the up position. This helps prevent blood
or pus from entering the good lung. Placement of the good lung up (and diseased lung down)
is also indicated with unilateral pulmonary interstitial emphysema (PIE).

Page 3
SAN PEDRO COLLEGE COMPETENCY APPRAISAL - OXYGENATION

12. A patient begins breathing and regains effective circulation after resuscitation but remains
unconscious. What position should you place this patient in?
A) Trendelenburg
B) supine
C) prone
D) lateral recumbent
Ans: D
Response:
The lateral recumbent or recovery position (victim placed on side and with the lower arm in
front of the body) helps maintain a patent airway and reduces the risk of airway obstruction
and aspiration in unresponsive adults with normal breathing and effective circulation.

13. As compared to the oral route, which of the following is the MAJOR disadvantage of nasal
endotracheal intubation?
A) increased incidence of sinusitis/otitis media
B) increased incidence of nasal tissue necrosis
C) greater airway resistance/work of breathing
D) difficulty in providing nutrition orally
Ans: C
Response:
The MAJOR disadvantage of nasal intubation is that a smaller tube is often needed, thereby
increasing the patient's airway resistance and work of breathing. Of lesser importance are the
increased potential for 1) sinusitis and otitis media; 2) necrosis of the nasal septum/external
meatus; and 3) nosebleed. As with the oral route, oral feedings are still very difficult, but
patients may be allowed to sip water in small quantities.

14. A conscious patient with an intact gag reflex requires an artificial airway solely to prevent
obstruction of the upper airway by the tongue. Which of the following type of airway is most
suitable in this situation?
A) tracheostomy tube
B) oropharyngeal airway
C) orotracheal tube
D) nasopharyngeal airway
Ans: D
Response:
An oropharyngeal airway can readily illicit a gag reflex and possible vomiting and should
therefore generally not be used in conscious patients. In this instance, a nasopharyngeal
airway should be used to maintain a patient airway.

15. A mechanically ventilated patient is breathing asynchronously with the ventilator. Breath
sounds are absent on the left, with dullness to percussion and a left shift of the trachea. Which
of the following is the most likely explanation for the problem?
A) the patient is experiencing diffuse bronchospasm
B) the endotracheal tube is in the right mainstem bronchus
C) a tension pneumothorax has developed on the left
D) a tracheoesophageal fistula has developed
Ans: B
Response:
A dull percussion note, tracheal shift toward the affected side and absent breath sounds are all
signs of atelectasis. In this instance, a right mainstem bronchus intubation would likely cause
left-sided atelectasis.

Page 4
SAN PEDRO COLLEGE COMPETENCY APPRAISAL - OXYGENATION

16. An intubated patient is receiving volume-cycled ventilation. During the patient-ventilator


check, you determine the presence of a large leak around the endotracheal tube cuff, which is
resulting in a loss of about a third of the set tidal volume on each breath. Which of the
following should you do?
A) Increase the set tidal volume to make up for the volume loss
B) Increase the set rate to maintain the same minute ventilation
C) Add air to the cuff until the leak is minimized at peak inspiratory pressure
D) Add air to the cuff until the cuff pressure equals the peak inspiratory pressure
Ans: C
Response:
A large leak around the endotracheal tube cuff usually indicates inadequate cuff inflation. To
properly adjust the cuff, you should add air until the leak is minimized

17. After a physician intubates a patient in the emergency room, your partner begins manual
ventilation with 100% O2. On auscultation, you note the absence of breath sounds, but hear
gurgling over the epigastrium. Which of the following has most likely occurred?
A) a right-sided tension pneumothorax
B) intubation of the left mainstem bronchus
C) intubation of the patient's esophagus
D) intubation of the right mainstem bronchus
Ans: C
Response:
After intubation, one should listen for equality of breath sounds as the patient is being
manually ventilated with oxygen. Air movement or gurgling sounds over the epigastrium
indicate possible esophageal intubation.

18. In order to insure adequate humidification for a patient with an artificial tracheal airway,
inspired gas at the proximal airway should be:
A) 100% saturated with water vapor at 32-35 °C
B) 80% saturated with water vapor at 32-35 °C
C) 100% saturated with water vapor at 37-40 °C
D) 100% saturated with water vapor at 100-110 °F
Ans: A
Response:
To insure adequate humidification for patients with artificial airways, inspired air should be
provided to the airway as near to body temperature and body humidity as possible. This may
be accomplished via a heated humidifier designed for high flows, or a heated jet type
nebulizer. These devices should be heated to a between 32-35 °C at the proximal end of the
airway. The air may need to be heated to a higher temperature at the humidification device,
since it will lose heat as it moves toward the patient.

19. A patient is intubated with an appropriate size endotracheal tube and is being ventilated with a
positive pressure ventilator. During inspiration, air is heard at the mouth. Which of the
following should be done?
A) check the cuff inflation
B) suction the patient
C) recommend a chest X-ray
D) replace the tube
Ans: A
Response:
Air leaking from the mouth of an orally intubated patient, particularly at peak inspiration, is
generally caused by insufficient air in the cuff. In this instance, air should be added just until
no leak is heard or to permit a minimal leak only at peak inspiration.

Page 5
SAN PEDRO COLLEGE COMPETENCY APPRAISAL - OXYGENATION

20. The use of heated humidification is important for a patient being treated with which of the
following?
A) 50% venturi mask
B) 2 L/min nasal cannula
C) endotracheal tube
D) nasopharyngeal airway
Ans: C
Response:
In order to minimize mucosal drying and maximize the absolute humidity delivered to
intubated patients, the source gas should be heated and humidified.

21. A comatose patient intermittently occludes his airway. You should recommend
A) immediately intubating and ventilating
B) inserting an oropharyngeal airway
C) performing a tracheostomy
D) ventilating with 100% oxygen
Ans: B
Response:
In comatose patients airway obstruction is often due to the tongue obstructing the posterior
pharynx. In this instance, the insertion of an oropharyngeal airway would help overcome this
problem.

22. A physician has requested your assistance in extubating an orally intubated patient. Which of
the following should be done BEFORE the tube itself is removed?

I. suction the pharynx


II. preoxygenate the patient
III. confirm cuff inflation
IV. suction the ET tube
A) II and IV only
B) I, II and IV only
C) III and IV only
D) I, II, III and IV
Ans: B
Response:
Before extubation, the therapist should 1) suction the endotracheal tube, 2) suction through the
pharynx to above the cuff, 3) oxygenate the patient well after suctioning, and 4) fully
DEFLATE the tube cuff.

23. The major complication associated with endotracheal tube extubation is:
A) bradycardia
B) tracheomalacia
C) laryngospasm
D) aspiration
Ans: C
Response:
The major complication associated with endotracheal tube extubation is laryngospasm. Post-
extubation laryngospasm is usually transient, lasting a matter of seconds. Should this occur,
oxygenation may be maintained with a high FIO2, and application of positive pressure. If the
spasm persists, a neuromuscular blocking agent may be needed, which will require manual
ventilation or reintubation.

Page 6
SAN PEDRO COLLEGE COMPETENCY APPRAISAL - OXYGENATION

24. The maximum time devoted to any intubation attempt should be no more than:
A) 60 seconds
B) 45 seconds
C) 30 seconds
D) 90 seconds
Ans: C
Response:
Prior to intubation, the patient is often apneic or in respiratory distress. Adequate ventilation
and oxygenation by bag and mask should be restored prior to attempting intubation. The use
of 100% oxygen provides a reserve during the intubation procedure. No more than 30 seconds
should be devoted to any intubation attempt. If intubation fails, the patient should immediately
be ventilated and oxygenated for 3 to 5 minutes before another attempt is made.

25. The average depth of proper oral endotracheal tube insertion from the teeth of an adult female
is:
A) 23 cm from the patient's teeth
B) 23 in from the patient's teeth
C) 21 cm from the patient's teeth
D) 32 cm from the patient's teeth
Ans: C
Response:
Once the glottis is visualized and the epiglottis moved, the endotracheal tube is inserted from
the right side and advanced without obscuring the glottic opening. The tip of the tube should
be seen passing between the cords and then advanced until the cuff has passed the cords by
two to three centimeters. The average insertion depth for an adult male is 23 cm from the
teeth; for a female 21 cm.

26. A physician asks you to assess the upper airway function of a patient with a fenestrated
tracheostomy tube. How should this be accomplished?
A) replace the inner cannula, plug the outer, inflate the cuff
B) remove the inner cannula, plug the outer, inflate the cuff
C) replace the inner cannula, plug the outer, deflate the cuff
D) remove the inner cannula, plug the outer, deflate the cuff
Ans: D
Response:
A fenestrated tracheotomy tube is a double cannulated tube that has an opening in the
posterior wall of the outer cannula, above the cuff. Removal of the inner cannula opens the
fenestration. Plugging of the proximal opening of the tube's outer cannula (with the cuff
deflated) allows for assessment of upper airway function. Removal of the plug allows access
for suctioning. If the need for mechanical ventilation occurs, the inner cannula can be
reinserted.

27. An adult male patient on ventilatory support has just been intubated with a 7.0 mm oral
endotracheal tube equipped with a high residual volume low pressure cuff. When sealing the
cuff to achieve a minimal occluding volume, you note a cuff pressure of 45 cm H2O. What is
the most likely problem?
A) the cuff pilot balloon and line is obstructed
B) the pressure manometer is out of calibration
C) the tube chosen is too small for the patient
D) the tube is in the right mainstem bronchus
Ans: C
Response:
Overinflation of a high volume, low pressure cuff changes its performance to a high pressure
cuff. This problem is common if the tube chosen is too small for that patient's trachea.

Page 7
SAN PEDRO COLLEGE COMPETENCY APPRAISAL - OXYGENATION

28. When checking a nondisposable steel laryngoscope blade prior to intubation, you note that the
bulb does not light when connected to the handle. Your first step should be to
A) recheck the handle/blade connection
B) replace the blade then
C) replace the batteries then
D) check/replace the bulbs
Ans: A
Response:
If the bulb does not light when checking a laryngoscope blade prior to intubation, the first
thing you should do is to recheck the handle/blade connection. If this does not work then
replace the blade, check/replace the bulb, or replace the batteries.

29. When using a disposable CO2 indicator confirm ET tube placement in airway, a false positive
finding (misleading color change) can occur
A) with esophageal intubation
B) during cardiac arrest
C) with mainstem bronchial intubation
D) during BVM ventilation
Ans: C
Response:
With disposable CO2 indicators, a failure to change color can occur even with proper tube
position during cardiac arrest (false negative). On the other hand, color change can occur with
improper tube placement if the ET tube is in the mainstem bronchus (a false positive).

30. Which of the following is false regarding the use of a tracheostomy tube obturator?
A) it should be disposed of after tube insertion
B) its blunt tip helps prevent tissue trauma during insertion
C) it is placed in the trach tube prior to insertion
D) it should be removed immediately after tube insertion
Ans: A
Response:
A tracheostomy tube obturator is placed inside a tracheostomy tube before insertion. Its
rounded, blunt end extends beyond the tube tip and thus helps prevent tissue trauma
("snowplowing") during insertion. You remove the obturator immediately after tube insertion,
but need to keep it at the bedside in case the trach tube comes out and needs to be reinserted.

31. You should provide tracheostomy care


A) whenever measuring trach tube cuff pressure
B) whenever changing the ventilator circuit
C) whenever changing the heat and moisture exchanger
D) whenever the stoma dressing becomes soiled
Ans: D
Response:
Optimum care of patient with tracheal airways involves provision of adequate humidification,
suctioning as needed, and regular cuff management. In addition, if the patients has a
tracheostomy, you also need to provide good tracheostomy care. In general, you should
provide trach care whenever the stoma dressing becomes soiled.

Page 8
SAN PEDRO COLLEGE COMPETENCY APPRAISAL - OXYGENATION

32. The most important safety consideration in providing tracheostomy care is to:
A) always use hydrogen peroxide for cleaning
B) always change the tube ties/holder
C) always make sure the tube is secure
D) always keep the inner cannula in place
Ans: C
Response:
Assuring that the tracheostomy tube is always secure is the mot important safety consideration
during provision of trach care. You do this by: (1) carefully remove the old dressing, making
sure the tube stays in tube in place; (2) using a second person to hold the tube in place when
changing the tube ties or holders; (3) never tying tube in place with a bow (always use a
square knot.instead); and (4) not leaving the bedside until your are sure that the tube is
secured in proper position.

33. Which of the following are indications for changing a tracheostomy tube?

I. the need for different size tube


II. a blown/damaged tube cuff
III. a soiled stoma dressing
A) I and II only
B) I and III only
C) II only
D) I, II and III
Ans: A
Response:
A tracheostomy tube change is indicated if the cuff is damaged and cannot be inflated or if the
physician wants to switch to a different size or different type tube, e.g., a fenestrated or
"talking" tube.

34. In order to allow a patient with a tracheostomy button to talk, eat, and cough normally, you
need to
A) use button spacers to assure appropriate position
B) insert a plug into the button's outer opening
C) attach a standard 15-mm connector to the button
D) attach a one-way valve to the button's outer opening
Ans: D
Response:
Tracheostomy buttons are small tubes used to maintain an open stoma after the tracheostomy
tube is removed. In order to allow a patient with a trach button to talk, eat, and cough
normally, you need to attach a one-way inspiratory valve to the button's outer opening.
Spacers are used to make sure the cannula is at the appropriate depth and does not obstruct the
airway. Plugging a trach button forces a patient to breathe and cough normally via the upper
airway, and a 15-mm connector can be used to deliver positive pressure ventilation if needed.

Page 9
SAN PEDRO COLLEGE COMPETENCY APPRAISAL - OXYGENATION

35. A doctor wants to see if a spontaneously breathing neuromuscular patient with a standard
tracheostomy tube can clear secretions and maintain upper airway function without the tube in
place. What should you recommend?
A) replace the tracheostomy tube with a trach button
B) extubate the patient and seal off the stoma
C) consider intubation with an oral ET tube
D) replace the cuffed tube with an uncuffed one
Ans: A
Response:
Tracheostomy buttons are small tubes used to maintain an open stoma after the tracheostomy
tube is removed. The patient can eat, breathe, and cough normally, but the stoma is available
in case the airway quickly needs to be reestablished, i.e., to bypass laryngeal or upper airway
obstruction or for suctioning or ventilatory support. An alternative approach in this scenario
would be replacing the standard tracheostomy tube with a fenestrated one.

36. The best way to assure the patency of a trach button is to


A) pass a suction catheter through the tube
B) insert a plug into the button's opening
C) attach a 15-mm connector to the button
D) attach a one-way valve to the button's opening
Ans: A
Response:
To prevent obstruction and ensure patency of a tracheostomy button, you or the patient should
regularly pass a suction catheter through the tube.

37. After removing a patient with an oral endotracheal tube in acute respiratory distress from a
ventilator, you find it difficult to provide manual ventilation via bagging. Your next step
should be to:
A) deflate the ET tube cuff
B) remove the bite block
C) try to pass a suction catheter
D) extubate the patient
Ans: C
Response:
In an obstructed airway scenario like this, if you cannot provide adequate ventilation via a
bag-valve system, the first thing you should do is try to pass a suction catheter. This may
relieve mucous plugging, if that is the problem causing obstruction.

38. When properly positioned and with the cuff inflated, the mask of a laryngeal mask airway
(LMA) seals off the:
A) esophagus
B) laryngeal inlet
C) soft palate
D) nasopharynx
Ans: B
Response:
A laryngeal mask airway (LMA) consists of a tube and mask with an inflatable cuff that is
blindly inserted into the pharynx. When properly positioned and with the cuff inflated, the
mask seals off the laryngeal inlet. This effectively bypasses the esophagus and provides a
direct route for bag-valve ventilation via the standard 15 mm connector.

Page 10
SAN PEDRO COLLEGE COMPETENCY APPRAISAL - OXYGENATION

39. An unconscious apneic patient know to have a full stomach cannot be orally intubated in the
emergency room. You would recommend
A) performing a percutaneous tracheotomy
B) intubating via the nasal route instead
C) inserting an esophageal-tracheal Combitube®
D) inserting a laryngeal mask airway (LMA)
Ans: C
Response:
When tracheal intubation cannot be performed and the risk of aspiration is known to be high
(as in unconscious patients with full stomachs), the esophageal-tracheal Combitube® is a
good choice for establishing an emergency airway.

40. You should avoid using a laryngeal mask airway (LMA) to establish an emergency airway in
patients who:

I. have intact gag reflexes


II. need tracheal suctioning
III. are conscious
A) I and II only
B) I and III only
C) II only
D) I, II and III
Ans: D
Response:
You should avoid using the LMA to establish an emergency airway in patients who are
conscious, have intact gag reflexes or who resist insertion. You also should avoid using an
LMA in patients who will need tracheal suctioning. The LMA should not be inserted in
patients with trauma to or obstructive lesions of the mouth or pharynx. Last, the LMA is a
poor choice in patients with decreased pulmonary or thoracic compliance who need positive
pressure ventilation, since the high peak pressures required to ventilate these patients will
breach the mask seal around the larynx.

41. A small adult (60 kg) requires intubation with a laryngeal mask airway (LMA). What size
LMA would you select for this patient?
A) 3
B) 4
C) 5
D) 6
Ans: B
Response:
Proper size selection is critical to effective use of the LMA, as well as maximum cuff inflation
volumes. The recommended size LMA for a 50-70 kg adult is a #4, with a maximum cuff
inflation volume of 30 mL. Often lesser volumes are sufficient to obtain a seal and/or achieve
60 cm H2O cuff pressure.

Page 11
SAN PEDRO COLLEGE COMPETENCY APPRAISAL - OXYGENATION

42. A 35 lb (16 kg) toddler requires intubation with a laryngeal mask airway (LMA). What size
LMA would you select for this patient?
A) 1
B) 1-1/2
C) 2
D) 2-1/2
Ans: C
Response:
Proper size selection is critical to effective use of the LMA, as well as maximum cuff inflation
volumes. The recommended size LMA for a 10-20 kg infant/children is a #2, with a maximum
cuff inflation volume of 10 mL. Often lesser volumes are sufficient to obtain a seal and/or
achieve 60 cm H2O cuff pressure.

43. A large adult (110 kg) requires intubation with a laryngeal mask airway (LMA). What size
LMA would you select for this patient?
A) 3
B) 4
C) 5
D) 6
Ans: D
Response:
Proper size selection is critical to effective use of the LMA, as well as maximum cuff inflation
volumes. The recommended size LMA for a large adult (> 100 kg) is a #6, with a maximum
cuff inflation volume of 50 mL. Often lesser volumes are sufficient to obtain a seal and/or
achieve 60 cm H2O cuff pressure.

44. An 80 kg adult has just been intubated with a #5 laryngeal mask airway (LMA). What is the
maximum cuff inflation volume you would use on this patient?
A) 10 mL
B) 20 mL
C) 30 mL
D) 40 mL
Ans: D
Response:
The proper us of LMAs requires adherence to maximum cuff inflation volumes. The
recommended maximum cuff inflation volume for a #5 LMA is 40 mL. Often lesser volumes
are sufficient to obtain a seal and/or achieve 60 cm H2O cuff pressure.

45. Immediately after insertion of a #4 laryngeal mask airway (LMA) in a 70 kg adult, you should
inflate the cuff to:
A) 60 cm H2O pressure
B) 10 mL volume
C) 30 cm H2O pressure
D) 60 mL volume
Ans: A
Response:
In general, regardless of the size of the LMA, you should inflate the cuff to 60 cm H2O
pressure, while at the dame time keeping the maximum inflation volume within that specified
by the manufacturer. For a #4 LMA, the maximum recommended cuff inflation volume is 30
mL.

Page 12
SAN PEDRO COLLEGE COMPETENCY APPRAISAL - OXYGENATION

46. All of the following indicate proper positioning of a laryngeal mask airway (LMA) except:
A) presence of smooth oval swelling in the neck area
B) observation of chest expansion during inspiration
C) clear visibility of the LMA cuff in the oral cavity
D) auscultation of good bilateral breath sounds
Ans: C
Response:
Primary indicators of correct LMA placement include observing chest expansion during
inspiration and auscultating good bilateral breath sounds. Additional indicators of proper
positioning include (a) not being able to see any portion of the cuff in the oral cavity, and (b)
observing a smooth oval swelling in the neck area around the thyroid cartilage. Visibility of
the LMA cuff in the oral cavity indicates that the tube is not inserted deeply enough.

47. To avoid leaks while manually bagging a patient with a laryngeal mask airway (LMA) in
place, you should do all of the following except:
A) keep pressures < 20-30 cm H2O
B) inflate the lungs slowly
C) keep volumes < 7-8 mL/kg
D) add more air to the cuff
Ans: D
Response:
To avoids leaks during positive pressure ventilation (PPV) with a laryngeal mask airway, you
should use slow inflation, keep peak inspiratory pressures less than 20-30 cm H2O, and limit
delivered tidal volume to no more than 8 mL/kg. Leaks during PPV most likely are due to
upward displacement of the mask. If a leak occurs: (1) ensure that the airway is securely taped
in place, (2) readjust the position of the airway by pressing the tube downward, and (3)
resecure the airway in its new position. Do not simply add more air to the cuff (this can
worsen the leak by pushing the cuff away from the larynx).

48. Which of the following actions are appropriate if a patient regurgitates after insertion of a
laryngeal mask airway (LMA)?

I. place the patient in side-lying position


II. immediately remove the airway
III. suction through the airway tube
A) I and II only
B) I and III only
C) II only
D) III only
Ans: B
Response:
If regurgitation occurs during use of an LMA, do not remove the LMA. Instead, place the
patient in a head down or side-lying position and temporarily disconnect all ventilation
equipment so that gastric contents are not forced into the lungs. Then reposition the device to
ensure the distal end is lying against the upper esophageal sphincter and secure it in place.
After your are sure the LMA is properly positioned, suction through its airway tube and
prepare for immediate tracheal intubation.

Page 13
SAN PEDRO COLLEGE COMPETENCY APPRAISAL - OXYGENATION

49. You should consider removing a laryngeal mask airway (LMA) only after the patient
A) exhibits good upper airway reflexes
B) is heavily sedated with a narcotic-analgesic
C) becomes stuporous or unconscious
D) develops an agonal breathing pattern
Ans: A
Response:
You should consider removing an LMA only after the patient's upper airway reflexes have
returned. Prior to removing an LMA, gather the equipment needed for suctioning and tracheal
intubation. Try to avoid suctioning via the airway tube since this can provoke laryngospasm.
Deflate the cuff, remove the device, and verify airway patency and unobstructed ventilation.

50. The esophageal-tracheal Combitube® is contraindicated in all of the following situations


except:
A) insertion in infants and small children
B) insertion in patients with esophageal trauma
C) insertion when you cannot visualize the vocal cords
D) insertion in patients with intact gag reflexes
Ans: C
Response:
The esophageal-tracheal Combitube® (ETC) it is a good choice in patients who are difficult to
intubate due to trauma, bleeding, vomiting, or other factors that make visualization of the
vocal cords impossible. The ETC should not be inserted in conscious patients or those with
intact gag reflexes. It is contraindicated in infants and small children and in patients with
esophageal trauma or disease.

51. After insertion of an esophageal-tracheal Combitube® (ETC) in an adult patient in the


Emergency Room, you cannot ventilate through either connector. In order to provide effective
ventilation to this patient, you should:
A) deflate the #1 pharyngeal cuff and ventilate through connector #2
B) withdraw the tube 2-3 cm at a time while ventilating through connector #1
C) remove the ETC and re-establish the airway by any alternative means
D) deflate the #2 tracheal/esophageal cuff and ventilate through the connector #1
Ans: B
Response:
If you cannot ventilate through either ETC connectors, the tube likely is inserted too far
(causing the large proximal cuff to obstruct the glottis). To correct this problem, withdraw the
tube 2-3 cm at a time while ventilating through connector #1 until breath sounds are heard
over the lungs. Only If this procedure fails should you remove the ETC and re-establish the
airway by an alternative means.

52. What approximate volume should be used to inflate the larger #1 cuff of an esophageal-
tracheal Combitube® (ETC)?
A) 12 - 15 mL
B) 20 - 30 mL
C) 40 - 60 mL
D) 80 - 100 mL
Ans: D
Response:
The ETC comes in two sizes: 37 and 41 French. Cuff inflation volumes for the 41 Fr: ETC
are: large cuff (#1) - 100 mL; small cuff (#2) - 15 mL. For the 37 Fr ETC, you fill the large
cuff with 85 mL and the small one with 12 mL.

Page 14
SAN PEDRO COLLEGE COMPETENCY APPRAISAL - OXYGENATION

53. What head-neck position is used for insertion of an esophageal-tracheal Combitube® (ETC)?
A) neck flexed
B) neutral position
C) neck extended
D) sniffing position
Ans: B
Response:
For insertion of an ETC you should place the head in a neutral position and pull the patient's
mandible and tongue forward between the thumb and fingers. You advance the tube until the
upper teeth or gums are aligned between the two black ringed insertion markers on the tube.

54. Which of the following is false regarding switching from an esophageal-tracheal Combitube®
(ETC) to an oral endotracheal tube?
A) the equipment needed is the same as for endotracheal intubation
B) the patient's stomach contents should be aspirate through the #2 tube
C) the body of the tube normally must be positioned in the trachea
D) the large #1 pharyngeal cuff must be deflated before laryngoscopy
Ans: C
Response:
To switch a patient from an ETC to an oral endotracheal tube, the airway normally must be in
the esophagus. If so, gather and prepare all equipment needed for endotracheal intubation, and
aspirate any stomach contents through the #2 tracheal/esophageal tube. To intubate while the
ETC in place, you must first deflate the large #1 pharyngeal cuff. This will allow for
laryngoscopy while the #2 ETC cuff keeps the esophagus occluded.

55. Which of the following humidification devices would be appropriate for a patient receiving
nasal oxygen therapy at 6 L/min?
A) unheated bubble humidifier
B) heated wick-type humidifier
C) heat and moisture exchanger (HME)
D) large volume jet nebulizer
Ans: A
Response:
For patients with intact upper airways with normal secretions receiving O2 at flows greater
than 4 L/min, a simple unheated bubble humidifier is all that is needed to provide adequate
humidification.

56. Which of the following airway appliances would be appropriate for short-term application of
bland aerosol therapy to a patient after extubation?
A) face tent
B) trach mask
C) T-tube
D) aerosol mask
Ans: D
Response:
Airway appliances used to deliver bland aerosol therapy include the aerosol mask, face tent,
T-tube and tracheostomy mask. The aerosol mask is the first choice for short-term application
of bland aerosol therapy to most patients with intact upper airways.

Page 15
SAN PEDRO COLLEGE COMPETENCY APPRAISAL - OXYGENATION

57. You should select an active water heated humidifier during mechanical ventilation instead of a
heat and moisture exchanger (HME) under all of the following circumstances except:
A) patient has thick secretions
B) patient receiving large tidal volumes
C) patient was intubated via the nasal route
D) patient-ventilator system has large leaks
Ans: C
Response:
You should start mechanically ventilated patients on a heated humidifier whenever one or
more contraindications exist against using an HME. HME contraindications include thick or
bloody secretions, hypothermia (< 32° C), large tidal volumes (> 1000 mL) and large system
leaks.

58. Removal of a tracheal tube should only be considered in patients who:

I. can maintain adequate oxygenation and ventilation without ventilatory support


II. are at minimal risk for upper airway obstruction
III. have adequate airway protection and are at minimal risk for aspiration
IV. can adequately clear pulmonary secretions on their own
A) I and II only
B) II and III only
C) I, III and IV only
D) I, II, III and IV
Ans: D
Response:
Removal of a tracheal tube should only be considered in patients who (1) can maintain
adequate oxygenation and ventilation while spontaneously breathing; (2) are at minimal risk
for upper airway obstruction; (3) have intact upper airway reflexes and are at minimal risk for
aspiration; and (4) can adequately clear secretions on their own.

59. All of the following indicate that a patient with an oral endotracheal tube is ready extubation
except:
A) a positive gag reflex
B) a negative cuff leak test
C) a successful spontaneous breathing trial
D) a deep cough on suctioning
Ans: B
Response:
In considering patients for extubation, they should first be able to maintain adequate
oxygenation and ventilation during a spontaneous breathing trial. To assess the risk for upper
airway obstruction, perform a cuff leak test (deflate the cuff and occlude the tube at its outlet).
If leakage occurs (a "positive" test), then the airway most likely is patent. A positive gag
reflex and the ability of the patient to raise the head off the bed indicate adequate airway
protection. Last, the ability to clear secretions is evident if the patient is alert, coughs deeply
on suctioning and can generate a maximum expiratory pressure (MEP) > 60 cm H2O.

Page 16
SAN PEDRO COLLEGE COMPETENCY APPRAISAL - OXYGENATION

60. To minimize the risk of aspiration of glottic secretions or cord damage during removal of an
oral endotracheal tube, you should:
A) have the patient cough while you quickly pull the tube
B) provide 100% oxygen for 1-2 minute before extubation
C) keep the tube cuff pressure below 25-30 cm H2O
D) fully occlude the ET tube while you quickly it out
Ans: A
Response:
Having the patient cough while you quickly pull out an endotracheal tube has two benefits: (1)
it minimizes the risk of aspiration of glottic secretions; and (2) it helps prevent damage to the
vocal cords (abducted during a cough).

Page 17
CHAPTER 11
Name: __________________________ Date: _____________

1. An adult male requires postural drainage of the posterior basal segments bilaterally. To
properly position this patient, you should:

I. elevate the foot of the bed 30 degrees


II. keep the bed flat but put a pillow under the patient’s hips
III. have the patient lie supine with a pillow under the hips
IV. have the patient lie prone with a pillow under the hips
A) I and II only
B) III and IV only
C) I and IV only
D) II and III only
Ans: C
Response:
In order to drain the posterior basal segments, a patient should be positioned in the prone
Trendelenburg position (30° down tilt) with a pillow under the hips.

2. By placing a patient in the prone position with a pillow under the abdomen and keeping the
bed level, you would be targeting what segment in postural drainage?
A) lateral basal segment of lower lobes
B) superior segment of lower lobes
C) apical segment of upper lobes
D) posterior segment of upper lobes
Ans: B
Response:
The superior segments of the lower lobes are drained in a flat prone position with a pillow
under the abdomen.

3. All of the following are needed for an effective cough except:


A) a closed glottis
B) compression phase
C) explosive exhalation
D) low inspiratory volumes
Ans: D
Response:
The effectiveness of a cough requires proper integration of three phases: inspiratory
component, compression phase, and finally the explosive exhalation. Energy is transferred to
the mucous and it is expelled from the airway. Strong abdominal muscles to generate large
volumes with glottis closure will generate high intra-alveolar pressures and a good cough.

Page 1
SAN PEDRO COLLEGE COMPETENCY APPRAISAL - OXYGENATION

4. Which of the following should you do in order to properly perform nasotracheal suctioning on
an adult patient?

I. lubricate the catheter


II. apply suction for less than 15 seconds
III. preoxygenate and postoxygenate the patient
IV. instruct patient to exhale and hold breath
A) I and IV only
B) I, II, and III only
C) III and IV only
D) I, III, and IV
Ans: B
Response:
When applying nasotracheal suctioning, the patient should be pre- and post-oxygenated, and
the suction catheter tip should be prelubricated. As with tracheal suctioning in general, the
time for applying suction should not exceed 15 seconds.

5. The administration of what aerosolized drug is most appropriate to thin secretions and help in
the removal of a mucous plug?
A) albuterol
B) ipratropium bromide
C) acetylcysteine (Mucomyst)
D) racemic epinephrine
Ans: C
Response:
Acetylcysteine (Mucomyst) breaks the disulfide bonds in mucous, thereby helping thin and
mobilize secretions in patients with mucous plugs.

6. A morning X-ray confirms bilateral infiltrates in the posterior segments of the upper lobes of a
30-year-old female patient. What postural drainage position should you use to drain these
segments?
A) prone with a pillow under her hips
B) sitting position leaning forward on a pillow
C) the foot of the bed raised 18 inches and the patient on her left side
D) supine with a pillow under her legs
Ans: B
Response:
To drain the posterior segments of the upper lobes, the proper patient position should be
sitting on a chair leaning forward on a pillow.

7. Which of the following bronchial hygiene techniques is most suitable for small infants?
A) postural drainage, percussion, and vibration
B) positive expiratory pressure
C) aggressive suctioning with a 14-Fr catheter
D) high-frequency oscillation
Ans: A
Response:
Of the choices available, the bronchial hygiene technique that is most suitable for small
infants is postural drainage, percussion, and vibration. Positive expiratory pressure and high-
frequency oscillation are generally not used on infants, and aggressive suctioning, especially
with a large catheter, would be potentially harmful.

Page 2
SAN PEDRO COLLEGE COMPETENCY APPRAISAL - OXYGENATION

8. During your morning patient assessments you suction a 7-month-old nasally intubated patient.
After assembling the needed equipment and following all precautions you begin the
procedure. For this patient, what are the appropriate pressure and time limits for this
procedure?
A) suction at a pressure range of –40 to –60 mm Hg, limiting the time to 30 seconds
B) suction at a pressure range of –80 to –100 mm Hg, limiting the time to less than 10–15
seconds
C) Suction at a pressure range of –100 to –120 mm Hg and continuing until you observe
secretions in the suction catheter
D) Suction at a pressure range of –60 to –80 mm Hg, limiting the time to less than 10–15
seconds
Ans: D
Response:
The normal pressure range for adults is –100 to –120 mm Hg, for children –80 to –100 mm
Hg, and for infants –60 to –80 mm Hg. The patient should be preoxygenated with 100%
oxygen for at least a minute, and the total suction time should be limited to no more than 10–
15 seconds on each attempt.

9. Which of the following is a possible complication of postural drainage, percussion, and


vibration?

I. pulmonary barotrauma
II. acute hypotension during procedure
III. dysrhythmias
IV. fractured ribs
A) I and III only
B) I, II, and III only
C) I, II, III, and IV
D) II, III, and IV only
Ans: D
Response:
Major hazards and complications of postural drainage, percussion, and vibration include
hypoxemia; acute hypotension during the procedure; pulmonary hemorrhage; pain or injury to
muscles, ribs, or spine; vomiting and aspiration; bronchospasm; dysrhythmias.

10. To remove accumulations of subglottic secretions from above the cuff of intubated patients,
you should recommend which of the following?
A) intrapulmonary percussive ventilation (IPV)
B) use of a tracheal tube with a suction port above the cuff
C) aggressive tracheal suctioning with saline lavage
D) frequent oropharyngeal suctioning with a Yankauer tip
Ans: B
Response:
A common problem in intubated patients is leakage of subglottic secretions past the tracheal
tube cuff. These secretions can contaminate the lower respiratory tract and are thought to be a
major contributing factor in the development of ventilator-associated pneumonia (VAP).

Page 3
SAN PEDRO COLLEGE COMPETENCY APPRAISAL - OXYGENATION

11. The maximum time that a therapist should suction an intubated patient in the intensive care
unit is:
A) 1 - 2 sec
B) 15 - 20 sec
C) 20 - 30 sec
D) 10 - 15 sec
Ans: D
Response:
While withdrawing the suction catheter the therapist should apply suction for a maximum of
10 to 15 seconds. The patient should be monitored for any changes. Suction should be stopped
with any sudden changes in the in the vital signs being monitored . Within 5 seconds of
beginning suctioning oxygenation can begin to decrease.

12. A 30 year-old male was found supine and unresponsive. In the ER it was confirmed he had
aspirated while on his back. After moving the patient to the ICU his physician wrote for chest
percussion with drainage every 4 hours because he had developed pneumonia. What is the
best position to place him in to drain the affected area?
A) prone with a pillow under his hips
B) prone with feet elevated 30 degrees
C) supine with a pillow under his hips
D) lying on his left side with feet elevated 15 inches above midline
Ans: A
Response:
This patient aspirated while laying flat on his back. Most commonly, this affects the superior
segments of both lower lobes. The position which facilitates drainage from this lung region is
a prone position with a pillow under the patient's hips.

13. A doctor orders a sputum induction to help in the diagnosis of tuberculosis in a patient. Which
of the following agents would you select for nebulization?
A) sterile water
B) albuterol
C) hypertonic saline solution
D) ipratropium bromide
Ans: C
Response:
There are a variety of solutions which may be nebulized for sputum induction. Hypertonic
saline will tend to draw fluid from the tissue into the mucus, thus thinning secretions and
aiding in its removal.

14. When acetylcysteine (Mucomyst) is appropriately ordered for a patient in the ICU, it can be
helpful in treating which of the following conditions?
A) mucous plugs
B) bronchospasm
C) pulmonary hypertension
D) sputum induction
Ans: A
Response:
Acetylcysteine breaks the disulfide bonds of mucus which thins and helps mobilize secretions
in patients with mucus plugs.

Page 4
SAN PEDRO COLLEGE COMPETENCY APPRAISAL - OXYGENATION

15. Morning X-ray rounds show a 25 year-old male post trauma patient to have infiltrates in the
anterior segment of the upper lobe. The physician's orders are to drain these segments. In what
postural drainage position should the patient be placed?
A) lying flat and supine with a pillow under his legs
B) lying in a supine Trendelenburg position with a pillow under his hips
C) lying on his side and turned 45 degrees onto his face
D) lying in a prone Trendelenburg position with a pillow under his hips
Ans: A
Response:
To facilitate the drainage of secretions from the anterior segment of the upper lobe, the patient
should be placed in a supine (flat) position with a pillow under his legs.

16. Which of the following cough methods would be best for helping a COPD patient clear
secretions?
A) cough with standard compression phase
B) the forced expiratory technique/huff coughing
C) abdominal thrust synchronize to explosive phase
D) autogenic drainage plus standard coughing
Ans: B
Response:
The forced expiratory technique or "huff coughing" is best suited for post-op patients for
whom explosive exhalation is very painful, and COPD patients prone to airway closure on
forced exhalation. It consists of 2-3 forced exhalations or huffs with the glottis open, followed
by a rest period. This process is repeated until the secretions have moved up into the pharynx,
where they can be cleared by the patient via expectoration.

17. The techniques recommend for airway clearance for a patient with a neurologic abnormality
(bulbar palsy) and an intact upper airway includes;

I. postural drainage, percussion, and vibration


II. positive expiratory pressure (PEP) therapy
III. mechanical insufflation-exsufflation
A) II and III only
B) I and III only
C) I and II only
D) I, II and III
Ans: B
Response:
Patients with neurologic disorders can have trouble mobilizing and clearing secretions. The
recommended airway clearance techniques for a patient with a neurologic abnormality (bulbar
palsy) and intact upper airway include postural drainage, percussion, and vibration to help
mobilize the secretion. The clearance of secretions with an ineffective cough can be
supplemented with mechanical insufflation-exsufflation and, if needed, nasotracheal
suctioning.

Page 5
SAN PEDRO COLLEGE COMPETENCY APPRAISAL - OXYGENATION

18. What is the proper vacuum pressure level for suctioning an adult patient with retained
secretions?
A) Vacuum pressure range of -40 to -60 mm Hg
B) Vacuum pressure range of -80 to -100 mm Hg
C) Vacuum pressure range of -100 to -120 mm Hg
D) Vacuum pressure range of -60 to -80 mm Hg
Ans: C
Response:
The normal pressure range for adults is -100 to -120 mm Hg , children - 80 to -100 mm Hg
and infants -60 to -80 mm Hg. Beforehand, the patient should be preoxygenated with 100%
for least 30 seconds.

19. All of the following are complications of postural drainage, percussion and vibration except
A) pulmonary hemorrhage
B) hypoxia
C) pulmonary barotrauma
D) fractured ribs
Ans: C
Response:
Major complications of postural drainage, percussion and vibration include pulmonary
hemorrhage and rib fractures. Lesser adverse effects include increased intracranial pressure,
hypoxemia, decreased cardiac output, and increased airway resistance.

20. A 20-year-old lethargic neuromuscular patient needs nasotracheal suctioning to remove


secretions retained as a result of an ineffective cough. A safe and effective procedure would
require which of the following;

I. limit suction time to 20 -30 seconds


II. lubricate the catheter before insertion
III. hyperoxygenate prior to suctioning
IV. use a ridged catheter
V. insert a nasopharyngeal airway.
A) I, II and IV only
B) I, III, IV and V
C) II, III and V only
D) I, II, III and V only
Ans: C
Response:
A nasopharyngeal airway is used to minimize mucosal trauma with nasotracheal suctioning.
Additionally when using the nasal route, it is recommended to apply a water-soluble lubricant
to the catheter tip to ease insertion. The patient should be hyper oxygenated before suctioning
and the suction time should be limited to not more than 10-15 seconds regardless of the route.

21. In instructing a patient how to breathe during a small volume nebulizer drug treatment, you
coach the patient to hold his breath at the end of each inspiration. The purpose of this
maneuver is to improve:
A) drug delivery
B) particle stability
C) aerosol penetration
D) inertial impaction
Ans: A
Response:
The ideal pattern for breathing during small volume nebulizer drug therapy is slow, deep
breathing with an end-inspiratory breath-hold. All three of these elements increase drug
delivery. Only the deep breathing component increases aerosol penetration.

Page 6
SAN PEDRO COLLEGE COMPETENCY APPRAISAL - OXYGENATION

22. Which of the following would facilitate clearance of pulmonary secretions in a patient with
cystic fibrosis?

I. Mucomyst
II. Flutter valve
III. Atropine
IV. DNase
A) I and III only
B) II and IV only
C) I, II, and IV only
D) II, III, and IV only
Ans: C
Response:
To help break-up and thin the thick secretions common in patients with cystic fibrosis, good
systemic hydration is essential. In addition, Mucomyst (acetylcysteine) and DNase
(deoxyribonuclease) are often prescribed for this purpose. To help mobilize these secretions
and make them easier to cough up, a bronchial hygiene method like the flutter valve is also
indicated. Atropine is contraindicated in these patients because as a cholinergic agent, it
decreases the watery output of the bronchial glands, making the secretions drier and harder to
remove.

23. A physician has ordered albuterol (Proventil) and deoxyribonuclease (DNase) by aerosol for a
cystic fibrosis patient who also receives postural drainage TID. You should administer these
therapies in which of the following sequences?
A) deoxyribonuclease (DNase), postural drainage, albuterol (Proventil)
B) albuterol (Proventil), deoxyribonuclease (DNase), postural drainage
C) postural drainage, albuterol (Proventil), deoxyribonuclease (DNase)
D) deoxyribonuclease (DNase), albuterol (Proventil), postural drainage
Ans: B
Response:
When a doctor orders a combination of therapies aimed at either getting drugs (steroids,
antibiotics) in or getting secretions out of the airway, you should generally perform them in
the follow order: (1) therapies to open the airways; (2) therapies to thin or break down the
secretions, and (3) therapies to help mobilize and remove the secretions. In this case that
would mean given the bronchodilator first (Proventil), followed by the mucolytic agent
(DNase), followed by the clearance procedure (postural drainage).

24. When instructing a patient in coughing techniques, the respiratory therapist should
recommend which of the following
A) "Lie flat, inhale quickly, hold your breath, and try to clear your throat"
B) "Sit upright, inhale fully, hold your breath, and then exhale forcefully"
C) "Sit in a semi-Fowler's position, inhale rapidly, and huff several times"
D) "Sit upright with your feet on the floor and huff several times"
Ans: B
Response:
When instructing patient on coughing techniques, they should be instructed to sit upright,
inhale fully, hold their breath, and then exhale forcefully.

Page 7
SAN PEDRO COLLEGE COMPETENCY APPRAISAL - OXYGENATION

25. What size catheter would you select to suction a child with an 5 mm endotracheal tube?
A) 5 Fr
B) 8 Fr
C) 10 Fr
D) 12 Fr
Ans: B
Response:
To suction a child with an 5 mm endotracheal tube, you would select an 8 Fr catheter. Using
the 'next smallest size rule of thumb, 2 x 5 = 10, next smallest catheter size = 10 Fr.

26. A patient about to receive postural drainage and percussion is attached to an ECG monitor,
and is receiving both IV solutions and oxygen (via nasal cannula). Which of the following
actions would be appropriate for this patient?
A) cancel the therapy because the patient cannot be repositioned
B) turn off the ECG monitor, but keep the IV and oxygen going
C) turn off the IV, but keep the monitor on and the oxygen going
D) inspect/adjust the equipment to ensure function during therapy
Ans: D
Response:
When performing postural drainage, any monitoring leads, IV tubing, and therapy equipment
connected to the patient should be inspected and adjusted to ensure continued function during
therapy.

27. If tolerated, a specified postural drainage position should be maintained for at least:
A) 3-5 minutes
B) 5-10 minutes
C) 10-20 minutes
D) 20-30 minutes
Ans: B
Response:
If tolerated, a specified drainage position should be maintained for at least 5-10 minutes, and
longer if good sputum production results. During therapy, the therapist should observe the
patient for signs of ill effects, and monitor the vital signs as needed. In general, total treatment
time should not exceed 30-40 minutes.

28. Strenuous patient coughing during postural drainage in head-down positions is contraindicated
because it can:
A) markedly increase intracranial pressure
B) impair the mucociliary clearance mechanism
C) increase expiratory airway resistance
D) cause air-trapping and pulmonary distention
Ans: A
Response:
Practitioners should ensure appropriate coughing technique during postural drainage, both
before and after positioning. When using a head-down position, strenuous coughing should be
avoided, since this can markedly raise intracranial pressure. Rather, the patient should use the
forced expiration technique.

Page 8
SAN PEDRO COLLEGE COMPETENCY APPRAISAL - OXYGENATION

29. A patient exhibiting coarse rhonchi and an ineffective cough could best be treated with
A) nasotracheal suction
B) oxygen therapy
C) incentive spirometry
D) bronchoscopy
Ans: A
Response:
The most common approach to removing secretions in a patient with an ineffective cough is
via nasotracheal suctioning.

30. Properly performed chest vibration is applied:


A) throughout inspiration
B) throughout expiration
C) during breath holding
D) at the end of expiration
Ans: B
Response:
When indicated, chest vibration is often used in conjunction with percussion, but limited to
application during the expiratory phase of breathing. After the patient take a deep breath, the
therapist exerts slight to moderate pressure on the chest wall, and initiates rapid vibrations wit
the hands throughout expiration.

31. During routine tracheostomy care, you palpate coarse vibrations on the anterior chest wall of a
patient on both inspiration and expiration. Which of the following actions would be most
appropriate at this time?
A) perform postural drainage and percussion
B) deflate the tracheotomy tube cuff
C) perform endotracheal suctioning
D) administer a bronchodilator by MDI
Ans: C
Response:
Palpation of coarse vibrations (tactile fremitus) on the chest wall is one sign of retained
secretions and the need for suctioning.

32. Which of the following clinical signs indicate that a patient is having a problem with retained
secretions?

I. lack of sputum production


II. labored breathing
III. development of a fever
IV. increased crackles/rhonchi
A) II and IV only
B) I, II and III only
C) III and IV only
D) I, II, III and IV
Ans: D
Response:
Indications that a patient may have a problem with retained secretions include 1) an
ineffective cough, 2) absent or increased sputum production, 3) a labored breathing pattern, 4)
decreased breath sounds, 5) crackles or rhonchi, 6) tachypnea, tachycardia, or 7) fever.

Page 9
SAN PEDRO COLLEGE COMPETENCY APPRAISAL - OXYGENATION

33. Patients should be suctioned:


A) at least once every two to three hours
B) whenever they are moved or ambulated
C) when physical findings support the need
D) whenever the charge nurse requests it
Ans: C
Response:
Routine suctioning of a patient should be discouraged. The decision to suction a patient
should be based on current physical assessment findings, including: coarse rhonchi, tactile
fremitus, and an ineffective cough.

Page 10
CHAPTER 12
Name: __________________________ Date: _____________

1. While suctioning a patient receiving ventilatory support, you note the heart rate increases
abruptly from 92 to 145 beats per minute. Which of the following actions could help prevent
or minimize this problem?
A) recommend an IV dose of atropine before suctioning
B) instill lidocaine (Xylocaine) into the trachea before suctioning
C) increase the oxygen concentration immediately before suctioning
D) give the patient two MDI puffs of beclomethasone before suctioning
Ans: C
Response:
Most cardiac arrhythmias during suctioning are due to arterial hypoxemia. The best way to
prevent or minimize arterial hypoxemia during suctioning is to preoxygenate the patient for 1–
2 minutes prior to beginning the procedure. You should also keep suction time below 10–15
seconds.

2. An ARDS patient is receiving ventilatory support with 100% oxygen and PEEP at the
recommended high limit of the protocol (25 cm H2O) but remains dangerously hypoxemic.
Long plateau pressures and inverse ratio ventilation have been tried, but without success.
Which of the following should you recommend to improve this patient's oxygenation?
A) increase the mandatory rate
B) put the patient in the prone position
C) administer a paralytic agent
D) increase the pressure limit
Ans: B
Response:
In patients with a generalized decrease in lung volume (as in ARDS), use of the prone position
can improve oxygenation. The prone position probably shifts blood flow to regions of the lung
that are less severely injured and thus better aerated. With the proper equipment, “proning”
may also allow the abdomen to expand more and facilitates better movement of the
diaphragm. The prone position also may decrease the likelihood of further lung injury
associated with positive pressure ventilation of patients with ARDS.

3. Which of the following is the appropriate load to establish for patients receiving inspiratory
muscle training?
A) at least 33% of the predicted inspiratory capacity (IC)
B) at least 10–15 ml/kg of predicted body weight (PBW)
C) at least 30% of the maximum inspiratory pressure (MIP/NIF)
D) at least –25 cm H2O, as measured by a calibrated manometer
Ans: C
Response:
For inspiratory training to be effective, the load against which the patient breathes must be
sufficient to increase muscle strength. The minimal resistance load to achieve this end is an
inspiratory pressure that is at least 30% of the MIP/NIF.

Page 1
SAN PEDRO COLLEGE COMPETENCY APPRAISAL - OXYGENATION

4. To obtain the most effective ventilation, a patient with severe emphysema should be instructed
to:
A) inhale slowly
B) exhale slowly
C) inhale quickly
D) exhale quickly
Ans: B
Response:
Patients with severe emphysema tend to have highly compliant small airways, which tend to
collapse during forced exhalation. To help prevent airway collapse and air trapping, they
should be instructed to exhale slowly, perhaps through pursed lips.

5. For which of the following patients should you carefully monitor cardiovascular function
during application of intermittent positive pressure breathing (IPPB)?

I. a patient with low blood pressure


II. a patient with poor vasomotor tone
III. a patient with cardiac insufficiency
A) I and II only
B) II and III only
C) I and III only
D) I, II, and III
Ans: D
Response:
IPPB can impede venous return to the heart by increasing intrathoracic pressures. Patients
with poor venomotor tone or those who are already hypotensive due to conditions such as
shock or cardiac insufficiency are particularly prone to this effect. For this reason, you should
assess the patient's cardiovascular status before administering IPPB and monitor high-risk
patients during treatment.

6. An adult patient in respiratory failure has the following ABGs on a simple O2 mask at 8
L/min: pH = 7.19; PaCO2 = 68 torr; HCO3 = 28 mEq/L; PaO2 = 85 torr. The attending
physician orders intubation and ventilatory support. Which of the following modes of support
are appropriate for this patient?

I. CMV at a frequency of 12/min


II. IMV at a frequency of 12/min
III. CPAP with 10 cm H2O pressure
A) I or II only
B) II or III only
C) I or III only
D) I, II, or III
Ans: A
Response:
The mode of ventilatory support initially chosen depends mainly upon the patient's underlying
pathophysiologic problem. When a patient's respiratory failure is associated with hypercapnea
due to inadequate alveolar ventilation—as in this case—either the CMV or IMV modes (with
equivalent rate settings) may be employed.

Page 2
SAN PEDRO COLLEGE COMPETENCY APPRAISAL - OXYGENATION

7. A doctor asks you to decrease the PaCO2 of a patient receiving high-frequency ventilation
oscillation ventilation (HFOV). You should consider all of the following adjustments
EXCEPT:
A) increasing the power/amplitude
B) decreasing the frequency
C) deflating the ET tube cuff
D) decreasing the bias flow
Ans: D
Response:
Increasing the HFOV power/amplitude is usually the first step to increase CO2
elimination/lower the PaCO2. Decreasing the frequency can also lower the PaCO2 (note that
frequency changes during HFOV affect CO2 elimination in a manner opposite to that observed
during conventional mechanical ventilation). If hypercapnia is severe despite use of the
maximum power/amplitude and lowest frequency settings, you can also consider creating a
cuff leak to enhance CO2 removal.

8. A patient with congestive heart failure (CHF) is coughing up large quantities of pink, frothy
sputum. ABG values on simple mask O2 at 7 L/min are as follows:

pH 7.44
PaCO2 29 torr
HCO3 20 mEq/L
BE –3 mEq/L
PaO2 46 torr
SaO2 76%

Which of the following treatments should you recommend?


A) nonrebreathing mask at 12 L/min and postural drainage therapy
B) intermittent positive pressure breathing (IPPB) with compressed air
C) starting intrapulmonary percussive ventilation to clear secretions
D) mask continuous positive airway pressure (CPAP) with 80% O2
Ans: D
Response:
A patient with CHF who is coughing up pink, frothy sputum is likely suffering from acute
cardiogenic pulmonary edema. The ABG indicates a fully compensated respiratory alkalosis
secondary to severe hypoxemia. The goal is to restore adequate oxygenation and maintain
alveolar inflation. A high FIO2 combined with noninvasive positive pressure (mask CPAP or
BiPAP™) are generally indicated in such instances. The positive pressure (1) helps keep
alveoli open, (2) reduces venous return to the right heart, and (3) lowers pulmonary vascular
pressures. This, in turn, decreases fluid movement into the interstitial space and alveoli and
improves oxygenation.

9. A patient during CPR can not have an intravenous line started. The physician wants to
administer nalaxone because the patient is suffering from a morphine overdose. What is an
alternative route of administration?
A) nasogastric (NG) tube
B) feeding tube
C) aerosol
D) endotracheal tube
Ans: D
Response:
Some medications can be delivered via endotracheal tube. Cardiovascular medications,
lidocaine, epinephrine, atropine, and nalaxone can be delivered via endotracheal tube safely.

Page 3
SAN PEDRO COLLEGE COMPETENCY APPRAISAL - OXYGENATION

10. Data for a patient being mechanically ventilated are below.

Ventilator Settings Blood Gases


Mode VCV-A/C pH 7.55
VT 900 mL PaCO2 20 torr
Set rate 10 HCO3 17 mEq/L
Actual rate 20 PaO2 125 torr
FIO2 0.35 SaO2 99%

Based on this information, you should suggest which of the following?


A) add 5 cm H2O PEEP
B) add deadse to the breathing circuit
C) change to SIMV at a rate of 10 breaths/min
D) set the FIO2 to 0.25
Ans: C
Response:
The ABG indicates a partially compensated respiratory alkalosis, most likely the result of the
patient initiating 10 machine breaths above the set rate. By changing to SIMV, the
spontaneously initiated breaths will not be delivered at the set tidal volume, but rather the
patient will receive only what he or she can draw from the circuit on such breaths. As a result,
the patient's minute ventilation will be reduced.

11. During a patient-ventilator check in the ICU, you observe the following settings and
monitored parameters on a 70-kg (X-lb.) patient receiving ventilator support:

Mode SIMV
VT 600 mL
Mandatory rate 10
Total rate 38
PEEP 8 cm H2O
Minute volume 10 L/min

Which of the following actions should you recommend at this time?


A) switch to assist/control mode
B) decrease the mandatory rate
C) add pressure support
D) increase the PEEP level
Ans: C
Response:
The key problem is the patient's rapid spontaneous rate and low spontaneous tidal volume.
The spontaneous tidal volume = [total minute vol – set min vol]/[total rate – set rate] = [10000
– 6000]/[38 – 10] = 4000/28 = 142 mL. To increase the spontaneous VT, you should add
pressure support. This will increase the efficiency of spontaneous ventilation and allow for a
lower spontaneous rate of breathing. In fact, you can judge the effect of pressure support by
the resulting decrease in rate. A good rule of thumb is to provide sufficient pressure support to
ensure a spontaneous rate less than 20/min.

Page 4
SAN PEDRO COLLEGE COMPETENCY APPRAISAL - OXYGENATION

12. Data for a 63-kg (140-lb.) patient receiving ventilatory support with 8 cm PEEP are as
follows:

Ventilator Settings Blood Gases


Mode SIMV pH 7.45
VT 600 mL PaCO2 36 torr
Rate 10 HCO3 25 mEq/L
FIO2 0.70 PaO2 55 torr
PEEP 8 cm H2O SaO2 100%

Which of the following changes should you recommend at this time?


A) lower the VT
B) increase the rate
C) increase PEEP
D) decrease the FIO2
Ans: C
Response:
The goal of PEEP is to achieve adequate oxygenation with a safe FIO2. In this case, the FIO2
is dangerously high, but shunting persists. Based on the 60/60 rule (PaO2 < 60 torr on FIO2 >
0.60), with a PaO2 of 55 torr on 70% O2 you should increase the PEEP level.

13. Which of the following best describes the way a patient should perform incentive spirometry?
A) the patient should exhale maximally and hold it for at least 5 seconds
B) the patient should inhale normally hold it for 5-10 seconds
C) the patient should inhale to inspiratory capacity and hold it for at least 5 seconds
D) the patient should inhale/exhale maximally for 10–15 seconds
Ans: C
Response:
The “sustained maximum inspiration” underlying incentive spirometry is essentially an
inspiratory capacity maneuver (IC), followed by a breath hold (10 seconds is ideal).

14. An ARDS patient is receiving ventilatory support with 100% oxygen and 20 cm H2O PEEP
with a plateau pressure of 30 cm H2O, but remains dangerously hypoxemic. Which of the
following should you recommend to improve this patient's oxygenation?
A) using inverse ratio ventilation (I:E > 1:1)
B) putting the patient in the prone position
C) administering a paralytic agent
D) increasing the pressure limit
Ans: B
Response:
In patients with a generalized decrease in lung volume (as in ARDS), use of the prone position
can improve oxygenation. The prone position probably shifts blood flow to regions of the lung
that are less severely injured and thus better aerated. With the proper equipment, “proning”
may also allow the abdomen to expand more and facilitates better movement of the
diaphragm. The prone position may also decrease the likelihood of further lung injury
associated with positive pressure ventilation of patients with ARDS.

Page 5
SAN PEDRO COLLEGE COMPETENCY APPRAISAL - OXYGENATION

15. You are called to the ER to help assess and care for a patient admitted with severe pulmonary
edema. While starting an intravenous line, the physician tells you to give the patient O2. You
should provide oxygen to this patient via a:
A) nonrebreathing mask at 15 L/min
B) nasal cannula at 6 L/min
C) simple face mask at 7 L/min
D) 50% air-entrainment mask
Ans: A
Response:
When treating patients with carbon monoxide poisoning, cyanide poisoning, acute pulmonary
edema, shock, trauma, or acute myocardial infarction in emergency settings, always provide
the highest possible FIO2. Of the devices listed, only the nonrebreathing mask can deliver high
FIO2s. You might also recommend CPAP with 100% O2 for this patient, since the elevated
airway pressure can help reduce venous return and alleviate pulmonary congestion.

16. A patient receiving bilevel positive airway pressure for acute respiratory failure has a PaO2 of
48 torr on 65% O2 with IPAP = 20 cm H2O and EPAP = 5 cm H2O. To raise this patient's
PaO2, you should recommend:
A) increasing the FIO2 to 0.80
B) increasing IPAP to 25 cm H2O
C) increasing EPAP to 10 cm H2O
D) decreasing IPAP to 15 cm H2O
Ans: C
Response:
This patient's hypoxemia is due to shunting (PaO2  50 torr, FIO2  0.50). If shunting is
present when administering NPPV for acute respiratory failure, you should increase the EPAP
level, while being sure to keep P (IPAP – EPAP)  5 cm H2O.

17. A patient who is receiving 0.5 mL isoetharine (Bronkosol) in 3 mL of normal saline three
times a day to relieve bronchospasm complains of nervousness and palpitations after therapy.
You should consider recommending all of the following to the patient's doctor EXCEPT:
A) substituting levalbuterol (Xopenex) for the isoetharine
B) decreasing the dose of isoetharine to 0.25 mL
C) substituting budesonide (Pulmicort) for the isoetharine
D) substituting albuterol (Proventil) for the isoetharine
Ans: C
Response:
This patient is exhibiting undesired cardiovascular and CNS effects associated with
isoetharine's moderate beta-1 adrenergic action. To help avoid these effects, you should
generally select or recommend those agents with the least beta1 and most beta-2 effect—e.g.,
albuterol or its isomer, levalbuterol. Alternatively, if cardiovascular/CNS effects are
pronounced, you can recommend decreasing the dose of the drug. Substituting budesonide for
the isoetharine makes no sense since budesonide is a steroid controller, not reliever.

18. A doctor institutes volume control ventilation for an 80-kg ARDS patient. Which of the
following is the maximum plateau pressure you should aim to achieve in this patient?
A) 50 cm H2O peak pressure
B) 30 cm H2O plateau pressure
C) 40 cm H2O peak pressure
D) 50 cm H2O plateau pressure
Ans: B
Response:
According to the NHLBI protocol, the target volume for ARDS patients is 4–6 mL/kg, with a
maximum plateau (alveolar) pressure of 30 cm H2O. The ventilator rate should initially be set
to match the prior VE but can be increased as needed up to a maximum of 35 breaths/min.

Page 6
SAN PEDRO COLLEGE COMPETENCY APPRAISAL - OXYGENATION

19. A 70 kg patient receiving mechanical ventilation has the following ventilator settings and
arterial blood gas results:

Ventilator Settings Blood Gases


Mode SIMV pH 7.26
VT 700 mL PaCO2 56 torr
Set rate 6 HCO3 22 mEq/L
Spon rate 0 PaCO2 92 torr
FIO2 0.55 SaO2 96%

Which of the following should you recommend?


A) increasing the inspiratory time
B) increasing the tidal volume to 800 mL
C) decreasing the FIO2 to 0.50
D) increasing the SIMV rate
Ans: D
Response:
The blood gas indicates uncompensated respiratory acidosis. As a result, the patient's minute
ventilation should be increased. Given that the patient has no spontaneous respirations and
that the tidal volume is adequate, you should increase the SIMV rate.

20. Thirty minutes into a spontaneous breathing trial to determine if a patient can be removed
from ventilatory support, you note a 15-torr rise in the PaCO2, an increase in the rate of
breathing from 20 to 35 breaths/min, and greater dependence on accessory muscle use. Based
on these observations, you should recommend:
A) continuing the trial and carefully monitoring the patient for an additional 30 minutes
B) ending the trial and returning the patient to a full ventilatory support mode
C) adding 10 cm H2O pressure support and obtaining another ABG to assess the effect
D) ending the trial and restoring the patient to partial support to exercise the diaphragm
Ans: B
Response:
Objective physiologic measures indicating a successful SBT include acceptable gas exchange,
and stable hemodynamics and ventilatory pattern. In addition, the patient should not exhibit
any subjective indicators of intolerance or failure. In this case the rise in PaCO2 exceeds the
limit (10 torr) for acceptable gas exchange, the increased rate of breathing indicates an
unstable ventilatory pattern, and the greater accessory muscle use suggests intolerance of the
procedure. For these reasons, you should return the patient to a level of ventilatory support
that maintains adequate oxygenation and ventilation while also preventing muscle fatigue—
i.e., usually a full ventilatory support mode.

21. An adult patient in respiratory failure has the following blood gases on a simple oxygen mask
at 8 L/min: pH=7.19; PaCO2=68 torr; HCO3=28 mEq/L; PaO2= 85 torr. The attending
physician orders intubation and ventilatory support. Which of the following FIO2 would you
initially select for this patient?
A) 0.25
B) 0.30
C) 0.45
D) 0.60
Ans: D
Response:
If ABG data is available before initiating ventilatory support, and the adequacy of
oxygenation is not in question, one can begin support at an FIO2 equal to or slightly higher
than that previously used. In this case (with a PaO2 of 85 torr on a simple mask at about 50-
60% O2) an initial FIO2 of 0.60 should suffice. Once on the ventilator, the P/F ratio should be
assessed and the need for PEEP determined.

Page 7
SAN PEDRO COLLEGE COMPETENCY APPRAISAL - OXYGENATION

22. A physician orders intubation and mechanical ventilation in the CMV-assist/control mode for
a 190 lb adult male patient with ARDS. Which of the following ventilatory parameters would
be most appropriate for this patient?

Rate/min VT (mL)
A 10 800
B 18 1300
C 8 900
D 20 450
A) A
B) B
C) C
D) D
Ans: D
Response:
In adult patients with ARDS, VTs between 4-6 ml/kg of body weight are used, with rates
between 8-20/min. For a 190 lb (86 kg) adult with normal lungs, a VT between 340-510 mL at
a rate between 8-20/min would be suitable. In the assist/control mode, the lower volumes and
higher rates are generally used.

23. Which of the following positions are appropriate for IPPB therapy?

I. semi-Fowler's
II. prone
III. standing
A) II and III only
B) I and III only
C) I, II and III
D) I and II only
Ans: B
Response:
For best results with IPPB, the patient should be positioned in as close to an upright posture as
possible. Slouching should be discouraged because it will hamper the excursion of the
diaphragm which will, in turn, decrease the volume of gas delivered to the patient's lungs. A
very obese patient should ideally be positioned standing upright next to the bed, but this is
often not practical.

24. Prior to suctioning a nine-week old infant receiving 40% oxygen via oxyhood, you would:
A) preoxygenate the infant with 50% oxygen
B) preoxygenate the infant with 30% oxygen
C) preoxygenate the infant with 80% oxygen
D) preoxygenate the infant with 90% oxygen
Ans: A
Response:
Procedures for nasopharyngeal and nasotracheal suctioning of infants and children are
comparable to those for adults, with a few key exceptions. In infants less than 6 months old,
preoxygenation with 100% oxygen is not recommended, due to the risk of retinopathies.
Instead, most clinicians recommend elevation of the FIO2 by no more than 10% above that
being delivered prior to suctioning.

Page 8
SAN PEDRO COLLEGE COMPETENCY APPRAISAL - OXYGENATION

25. You are monitoring a 77 kg (169 lb) patient being weaning using the SIMV mode. When you
decrease the SIMV rate from 10/min to 6/mim, the patient's heart rate increases from 85/min
to 99/min. Which of the following actions would you take at this time?
A) Switch to pressure support weaning
B) Continue to observe and monitor the patient
C) Recommend a mild sedative (e.g., Versed)
D) Restore the SIMV rate back to 10/min
Ans: B
Response:
Heat rate increases of 15-20/min are common during weaning from ventilatory support and do
not necessarily indicate patient harm. In this case, you should continue to observe and
carefully monitor the patient.

26. A physician orders a "T-tube trial" for a patient receiving ventilatory support in the assist-
control mode with an FIO2 of 0.50. What FIO2 would you deliver to the patient during her
spontaneous breathing period?
A) 0.30
B) 0.70
C) 0.50
D) 0.60
Ans: D
Response:
When using periods of spontaneous breathing as the weaning method, patient should generally
be provided with a humidified oxygen via a T-tube at an FIO2 about 10% higher than had been
delivered by the ventilator.

27. Bronchodilation is most commonly achieved via the use of which of the following drug
categories?
A) cholinergic agents
B) prostaglandins
C) adrenergic agents
D) anticholinergic agents
Ans: C
Response:
Bronchodilation is usually achieved using beta adrenergic agents. Supplementing this
approach are selected anticholinergic drugs, the xanthines, and (more recently) the
prostaglandins.

28. A doctor institutes pressure control ventilation for a 70 kg ARDS patient. Which of the
following tidal volumes would you aim to achieve in this patient when using the NHLBI
protocol?
A) 630 mL
B) 700 mL
C) 560 mL
D) 420 mL
Ans: D
Response:
According to the NHLBI Protocol, you begin ventilation of ARDS patients with an initial tidal
volume of 8 mL/kg IBW, then reduce it by 1 mL/kg every two hours until you achieve a V T of
6 mL/kg (minimum of 4 mL/kg). The ventilator rate should initially be set to match the prior
VE, but can be increased as needed up to a maximum of 35/min.

Page 9
SAN PEDRO COLLEGE COMPETENCY APPRAISAL - OXYGENATION

29. A patient receiving mechanical ventilation has the following ventilator settings and arterial
blood gas results:

Ventilator Settings Blood Gases


Mode SIMV pH 7.56
VT 750 ml PaCO2 26 torr
Set Rate 12 HCO3 22 mEq/L
Spon Rate 0 PaO2 92 torr
FIO2 0.55 SaO2 96%

Which of the following should you recommend?


A) increase the peak flow
B) increase the tidal volume to 800 mL
C) decrease the FIO2 to 0.50
D) decrease the SIMV rate
Ans: D
Response:
The blood gas indicates uncompensated respiratory alkalosis. As a result, the patient's minute
ventilation should be reduced by decreasing the SIMV rate.

30. A patient under your care has X-ray and clinical evidence of severe unilateral right lung
infiltrates. His PaO2 on a non-rebreathing mask is 52 torr. The attending physician asks your
advice as to how best to improve this patient's oxygenation without committing to ventilatory
support. Which of the following would you recommend?
A) place the patient on his left side (left lung down)
B) place the patient on his right side (right lung down)
C) turn the patient from the supine to prone position
D) institute a regimen of inspiratory resistive breathing
Ans: A
Response:
Dependent positioning can improve the distribution of ventilation in patients with V/Q
imbalances, especially those with local conditions such as unilateral pneumonias. Placing the
good lung in the dependent or down position (in this case the left lung) can significantly
improve oxygenation without a change in FIO2, since the 'down' lung will receive the best
ventilation and blood flow.

31. A doctor orders inverse ratio (2:1) BiPAP™ via a Respironics S/T-D unit for a patient in
hypoxemic respiratory failure. Which of the following settings would achieve this goal:
A) mode = T; %IPAP = 66%
B) mode = S/T; %IPAP = 66%
C) mode = T; %IPAP = 20%
D) mode = IPAP; %IPAP = 75%
Ans: A
Response:
To provide inverse ratio BiPAP via a Respironics S/T-D unit, you must set the device to the T
mode and set a %IPAP > 50%.

Page 10
SAN PEDRO COLLEGE COMPETENCY APPRAISAL - OXYGENATION

32. Which of the following BiPAP™ settings would you initially select for a 165 lb COPD patient
in hypercapnic respiratory failure needing noninvasive ventilation in the emergency
department?
A) EPAP = 5 cm H2O; IPAP = 15 cm H2O; mode S/T; rate 10
B) EPAP = 0 cm H2O; IPAP = 10 cm H2O; mode S/T; rate 10
C) EPAP = 5 cm H2O; IPAP = 15 cm H2O; spon mode only
D) EPAP = 20 cm H2O; IPAP = 30 cm H2O; mode S/T; rate 10
Ans: A
Response:
For a patient in hypercapnic respiratory failure needing NPPV in the emergency department
the following settings are a good starting point: EPAP = 5 cm H2O; IPAP = 15 cm H2O; mode
S/T; rate 10. Setting an EPAP of 5 cm H2O helps prevent auto-PEEP and air trapping, the
IPAP of 15 cm H2O should increase the patients tidal volume and using the S/T mode with a
rate of 10 assure that the patient will receive adequate ventilatory support even if her or she
becomes apneic.

33. In order to stabilize a patient during the initial application of ventilatory support, which of the
following parameters must be set?

I. ventilatory support mode


II. oxygen concentration (FIO2)
III. minute ventilation (f, VT)
IV. pressure support
A) I and III only
B) I, II and III only
C) III and IV only
D) I, II, III and IV
Ans: B
Response:
In order to stabilize a patient when first applying ventilatory support, one must carefully select
the ventilatory support mode, FIO2, and the parameters of minute ventilation.

34. Which of the following inspiratory pressure patterns would have the worst effect on a patient's
cardiac output?
A) low pressure, short duration curve with PEEP
B) high pressure, short duration curve with inspiratory hold
C) high pressure, long duration square wave with PEEP
D) high pressure, increasing force curve with expiratory retard
Ans: C
Response:
In general, the higher the inspiratory pressure and the longer its duration, the greater the
harmful effect of positive pressure ventilation on a patient's cardiac output. These negative
effects will be compounded by the use of a constant pressure (square) waveform during
inspiration and PEEP.

Page 11
SAN PEDRO COLLEGE COMPETENCY APPRAISAL - OXYGENATION

35. A doctor asks you to assess if a 65 kg (143 lb) patient with a neuromuscular disorder being
mechanically ventilated in the SIMV mode is ready for weaning. You obtain the following
data during a bedside ventilatory assessment:

Spontaneous tidal volume 250 mL


Minute ventilation 10 L/min
Vital capacity 700 mL
Max inspir pressure (MIP) -28 cm H2O

Based on this information, which of the following would you recommend?


A) begin a spontaneous breathing T-piece trial
B) postpone weaning and re-evaluate the patient
C) begin weaning using a pressure support protocol
D) begin weaning by decreasing the SIMV rate
Ans: B
Response:
The patient's vital capacity (> 10 mL/kg) and MIP ( < -25 cm H2O) are borderline adequate, as
is the minute ventilation (10 L/min). However, close inspection of the data indicates a
abnormally low tidal volume. Dividing the minute ventilation by the tidal volume yields the
spontaneous rate of breathing rate (rate = 10 L/min ÷ 250 mL/breath = 40 breaths/min). This
yields a rapid shallow breathing index of 40/0.25 = 160/min, far above the threshold value of
100 that indicates a potential weaning problem and likely weaning failure. Based on discovery
and analysis of the missing data, you should recommend postponing weaning and re-
evaluating the patient.

36. A surgeon orders an increase in PEEP from 6 to 10 cm H2O for a post-op patient receiving
mechanical ventilation. After you adjust the PEEP setting, you note a rapid fall in the patient's
arterial blood pressure and a rapid rise in her heart rate. Which of the following actions would
your recommend to the surgeon?
A) increase the FIO2 by 10%
B) administer a vasopressor
C) return the PEEP to 6 cm H2O
D) obtain a stat arterial blood gas
Ans: C
Response:
Because application of PEEP increases intrathoracic pressure, it can impede venous return and
decrease cardiac output is some patients. The drop in this patient's blood pressure and rise in
her heart rate indicates that the increase in PEEP is decreasing her cardiac output. In general,
whenever an intervention has a bad effect, the first action should be to reverse the
intervention, in this case by returning the PEEP to 6 cm H2O.

37. You should instruct a postoperative patient who is to receive incentive spirometry to
A) exhale maximally prior to inhalation
B) exhale maximally after a normal inhalation
C) inhale maximally following maximum exhalation
D) inhale maximally after a normal exhalation
Ans: D
Response:
When coaching patients on the use of an incentive spirometer to perform sustained maximum
inspiratory exercises, they should exhale normally, then take as deep a breath as they can and
hold it for about at least 5 seconds.

Page 12
SAN PEDRO COLLEGE COMPETENCY APPRAISAL - OXYGENATION

38. A 43-year-old 70 kg woman receiving 40% oxygen via T-tube has been admitted to the ICU
following gastric surgery. Here attending doctor orders volume control ventilation in the
assist/control mode. Which of the following would be the most appropriate initial ventilator
settings for this patient?

VT Rate FIO2
A 500 10 0.70
B 650 16 0.95
C 700 12 0.50
D 900 15 0.40
A) A
B) B
C) C
D) D
Ans: C
Response:
Given this patient's weight, a tidal volume of about 700 mL is in order (10 mL/kg). For
assist/control ventilation of an adult, a starting rate of 8 to 12/min would be satisfactory.
Given that she is already receiving 40% oxygen, it would be wise to provide an initial small
increment in FIO2 as she is first placed on the ventilator.

39. A patient is being mechanically ventilated in the SIMV mode at a rate of 4/min. The
spontaneous respiratory rate increases from 12 to 35/min. Which of the following is the most
appropriate action?
A) Increase the SIMV rate
B) Add 10 cm H2O PEEP
C) Initiate pressure control
D) Sedate the patient
Ans: A
Response:
In the SIMV mode, a significant increase in the total respiratory rate indicates increased work-
of-breathing. Therefore, the machine rate setting should be increased.

40. Which of the following procedures will best prevent atelectasis in a cooperative postoperative
patient?
A) metered dose inhaler with albuterol (Proventil)
B) continuous pulse oximetry
C) incentive spirometry
D) pursed-lip breathing
Ans: C
Response:
Incentive spirometry is an excellent way to help prevent atelectasis in postoperative patients.

41. The best position for initiating diaphragmatic breathing exercises is:
A) supine with knees bent, head supported
B) sitting with feet supported on stool
C) prone Trendelenburg with abdomen supported
D) semi-Fowler's position with knees bent
Ans: D
Response:
The best position for diaphragmatic breathing exercises is similar to that used for cough
training, i.e. a 45ø sitting position (semi-Fowler's), with shoulder rotated inward, the head and
spine slightly flexed, the forearms relaxed or supported, and the knees bent. This position is
designed to help relax the abdominal muscles and facilitate maximum diaphragm movement.

Page 13
SAN PEDRO COLLEGE COMPETENCY APPRAISAL - OXYGENATION

42. The optimal IPPB breathing pattern for treatment of atelectasis is:
A) rapid, deep breaths held at end-inspiration
B) slow, shallow breaths held at end-inspiration
C) slow, deep breaths held at end-inspiration
D) rapid, shallow breaths held at end-inspiration
Ans: C
Response:
The optimal breathing pattern to reinflate collapsed lung units with IPPB consists of slow,
deep breaths that are held at end-inspiration. This type of maneuver increases the distribution
of inspired gas to areas of the lung with low compliance, specifically, the atelectatic areas.

43. A postop patient receiving incentive spirometry treatments complains of dizziness and
tingling in the extremities following therapy. Which of the following is the most likely cause
of these symptoms?
A) The patient is hypoxemic
B) The patient is inhaling too rapidly
C) The patient is hyperventilating
D) The patient is breathing too slow
Ans: C
Response:
Dizziness and tingling in the extremities (paresthesia) are symptoms of respiratory alkalosis
(hyperventilation). This is a common complication of incentive spirometry treatments when
the patient breathes too fast. To avoid hyperventilation during incentive spirometry, patients
should be taught to take 6 or fewer breaths per minute through the device.

44. When initially placed on a ventilator at an FIO2 of 1.0, a patient has a PaO2 of 185 mm Hg.
Which of the following conclusions can you draw regarding this patient's oxygenation?
A) the patient has efficient oxygenation
B) the patient has a low arterial O2 content
C) the patient has a diffusion defect
D) the patient has a large physiologic shunt
Ans: D
Response:
Assuming a normal PCO2 and barometric pressure, with a PaO2 of 185 mm Hg on an FIO2 of
1.0 this patient has a P(A-a)O2 close to 500 mm Hg. Oxygenation is minimally acceptable as
long as the P(A-a)O2 is less than 350 mm Hg. P(A-a)O2s above 350 mm Hg indicate severe
impairment in oxygen exchange, due mainly to physiologic shunting.

45. A doctor wants to provide full ventilatory support but use low tidal volumes and permissive
hypercapnia in an ARDS patient. Which of the following modes of mechanical ventilation
would you choose for this patient?
A) control mode
B) assist/control
C) pressure support ventilation
D) SIMV
Ans: A
Response:
Control mode is indicated when full ventilator support is needed but the rate and/or ventilatory
pattern must be controlled or patient effort eliminated, as in the following clinical situations:
inverse I:E ratio ventilation, permissive hypercapnia, hyperventilation in brain injury, flail
chest, hyperventilation syndrome, seizure disorders.

Page 14
SAN PEDRO COLLEGE COMPETENCY APPRAISAL - OXYGENATION

46. Breathing exercises are most likely to help a patient who has
A) pneumothorax
B) emphysema
C) bronchiectasis
D) pulmonary edema
Ans: B
Response:
Breathing exercises such as pursed-lipped and diaphragmatic breathing exercises can help
reduce air-trapping and decrease dyspnea in patients with emphysema.

47. The PEEP/CPAP level is optimum when:


A) the pulmonary wedge pressure exceeds 15 mm Hg
B) the arterial-venous O2 difference is maximized
C) oxygen delivery to the tissues is maximized
D) the PaO2 is between 60 to 100 mm Hg
Ans: C
Response:
When PEEP/CPAP is instituted, the goal should be to achieve adequate oxygenation, with an
acceptable FIO2, without compromising cardiovascular function. In principle, optimum PEEP
or CPAP occurs when oxygen delivery to the tissues is maximized.

48. All of the following are TRUE regarding control mode ventilation EXCEPT:
A) control mode is poorly tolerated by most patients
B) during control mode, patient efforts result in asynchrony
C) patients in control mode may require heavy sedation/paralysis
D) control mode cannot support inverse I:E ratio ventilation
Ans: D
Response:
Control mode is poorly tolerated by most patients and thus may require heavy sedation or
neuromuscular paralysis to be effective. If not, patient efforts can result in asynchrony
("fighting the ventilator") and increased work of breathing. Nonetheless, control mode is
required when the patient's rate or ventilatory pattern must be controlled or patient effort
eliminated, as when using 'abnormal' patterns such as inverse I:E ratio.

49. When a patient is receiving positive-pressure ventilation, the respiratory therapist should
strive to keep the alveolar (plateau) airway pressure below:
A) 20 cm H2O
B) 30 cm H2O
C) 50 cm H2O
D) 65 cm H2O
Ans: B
Response:
Research shows that the risk of lung injury due to barotrauma can be minimized by
maintaining plateau pressures below 30 cm H2O.

Page 15
SAN PEDRO COLLEGE COMPETENCY APPRAISAL - OXYGENATION

50. You increase the PEEP level on a patient receiving ventilatory support. Which of the
following would indicate that too much PEEP is being used?
A) a decrease in pulmonary wedge pressure from 16 to 10 mm Hg
B) an increase in effective compliance from 19 to 31 mL/cm H2O
C) a decrease in cardiac output from 5.1 to 3.2 liters/min
D) a decrease in central venous pressure from 9 to 4 mm Hg
Ans: C
Response:
During initial application of PEEP, PaO2, FRC and static compliance normally increase.
Cardiac output may increase slightly, stay the same or decrease. A significant decrease in
cardiac output indicates excessive PEEP.

51. Data for a 78 kg (172 lb) patient receiving ventilatory support with 12 cm PEEP are as
follows:

Ventilator Settings Blood Gases


Mode SIMV pH 7.34
VT 750 ml PaCO2 43 mm Hg
Rate 8 HCO3 22 mEq/L
FIO2 0.45 PaO2 145 mm Hg
PEEP 12 cm H2O SaO2 99%

Which of the following changes would you recommend at this time?


A) lower the VT
B) increase the rate
C) decrease PEEP
D) decrease the FIO2
Ans: C
Response:
The goal of PEEP is to achieve adequate oxygenation with a safe FIO2. In this case, the FIO2 is
acceptable but the PaO2 is excessive. Thus the PEEP level should be lowered.

52. All of the following are TRUE regarding intermittent mandatory ventilation (IMV) EXCEPT:
A) machine breaths may target either volume or pressure
B) spontaneous breathing allowed; patient controls rate and pattern
C) hypoventilation can occur at low set machine rates
D) asynchronous breathing prevented during machine breaths
Ans: D
Response:
IMV allows spontaneous breathing between machine breaths, so that the patient can control
both the overall rate and pattern. IMV provides full ventilatory support at normal rates and
partial support at lower rates. Machine breaths may target either volume or pressure and
spontaneous breaths may be pressure supported. Asynchronous breathing can still occur
during machine breaths, usually due to improper machine sensitivity or flow settings.

Page 16
SAN PEDRO COLLEGE COMPETENCY APPRAISAL - OXYGENATION

53. Despite manual hyperoxygenation/hyperinflation, a patient on mechanical ventilatory support


with PEEP tends to easily develop hypoxemia during suctioning. Techniques that could help
minimize this problem include:

I. perform hyperoxygenation/inflation through the ventilator


II. use a closed suction system that does not require disconnection
III. using a special catheter that provide both suction and oxygen
A) I and II only
B) I, II and III
C) I and III only
D) II and III only
Ans: B
Response:
Hyperoxygenation/hyperinflation are probably more effective when done through the
ventilator (as opposed to manually). Use of a closed suction system that does not require
disconnection of the circuit from the airway may also be helpful. A double lumen catheter that
can provide both suction pressure and oxygen delivery can also help.

54. In order to minimize the likelihood of acute respiratory alkalosis during IPPB, what rate of
breathing should you strive for?
A) 3-4/min
B) 6-8/min
C) 12-14/min
D) 16-18/min
Ans: B
Response:
In general, keeping the respiratory rate between 6 to 8 breaths per minute will minimize the
likelihood of hyperventilation during IPPB. Nonetheless, it is vital that the therapist remain
with the patient throughout the treatment and be on guard for the signs and symptoms
hypocapnea. These signs and symptoms include dizziness and numbness or tingling of the
extremities (parasthesia).

55. Indicators of successful application of noninvasive positive pressure ventilation (NPPV)


include all of the following EXCEPT:
A) reduced dyspnea
B) decreased respiratory rate
C) increased use of accessory muscles
D) decreased PaCO2
Ans: C
Response:
Indicators of successful application of NPPV include a rapid improvement in patient comfort,
as evidenced by reduced dyspnea, decreased respiratory rate, decreased use of accessory
muscles, and patient synchronization with device. In addition, we should expect to see a
reduced PaCO2 in patients with respiratory acidosis or improved oxygenation in patients with
hypoxemic respiratory failure.

Page 17
SAN PEDRO COLLEGE COMPETENCY APPRAISAL - OXYGENATION

56. Assuming all else being equal, which of the following BiPAP settings would result in the
greatest tidal volume?
A) EPAP = 5 cm H2O; IPAP = 15 cm H2O
B) EPAP = 10 cm H2O; IPAP = 20 cm H2O
C) EPAP = 5 cm H2O; IPAP = 25 cm H2O
D) EPAP = 15 cm H2O; IPAP = 30 cm H2O
Ans: C
Response:
Tidal volume during pressure-limed breathing is primarily a function of the difference
between the baseline and peak pressures. During BiPAP, the baseline pressure is the EPAP
setting and the peak pressure is the IPAP setting. Thus the greater the difference between
EPAP and IPAP, the greater the expected tidal volume.

57. A patient is breathing spontaneously, but pressure above atmospheric is maintained at the
airway opening throughout the breathing cycle. What mode of ventilatory support is being
delivered?
A) continuous positive airway pressure (CPAP)
B) continuous mandatory ventilation-control mode
C) pressure support ventilation (PSV)
D) intermittent mandatory ventilation (IMV)
Ans: A
Response:
The maintenance of a pressure above atmospheric at the airway opening throughout a
spontaneous breathing cycle best describes continuous positive airway pressure, or CPAP.
CPAP can be used alone or in combination with other modes of ventilatory support, such as
CMV or IMV. When used in combination with ventilatory support, we usually refer to this
technique as positive end-expiratory pressure or PEEP.

58. All of the following are indications for positive-end expiratory pressure (PEEP) EXCEPT:
A) to decrease hypoxemia due to shunting (e.g., ARDS, IRDS)
B) to lower FIO2 needs in patients with refractory hypoxemia
C) to allow for graded levels of ventilatory support
D) to decrease auto-PEEP in patients with airway obstruction
Ans: C
Response:
The primary indication for PEEP is to decrease hypoxemia due to shunting in conditions like
ARDS and IRDS. PEEP can also lower FIO2 needs in patients with refractory hypoxemia and
help maintain or increase the FRC (e.g., in thoracic surgery). Last, extrinsic PEEP can be
applied to decrease auto-PEEP in patients with airway obstruction receiving ventilatory
support.

59. In order for continuous positive airway pressure (CPAP) to be successful, the patient must
have:
A) an adequate PaO2 on less than 50% O2
B) a secure artificial airway in place
C) adequate spontaneous ventilation
D) a cardiac output greater than 5 L/min
Ans: C
Response:
CPAP alone does not provide ventilation, serving only to maintain alveoli at greater inflation
volumes. Thus flow into and out of the lung depends on spontaneously generated changes in
pleural pressure, as with normal breathing.

Page 18
SAN PEDRO COLLEGE COMPETENCY APPRAISAL - OXYGENATION

60. A doctor institutes volume control ventilation for a 70 kg ARDS patient with a targeted tidal
volume of 420 mL. To maintain adequate ventilation with this tidal volume, you would allow
a machine respiratory rate as high as:
A) 20/min
B) 25/min
C) 30/min
D) 35/min
Ans: D
Response:
According to the NHLBI Protocol, you begin ventilation of ARDS patients with an initial tidal
volume of 8 mL/kg IBW, then reduce it by 1 mL/kg/2hrs until you achieve a VT of 6 mL/kg
(minimum of 4 mL/kg). The ventilator rate should initially be set to match the prior VE, but
can be increased as needed up to a maximum of 35/min.

61. A doctor want to apply inverse I:E ratio ventilation for an ARDS patient. Which of the
following modes of mechanical ventilation would you choose for this patient?
A) assist/control
B) CPAP
C) control mode
D) SIMV
Ans: C
Response:
Control mode is indicated when full ventilator support is needed but the rate and/or ventilatory
pattern must be controlled or patient effort eliminated, as in the following clinical situations:
inverse I:E ratio ventilation, permissive hypercapnia, hyperventilation in brain injury, flail
chest, hyperventilation syndrome, seizure disorders.

62. When volume control ventilation is initiated for a patient in hypercapnic respiratory failure
and no settings are specified, which of the following would be the most acceptable initial tidal
volume setting?
A) 2-4 mL/kg predicted body weight
B) 4-6 mL/kg predicted body weight
C) 8-10 mL/kg predicted body weight
D) 12-15 mL/kg predicted body weight
Ans: B
Response:
Typically, initial tidal volumes are set to 8-10 mL/kg predicted body weight (PBW). The
exception for patients with ARDS/acute respiratory failure in which the ARDS Protocol may
start with 8 mL/kg, but aims to achieve a tidal volume of 4-6 mL/kg PBW.

63. In establishing initial ventilatory support settings, the most important consideration is the:
A) patient's size and clinical condition
B) type of ventilatory support device used
C) quality and amount of nursing supervision
D) amount of spontaneous ventilatory effort
Ans: A
Response:
The first step in the management of patients receiving ventilatory support is to establish the
initial ventilator settings. Initial ventilator settings are based first and foremost on the patient's
size and clinical condition. Settings are entered and proper ventilator function verified before
connection to the patient.

Page 19
SAN PEDRO COLLEGE COMPETENCY APPRAISAL - OXYGENATION

64. An adult patient in respiratory failure has the following ABGs on a simple O2 mask at 8
L/min: pH=7.19; PaCO2=68 mm Hg; HCO3=28 mEq/L; PaO2= 85 mm Hg. The attending
physician orders intubation and ventilatory support. Which of the following modes of support
would you consider appropriate for this patient?

I. CMV at a frequency of 12/min


II. IMV at a frequency of 12/min
III. CPAP with 10 cm H2O pressure
A) I or II only
B) II or III only
C) I or III only
D) I, II or III
Ans: A
Response:
The mode of ventilatory support initially chosen depends mainly upon the patient's underlying
pathophysiologic problem. When a patient's respiratory failure is associated with hypercapnia
due to inadequate alveolar ventilation -- as in this case -- either the CMV or IMV modes (with
equivalent rate settings) may be employed.

65. A physician orders intubation and mechanical ventilation in the CMV-assist/control mode for
a 90 lb, 12 year-old child. Which of the following ventilatory settings would be most
appropriate for this patient?

Rate/min VT (ml)
A 12 400
B 14 300
C 8 900
D 25 350
A) A
B) B
C) C
D) D
Ans: D
Response:
With adolescents in the 8-16 year-old age range, VTs between 8-10 ml/kg, at rate between 20-
30/min are usually sufficient. For a 90 lb (41 kg) child, VTs between 325-400 ml at rates of
20-30/min would be appropriate.

Page 20
SAN PEDRO COLLEGE COMPETENCY APPRAISAL - OXYGENATION

66. A patient is admitted through the emergency room with an initial diagnosis of CO poisoning.
ABGs are drawn with her breathing room air and show the following:

pH 7.46
PaCO2 32 mm Hg
PaO2 85 mm Hg

Based on this, what change would you recommend for the patient's respiratory care plan?
A) Give the patient 100% O2 by a nonrebreathing mask
B) Institute mechanical ventilation with 50% O2
C) Give the patient 28% O2 by an air-entrainment mask
D) Discharge her because she is not hypoxemic
Ans: A
Response:
Remember that the PaO2 only represents dissolved O2 only! CO poisoning affect HbO2
saturation, not dissolved oxygen levels. Indeed, the PaO2 in CO poisoning can be normal or
even high (if the patient is breathing supplemental O2). Any patient suspected of CO
poisoning should be given as high an FIO2 as possible (thus the nonrebreather). In addition,
CO-oximetry should be performed on an arterial sample to verify the Hb O2 and COHb levels
and confirm the diagnosis.

67. A doctor orders pressure control ventilation for a 50 kg postop female patient. Which of the
following settings would you choose for this patient?
A) pressure limit/PIP = 35 cm H2O; rate = 20/min; inspiratory time = 2 seconds
B) pressure limit/PIP = 40 cm H2O; rate = 20/min; inspiratory time = 1 second
C) pressure limit/PIP = 25 cm H2O; rate = 15/min; inspiratory time = 1 second
D) pressure limit/PIP = 30 cm H2O; rate = 30/min; inspiratory time = 1 second
Ans: C
Response:
Initial settings for pressure control ventilation of adults are pressure limit: 20-30 cm H2O, rate:
8-24/min, and inspiratory time or I:E set to achieve I:E less than or equal to 1:1 (e.g., 1:2, 1:3).

68. Which of the following parameters would you set to establish the minute volume for a patient
being ventilated via a volume-cycled ventilator?

I. Expiratory time
II. Rate per minute
III. Tidal volume
IV. Flow
A) II, III only
B) I, III only
C) I, II, IV only
D) I, II, III, IV
Ans: A
Response:
The minute volume (VE) on a volume-cycled ventilator is determined by the respiratory rate
or frequency (f) and tidal volume (VT) setting (VT). VE = f x VT

Page 21
SAN PEDRO COLLEGE COMPETENCY APPRAISAL - OXYGENATION

69. Pursed-lip breathing would be most beneficial for a patient who has
A) asthma
B) croup
C) emphysema
D) pneumonia
Ans: C
Response:
Patients with severe emphysema tend to have highly compliant (floppy) airways. As a result,
to help prevent airway collapsing and air trapping, they should be instructed to exhale slowly,
through pursed-lips.

70. You cannot maintain satisfactory oxygenation on an ARDS patient receiving 100% O2 and
being ventilated with pressure control ventilation at a plateau pressure of 40 cm H2O and 15
cm H2O PEEP. Which of the following modes of ventilation would you recommend at this
time?
A) synchronous intermittent mandatory ventilation (SIMV)
B) airway pressure release ventilation (APRV)
C) pressure regulated volume control (PRVC)
D) continuous positive airway pressure (CPAP)
Ans: B
Response:
Airway pressure release ventilation (APRV) is equivalent to CPAP with regular, brief,
intermittent releases in airway pressure to baseline. Often referred to as "inverted IMV"
(based on graphic appearance), APRV is indicated for patients with (1) ALI/ARDS, especially
when Pplat > 30 cm H2O; (2) refractory hypoxemia due to collapsed alveoli; or (3) massive
atelectasis.

71. Which of the following modes of ventilation will require the lowest possible peak pressures
yet tend to assure a consistent tidal volume in patients with changing compliance or
resistance?
A) pressure control ventilation (PCV)
B) airway pressure release ventilation (APRV)
C) pressure regulated volume control (PRVC)
D) continuous positive airway pressure (CPAP)
Ans: C
Response:
Pressure regulated volume control (PRVC) is a form of pressure control (patient- or time-
triggered, pressure-limited and time-cycled ventilation) in which the pressure limit is
automatically adjusted breath-to-breath to maintain a target tidal volume. It is indicated for
patients: (1) requiring the lowest possible pressure and a guaranteed consistent VT; (2) with
Acute Lung Injury/ARDS; (3) requiring high and/or variable inspiratory flow; and (4) those
with changing compliance or resistance.

72. Which of the following ventilator graphics displays would be the best choice to identify the
presence of a circuit leak?
A) volume vs. time display
B) pressure vs. time display
C) flow vs. time display
D) pressure vs. volume display
Ans: A
Response:
The best choice to identify the presence of a circuit leak using ventilator graphics would be a
volume vs. time display. A volume vs. flow X-Y loop can also be used to detect a circuit leak.

Page 22
SAN PEDRO COLLEGE COMPETENCY APPRAISAL - OXYGENATION

73. Which of the following ventilator graphics displays would be the best choice to identify the
presence of auto-PEEP?
A) volume vs. time display
B) pressure vs. time display
C) flow vs. time display
D) pressure vs. volume display
Ans: C
Response:
The best choice to identify the presence of auto-PEEP using ventilator graphics would be a
flow vs. time display. A volume vs. flow X-Y loop can also be used to detected auto-PEEP.

74. Which of the following ventilator graphics displays would be the best choice to assess the
work of breathing associated with patient triggering?
A) volume vs. time display
B) flow vs. volume display
C) flow vs. time display
D) pressure vs. volume display
Ans: D
Response:
The best choice to assess the work of breathing associated with patient triggering using
ventilator graphics would be a pressure vs. volume loop.

75. Which of the following ventilator graphics displays would be the best choice to assess a
patient for overdistension during positive pressure ventilation?
A) volume vs. time display
B) flow vs. volume display
C) flow vs. time display
D) pressure vs. volume display
Ans: D
Response:
The best graphic display to assess to assess a patient for overdistension during positive
pressure ventilation would be a pressure vs. volume loop.

76. Which of the following patients receiving volume- or pressure-control ventilation should be
considered for high-frequency oscillation ventilation (HFOV)?

I. a patient requiring mean airway pressures > 25 cm H2O


II. a patient who remains hypoxemic > 20 cm H2O PEEP
III. a patient needing high FIO2s (> 60% O2)
A) I and II only
B) II and III only
C) I and III only
D) I, II, and III
Ans: D
Response:
In general, HFOV is indicated for infants, children and adults with hypoxemic respiratory
failure who have not responded to more conventional methods of improving oxygenation.
This includes patients receiving VCV or PCV who: (1) need high concentrations of O2 (FIO2 >
0.60; P/F ratio < 200); (2) require high airway pressures or reverse I:E (Pmean > 25 cm H2O);
and (3) remain hypoxemic on high PEEP (20 – 25 cmH2O).

Page 23
SAN PEDRO COLLEGE COMPETENCY APPRAISAL - OXYGENATION

77. A doctor orders a preterm infant set up on high-frequency oscillation ventilation (HFOV).
Which of the following frequencies would you initially set for this infant?
A) 15 Hz
B) 10 Hz
C) 7 Hz
D) 5 Hz
Ans: A
Response:
With high-frequency oscillation ventilation (HFOV), you set the frequency in Hz (cycles/sec).
The recommended starting frequency for preterm infants on HFOV is 15 Hz, which equal 15
cycles/sec x 60 sec/min = 900/min. Recommended HFOV frequencies vary inversely with
patient size/weight.

78. To adjust patient oxygenation during high-frequency oscillation ventilation (HFOV), you
manipulate which of the following settings:
A) oscillation frequency
B) mean pressure (Pmean)
C) % inspiratory time
D) amplitude/power
Ans: A
Response:
The main determinant of oxygenation during HFOV is the mean airway pressure (Pmean). In
general, the higher Pmean, the larger the FRC and the better the oxygenation for a given FIO2.

79. To lower the PaCO2 of a patient receiving high-frequency oscillation ventilation (HFOV), you
would:
A) decrease the oscillation frequency
B) increase mean pressure (Pmean)
C) increase the oscillation frequency
D) increase the % inspiratory time
Ans: A
Response:
The settings that affect ventilation/CO2 elimination during HFOV are the amplitude and
frequency of the pressure oscillations. Increasing the amplitude and decreasing the oscillation
frequency lowers the PaCO2, whereas decreasing the amplitude and increasing frequency
raises the PaCO2).

Page 24
CHAPTER 13
Name: __________________________ Date: _____________

1. The ER physician asks you to review a chest radiograph from a patient with history of severe
emphysema. Which of the following findings would you expect to observe on this film?

I. a wide mediastinum
II. an increase in peripheral vascular markings
III. lowered, flattened diaphragm
IV. an increased radiolucency in the lung fields
V. presence of bullae and blebs
A) II and III only
B) III, IV, and V only
C) II, III, and V only
D) I, II, III, IV, and V
Ans: B
Response:
Flat diaphragms and increased radiolucency throughout the lung fields are common in the
chest X-ray of a any patient with a COPD condition, as are a decrease in peripheral vascular
markings, an increased retrosternal airspace, and a narrow mediastinum.

2. When using a pulse oximetry device, the most common source of error and false alarms is:
A) patient motion artifact
B) presence of HbCO
C) presence of vascular dyes
D) ambient light detection
Ans: A
Response:
The most common source of error and false alarms with pulse oximetry is motion artifact.
Securing the sensor properly or relocating of the sensor to an earlobe, a toe, or an external
naris can help minimize this problem.

3. A patient receiving mechanical ventilation in the CMV control mode is making asynchronous
breathing efforts against the ventilator's controlled breaths. This will result in:
A) decreased ventilatory drive
B) increased physiologic deadspace
C) increased work of breathing
D) acute metabolic acidosis
Ans: C
Response:
Controlled ventilation is poorly tolerated by many patients, often resulting in asynchronous
breathing efforts or strenuous attempts to breathe spontaneously. Both conditions can increase
the work of breathing, and with it, the oxygen consumption of the respiratory muscles. It is for
this reason that sedation or paralysis is often required when controlled ventilation is necessary.

Page 1
SAN PEDRO COLLEGE COMPETENCY APPRAISAL - OXYGENATION

4. A patient on a 30% aerosol oxygen mask has the following arterial blood gas results:

pH 7.54
PaCO2 27 torr
PaO2 80 torr
HCO3 23 mEq/L
BE –2 mEq/L

Which of the following is the correct interpretation of the arterial blood gas?
A) acute alveolar hyperventilation without hypoxemia
B) partially compensated respiratory alkalosis
C) respiratory acidemia with hypoxemia
D) hypochloremic metabolic alkalosis
Ans: A
Response:
The high pH indicates alkalemia. The low PaCO2 indicates hyperventilation, consistent with
the high pH (respiratory alkalosis). The normal HCO3 and BE indicate no metabolic
involvement, hence compensation has not begun yet (acute process). The PaO2 is in the
acceptable range at a low FIO2 (no hypoxemia present). Conclusion: acute respiratory
alkalosis due to alveolar hyperventilation without hypoxemia.

5. You find a patient receiving SIMV with a preset rate of 6 breaths/min, a VT of 1000 mL, and a
PEEP of 10 cm H2O. You note a peak inspiratory pressure of 50 cm H2O for each SIMV
mechanical breath. Which of the following alarm settings are appropriate for this patient?

I. low exhaled minute ventilation at 8 L/min


II. high inspiratory pressure limit at 65 cm H2O
III. low PEEP/CPAP pressure alarm at 5 cm H2O
A) I, II, and III only
B) II and III only
C) I and III only
D) II only
Ans: B
Response:
The high-pressure-limit alarm should be set about 10–15 cm H2O above the peak airway
pressure of the mechanical controlled breaths. The low-PEEP/CPAP alarm should always be
set 3–5 cm H2O or 20% below the set baseline pressure. A low exhaled volume alarm should
trigger when either the VT or <V>E falls 20% below preset values (6 L/min in this question).

6. Arterial hemoglobin saturation (%HbO2) should be kept above what level in order to
guarantee adequate oxygen delivery to the tissues?
A) 65%
B) 70%
C) 75%
D) 90%
Ans: D
Response:
Normal SaO2 should be more than 95% breathing room air. Levels below 90% indicate the
need for supplemental O2 therapy. Drops in oxyhemoglobin content are usually the result of
cardiac, pulmonary, or combined cardiopulmonary disease. Hb saturation data must always be
interpreted with knowledge of Hb/Hct levels. For example, a patient with an SpO2 of 97% and
severe anemia (Hb < 7 g/dL) is still suffering from hypoxemia, due to reduced blood O2
content.

Page 2
SAN PEDRO COLLEGE COMPETENCY APPRAISAL - OXYGENATION

7. A new medical resident asks for your help in calculating the static lung compliance for an ICU
patient receiving volume-cycled ventilation. The patient has the following settings and
monitoring data:

VT 700 mL
Rate 12/min
Peak pressure 50 cm H2O
Plateau pressure 30 cm H2O
PEEP 10 cm H2O
Mechanical deadspace 100 mL

The patient's static lung compliance is:


A) 18 mL/cm H2O
B) 35 mL/cm H2O
C) 22 mL/cm H2O
D) 26 mL/cm H2O
Ans: B
Response:
Static compliance equals corrected tidal volume divided by the plateau pressure – PEEP. In
this instance, static compliance = 700 ÷ (30 – 10) = 700 ÷ 20 = 35 mL/cm H2O.

8. A doctor wants your recommendation on how to monitor the cardiopulmonary status of a


patient undergoing a bronchoscopy procedure during moderate sedation. You should
recommend the following:
A) pulmonary function testing
B) noninvasive pulse oximetry
C) frequent ABGs via radial puncture
D) transcutaneous PaO2 monitoring
Ans: B
Response:
Besides vital signs, a noninvasive techniques like continuous pulse oximetry is the standard of
care to monitor the cardiopulmonary status of a patient during moderate sedation procedures.
More expensive, invasive techniques such as PFTs or an A-line insertion are required only
when more critical, invasive procedures are being done.

9. A patient is receiving continuous mandatory ventilation (CMV) in the control mode at a rate
of 12 breaths/min. The percent inspiratory time (%I-time) is set at 20%. What is the patient's
inspiratory time?
A) 0.75 sec
B) 1.00 sec
C) 1.25 sec
D) 1.50 sec
Ans: B
Response:
Given the rate (f) and percent inspiratory time (%I-time), the inspiratory time (I-time) can be
computed as: total cycle time × %I-time. Total cycle time = 60/f = 60/12 = 5 sec. I-time = 5
sec × 0.20 = 1.00 sec.

Page 3
SAN PEDRO COLLEGE COMPETENCY APPRAISAL - OXYGENATION

10. A mechanically ventilated patient is being monitored by a capnograph in the ICU. The nurse
calls you STAT to the room and you note that the PETCO2 dropped suddenly from 36 to 0 torr.
All of the following are possible causes of this finding except:
A) ventilator disconnection
B) increased cardiac output
C) obstructed artificial airway
D) cardiac arrest
Ans: B
Response:
Causes of a PETCO2 of zero include (1) a large system leak or disconnection, (2) esophageal
intubation, (3) cardiac arrest, and (4) a totally obstructed/kinked artificial airway. Increased
cardiac output would cause a rise in end-tidal CO2.

11. During a patient-ventilator system check you notice the following airway pressures on an
adult mechanically ventilated patient receiving 5 cm H2O of PEEP:

Time
Measure 0400 0500 0600
Peak pressure (cm H2O) 42 47 53
Plateau pressure (cm H2O) 32 36 42

Knowing that no ventilator setting changes have been made, what is the most likely cause of
these changes?
A) the patient is developing bronchospasm
B) the patient's lungs are becoming more compliant
C) the patient is performing a Valsalva maneuver
D) the patient is developing atelectasis
Ans: D
Response:
The cause of the increased peak inspiratory pressures cannot be increased airway resistance
since the (peak – plateau) pressure difference remains constant at ~10 cm H2O. What is
changing is the (plateau – PEEP) pressure difference, which is increasing due to a gradual rise
in plateau pressures. This indicates a decrease in either lung or thoracic compliance.
Atelectasis, which causes consolidation, decreases lung compliance. Bronchospasm causes an
increase in airway resistance with a widening of the PIP-Pplat difference.

12. Common arterial sites used for percutaneous arterial blood sampling include all of the
following except:
A) carotid
B) radial
C) brachial
D) femoral
Ans: A
Response:
The radial artery is the preferred site for arterial blood sampling because (1) it is near the skin
surface, (2) the ulnar artery provides for good collateral circulation, and (3) the artery is not
near any large veins. Other sites include the brachial, femoral, and dorsalis pedis arteries.
These sites carry greater risk and should be used only by those with training in alternative site
sampling. Carotid arteries are never to be used for arterial puncture.

Page 4
SAN PEDRO COLLEGE COMPETENCY APPRAISAL - OXYGENATION

13. On reviewing the blood gas report on a patient, you note a PaCO2 of 25 torr, a base excess
(BE) of –10 mEq/L, and a pH of 7.35. You would characterize this acid-based abnormality as:
A) compensated metabolic acidosis
B) acute (uncompensated) metabolic acidosis
C) compensated respiratory alkalosis
D) acute (uncompensated) respiratory alkalosis
Ans: A
Response:
First, you should recognize that compensation is occurring with this patient because both the
PaCO2 and BE are abnormally low Second, because the pH is less than 7.40, you can
conclude that the primary problem is the one causing acidosis, in this case the low BE (–10
mEq/L). Therefore, the low PaCO2 must represent compensation for the low BE. Conclusion:
the patient has a compensated metabolic acidosis.

14. A first-year resident has just inserted an indwelling arterial catheter in an ICU patient. A good
indication that the catheter has been successfully inserted in an artery is:
A) a positive Allen test
B) a good blood return
C) ability to flush the line
D) proper blood pressure and waveform
Ans: D
Response:
The Allen test is indicated only to assess for collateral circulation on the radial artery site
before performing the procedure. Blood return and ability to flush the line can also occur if
the catheter has been inserted on a venous vessel. The best indication that the line has been
properly inserted on an artery is the return of arterial blood pressure values accompanied by a
good arterial waveform once the line is connected to the transducer and the monitor.

15. A sample obtained from the distal port of a pulmonary artery catheter has a PO2 of 95 torr and
an Hb oxygen saturation of 97%. Which of the following statements could explain these
results?

I. the catheter balloon remained inflated during sampling


II. the catheter is misplaced in the right ventricle
III. the blood sample was withdrawn too quickly
IV. the patient has an abnormally low cardiac output
A) I, II, and IV only
B) II, III, and IV only
C) I and III only
D) I, II, and III only
Ans: C
Response:
When obtaining a mixed venous sample, if the balloon is not deflated or the sample is
withdrawn too quickly, you may contaminate the venous blood with blood from the
pulmonary capillaries (oxygenated blood). The result is always a falsely high oxygen level.
Rapid flow of IV fluid can also dilute the blood sample and affect oxygen content measures.

Page 5
SAN PEDRO COLLEGE COMPETENCY APPRAISAL - OXYGENATION

16. You need to provide continuous monitoring of the FIO2 for a ventilator that uses a heated
humidifier delivery system. The only analyzer available is a galvanic cell analyzer. Where
should you place the analyzer's sensor?
A) distal to the heated humidifier
B) on the expiratory side of the circuit
C) proximal to the heated humidifier
D) as close to the patient as possible
Ans: C
Response:
Inaccurate readings can occur with electrochemical oxygen analyzers due to either condensed
water vapor or pressure fluctuations. Galvanic cells are particularly sensitive to condensation.
To avoid this problem during continuous use in humidified ventilator circuits, place the
analyzer sensor proximal to any humidification device.

17. All of the following statements regarding a capillary blood gas sample are true, except:
A) it should not be performed to assess oxygenation status
B) it should not be performed in infants less than 72 hrs old
C) it should not be performed in swollen or edematous tissue
D) it should not be performed in the posterior curvature of the heel
Ans: B
Response:
A capillary blood puncture should not be performed in infants less than 24 hours old due to
the immaturity of the capillary bed after birth. Capillary blood sampling also should be
avoided on the heels of infants that have just begun walking; on inflamed, swollen, or
edematous tissue; on cyanotic or poorly perfused areas; and when accurate analysis of the
oxygenation status is needed.

18. After performing a modified Allen test on the left hand of a patient, you note that his palm and
fingers do not become pink for more than 15 seconds after releasing pressure on the ulnar
artery. At this point you should:
A) use the left brachial site for sampling
B) repeat the test on the right hand
C) use the femoral site for sampling
D) go ahead and draw the sample from that site
Ans: B
Response:
The results of the initial Allen test indicate lack of collateral circulation on the left hand. You
should repeat the Allen test on the opposite hand and proceed accordingly. Brachial puncture
should be considered if the Allen test fails to show proper collateral circulation in both radial
arteries.

Page 6
SAN PEDRO COLLEGE COMPETENCY APPRAISAL - OXYGENATION

19. A 20-year-old 65-kg (143-lb.) patient is receiving volume-oriented SIMV with a set rate of 14
breaths/min, a total rate of 14 breaths/min, a VT of 500 mL, and an FIO2 of 0.50. Blood gas
results are as follows:

pH 7.52
PaCO2 26 torr
HCO3 23 mEq/L
PaO2 94 torr

What are the appropriate recommendations for you to make?


A) decrease the SIMV rate
B) add mechanical deadspace
C) decrease the FIO2
D) add pressure support
Ans: A
Response:
The blood gas results suggest normal oxygenation with an uncompensated respiratory
alkalosis due to hyperventilation. The fact that there is no spontaneous ventilation (total rate =
set rate) indicates suppression of the respiratory drive probably due to hypocapnia. In order to
stimulate the patient to breathe spontaneously, you need to eliminate the hypocapnia. On the
SIMV mode this is best done by decreasing the set respiratory rate on the ventilator.

20. You observe a sudden drop in the peak inspiratory pressure when monitoring a patient on
volume-targeted ventilation. Which of the following may explain this change?

I. a defective exhalation valve


II. a burst endotracheal tube cuff
III. a high VT setting
IV. patient disconnection
A) II and IV only
B) III only
C) I, II, and IV only
D) II, III, and IV only
Ans: C
Response:
During volume-controlled ventilation a sudden fall in peak inspiratory pressure can be caused
by any of the following events: (1) improved compliance or resistance; (2) a decrease in either
the volume or flow setting; and (3) patient-ventilator system leaks, such as an ET tube cuff
leak, a malfunctioning exhalation valve, or tubing disconnection/leak.

21. The ER physician asks you to evaluate a trauma patient who was the victim of a house fire. In
order to properly evaluate the cardiopulmonary status of this patient you should perform all of
the following procedures except:
A) auscultation of breath sounds
B) pulse oximetry
C) assessment of sensorium
D) rate, depth, and pattern of breathing
Ans: B
Response:
Due to the patient's involvement in a house fire you should immediately suspect the presence
of carbon monoxide poisoning. Carbon monoxide's high affinity for hemoglobin will cause
profound hypoxemia. Pulse oximetry is unable to measure carbon monoxide saturations and is
contraindicated to assess patients with suspected smoke inhalation. In order to assess for the
presence of carbon monoxide in the blood you must run a CO-oximetry blood gas test.

Page 7
SAN PEDRO COLLEGE COMPETENCY APPRAISAL - OXYGENATION

22. During the assessment of a mechanically ventilated patient in the ICU you notice the
following vital signs:

Heart rate: 118/min


BP: 135/90
Set resp rate: 8/min
Total resp rate: 35/min
Temp: 99.3 °F

A surgical resident has just inserted a right pleural chest tube to drain a significant pleural
effusion. At this point you should recommend that the resident:
A) paralyze the patient
B) reposition the chest tube
C) ask the patient to relax
D) assess for pain
Ans: D
Response:
It is very common for a patient after any invasive procedure to develop surgical pain. Since
the patient is mechanically ventilated and cannot verbally communicate, abnormal vital signs
(usually on the high side of normal) are a common indication of the presence of pain. Asking
the patient to indicate if he or she is in pain and providing for proper pain management is the
right course of action. Paralytics are indicated for patient-ventilator asynchrony. A chest tube
should not be repositioned without assessing a chest X-ray first; asking the patient to relax
will not alleviate the pain.

23. A patient with a size 8 tracheostomy tube is being suctioned by the nurse. While suctioning
the patient you observe several PVCs on the patient's monitor. You should recommend that
the nurse:
A) use a larger suction catheter
B) preoxygenate the patient with 100% O2
C) sedate the patient prior to suction
D) suction less often
Ans: B
Response:
Hypoxia and mechanical stimulation of the myocardium are common causes of premature
ventricular contractions (PVCs). Several respiratory procedures can cause hypoxia and
produce PVCs. Preoxygenating and hyperinflating the patient before suctioning is always
required to avoid hypoxia and myocardial irritability, especially in patients suffering from pre-
existing cardiac diseases. Using a larger catheter, sedating the patient, and suctioning less
often will not prevent the hypoxia and the PVCs caused by the hypoxia itself.

24. A pulmonologist asks you to assess airway responsiveness during a pulmonary function exam.
He wants to rule out asthma from chronic bronchitis in a patient complaining of nocturnal
wheezing. You should consider all of the following tests except:
A) thoracic gas volume
B) graded exercise test
C) histamine challenge test
D) methacholine bronchoprovocation test
Ans: A
Response:
Tests that are indicated to assess for the presence and the degree of airway responsiveness are:
methacholine bronchoprovocation studies, histamine challenge, and exercise challenge tests.
They are also indicated to screen individuals who may be at risk from environmental or
occupational exposure to allergens. Thoracic gas volume (via body plethysmography) does
not assess for airway responsiveness and reactivity.

Page 8
SAN PEDRO COLLEGE COMPETENCY APPRAISAL - OXYGENATION

25. Which of the following tests should you recommend for a patient with suspected hepatitis and
history of alcohol and drug abuse?
A) cardiac enzymes
B) complete blood count
C) liver enzymes
D) partial prothrombin time
Ans: C
Response:
Liver enzymes are indicated to assess suspected liver damage due to infections, alcohol and
drug abuse, among others. Cardiac enzymes are indicated when myocardial damage is
suspected due to an MI or ischemia. Complete blood count and partial prothrombin time are
indicated to evaluate red and white blood cell counts and coagulation status of the blood.

26. You are assisting with the endobronchial intubation of an adult patient in the ICU. You
confirm the presence of bilateral breath sounds. In order to properly assess tracheal tube
placement you should order the following procedure:
A) CAT Scan test
B) laryngoscopy
C) AP chest radiograph
D) bedside spirometry
Ans: C
Response:
Taking a chest X-ray is the most common method used to confirm proper placement of an ET
or tracheostomy tube after endotracheal intubation has been confirmed. On the X-ray, the tube
tip should be positioned about 4 to 6 cm above the carina or between T2 and T4. This position
minimizes the chance of the tube moving down into the mainstem bronchi (endobronchial
intubation) or up into the larynx (extubation).

27. A sudden rise in end-tidal CO2 levels can be caused for all of the following except:
A) sudden release of a tourniquet
B) injection of sodium bicarbonate
C) sudden increase in cardiac output
D) sudden malignant hypothermia
Ans: D
Response:
Causes of a sudden rise in end-tidal CO2 levels include (1) a sudden increase in cardiac
output, (2) a sudden release of a tourniquet, (3) seizure, shivering, pain, and (4) injection of
sodium bicarbonate. Hypothermia would cause a FALL in end-tidal CO2 levels.

Page 9
SAN PEDRO COLLEGE COMPETENCY APPRAISAL - OXYGENATION

28. The following arterial blood gases are obtained on five patients. Which of these patients is
most in need of ventilatory support?

Patient pH PaCO2 HCO3


mm Hg mEq/L
A 7.33 60 33
B 7.36 50 28
C 7.38 56 32
D 7.20 65 23
A) A
B) B
C) C
D) D
Ans: D
Response:
All patients have PCO2s above 50 mm Hg, however, only patient D has a life-threatening
UNCOMPENSATED respiratory acidosis (with a pH of 7.20). The other patients all exhibit
varying degrees of COMPENSATED respiratory acidosis, and are thus suffering from chronic
or acute-on-chronic (as opposed to acute) hypercapnic respiratory failure.

29. While doing a ventilator-patient assessment you observe 'scalloping' of the inspiratory airway
pressure waveform (Paw) occurring after the beginning of inspiration. Which of the following
can explain this finding?
A) improper sensitivity setting
B) presence of auto-PEEP/air-trapping
C) a leak in the patient-ventilator system
D) inadequate inspiratory flow setting
Ans: D
Response:
A drop in pressure ('scalloping') during flow-limited ventilation indicates inadequate
inspiratory flow. Normally, pressure should rise after inspiration begins. To correct this
problem during flow-limited ventilation adjust the inspiratory flow until the “scalloping” of
the pressure waveform disappear.

30. Which of the following characteristics can be found in the sputum of a patient with acute
pulmonary edema?

I. offensive in odor
II. watery
III. pinkish
IV. frothy
A) I and II only
B) II and III only
C) II, III and IV only
D) I, II and III only
Ans: C
Response:
Pulmonary edema secretions are mainly watery, often tainted with blood. The resulting
sputum is watery, pink and frothy. No odor is associated with pulmonary edema unless there
is an underlying infection.

Page 10
SAN PEDRO COLLEGE COMPETENCY APPRAISAL - OXYGENATION

31. Which of the following statements best describes the correct steps for an Allen test before an
arterial puncture is performed?
A) compress both the radial and ulnar arteries then release the radial artery
B) compress the brachial artery only and observe circulation to the hand
C) compress both the radial and ulnar arteries then release both arteries at once
D) compress both the radial and ulnar arteries then release the ulnar artery
Ans: D
Response:
To perform an Allen's test: (1) both the radial and ulnar arteries should be compressed at the
same time while the patient clenches the fist three times for about 5 seconds, (2) the patient
then opens his/her hand, (3) the therapist then release compression on the ulnar artery only
while maintaining pressure on the Radial artery (4) The palmar surface should flush within 5
seconds, prolonged delay before flushing indicates decreased ulnar artery flow. If the radial
artery is unsuitable as a puncture site, the other wrist should be assessed. If both radial arteries
lack collateral circulation, the brachial artery is the second choice, followed by the femoral
artery.

32. All of the following are recommended alarms settings for an adult patient on the SIMV +
pressure support mode except:
A) low pressure 5-10 cm H2O below mechanical peak pressures
B) oxygen analyzer alarm ± 5 or 6% of set FIO2
C) low PEEP/CPAP alarm 3 to 5 cm H2O below set PEEP
D) high PIP alarm 10-15 cm H2O above mechanical breath pressures
Ans: A
Response:
During SIMV mode with pressure support (PSV), peak machine breath pressures are
significantly higher than those of the PSV spontaneous breaths. Hence, the low inspiratory
pressure alarm should be set 5 to 10 cm H2O pressure below the patient's SPONTANEOUS
generated pressures. Otherwise the ventilator will alarm every time the patient generates a
PSV breath.

33. While obtaining the vital signs on a respiratory patient you notice an SpO2 measurement of
70%. If this is an accurate measure of this patient's hemoglobin saturation, what should be the
patient's approximate PaO2?
A) 40 mm Hg
B) 50 mm Hg
C) 60 mm Hg
D) 70 mm Hg
Ans: A
Response:
A good rule of thumb to remember when measuring SpO2 is the "40-50-60"/ "70-80-90" rule.
The first set of numbers is the approximate PaO2 corresponding to the second set of Hb
saturation. In this case, a SpO2 or 70% corresponds to a PaO2 of about 40 mm Hg.

Page 11
SAN PEDRO COLLEGE COMPETENCY APPRAISAL - OXYGENATION

34. You are assisting with the nasal intubation of an adult patient. After positive end-tidal
colorimetry confirmation, you notice that breath sounds are diminished on the left compared
with the right lung. The most likely cause of this finding is:
A) The cuff of the endotracheal tube has been over-inflated
B) The tip of the tube is in the right mainstem bronchus
C) The endotracheal tube has been inserted into the esophagus
D) The tip of the tube is in the left mainstem bronchus
Ans: B
Response:
The right mainstem bronchus is more in line with the trachea than the left, therefore, right
mainstem intubations are more common than left side intubations. If right mainstem
intubation occurs, breath sounds will be significantly decreased on the left side together with
decreased chest expansion on that side. Proper ET tube position must be confirmed with a
chest X-ray and corrected by withdrawing the ET tube until it is 4 to 6 cm above the carina.

35. Possible hazards and complications of arterial blood gas puncture include:

I. arteriospasm
II. nerve damage
III. infection
IV. hemorrhage
A) I, II and III only
B) II and IV only
C) I, II, III and IV
D) II and III only
Ans: C
Response:
Complications of arterial puncture include arteriospasm, air or clotted-blood emboli,
anaphylaxis from local anesthetic, patient or sample contamination, hematoma, hemorrhage,
trauma to the vessel or nerve, arterial occlusion, vasovagal response and pain.

36. You are called to the ER to perform a blood gas puncture in a patient breathing room air
complaining of shortness of breath. Blood gas results are as follows:

pH 7.31
PCO2 51 mm Hg
PaO2 62 mm Hg
HCO3 24 mEq

The most likely cause of this patient's mild hypoxia is:


A) alveolar hypoventilation
B) moderate shunting
C) severe V/Q imbalance
D) metabolic acidosis
Ans: A
Response:
Hypoxemia due to alveolar hypoventilation occurs in the presence of (1) normal a/A ratios,
(2) hypercapnia, and (3) acidemia. This type of hypoxemia can be corrected by increasing the
patient's alveolar ventilation and normalizing the PaCO2.

Page 12
SAN PEDRO COLLEGE COMPETENCY APPRAISAL - OXYGENATION

37. All of the following are indications for arterial blood sampling except:
A) the need to monitor the severity of airway obstruction
B) the need to assess the adequacy of tissue oxygenation
C) the need to evaluate ventilation and acid-base status
D) the need to evaluate a patient's response to therapy
Ans: A
Response:
Some of the indications for arterial blood sampling include the following: (1) the need to
evaluate ventilation (PaCO2), acid-base (pH and PaCO2), and oxygenation (PaO2 and SaO2)
status, and the oxygen-carrying capacity of blood (PaO2, HbO2, total Hb, and
dyshemoglobins); (2) the need to assess the patient's response to therapy and/or diagnostic
tests (e.g., O2 therapy, exercise testing); and (3) the need to monitor severity and progression
of a documented disease process.

38. You are asked to assess a mechanically ventilated patient for his readiness to be weaned off
the ventilator. What minimum maximum inspiratory pressure (MIP; NIF) the patient needs to
achieve before any weaning attempt is made?
A) -15 cm H2O
B) -10 cm H2O
C) -35 cm H2O
D) -20 cm H2O
Ans: D
Response:
For MIP/NIF normal values should be in the range of -80 to -100 cm H2O pressure, actual
values will depend on patient age and sex. When the absolute MIP value is less than 20 to 30
cm H2O, it is unlikely that the patient has sufficient muscle strength to support adequate
spontaneous ventilation and weaning should not be attempted.

39. Which of the following measurements done before and after would best determine the
effectiveness of an aerosolized albuterol (Proventil) treatment administered to an asthmatic
patient?
A) arterial blood gas analysis
B) maximum inspiratory pressure (MIP)
C) peak expiratory flow rate measurement
D) vital capacity
Ans: C
Response:
When monitoring the effectiveness of a bronchodilator treatment pre and post PEFR
measurement should be done. The highest pre/post results should be use to calculate percent
change (%) or percent improvement. A change of 15 to 20% indicates a significant
improvement of the bronchospasm. If no improvement is seen, you may want to recommend a
change on the bronchodilator, the dose or the frequency.

Page 13
SAN PEDRO COLLEGE COMPETENCY APPRAISAL - OXYGENATION

40. Co-oximetry analysis should be performed whenever the following information is needed,
except:
A) actual blood oxygen content
B) levels of abnormal hemoglobins
C) whenever pulse oximetry results (SpO2) need validation
D) evaluation of acid-base status
Ans: D
Response:
Co-oximetry offers a true measurement of the actual levels of abnormal hemoglobin using the
principle of spectrophotometry. Unlike SpO2 and PaO2 procedures, co-oximetry measures the
total hemoglobin in a blood sample and fractions of the total bound to oxygen and other
chemicals. Typically, the sample undergoes analysis for total hemoglobin (THb, g/dL),
percent oxyhemoglobin saturation (HbO2% or SaO2), percent carboxyhemoglobin saturation
(HbCO%), percent methemoglobin saturation (metHb%) , and percent sulfhemoglobin
saturation (SHb%) . In addition, total O2 content (CaO2 in mL/dL) of the sample is calculated
(total Hb x 1.34 x HbO2%).

41. In assessing a patient receiving ventilatory support in the assist-control, volume mode you
hear the high pressure limit alarm sounding and note a decrease in expired volume. No
settings has been changed on the ventilator. Which of the following best explains these
findings?
A) an increase in patient-triggered respiratory frequency
B) the presence of a leak in the patient-ventilator system
C) increased airway resistance or decreased compliance
D) improperly set trigger sensitivity level
Ans: C
Response:
On a volume-cycled ventilator a decrease in expired volume occurring together with an
INCREASED airway pressure (high pressure limit activated) usually indicates an increase in
total impedance, as occurs with either an increase in airway resistance or a decrease in
compliance. Tube kinking or obstruction, patient-ventilator asynchrony or cough would have
a similar effect.

42. Which of the following procedures would be most helpful in identifying the presence of a
pneumothorax?
A) arterial blood gas analysis
B) auscultation
C) chest palpation and percussion
D) chest X-ray
Ans: D
Response:
The only choice that will allow you to "see" the pneumothorax an officially confirm it is a
chest X-ray. Auscultation helps to the initial diagnosis of a pneumothorax but it does not
confirm its presence.

Page 14
SAN PEDRO COLLEGE COMPETENCY APPRAISAL - OXYGENATION

43. Pulse oximetry when use to monitor patient's oxygenation status has the following major
disadvantage:
A) skin burns due to using incompatible probes
B) pressure sores at the measuring site
C) false results leading to incorrect decisions
D) electrical shock at the measuring site
Ans: C
Response:
When using pulse oximetry to asses and monitor patient's response to therapy the greatest
hazard is not understanding the device limitations. Device limitations can result in false-
negative results for hypoxemia and/or false-positive results for normal oxygen levels or
hyperoxia (PaO2 > 100 mm Hg). False readings may then lead to inappropriate treatment of
the patient.

44. You are about to switch a patient from volume-oriented assist/control ventilation to pressure
control ventilation with inverse I:E ratio. The patient will be paralyzed in order to avoid
ventilator dyssynchrony. Which of the following alarms will be most important to the patient
safety in this new mode of ventilation?
A) inverse I:E ratio alarm
B) low pressure/disconnect alarm
C) FIO2 alarm
D) High pressure alarm
Ans: B
Response:
Since unrecognized disconnection of a paralyzed patient from a ventilator can quickly lead to
injury and death, the low pressure/disconnect alarm setting is critical in this scenario. This
alarm will sound when the airway pressure during machine breaths drops below the set level
(5 - 10 cm H2O below the set PIP in PCV). Such a pressure drop signals either a significant
leak or patient disconnection from the ventilator.

45. You note an increase in I:E ratio from 1:3 to 1:2 in a patient receiving CMV in the assist-
control mode via a volume-cycled ventilator. Which of the following changes can explain this
finding?

I. a change in the flow setting


II. a change in patient's breathing rate
III. a change in the FIO2
IV. a change in the tidal volume
A) II and IV only
B) I, II and III only
C) I, II and IV only
D) I, II, III and IV
Ans: C
Response:
In the CMV assist-control mode, the I:E ratio can change whenever any time-related machine
parameter changes (including the preset flow, volume and rate), or when the patient's assisted
breath rate changes altering the total cycle time.

Page 15
SAN PEDRO COLLEGE COMPETENCY APPRAISAL - OXYGENATION

46. An intubated patient is receiving volume control ventilation. The patient's condition has not
changed, but you observe higher peak inspiratory pressures than before. Which of the
following is the most likely cause of this problem?
A) there is a leak in the patient-ventilator system
B) the endotracheal tube cuff is deflated or burst
C) the endotracheal tube is partially obstructed
D) the endotracheal tube is displaced into the pharynx
Ans: C
Response:
A rise in airway pressure during volume control ventilation indicates either (1) an increase in
impedance (increased airway resistance, decreased compliance), or (2) a mechanical
obstruction to gas flow. Applying this knowledge to this case, the most likely cause of the rise
in airway pressure is a partially obstructed ET tube. All the other possibilities would tend to
cause leaks, which would cause the peak inspiratory pressure to fall, not rise.

47. Over a 3 hour period, you note that a patient's plateau pressure has remained stable, but her
peak pressure has been steadily increasing. Which of the following is the best explanation for
this observation?
A) the patient's airway resistance has increased
B) the patient is developing atelectasis
C) the patient's compliance has decreased
D) the patient is developing pulmonary edema
Ans: A
Response:
With a constant flow, differences between the peak and plateau pressure are directly
proportional to the airway resistance. In this case, an increase in the peak - plateau pressure
difference signals an INCREASE in airway resistance. All other choices suggest decreased
compliance, which would affect the plateau – PEEP pressure difference, not the peak - plateau
pressure. Remember "R-C-P": R = Resistance (peak pressure); C = compliance (plateau
pressure); P = PEEP (baseline pressure).

48. A patient receiving long-term positive pressure ventilatory support exhibits a progressive
weight gain and a reduction in the hematocrit. Which of the following is the most likely cause
of this problem?
A) leukocytosis
B) chronic hypoxemia
C) water retention
D) leukocytopenia
Ans: C
Response:
Many patients receiving long-term positive pressure ventilatory support exhibit significant salt
and water retention, as manifested by either a weight gain or failure to lose weight as
anticipated. In addition, such patients typically show a reduction in hematocrit, consistent with
hypervolemia due to water retention.

Page 16
SAN PEDRO COLLEGE COMPETENCY APPRAISAL - OXYGENATION

49. You obtain an SpO2 measurement on a patient of 80%. Assuming this is an accurate measure
of hemoglobin saturation, what is the patient's approximate PaO2?
A) 40 torr
B) 50 torr
C) 60 torr
D) 70 torr
Ans: B
Response:
The rule of thumb used to equate hemoglobin saturation to PO2 is "40-50-60 (PO2) = 70-80-90
(saturation)." So with a SpO2 of 80%, this patient's PO2 would be approximately 50 torr (mm
Hg).

50. At the bedside of a patient receiving volume-control ventilation, you suddenly observe the
simultaneous sounding of the high pressure and low volume alarms. Which of following is the
most likely cause of this problem?
A) a leak in the ET tube cuff
B) a mucous plug in the ET tube
C) ventilator circuit disconnection
D) development of pulmonary edema
Ans: B
Response:
During volume-control ventilation, a high pressure/low volume condition signals an
obstruction (increased impedance). Although either the mucous plug or the development of
pulmonary edema increases impedance, only a plugged ET tube would cause a sudden rise in
airway pressure.

51. During auscultation of a patient's chest, you hear intermittent "bubbling" sounds at the lung
bases. Which of the following chart entries best describe this finding?
A) "bronchial sounds heard at lung bases"
B) "wheezes heard at lung bases"
C) "rhonchi heard at lung bases"
D) "crackles (rales) heard at lung bases"
Ans: D
Response:
The preferred term for short, discontinuous adventitious lung sounds that are crackling or
bubbling in nature is crackles. Many clinicians still use the term rales for these sounds.
Crackles are caused either by movement of excessive secretions in the airways, or by
collapsed airways opening during inspiration.

52. You obtain an SpO2 reading of 90% using an oximeter with an accuracy of ±5%. This could
indicate a PO2 as low as:
A) 70 mm Hg
B) 65 mm Hg
C) 60 mm Hg
D) 55 mm Hg
Ans: D
Response:
With some oximeters' accuracy being only ±5%, an SpO2 reading of 90% could mean an
actual SaO2 of as low as 85%, corresponding to a PO2 of 55 mm Hg or less!

Page 17
SAN PEDRO COLLEGE COMPETENCY APPRAISAL - OXYGENATION

53. You obtain an SpO2 reading of 100% on a patient receiving oxygen via a nonrebreathing
mask. What range of arterial PO2s is possible in this patient?
A) 60-90 mm Hg
B) 90-100 mm Hg
C) 100-200 mm Hg
D) 100-600 mm Hg
Ans: D
Response:
At the high end, pulse oximetry data can be meaningless. Due to the characteristics of the
oxyhemoglobin dissociation curve, a patient with a SpO2 of 100% could have a PaO2
anywhere between about 100 and over 600 mm Hg! It is for this reason that pulse oximeters
should never be use to monitor for hyperoxia (as may be important in neonates).

54. You observe the following on the bedside capnograph display of a patient receiving
ventilatory support. What is your interpretation of this display data?

A) ventilator disconnection
B) hypoventilation
C) rebreathing
D) increased cardiac output
Ans: A
Response:
This capnogram shows disconnection, indicated by the immediate transition from a normal
pattern to PCO2 = 0 torr baseline. Other problems that this display could indicate include
esophageal intubation, ventilator malfunction/failure, or an obstructed /kinked ET tube.

Page 18
SAN PEDRO COLLEGE COMPETENCY APPRAISAL - OXYGENATION

55. You observe the following on the bedside capnograph display of a patient receiving
ventilatory support. What is your interpretation of this display data?

A) ventilator disconnection
B) hyperventilation
C) rebreathing
D) increased cardiac output
Ans: B
Response:
This capnogram shows a progressive reduction in expired CO2, most commonly indicating
hyperventilation. Other problems that this display could indicate include hypothermia/reduced
metabolism, or sedation/neuromuscular paralysis.

56. You observe the following on the bedside capnograph display of a patient receiving
ventilatory support. What is your interpretation of this display data?

A) ventilator disconnection
B) hyperventilation
C) rebreathing
D) increased cardiac output
Ans: C
Response:
This capnogram shows a progressive rise in the baseline end-expired PCO2, most commonly
indicating rebreathing/increased mechanical dead space.

Page 19
SAN PEDRO COLLEGE COMPETENCY APPRAISAL - OXYGENATION

57. While working in the intensive care unit, you notice the following airway pressures on a
mechanically ventilated adult patient receiving 5 cm H2O PEEP:

Time Plateau Peak


Pressure Pressure
cm H2O cm H2O
0900 34 44
1000 38 49
1100 44 55
Knowing that no ventilator setting changes have been made, what is the most likely cause of
these changes?
A) endotracheal suctioning is needed
B) the patient's lungs are becoming more compliant
C) the patient is developing pneumonia
D) the patient is developing bronchospasm
Ans: C
Response:
Peak pressures are progressively increasing, indicating increased impedance to inflation. The
cause of the increased impedance cannot be increased airway resistance, since the PIP-Pplat)
pressure difference remains relatively constant at 10 cm H2O. What is changing is the (Pplat -
PEEP) pressure difference, which is increasing due to a gradual rise in plateau pressures. This
indicates a decrease in either lung or thoracic compliance. Pneumonia causes consolidation,
which decreases lung compliance.

58. Before connecting the sample syringe to an adult's arterial line stopcock, you would:
A) flush the line and stopcock with the heparinized IV solution
B) aspirate the deadspace volume of fluid/blood using a waste syringe
C) align the stopcock off to the patient, on to the flush solution
D) increase the flush solution bag pressure by 20-30 mm Hg
Ans: B
Response:
Before connecting a sample syringe to an adult's arterial line stopcock, you should aspirate the
deadspace volume of fluid/blood using a waste syringe, reposition the stopcock handle to
close off all ports and disconnect and properly discard the waste syringe.

59. After placing a patient on a volume-cycled ventilator in the CMV assist/control mode, you
note that 45 cm H2O pressure are required to deliver the preset tidal volume of 700 ml. What
high pressure limit would you now set for this patient?
A) 50 cm H2O
B) 60 cm H2O
C) 70 cm H2O
D) 80 cm H2O
Ans: B
Response:
The high pressure limit on a volume-cycled should generally be set about 10-15 cm H2O
above the peak pressure needed to deliver the volume. In this case, the high pressure limit
would be set to about 60 cm H2O (45+15).

Page 20
CHAPTER 14
Name: __________________________ Date: _____________

1. While suctioning a patient, you observe an abrupt change in the ECG waveform being
displayed on the cardiac monitor and a drop in SpO2. Which of the following actions should
you take?
A) change to a smaller catheter and repeat the procedure
B) stop suctioning and immediately administer oxygen
C) decrease the amount of negative pressure being used
D) instill 10 mL normal saline directly into the trachea
Ans: B
Response:
If you observe any major change in a patient's heart rate or rhythm or other adverse effect
during suctioning, immediately stop the procedure and give oxygen to the patient, while
providing manual ventilation as necessary.

2. After initiating a bronchodilator aerosol via IPPB to an asthmatic, the patient complains of
fatigue and shortness of breath. On inspection of the patient, you note increased use of
accessory muscles, an increase in heart rate from 90 to 122, and other signs of mild to
moderate distress. Which of the following is the appropriate action at this time?
A) stop the treatment and immediately chart this untoward reaction
B) decrease the flow and have the patient exhale more forcibly
C) increase the pressure limit and get the patient to breathe slower
D) stop the treatment and stay with the patient until she improves
Ans: D
Response:
Depending on the goals of therapy and the condition of the patient, IPPB treatments typically
vary in duration from 10 to 20 minutes. Should you observe any untoward effects, or should
the patient exhibit signs of tiring or other adverse effects, immediately stop the treatment and
stay at the bedside until the patient is stabilized.

3. A patient is receiving appropriate oxygen therapy via a simple mask at 5 L/min but complains
that the mask is confining and interferes with eating. Which of the following oxygen-delivery
devices is a suitable alternative?
A) nasal cannula at 4–5 L/min
B) nasal cannula at 2 L/min
C) nonrebreather mask at 10 L/min
D) a 28% venture mask at 10 L/min
Ans: A
Response:
A simple mask with an input flow of 5 L/min delivers an oxygen concentration of about 35–
40%. A nasal cannula with an input flow of 4–5 L/min would yield an FIO2 of 36–40% and is
therefore the best answer. All other choices would result in a substantial change in the FIO2,
and the choice involving a mask would not address the issues of confinement and difficulty
eating.

Page 1
SAN PEDRO COLLEGE COMPETENCY APPRAISAL - OXYGENATION

4. During an IPPB treatment being given to a 66-year-old COPD patient, you note signs of
further air trapping during exhalation. Which of the following changes in technique should
you consider?

I. instructing the patient to prolong exhalation


II. increasing the inspiratory flow rate
III. increasing the preset pressure limit
IV. retarding exhalation via a PEEP valve
A) I, II, and III only
B) II and IV only
C) II, III, and IV only
D) I, II, and IV only
Ans: D
Response:
Increased expiratory times may be appropriate during IPPB therapy given to patients with
certain forms of COPD. This can be achieved by (1) coaching the patient to prolong the
expiratory phase, (2) increasing the inspiratory flow, or (3) mechanically retarding exhalation
via a PEEP valve placed distal to the expiratory port.

5. After initiating nasal CPAP on a patient, you set the prescribed pressure and turn the flow
generator on. At this point the mask pressure reading is 0 cm H2O. The most likely cause of
this problem is:
A) electrical failure
B) a large system leak
C) patient asynchrony
D) too high a flow
Ans: B
Response:
After a patient is fitted with a nasal CPAP mask and has the prescribed pressure valve
attached, the unit is turned on and checked for leaks. An inability to generate any pressure
indicates that a large system leak is present.

6. You notice that the air-entrainment ports of a venturi mask are occluded by a patient's
bedding. What effect would this have on total flow and FIO2?
A) increase total output flow and decrease FIO2
B) increase both total output flow and FIO2
C) decrease both total flow output and FIO2
D) decrease total output flow and increase FIO2
Ans: D
Response:
Occlusion of the entrainment port would decrease (or eliminate) the entrainment of air.
Because the entrainment of air accounts for a significant amount of output, the total flow of
the device would drop. The decrease in entrained air would also mean that less air would be
available to dilute the oxygen. Hence, the FIO2 would increase.

7. To prevent hypoxemia during suctioning an orally intubated patient, you should do which of
the following?
A) press alarm silence prior to suctioning
B) set vacuum pressure to 100–120 mm Hg before procedure
C) administer 100% oxygen through the ventilator for 1–2 minutes before suctioning
D) maintain the set FIO2 and increase PEEP prior to suctioning
Ans: C
Response:
To prevent hypoxemia during suctioning of an intubated patient, you should hyperinflate and
hyperoxygenate the patient with 100% oxygen via resuscitation bag or through the ventilator.

Page 2
SAN PEDRO COLLEGE COMPETENCY APPRAISAL - OXYGENATION

8. A 48-year-old male is orally intubated, receiving mechanical ventilation with a #7.0


endotracheal tube secured in place. Cuff pressure is found to be 34 cm H2O during peak
inspiration using the minimal occluding volume technique. You should suggest:
A) replacing the endotracheal tube with a smaller size
B) performing a minimal leak technique
C) replacing the endotracheal tube with a larger size
D) deflating and reinflating the cuff
Ans: C
Response:
You should use a “tube exchanger” to change the endotracheal tube or recommend
reintubating the patient with a larger endotracheal tube in order to prevent excessive cuff
pressures and mucosal damage.

9. An adult patient is being mechanically ventilated on the assist/control mode with a heat and
moisture exchanger (HME) being used as humidification. Over the course of 4 hours, you
notice that the peak pressure has increased by 12 cm H2O, but the plateau pressure is
unchanged. It also has become more difficult to suction the patient's tracheal secretions. What
should you recommend at this time?
A) switching the patient to the SIMV mode
B) changing to a heated wick humidifier
C) instilling normal saline solution before suctioning
D) switching to a closed catheter suction system
Ans: B
Response:
An increase in the peak pressure without an increase in the plateau pressure indicates an
increase in airway resistance. Difficulty in suctioning secretions suggests that the increase in
resistance is caused by retained secretions. Since thick or bloody secretions are
contraindications to using a heat and moisture exchanger, you should switch the patient over
to a humidifier capable of providing higher absolute humidity, such as a heated wick-type
device.

10. For a patient who has been weaned from mechanical ventilation but has not been extubated,
full ventilatory support should be reinstituted if which of the following occurs?

I. the blood pressure increases or decreases by 5 mm Hg


II. the pulse increases by 30 beats per minute
III. the respiratory rate increases by 10 or exceeds 30 breaths per minute
IV. the PaCO2 increases by 12 torr
A) II and III only
B) III and IV only
C) I, II, and III only
D) II, III, and IV only
Ans: D
Response:
The significant increase in pulse, respiratory rate, and PaCO2 indicate that the patient is
having significant problems and needs to be placed back on the ventilator. The change in
blood pressure is not significant.

Page 3
SAN PEDRO COLLEGE COMPETENCY APPRAISAL - OXYGENATION

11. After initiating assist-control mechanical ventilation, the inverse I:E ratio alarm is triggered.
Which of the following should be increased to correct this problem?
A) inspiratory flow
B) pressure limit
C) tidal volume
D) respiratory rate
Ans: A
Response:
During routine assist-control ventilation, I:E ratios are normally maintained in the 1:2 to 1:4
range. An inverse I:E ratio alarm indicates that the I-time exceeds the E-time. This is often
due to a peak flow setting that is too low. You should either increase the flow, decrease the
volume, or decrease the respiratory rate to give more time for exhalation.

12. A college student is brought to the emergency department following a motor vehicle accident.
He is tachypneic and tachycardic. He is receiving oxygen via a nonrebreathing mask at 10
L/min. You observe that the mask bag fully deflates on each inspiration. What action should
you take?
A) change to a simple mask at 4 L/min
B) increase the flow rate to 12–15 L/min
C) change to a nasal cannula at 8 L/min
D) continue therapy and monitor SpO2
Ans: B
Response:
Reservoir bag deflation on inspiration is a sign of inadequate flow; increase the flow rate to
prevent room air entrainment and FIO2 reduction.

13. A physician orders oxygen per protocol for an otherwise healthy postoperative patient who
has a PaO2 of 52 torr on room air. After initiating a nasal cannula at 2 L/min and repeating an
arterial blood gas, the patient's PaO2 is now 59 torr. What action should you recommend to the
physician?
A) increase the oxygen liter flow and reassess the patient
B) intubate the patient and institute mechanical ventilation
C) decrease the oxygen liter flow and reassess the patient
D) institute continuous positive airway pressure by mask
Ans: A
Response:
Based on the data given, O2 has been prescribed per protocol to correct arterial hypoxemia on
an otherwise healthy patient. The initial response indicates that this objective is not being fully
met because the PaO2 is still below the normal range of 80–100 torr. In light of this patient's
hypoxemic state and the use of an oxygen therapy protocol, the FIO2 should be gradually
increased and the patient reassessed until the targeted PaO2 or 80–100 torr or an SpO2 of
greater than 92–95% is achieved. When assessment reveals that supplemental O2 is no longer
needed, most protocols permit it to be discontinued or placed on standby.

14. A 70-year-old male patient in the emergency department complains of shortness of breath,
chest pain, and diaphoresis. The physician with a trauma patient in the room next door asks
you to begin an assessment. Your initial action should include all of the following EXCEPT:
A) monitoring their clinical status, including SpO2
B) quickly recommending moderate to high FIO2
C) promptly informing the nurse and physician
D) measuring maximum inspiratory pressure
Ans: D
Response:
All of the above actions except maximum inspiratory pressure (MIP) measurement are
indicated in this emergency situation.

Page 4
SAN PEDRO COLLEGE COMPETENCY APPRAISAL - OXYGENATION

15. You are asked to assess a 16-year-old patient with a severe head cold on 4 L/min of oxygen
via nasal cannula. The patient is alert and awake and is complaining that he can't breathe
through his nose. His pulse oximeter reads 84% saturation. What action should you take?
A) decrease the O2 flow until the patient is more comfortable
B) increase the O2 flow until the SpO2 equals or exceeds 90%
C) change to a simple mask at 5–7 L/min
D) recommend an arterial blood gas before considering any changes
Ans: C
Response:
An SpO2 of 84% is substantially below normal. However, the nasal oxygen therapy is
probably not effective because this patient is probably breathing through his mouth. The
oxygen therapy should therefore be switched to a mask capable of approximating the FIO2 of
nasal cannula at 4 L/min.

16. Soon after a severe asthmatic patient is started on heliox therapy with an 80/20 mix, his SpO2
drops from 94% to 88%. All of the following actions should be considered EXCEPT:
A) checking oxygen analyzer to ensure delivery of prescribed FIO2
B) checking to ensure an adequate seal of the face mask
C) recommending a more oxygen-enriched heliox mixture, such as 70/30
D) reducing the inspiratory flow
Ans: D
Response:
Oxygenation can be a problem for some patients receiving heliox therapy, particularly an
80/20 mixture, which has about the same FIO2 as room air. After checking the seal of the
mask and confirming the FIO2, you may consider recommending a more oxygen-enriched
heliox mixture such as 70/30. In such instances, you may also need to increase the inspiratory
gas flow to prevent collapse of the reservoir bag. However, decreasing the inspiratory flow
would be inappropriate in this situation.

17. A physician prescribes incentive spirometry for a postoperative patient who complains of
dizziness when performing five inspiratory maneuvers in a row. What action should you take?
A) recommend that the therapy be discontinued
B) coach the patient to pause before each maneuver
C) begin CPR on the patient
D) begin oxygen therapy via protocol
Ans: B
Response:
To avoid light-headedness or dizziness associated with hyperventilation, a patient using
incentive spirometry should be coached to perform one or two maneuvers and then to breathe
normally for 30–60 seconds before initiating another maneuver.

Page 5
SAN PEDRO COLLEGE COMPETENCY APPRAISAL - OXYGENATION

18. A toddler is receiving volume-controlled ventilation. To minimize volume loss due to


compression and tubing expansion, which of the following should you select?
A) longer breathing circuit
B) low compliance tubing
C) large diameter tubing
D) smaller-diameter ET tube connector
Ans: B
Response:
Compressed volume” is machine-delivered volume that the patient does not receive due to gas
compression (following Boyle's law) and circuit expansion. Compressed volume is most
critical when delivering small volumes to infants and toddlers. The larger the volume and
more compliant the full gas delivery system is (including ventilator “innards,” humidifier
volume, and delivery tubing), the greater the compressed volume loss will be. Therefore, to
minimize compressed volume loss in infants and toddlers, you should use small-diameter, stiff
delivery tubing and humidifiers and ventilators with low internal volumes.

19. A recently intubated 25-year-old female patient has no breath sounds over the left side of her
chest. Her SpO2 on 40% O2 has dropped from 96% to 90%. At the same time, the peak
inspiratory pressure on the ventilator has increased from 35 cm H2O to 45 cm H2O. You note
a tube length marking of 26 cm at the teeth. What action should you take?
A) increase the FIO2 and the flow rate
B) recommend a stat chest X-ray
C) administrator a bronchodilator
D) retract the ET tube up by 3–4 cm
Ans: D
Response:
The most likely problem is right-sided mainstem bronchus intubation, as suggested by the
clinical findings including decreased ventilation to the left chest and a reduction in PaO2.
Given that the tube is secured at 26 cm, which is quite deep for this patient, the ET tube
should be retracted by about 3 cm.

20. An 88-year-old patient is having a problem holding the small-volume nebulizer mouthpiece
for her bronchodilator treatment. What action should you take?
A) change to a dry powder inhaler (DPI)
B) change to a metered-dose inhaler (MDI)
C) hold the nebulizer for the patient
D) change to an aerosol mask
Ans: D
Response:
Changing to a mask will eliminate the need for the patient to hold the mouthpiece in place,
thus benefiting the patient and saving you time.

Page 6
SAN PEDRO COLLEGE COMPETENCY APPRAISAL - OXYGENATION

21. Each time a patient receiving continuous-flow IMV spontaneously inhales, the pressure drops
6 cm H2O. Which of the following actions would correct this problem?

I. changing to the assist/control mode


II. increasing the flow to the IMV reservoir bag
III. paralyzing the patient
A) II only
B) III only
C) I and II only
D) II and III only
Ans: A
Response:
A drop in pressure below baseline that coincides with the patient's inspiration with a
continuous-flow system indicates inadequate flow. In general, a drop in pressure of 2 cm H2O
is acceptable; any greater decrease in pressure means that system flow should be increased.

22. An infant is placed on continuous-flow CPAP for treatment of hypoxemia. The initial CPAP
level is 6 cm H2O; however, you notice that the pressure falls to 3 cm H2O with each
inspiration. What should you do to correct the problem?
A) tell the patient to relax and breathe more slowly
B) sedate the patient with diazepam or midazolam
C) increase the CPAP to 9 cm H2O to offset the loss
D) increase the system flow to maintain the pressure
Ans: D
Response:
A fall in inspiratory pressure during continuous-flow CPAP suggests that the patient's flow is
exceeding the system flow (insufficient flow). To overcome this problem, increase the flow
until the pressure drop is no more than 1–2 cm H2O.

23. A patient coughs vigorously while receiving postural drainage and percussion on the superior
segment of the left lower lobe. You then note that the sputum is mixed with a large amount of
bright red blood. You should:
A) stop the treatment, stabilize the patient, and inform the physician
B) continue the treatment and make a note of the sputum in the chart
C) give the patient O2 by simple mask and continue the treatment
D) quickly discard the sputum so the patient does not see it and become upset
Ans: A
Response:
Hemoptysis or coughing up blood is one potentially rare but serious side effect of postural
drainage, percussion, and vibration. As with all serious side effects, the treatment should be
stopped, the patient stabilized and monitored closely, and the physician notified.

24. Which of the following represents the primary indication for and approved use of inhaled
nitric oxide?
A) hypoxemia associated with obstructive sleep apnea
B) hypoxemia in neonates with persistent pulmonary hypertension
C) ventilatory failure in premature neonates
D) hypoxemia associated with hyaline membrane disease
Ans: B
Response:
The primary indication and approved use of inhaled nitric oxide is for the treatment of term
and near-term neonates with hypoxemic respiratory failure due to persistent pulmonary
hypertension of the newborn (PPHN).

Page 7
SAN PEDRO COLLEGE COMPETENCY APPRAISAL - OXYGENATION

25. A patient is receiving IPPB therapy for atelectasis with a set pressure of 25 cm H2O. During
therapy, the patient's pulse becomes thready and the blood pressure drops from 120/80 mm Hg
to 90/50 mm Hg. You should:
A) decrease the IPPB pressure to 10 cm H2O and monitor the patient
B) increase the IPPB pressure to 45 cm H2O and continue the treatment
C) discontinue the treatment and notify the physician
D) change the treatment to intermittent CPAP with 10 cm H2O PEEP
Ans: C
Response:
Positive pressure can adversely affect the hemodynamic status of a patient. When that occurs,
you should stop the treatment, stabilize the patient, and then notify and suggest alternative
therapies to the physician.

26. Prior to beginning an adrenergic aerosol bronchodilator treatment on an adult patient, you
record a resting heart rate of 132 beats/min. Which of the following is the correct action in this
case?
A) double the drug diluent and prolong the administration time
B) have the patient self-administer the aerosol treatment
C) postpone therapy until you are able to contact the ordering physician
D) use half the standard dosage listed in the package insert
Ans: C
Response:
In general, an adrenergic bronchodilator should not be given if a patient is already
experiencing tachycardia. Moreover, once a treatment is begun with an adrenergic drug, it
should be terminated if the pulse rate increases more than about 20 beats/min.

27. Following administration of a bland aerosol treatment, auscultation reveals rhonchi


throughout middle and upper lung fields. You should:
A) encourage the patient to cough
B) recommend administration of a bronchodilator
C) recommend discontinuation of therapy
D) discontinue the treatment and administer oxygen
Ans: A
Response:
Rhonchi are caused by air moving through secretions in the large airways. Patients with
rhonchi after receiving a bland aerosol should be encouraged to cough to clear the loose
secretions.

28. A patient was recently changed from assist/control mode to pressure support ventilation, and
the high respiratory rate alarm, which is set at 25, is sounding because the patient is breathing
between 25 and 28 breaths per minute. What change should you make to the ventilator
alarms?
A) increase the high respiratory rate alarm to 50
B) increase the high pressure alarm to 50 cm H2O
C) increase the high respiratory rate alarm to 30–35
D) stop weaning the patient immediately
Ans: C
Response:
When weaning on pressure support, a modest increase in respiratory rate is common and
generally should be tolerated up to a maximum of 30–35 breaths per minute. In this instance,
the high respiratory rate alarm should be increased to 30–35 breaths per minute.

Page 8
SAN PEDRO COLLEGE COMPETENCY APPRAISAL - OXYGENATION

29. While monitoring a patient during a T-tube weaning trial, you note increased patient agitation,
increased heart rate (from 85 to 110 beats/min) and respiratory rate (from 15 to 34
breaths/min), and PVCs increasing to an average of 4 per minute. You should:
A) encourage the patient to relax and continue careful monitoring
B) request that the patient be given a stat bolus of lidocaine
C) reconnect the patient to the ventilator with prior settings
D) request that the patient be given a strong sedative/hypnotic
Ans: C
Response:
Development of severe agitation, tachypnea, tachycardia, bradycardia, hypotension,
asynchronous or paradoxical breathing, angina, or cardiac arrhythmias during a T-tube
weaning trial usually indicate that the patient is not tolerating it well and that ventilatory
support should be reinstituted.

30. After a patient has been receiving bland aerosol therapy via an ultrasonic nebulizer for 5
minutes, she begins to wheeze. What should you do at this time?
A) recommend that the patient be given IV epinephrine
B) stop the treatment, monitor the patient, and notify the doctor
C) add 0.5 mL (2.5 mg) of albuterol to the nebulizer solution
D) switch the ultrasonic nebulizer source gas to 100% O2
Ans: B
Response:
A patient who starts wheezing during an ultrasonic nebulizer treatment is likely developing a
bronchospasm in response to the high-density aerosol (an adverse reaction). In general, when
a patient experiences an adverse reaction to therapy, you should stop the therapy, monitor the
patient closely, and then contact the physician.

31. When administering a treatment with a Bennett AP-5 IPPB machine, you notice that after
triggering, airway pressures first drops into the negative range, then moves into the positive
range later during inspiration, just before the machine cycles off. What would you do to
correct this problem?
A) increase the pressure setting
B) increase the flow rate
C) push in the air-mix knob
D) tell the patient to inhale faster
Ans: B
Response:
During any form of positive pressure breathing such as IPPB or CPAP, a drop in pressure
BELOW baseline during inspiration (scalloping' of the pressure waveform) indicates
insufficient flow. The inspiratory flow should be increased in this case.

32. An infant receiving a moderate concentration of O2 in an Isolette must be removed in order to


insert an umbilical artery catheter. Which of the following devices would you recommend to
provide supplemental O2 to the infant during this procedure?
A) catheter
B) simple mask
C) cannula
D) oxyhood
Ans: B
Response:
A simple mask is best choice for delivering short-term O2 therapy to infants during transport
or when they must be removed from an O2 enclosure for special procedures. However, masks
are usually not well tolerated by infants over the long-term and can easily result in pressure
necrosis of their delicate skin.

Page 9
SAN PEDRO COLLEGE COMPETENCY APPRAISAL - OXYGENATION

33. Immediately following the initiation of mechanical ventilation, an inverse I:E ratio is detected
and not intended as part of the therapy. Which of the following should be done to correct this
problem?
A) increase the inspiratory flow
B) increase the pressure limit
C) increase the tidal volume
D) increase the respiratory rate
Ans: A
Response:
An inverse I:E ratio indicates that the inspiratory time is exceeding the expiratory time.
Though occasionally intentional to help improve oxygenation, it is often an abnormal
situation. The most common cause of this alarm condition is an excessively long I-time, due
to inadequate inspiratory flow. By increasing the inspiratory flow, you can alleviate this
problem.

34. If the reservoir bag of a nonrebreather mask collapses completely during inspiration, you
should do which of the following?
A) use an entrainment mask
B) increase the flow to the bag
C) remove the valve between the reservoir bag and the mask
D) loosen the mask around the patient's face
Ans: B
Response:
A nonrebreather mask is a reservoir device which only works properly if the reservoir does
not completely collapse during a deep breath. To ensure adequate flow to a reservoir device,
the therapist must assure that the reservoir remains partially inflated during inspiration. In this
case, you should increase the flow to the bag.

35. The high pressure limit and low exhaled volume alarms on an adult male patient receiving
volume ventilation suddenly activates. Auscultation reveals no breath sounds on the left side.
The oral endotracheal tube is observed to be at the 28 cm mark at the patient's lips. Which of
the following should you do?
A) recommend insertion of a left chest tube
B) measure the patient's static lung compliance
C) add 10 mL of air to the endotracheal tube cuff
D) withdraw the tube until breath sounds are heard bilaterally
Ans: D
Response:
An important clue is the that the tube appears to be deep, being secured at the 28 cm mark,
rather than the typical level of 22-23 to the lip for an average adult. In addition, simultaneous
sounding of the high pressure and low exhaled volume alarms combined with loss of
ventilation to the left lung strongly suggest a right mainstem intubation. Withdrawing the tube
until bilateral breath sounds are heard, then securing it is the best action. From there, you
should recommend a chest x-ray.

Page 10
SAN PEDRO COLLEGE COMPETENCY APPRAISAL - OXYGENATION

36. Before giving a breathing treatment with racemic epinephrine for laryngeal edema, you note
the patient's heart rate is 85/min and respiratory rate is 16/min. Five minutes into the
treatment, his pulse rate climbs to 125/min and his respiratory rate rises to 22/min. What is the
best action in this case?
A) continue the treatment as ordered to completion
B) use albuterol (Proventil) instead of racemic epinephrine and continue the treatment
C) stop the treatment, monitor the patient, inform the physician and ask for further
guidance
D) stop the treatment; ask the nurse to give the remaining medication it intravenously
Ans: C
Response:
The significant rise in heart rate and respiratory rate suggests that the patient is experiencing
an adverse reaction to the racemic epinephrine. As a result, the therapy should be stopped, the
patient closely monitored and the physician contacted.

37. A patient in the ICU is receiving an aerosol bronchodilator therapy. The treatments have been
administered with a mouthpiece but since the last treatment, the patient has become somewhat
lethargic. You should
A) continue the treatment using a mouthpiece
B) recommend discontinuing the therapy
C) change to an aerosol mask
D) administer the treatment using compressed air
Ans: C
Response:
A lethargic patient will often not be able to hold the nebulizer and utilize the mouthpiece. In
this case, a face mask is indicated.

38. A patient who is receiving postural drainage says that she becomes short of breath when she
lies in a head-down position. Your most appropriate action would be to:
A) administer oxygen prior to treatments
B) modify therapy positions according to the patient's tolerance
C) perform drainage no closer than 15 minutes before or after meals
D) discontinue postural drainage
Ans: B
Response:
Since therapy can not work if the patient is not able to tolerate it, you should consider
modifying any therapy to help assure patient compliance.

39. An alert patient with emphysema and an elevated CO2 level is given 50% O2 by an air-
entrainment mask. One hour later the nurse calls you to evaluate the patient who is now very
lethargic. Which of the following is the most likely cause of this?
A) muscle fatigue
B) cerebral hypoxia
C) hypotension
D) O2-induced hypoventilation
Ans: D
Response:
Many patients with severe COPD are chronic CO2 retainers, characterized on blood gas
analysis as a fully compensated respiratory acidosis. Because their stimulus to breathe is low
blood oxygen (the 'hypoxic-drive'), a PaO2 in excess of 60 to 70 mm Hg can cause some of
these patients to hypoventilate. For these reasons, many clinicians recommend avoiding FIO2s
greater than 0.30 to 0.40 in these patients. However, it is the PaO2, not the FIO2 which
depresses the 'hypoxic drive' in such patients so you must NEVER deprive O2 from any
patient in need.

Page 11
SAN PEDRO COLLEGE COMPETENCY APPRAISAL - OXYGENATION

40. For adults with otherwise normal lungs who are receiving ventilatory support in the CMV
control or assist/control modes, inspiratory flow should be set or modified so the I:E ratio is
between:
A) 1:1 - 1:2
B) 1:2 - 1:3
C) 1:5 - 1:6
D) 2:1 - 1:1
Ans: B
Response:
In the CMV control or assist/control modes, inspiratory flow should generally be set to
provide an I:E ratio of between 1:2 - 1:3. In patients with normal lungs, this will provide
sufficient time for full exhalation and a perceptible pause before the next breath, without
compromising cardiovascular function. However, it is often necessary to modify this approach
for patients with expiratory airway obstruction (COPD, asthma) to achieve lower ratios (1:4 or
1:5) to prevent air-trapping and auto-PEEP.

41. For a patient with a history of COPD, the ventilator rate and inspiratory flow during CMV in
the assist/control mode should be set or modified to provide an I:E ratio no more than:
A) 1:1
B) 1:2
C) 1:4
D) 2:1
Ans: C
Response:
In many patients receiving CMV with chronic airway obstruction due to conditions such as
COPD, I:E ratios of 1:4, 1:5 or even less are needed to provide sufficient time for exhalation
and to prevent air trapping/auto-PEEP.

42. For a critically ill infant or child's, monitoring during application of postural drainage,
percussion and vibration should include:
A) pulse oximetry
B) arterial blood gases
C) capnography
D) peak flow
Ans: A
Response:
Given the inherent instability of the critically ill young pediatric or infant patients, the
traditional assessment of the respiratory rate, color, pulse, and blood pressure before, during,
and after chest physical therapy should be supplemented with either pulse oximetry or
transcutaneous PO2 monitoring.

43. The SpO2 of an infant receiving CPAP via nasal prongs with an FIO2 of 0.40 drops
substantially during frequent episodes of crying. Which of the following actions would you
recommend in this situation?
A) switch to CPAP via an endotracheal tube
B) increase the CPAP level by 2-4 cm H2O and increase the FIO2 to 60%
C) continue CPAP but also place the infant in an oxyhood with 40% O2
D) increase the oxygen concentration to 50%
Ans: C
Response:
A crying infant is likely to inspire large volumes of air through the mouth. For this reason,
infants receiving nasal CPAP should also be enclosed in an oxyhood delivering an FIO2 equal
to that being received via the nasal route. This will ensure a stable FIO2 regardless of the
infant's breathing pattern.

Page 12
SAN PEDRO COLLEGE COMPETENCY APPRAISAL - OXYGENATION

44. Following extubation, a patient is placed on a heated nebulizer with an FIO2 of 21% but
begins to develop mild stridor. Which of the following specific approaches would you
recommend?
A) switch to cool mist therapy via jet nebulizer
B) increase the heat setting on the nebulizer to high
C) switch to oxygen therapy via nasal cannula and bubble humidifier
D) oxygen therapy via 'venti-mask' and bubble humidifier
Ans: A
Response:
If humidity or aerosol therapy is indicated, most clinicians recommend a cool (as opposed to
heated) mist following extubation. This is because a heated mist may worsen post-extubation
mucosal swelling, thereby worsening airway obstruction.

45. A patient receiving a bland ultrasonic nebulizer treatment begins to wheeze. Which of the
following should you do at this time?
A) Discontinue therapy, monitor patient and notify the physician
B) Decrease the nebulizer output
C) Add oxygen to the nebulizer circuit
D) Continue therapy and reassure the patient
Ans: A
Response:
Even bland aerosol therapy, particularly the high output of an ultrasonic nebulizer, may
irritate the airways and cause wheezing. This could be occurring in this instance, and as a
result the treatment should be discontinued, the patient monitored and the physician notified.

46. A patient with chronic bronchitis is receiving O2 therapy via a 24% air-entrainment mask.
Over the last day, you note an increase in the volume and thickness of his secretions. Which
of the following actions would you recommend?
A) providing an air-generated water aerosol via a collar adapter
B) switching to a nasal cannula at 1 L/min flow
C) switching to a 28% air entrainment (venturi) mask
D) attaching the air-entrainment mask to a bubble humidifier
Ans: A
Response:
The high flows produced by low concentration air entrainment mask can be quite drying. In
cases such as this one, additional humidity/aerosol should be provided. While a simple bubble
humidifier can saturate the input O2, it does not add to the water vapor content of the entrained
air. In order to do so, you can add aerosol to the entrained air through a standard 22 mm
collar, using either an air-powered jet nebulizer or ultrasonic.

Page 13
SAN PEDRO COLLEGE COMPETENCY APPRAISAL - OXYGENATION

47. For which of the following patients would you consider modifying any head-down positions
used for postural drainage?

I. a patient with unstable blood pressure


II. a patient with a cerebrovascular disorder
III. a patient with systemic hypertension
IV. a patient with orthopnea
A) II and IV only
B) I, II, III and IV
C) I and IV only
D) II, III and IV only
Ans: B
Response:
As part of patient assessment prior to initiating postural drainage, you should evaluate the
potential need for modification of the position(s) chosen. Modification of head-down positions
may be needed in patients with unstable cardiovascular status, hypertension, cerebrovascular
disorders, and orthopnea.

48. Soon after initiating postural drainage in a Trendelenburg position, the patient develops a
vigorous and productive cough. Which of the following actions would be appropriate at this
time?
A) maintain the drainage position while carefully watching the patient
B) ask the patient to swallow the sputum until you switch positions
C) stop the treatment at once and report the incident to the nurse
D) move the patient to the sitting position until the cough subsides
Ans: D
Response:
Postural drainage does not always result in the immediate production of secretions. More
often, secretions are mobilized toward the trachea for easier removal with a less forceful
cough effort. However, if drainage is effective at once, it may precipitate vigorous coughing
during the therapy. If this occurs, have the patient sit up immediately until the cough subsides.

49. A patient recovering from recent abdominal surgery is having difficulty developing an
effective cough. Which of the following actions would you recommend to help this patient
generate a more effective cough?

I. coordinating coughing with pain medication


II. staging expiration or a huffing cough
III. "splinting" the operative site
IV. applying manual chest compression
A) II and IV only
B) I, II and III only
C) III and IV only
D) II, III and IV only
Ans: B
Response:
For many postoperative patients, coughing effectiveness can be enhanced by coordinating
therapy sessions with pain medications and assisting the patient in "splinting" the operative
site. 'Staging' the expiratory effort or using a huffing cough may be helpful in these patients.

Page 14
SAN PEDRO COLLEGE COMPETENCY APPRAISAL - OXYGENATION

50. A nurse indicates a neuromuscular patient under her care cannot develop a good cough.
Which of the following would you recommend as best able to aid this patient in clearing
secretions?
A) combining mechanical insufflation/exsufflation with suctioning
B) applying forward waist flexion to aid expiratory flow
C) using long expiratory bursts
D) employing the forced expiration technique (FET)
Ans: A
Response:
In neuromuscular patients who cannot generate a forceful expulsion, mechanical
insufflation/exsufflation is a viable option. In this technique, the device provides a positive
pressure breath, followed immediately by application of negative airway pressure. This can
help increase the volume and velocity of expired air, and may help move secretions toward the
trachea, where they can be suctioned out.

51. An adult is being mechanically ventilated through an 8 mm ID endotracheal tube. The tube is
taped at the 18 cm mark. A significant leak is heard every time a breath is delivered and both
the low pressure and low tidal volume alarms are sounding. The addition of 10 mL of air does
not help and the pilot balloon is firm. Which of the following is the most likely reason for this
finding?
A) The cuff is at or above the patient's vocal cords
B) The patient has a right mainstem intubation
C) The patient has a bronchopleural fistula
D) The cuff requires more air
Ans: A
Response:
In a normal adult male, a properly positioned oral ET tube will be secured at about the 22-23
cm mark at the lips. At 18 cm, the ET tube could be too high, possibly at or above the vocal
cords. This would account for the leak that is evident in the volume loss and concurrent
alarms and not reversed with more air in the cuff. An initial attempt may be made to deflate
the cuff and gently advance the ET tube to about the 23 cm mark. If unsuccessful, this patient
may need to be reintubated.

52. A 80 kg (176 lb) adult male is being ventilated with a Puritan Bennett 840 ventilator following
a motor vehicle accident. Pertinent data are below.

Mode Assist/control
FIO2 0.40
Set Rate 12/min
Actual Rate 30/min
VT 700 ml
Flow 60 L/min
I:E ratio 1:1

To improve the I:E ratio, you should


A) add 10 cm H2O PEEP
B) increase the rate setting
C) add inspiratory hold
D) increase the flow
Ans: D
Response:
To improve the I:E ratio, you should increase the inspiratory flow. This will deliver the tidal
volume quicker and allow more time for exhalation. Another option, not provided, would be
to decrease the patient's assisted rate, perhaps by using a sedative/hypnotic (unless head injury
is a problem).

Page 15
SAN PEDRO COLLEGE COMPETENCY APPRAISAL - OXYGENATION

53. A patient with a head injury is being hyperventilated on a ventilator. Although a heat moisture
exchanger (HME) is being used, the patient develops thick secretions which are difficult to
suction. Which of the following should you suggest?
A) Instill acetylcysteine (Mucomyst)
B) Increase I.V. fluids
C) Start postural drainage and percussion
D) Replace the exchanger with a heated humidifier
Ans: D
Response:
While increasing IV fluids will eventually help the patient, an immediate remedy is to use a
heated humidifier. This will decrease the insensible water loss and keep the secretions moist.

54. You are instructing a preoperative patient to perform sustained maximum inspiratory
maneuvers. The patient has a history of asthma and is unable to achieve enough inspiratory
flow to obtain the desired volume. The therapist should
A) perform postural drainage and percussion
B) change to a volume-oriented device
C) coach the patient to breathe deeper
D) coach the patient to inhale faster
Ans: B
Response:
Since a volume-oriented incentive spirometer does not require a threshold flow to operate, it is
recommended for patients who can not generate adequate flows (such as some asthmatic
patients).

55. A patient is receiving high frequency jet ventilation with a frequency of 150 breaths/min and
the I:E ratio is set at 1:1. Sequential chest x-rays indicate that the patient is air trapping. The
appropriate action for you to take would be to
A) change the I:E ratio to 1:2.5
B) increase the frequency to 200 breaths/min
C) ensure that the delivered volume is at least 10 - 12 mL/kg
D) adjust the amount of driving pressure to 5 psig
Ans: A
Response:
The presence of air-trapping/auto-PEEP is often caused by insufficient exhalation time. By
changing the I:E ratio from 1:1 to 1:2.5, you will give the patient more time to exhale before
the next breath is delivered and thus decrease the amount of trapped gas.

56. In accordance with many ventilator weaning protocols, all of the following indications would
suggest that the adult patient is ready to be weaned from a mechanical ventilator, EXCEPT:
A) spontaneous rate of 19/min
B) spontaneous VT of 200 mL
C) vital capacity of 1.5 L
D) resting minute volume of 8.5 L/min
Ans: B
Response:
Weaning protocols permit you to begin weaning once selected criteria are met and the order
to “wean per protocol” is written by the physician. However, a spontaneous tidal volume of
200 is too small and not consistent with the ability of the patient to wean since that volume is
only a few mLs above anatomical deadspace.

Page 16
SAN PEDRO COLLEGE COMPETENCY APPRAISAL - OXYGENATION

57. You are called to the ICU to check on a patient being supported by a volume-cycled
ventilator. You note that both the low tidal volume and high pressure limit alarm are sounding
on each inspiration and that the patient's SPO2 is 85% and their heart rate is 148. You first
action should be to:
A) call the attending physician for further patient information
B) increase the preset tidal volume to 1.5x the current setting
C) disconnect patient and provide manual ventilation with 100% O2
D) check the full patient-ventilator system for malfunctions
Ans: C
Response:
Whenever a potential problem arises during ventilatory support, the first priority is to assure
adequate patient ventilation and oxygenation. If there is any doubt about the proper function
of the support system, particularly if the patient shows signs of distress, the patient should be
disconnected from the device and ventilated with 100% oxygen via a manual resuscitator until
the problem is resolved.

58. A ventilator low tidal volume alarm and high pressure alarms just started sounding but the
patient appears stable and their SPO2 is 98%. Your first action should be to:
A) disconnect the patient and provide 100% FIO2
B) attempt to pass a suction catheter to determine airway patency
C) give an albuterol treatment
D) call a “code blue”
Ans: B
Response:
The combination of a low tidal volume and high pressure alarm suggest an acute airway
obstruction. Often this is caused by mucus or an agitated patient who may be biting the ET
tube. For patient in no immediate distress, you should initially attempt to pass a suction
catheter and suction the airway, while assessing other possible causes.

59. During noninvasive ventilation (NPPV) via nasal mask, the patient mouth is wide open and
the manometer fails to reach the set inspiratory pressure. What modification should the
therapist make to resolve this problem.
A) consider adding a chin strap or using a full-facemask.
B) decrease the flow
C) increase the FIO2
D) switch to a nasal catheter
Ans: A
Response:
Many ventilators used for NPPV can compensate for small leaks quite well. However,
massive leaks through the mouth are indicated by the inability to reach the set pressure and
can generally only be resolved by adding a chin strap or changing a full face mask.

60. During postural drainage of the left lower lobe, a patient complains of acute chest pain. Which
of the following should you do?
A) give the patient supplemental oxygen
B) continue the treatment with the bed flat
C) ask the nurse to administer pain medication
D) discontinue the treatment and monitor the patient
Ans: D
Response:
The occurrence of a major unexpected and potentially harmful hazard effect such as acute
chest pain is reason enough to discontinue any treatment. According to the AARC, when a
patient exhibits pain during postural drainage, you should stop therapy, exercise care in
moving the patient, and consult the ordering physician.

Page 17
SAN PEDRO COLLEGE COMPETENCY APPRAISAL - OXYGENATION

61. A 45-year-old patient with asthma is prescribed 0.3 mL of albuterol (Proventil) in 3 mL


normal saline via small volume nebulizer. Before initiating therapy, you note from chart
review that the patient is severely hypertensive and has been experiencing episodes of
superventricular tachycardia. You should do which of the following?
A) administer the treatment as ordered
B) postpone the treatment and consult the physician
C) dilute the albuterol with extra normal saline
D) decrease the amount of albuterol administered
Ans: B
Response:
Albuterol is a beta-adrenergic drug that can increase heart rate and blood pressure. For this
reason, hypertension and tachycardia are contraindications to its administration. Whenever a
contraindication exists to drug administration, you normally should postpone the treatment
and consult with the ordering physician.

62. A patient is admitted to the Emergency Department comatose with suspected smoke
inhalation. After confirming airway patency, which of the following should you do FIRST?
A) measure the SpO2
B) initiate 100% oxygen
C) obtain an arterial blood gas
D) request a stat chest X-ray
Ans: B
Response:
In cases of suspected smoke inhalation, it is likely that the patient is suffering from hypoxemia
due to a high concentration of carboxyhemoglobin (COHb) in his blood, which requires
immediate treatment. Unless a hyperbaric chamber is available, the only way to treat carbon
monoxide poisoning is to provide supplementary O2, ideally 100%. Note also that standard
pulse oximetry cannot reveal the presence of COHb and the SaO2 reported by blood gas
analysis is simply a calculated value based on the PO2 and pH. To measure the actual amount
of COHb in the patient's blood (after administering 100% O2), you would need to perform co-
oximetry.

63. A 30 kg (66 lb) child is being mechanically ventilated in the SIMV mode. The following data
are available:

Ventilator Settings Blood Gases


FIO2 0.40 pH 7.38
Mandatory rate 18 PaCO2 42 torr
Total rate 23 PaO2 130 torr
VT 350 mL HCO3 23 mEq/L
PEEP 12 cm H2O BE 0 mEq/L

Based on these results, which of the following should you do?


A) Decrease tidal volume
B) Reduce the PEEP
C) Decrease the rate
D) Lower the FIO2
Ans: B
Response:
The blood gas indicates normal acid-base balance, so you don't want to recommend changing
any ventilatory parameters. In terms of oxygenation, the PaO2 is higher than needed to
maintain adequate Hb saturation. To lower the PaO2, you could either lower the FIO2 or
reduce the PEEP. Since a PEEP of 12 cm H2O is potentially more harmful that an FIO2 of
0.40, you should first reduce the PEEP.

Page 18
SAN PEDRO COLLEGE COMPETENCY APPRAISAL - OXYGENATION

64. At the bedside of a patient receiving volume-control ventilation, you observe the simultaneous
sounding of the low pressure and low volume alarms. Which of following is the most likely
cause of this problem?
A) a leak in the ET tube cuff
B) a mucous plug in the ET tube
C) right mainstem intubation
D) development of pulmonary edema
Ans: A
Response:
The development of a mucous plug, right mainstem intubation, or pulmonary edema all would
cause a high pressure/low volume condition. During volume-control ventilation, a low
pressure/low volume condition signals a leak, such as that which would occur with a blown
ET tube cuff.

65. You observe the following flow vs. time display on a patient receiving volume-control
ventilation. Which of the following actions would be appropriate?

A) decrease the inspiratory flow


B) increase the expiratory time
C) decrease the PEEP level
D) increase the tidal volume
Ans: B
Response:
This display indicates a failure of the expiratory flow to return to the 0 baseline, likely
resulting in auto-PEEP. Once its presence is confirmed, auto-PEEP needs to be managed
according to its cause. If the cause is insufficient exhalation time, you should either increase
the inspiratory flow, decrease the inspiratory time or increase the expiratory time. If auto-
PEEP is associated with excessive airway resistance, you can often reduce or eliminate it with
by suctioning or administering bronchodilators.

Page 19
SAN PEDRO COLLEGE COMPETENCY APPRAISAL - OXYGENATION

66. You observe the following pressure-volume loop display on a patient receiving volume-
control ventilation. Which of the following actions would be appropriate?

A) decrease the delivered volume


B) increase the inspiratory flow
C) decrease the I:E ratio
D) increase the PEEP level
Ans: A
Response:
This pressure-volume loop exhibits significant flattening beyond its upper inflection point,
indicating overdistention of the lungs. Due to its resemblance to a bird, this is sometimes
called a “beaked” pressure-volume loop. When you observe this problem, you generally can
resolve it by either reducing the volume (in volume ventilation) or the pressure setting

67. A physician orders an 70% He/30% O2 mixture to be delivered to a patient having an acute
asthmatic attack. Assuming use of a 70% He/30% O2 bulk cylinder, which of the following
systems would be most appropriate to deliver this mixture?
A) nebulizer set at 100% oxygen + aerosol mask
B) tight-fitting nonrebreathing mask at 10 L/min
C) simple oxygen mask at 10 L/min oxygen
D) nasal cannula at 10 L/min
Ans: B
Response:
Due to its high diffusibility, helium mixtures need to be administered via closed systems or at
least those with minimal leaks. Because a tight-fitting nonrebreathing mask with competent
valving can deliver close to 100% source gas, it approximates the characteristics of a closed
delivery system. For this reason, a nonrebreathing mask is the first choice for short-term
administration of helium-oxygen mixtures to spontaneously breathing patients.

Page 20
SAN PEDRO COLLEGE COMPETENCY APPRAISAL - OXYGENATION

68. An asthmatic patient started on a 70% He/30% O2 mixture via a pneumatically powered
volume ventilator through a cuffed ET tube exhibits worsening hypercapnia despite higher
than normal volume settings. Which of the following actions should you consider?
A) create a larger cuff leak at peak inspiration to allow the extra helium to escape
B) switch to an electrically-powered, time-cycled pressure-limited ventilator
C) switch to a 80% He/20% O2 mixture and obtain a new blood gas sample
D) use density conversion factors to adjust the ventilator volume and flow settings
Ans: D
Response:
The performance of ventilators delivering heliox varies significantly by model. In general,
only ventilators approved by the FDA for delivering heliox should be used. Even with
approved ventilators, you may need to add special modules or use conversion factors to adjust
volume and/or flow settings.

69. The recommended initial dose of inhaled nitric oxide (INO) is:
A) 2 ppm
B) 20 ppm
C) 200 ppm
D) 2%
Ans: B
Response:
INO is approved for the treatment of term and near-term (>34 weeks) neonates with
hypoxemic respiratory failure associated with persistent pulmonary hypertension of the
newborn (PPHN). The recommended initial dose of inhaled nitric oxide is 20 parts per
million, ppm but can often be quickly reduced to 5-6 ppm.

70. Which of the following gas concentrations must be monitored during inhaled nitric oxide
(NO) therapy?

I. NO
II. NO2
III. O2
A) I only
B) I and III only
C) II and II only
D) I, II and III
Ans: D
Response:
When nitric oxide (NO) comes in contact with oxygen, the toxic gas nitrogen dioxide (NO2) is
produced. NO2 levels should not exceed 2-3 ppm. For this reasons, Nitric oxide therapy
systems must continuously monitor not only NO and O2 levels, but also and NO2 levels.
Alarms are used to detect excessive levels of nitric oxide and NO2, or undesired changes in
FIO2.

Page 21
SAN PEDRO COLLEGE COMPETENCY APPRAISAL - OXYGENATION

71. After withdrawal from inhaled nitric oxide (NO) therapy, an infant suddenly becomes
hemodynamically unstable. You should:
A) return the infant to the prior NO dosage
B) recommend administration of vasodilators
C) decrease to FIO2 o the lowest level tolerable
D) initiate rapid chest compressions
Ans: A
Response:
During weaning or immediately after withdrawing NO, some patients can becomes
hemodynamically unstable and/or develop severe hypoxemia. The best solution in these cases
is to restore the NO therapy at the level previously being administered. Additional
hemodynamic support (e.g., vasopressors) and supplemental oxygen may also be necessary, as
well as close patient monitoring.

72. You are monitoring a recent postoperative craniotomy patient who is being mechanically
ventilated and has an ICP of 22 mm Hg. The latest ABG results are as follows:

pH 7.35
PaCO2 47 mm Hg
HCO3 25 mEq/L
BE 0
PaO2 89 mm Hg
SaO2 96%

Based on this information, which of the following is the best action?


A) maintain the current settings
B) decrease the tidal volume
C) increase the minute ventilation
D) add 10 cm H2O PEEP
Ans: C
Response:
There is evidence that PaCO2 levels of about 25 -30 mm Hg result in short-term cerebral
vasconstriction, thus decreasing intracranial pressure (ICP) in patients with head trauma. With
a higher than normal ICP (normal = 10-12 mm Hg), this patient should be hyperventilated to
achieve PCO2's of approximately 25 -30 mm Hg for 24 to 48 hour post surgery.

Page 22
SAN PEDRO COLLEGE COMPETENCY APPRAISAL - OXYGENATION

73. An 80 kg adult male patient with aspiration pneumonia who is being mechanically ventilated
via volume ventilator has the following initial settings and blood gas results.

Ventilator Settings Blood Gases


Mode SIMV pH 7.29
VT 500 ml PaCO2 52 mm Hg
Rate 12 HCO3 25 mEq/L
FIO2 0.50 PaO2 63 mm Hg
PEEP 5 cm H2O SaO2 91%

Based on this information, you should now do which of the following?


A) increase the FIO2
B) increase the tidal volume
C) add pressure support
D) increase PEEP
Ans: B
Response:
The blood gas reveals uncompensated respiratory acidosis with mild hypoxemia. Both are
probably the result of hypoventilation. While the respiratory rate setting is within the
recommended range, the tidal volume may be somewhat low for a large adult male. Of the
available choices, increasing the tidal volume would increase the patient's alveolar ventilation
and provide a small increment in PaO2, thus, helping normalize the patient's acid-base AND
oxygenation status.

74. A patient is being mechanically ventilated in the pressure control mode with an FIO2 of 0.35, a
set rate of 18, and a pressure limit of 25 cm H2O. Results of an arterial blood gas analysis are
below.

pH 7.45
PaCO2 34 mm Hg
HCO3 23 mEq/L
SaO2 83%

On the basis of these results, the most appropriate action would be to


A) increase the FIO2
B) add mechanical dead space
C) increase the ventilator rate
D) decrease the pressure limit
Ans: A
Response:
The patient's saturation indicates an oxygenation problem (a SaO2 of 84% indicates a PaO2 in
the 50-60 mm Hg range). To correct this, increase the FIO2.

75. When 15 cm H2O PEEP is initiated, a patient's cardiac output decreases from 4 to 2 L/min.
Which of the following actions is appropriate?
A) maintain the present settings and then check the ABGs in one hour
B) decrease the respiratory rate to extend cardiac filling time
C) increase PEEP slightly to reach the "optimal" PEEP
D) decrease PEEP to 10 cm H2O and recheck the cardiac output
Ans: D
Response:
One of the most common problems with PEEP is that it adversely affect cardiac output. In this
case, the drop from in cardiac output is life-threatening. The PEEP level must be decreased
and the patient's cardiac output remeasured.

Page 23
SAN PEDRO COLLEGE COMPETENCY APPRAISAL - OXYGENATION

76. A 50-year-old male is brought to the emergency room with suspicion of carbon monoxide
poisoning. He is not alert and does not respond to commands. The ABG on a nonrebreathing
mask shows a PaO2 of 350 torr and a PaCO2 of 45 torr. What action should you take?
A) change to a nasal cannula at 3L/min
B) change to a Venturi mask at 40%
C) maintain present therapy
D) intubate and initiate mechanical ventilation
Ans: C
Response:
The high PaO2 (350 torr) is a measure of dissolved O2 only. Carbon monoxide poisoning
lowers the HbO2, not the PaO2 For this reason, you should continue therapy with the
nonrebreathing mask until the patient is alert and the COHb% levels reach 10% or less (as
measured by CO-oximetry).

77. A paralyzed patient being mechanically ventilated in the volume-control mode has an
abnormally low PaCO2 and high pH. Which of the following would be the best way to
increase the patient's PaCO2 without changing the minute ventilation?
A) decrease the tidal volume and increase the rate
B) increase the tidal volume and decrease the rate
C) add mechanical dead space to the ventilator circuit
D) increase the inspiratory flow
Ans: C
Response:
Minute ventilation equals alveolar ventilation + deadspace ventilation. By increasing the
mechanical deadspace, you can decrease alveolar ventilation, raise the PaCO2 and lower the
pH without changing minute ventilation.

78. As part of a weaning protocol, you switch a 85 kg (187 lb) patient from SIMV at 4/min to 5
cm H2O CPAP and an FIO2 of 0.45. Twenty minutes after switching to CPAP, you obtain the
following data:

pH 7.25 Resp Rate 32/min


PaCO2 52 mm Hg MIP -23 cm H2O
PaO2 49 mm Hg Vital Capacity 1.2 L
HCO3 22 mEq/L

Which of the following should actions would be appropriate at this time?


A) Administer sodium bicarbonate
B) Continue CPAP, but raise the FIO2 to 0.55
C) Increase the CPAP to 8 cm H2O
D) Put the patient back on SIMV
Ans: D
Response:
Although the MIP and VC are adequate, the high PCO2 and respiratory rate and dangerously
low pH and PaO2 indicate a failed weaning attempt. You should put the patient back on
ventilatory support and try again when appropriate.

Page 24
CHAPTER 15
Name: __________________________ Date: _____________

1. A patient with acute bronchitis is receiving mechanical ventilation. Wheezing is heard over all
lung fields, and rhonchi are heard over the central airways. Previously suctioned secretions
have been quite thick. The patient's peak pressure is 45 cm H2O, and plateau pressure is 20 cm
H2O. All of the following would be useful to treat the patient's condition EXCEPT:
A) albuterol (Proventil)
B) ipratropium bromide (Atrovent)
C) acetylcysteine (Mucomyst)
D) pancuronium bromide (Pavulon)
Ans: D
Response:
Based on the clinical presentation, the immediate problem appears to be bronchospasm
(wheezing) and thick secretions (rhonchi) causing an increase in airway resistance (increased
PIP - plateau). A bronchodilator (albuterol or ipratropium) and a mucolytic (acetylcysteine)
are indicated to treat this situation. Suctioning this patient will also be helpful in removing
excessive secretions. Pancuronium bromide (Pavulon) is a non-depolarizing neuromuscular
blocking agent that will not achieve bronchodilation or clear the airways. In fact, Pavulon can
cause release of histamine, which can cause bronchospasm.

2. A 75-year-old female patient with a fractured hip has been bedridden for at least one week.
The patient has clear breath sounds, but they are diminished slightly in the bases. The patient
has normal pulmonary function based on bedside spirometry. Which of the following should
you recommend?

I. bronchodilator therapy
II. deep suctioning
III. incentive spirometry treatments
IV. coughing and deep breathing
A) I and III only
B) II and IV only
C) III and IV only
D) I, II, and IV only
Ans: C
Response:
At this point, the patient is at high risk for developing atelectasis due to retained secretions
and immobility. An initial regimen of incentive spirometry and deep breathing and coughing
may well prevent atelectasis and aid secretion clearance.

3. An intubated patient in the ICU needs to undergo bedside bronchoscopy and is in need of
short-term moderate sedation. Which of the following agents would you recommend for this
procedure?
A) propofol (Diprivan)
B) haloperidol (Haldol)
C) lorazepam (Ativan)
D) cisatracurium (Nimbex)
Ans: A
Response:
Propofol is the agent of choice for rapid sedation of patients undergoing minor invasive
procedures. It has a quick and short half-life (< 30 minutes). Haloperidol (Haldol) is a
neuroleptic/antipsychotic agent, not a sedative. Lorazepam (Ativan) is a sedative used for
long-term sedation. Cisatracurium (Nimbex) is a neuromuscular blocking agent, not a
sedative.

Page 1
SAN PEDRO COLLEGE COMPETENCY APPRAISAL - OXYGENATION

4. You are assisting a medical resident performing an emergent intubation on a somewhat


combative patient. The resident wants to briefly paralyze the patient to facilitate this
procedure. Which drug would you recommend for this purpose?
A) pancuronium (Pavulon)
B) succinylcholine (Anectine)
C) vecuronium (Norcuron)
D) cisatracurium (Nimbex)
Ans: B
Response:
For relatively short-term paralysis during procedures such as endotracheal intubation, a
depolarizing agent like succinylcholine (Anectine) is recommended. Non-depolarizing agents
such as pancuronium (Pavulon), vecuronium (Norcuron), and cisatracurium (Nimbex)
produce prolonged paralysis and are generally used for long-term management of
mechanically ventilated patients.

5. A physician is having difficulty visualizing the airway of an obese patient during an


emergency intubation procedure. He asks for your recommendation to quickly secure the
airway and provide ventilation. You should recommend:
A) a cricothyrotomy
B) inserting an LMA
C) sedating the patient
D) using a double-lumen ET tube
Ans: B
Response:
You should recommend a laryngeal mask airway (LMA) when the physician is unable to
properly visualize the vocal cords and needs to secure the airway quickly. An emergency
cricothyrotomy is a surgical procedure indicated only when the oral and nasal routes are
unavailable. Sedation may help, but agitation is not the problem with this patient. Likewise,
changing the type of ET tube used will not help with visualizing the airway either.

6. A 90-kg male patient with a flail chest injury is being mechanically ventilated in the
assist/control mode with an FIO2 of 0.5, a set rate of 18, and a tidal volume of 600 mL. He is
involuntarily breathing above the set rate for a total respiratory rate of 28–30 breaths/min.
Results of an arterial blood gas analysis are as follows:

pH 7.52
PaCO2 27 torr
HCO3 21 mEq/L
BE –2
PaO2 81 torr
SaO2 96%

On the basis of these results, the most appropriate action is to:


A) increase the FIO2
B) add mechanical deadspace
C) increase the ventilator rate
D) increase the tidal volume
Ans: B
Response:
Adding mechanical deadspace will cause rebreathing of CO2, and thus PaCO2 will increase.
Another potential answer would have been to switch to the SIMV mode or to sedate the
patient, but these are not given as a choice.

Page 2
SAN PEDRO COLLEGE COMPETENCY APPRAISAL - OXYGENATION

7. A 87-year-old nursing home patient is admitted with pneumonia. On assessment the patient
presents with a 103.2 °F temperature, dry mucous membranes, urine output of 10 mL/hr for
the past two hours, mild hypotension, and decreased hematocrit on his CBC. You should
recommend to the ER physician all of the following EXCEPT:
A) initiating IV fluids immediately
B) beginning diuretic therapy
C) minimizing insensible water loss
D) documenting fluid intake/output every hour
Ans: B
Response:
Dehydration is very common in the elderly due to improper fluid intake and altered fluid
metabolism. Common signs of dehydration are: dry mucous membranes, hypotension,
diminished urine output, decreased skin turgor, increased hematocrit, thick and tenacious
secretions, decreased central venous pressure (CVP), and decreased pulmonary capillary
wedge pressure (PCWP).

8. Which of the following airway routes would you recommend to immediately ventilate a
trauma patient in a neck brace with a suspected cervical spine injury who was admitted to the
emergency department?

I. orotracheal intubation
II. nasotracheal intubation
III. laryngeal mask airway
A) II only
B) III only
C) II or III only
D) I, II, or III
Ans: C
Response:
Generally, patients with cervical spine injuries cannot be intubated via the oral route. For
these patients, either nasotracheal intubation or insertion of a laryngeal mask airway are good
alternatives, with the nasotracheal route preferred if the need for positive pressure ventilation
is likely to be long term.

9. A COPD patient being mechanically ventilated appears to be developing auto-PEEP. Which


of the following should you recommend to improve this situation?
A) decreasing the I:E ratio
B) adding an inspiratory hold
C) using an inverse I:E ratio
D) using a decelerating flow pattern
Ans: A
Response:
Due to their high expiratory flow resistance, COPD patients are more likely to develop auto-
PEEP during mechanical ventilation. If the problem is associated with bronchospasm, you
should recommend a bronchodilator. If not, you can recommend decreasing the I:E ratio by
(a) shortening inspiration (by using higher flows and/or lower tidal volumes) or (b)
lengthening the expiratory time by using lower rates or switching to SIMV mode. Another
alternative is to apply external PEEP in an amount less than the auto-PEEP level. This helps
prevent the small airway closure during exhalation that can cause air trapping.

Page 3
SAN PEDRO COLLEGE COMPETENCY APPRAISAL - OXYGENATION

10. You are managing a 49-year-old male patient who weighs about 80 kg (175 lbs.) who is on a
mechanical ventilator with the following settings and blood gases:

Ventilator Settings Blood Gases


VT 750 mL pH 7.42
Rate 12/min PaCO2 36 torr
FIO2 0.35 PaO2 58 torr
Mode Assist/control HCO3 23 mEq/L
PEEP 5 cm H2O

What changes should you now recommend to the physician?


A) increasing the FIO2 to 0.45
B) changing the mode to SIMV
C) adding 150 mL of deadspace
D) increasing the rate to 16/min
Ans: A
Response:
The blood gas analysis indicates normal ventilation and acid-base balance with moderate
hypoxemia. With an FIO2 below 0.50, the hypoxemia is most likely due to a V/Q imbalance.
In this instance, you should increase the FIO2 to 0.45. If this does not adequately address the
problem, you may need to consider an increase in PEEP.

11. A patient with heart failure is receiving volume-cycled ventilation and has a pulmonary artery
catheter in place. The ventilator peak pressure is 45 cm H2O, and plateau pressure is 25 cm
H2O. The patient's pulmonary artery pressure is 42/33 mm Hg, and pulmonary capillary
wedge pressure (PCWP) is 28 mm Hg. Lung sounds indicate dependent crackles and
wheezing. Which of the following should you recommend?
A) administering albuterol (Proventil)
B) decreasing the mean airway pressure
C) administering furosemide (Lasix)
D) removing the PA catheter—it is malfunctioning
Ans: C
Response:
The diagnosis of heart failure and the increased PA and PCWP suggest that this patient is in
congestive heart failure. In order to alleviate the backup of fluid in the lungs, the therapy plan
should generally include a rapid-acting diuretic such as furosemide (Lasix) as well as positive
inotropic agents as Digoxin and Dopamine.

12. A patient is receiving a treatment with 2.5 mg of albuterol and 3.0 mL of normal saline in the
emergency department. The heart rate prior to therapy is 80 beats/min, and at the end of
therapy is 128 beats/min. You should recommend:
A) adding acetylcysteine (Mucomyst) to the treatment
B) decreasing the dosage of albuterol
C) increasing the amount of saline per treatment to 5 mL
D) changing to ipratropium bromide (Atrovent)
Ans: B
Response:
The significant increase in heart rate (above 20 from baseline) indicates that this dosage of
albuterol (Proventil) is provoking systemic side effects in this patient. Before considering a
different drug, you should recommend trying a reduced dosage of albuterol and monitor
carefully for both desired effects and side effects.

Page 4
SAN PEDRO COLLEGE COMPETENCY APPRAISAL - OXYGENATION

13. An adult patient who suffered a cerebral contusion and resulting cerebral edema from an
automobile accident has just been placed on volume-cycled mechanical ventilation while in
the emergency department. Initial ABG values are as follows:

pH 7.39
PaCO2 42 torr
HCO3 25 mEq/L
BE 0 mEq/L
PaO2 92 torr
SaO2 95%

What should you recommend for the management of this patient?


A) maintaining the present settings and monitor the patient
B) increasing the minute volume on the ventilator
C) increasing the inspired O2 percentage
D) changing to pressure control ventilation
Ans: B
Response:
Although the ABG values are within normal range, some evidence suggests that
hyperventilating closed-head trauma patients may be beneficial during the first 24–48 hours of
their management. Typically, the target is a PaCO2 of approximately 25–30 torr. This causes
cerebral vasoconstriction, resulting in a reduction in intracranial pressure and swelling.

14. Which of the following drugs would be most appropriate to recommend as a substitute for
albuterol (Proventil) for a patient who has bronchospasm and whose cardiac rate increases by
50 beats/min with each treatment?
A) isoetharine (Bronkosol)
B) isoproterenol (Isuprel)
C) racemic epinephrine
D) ipratropium bromide (Atrovent)
Ans: D
Response:
Most sympathetic bronchodilators will result in some beta1 stimulation, causing an increase in
heart rate. Though albuterol has fewer beta1 effects than earlier adrenergic bronchodilators, it
may result in a significant increased heart rate in some patients. In this instance, a therapist
might recommend an anticholinergic bronchodilator such as ipratropium bromide (Atrovent).
An alternative may have been to recommend one of the newest sympathetic bronchodilators,
levalbuterol (Xopenex) with fewer beta1 effects, but that was not a choice.

Page 5
SAN PEDRO COLLEGE COMPETENCY APPRAISAL - OXYGENATION

15. A patient with neuromuscular disease has been on ventilatory support for 4 months via
tracheostomy. At this point, she requires only nighttime ventilator support. Which of the
following artificial airways should you recommend?
A) tracheostomy button
B) Bivona tracheostomy tube
C) cuffed, fenestrated tracheostomy tube
D) uncuffed, standard tracheostomy tube
Ans: C
Response:
For a patient with a tracheostomy on long-term mechanical ventilation who still requires
intermittent support, a cuffed, fenestrated tracheostomy tube is the ideal airway. With the
inner cannula in place and cuff inflated, a fenestrated trach tube performs like a standard tube
and can be used for positive pressure ventilation. However, fenestrated trach tubes have one or
more openings in their posterior curvature that permits air movement through the tube (from
trachea to larynx) when the inner cannula is removed. This air flow allows phonation (talking)
and facilitates communication. In addition, by removing a fenestrated tube's inner cannula,
plugging the outer cannula, and deflating its cuff, you can test the patient's ability to resume
normal upper airway function.

16. A physician orders 3 L/min O2 via simple mask to a 33-year-old post-op female patient with
moderate hypoxemia breathing room air (PaO2 = 52 torr). The correct action at this time is to:
A) carry out the physician's prescription exactly as written
B) recommend a flow of at least 5 L/min to wash out CO2
C) recommend that the mask be changed to a cannula at 2 L/min
D) not apply the oxygen until contacting the medical director
Ans: B
Response:
With this level of hypoxemia, this patient should immediately receive moderate O2
concentrations, as available by simple mask. However, a simple mask requires a flow of at
least 5 L/min in order to replenish the O2 and prevent CO2 rebreathing.

17. A patient in combined hypoxemic and hypercapnic respiratory failure due to an acute
restrictive disorder is placed on a ventilator in the SIMV mode at a rate of 12 breaths/min and
a PEEP of 10 cm H2O. Soon thereafter, she begins to exhibit a paradoxical breathing pattern
with intercostal retractions. Which of the following changes should you recommend?
A) decreasing the PEEP level to 5 cm H2O
B) decreasing the rate to 8 breaths/min
C) switching over to the pure CPAP mode
D) providing supplemental pressure support
Ans: D
Response:
SIMV with PEEP is an appropriate choice for a patient in combined hypoxemic and
hypercapnic respiratory failure due to an acute restrictive disorder. Coexisting respiratory
muscle fatigue in these patients may necessitate the addition of pressure support ventilation to
supplement the spontaneous breaths during SIMV and overcome the resistance imposed by
the artificial airway.

Page 6
SAN PEDRO COLLEGE COMPETENCY APPRAISAL - OXYGENATION

18. You are asked to assess whether a 65-kg (143-lb.) patient with a neuromuscular disorder being
mechanically ventilated in the SIMV mode is ready for weaning. After obtaining the
following data during a bedside spontaneous breathing assessment, what would you
recommend?

Spontaneous tidal volume 250 mL


Minute ventilation 10 L/min
Vital capacity 650 mL
Max inspiratory pressure (MIP) –20 cm H2O
A) beginning a spontaneous breathing T-piece trial
B) postponing weaning and reevaluating the patient
C) beginning weaning using a pressure support protocol
D) beginning weaning by decreasing the SIMV rate
Ans: B
Response:
Although the patient's vital capacity and MIP are borderline adequate, the tidal volume is very
low (3.8 mL/kg). The (missing) spontaneous breathing rate calculated by 10 L/min ÷ 0.250
L/breath = 40 breaths/min is too high. This yields a rapid shallow breathing index of 40/0.25 =
160, far above the threshold of 100. You should recommend postponing weaning and
reevaluating the patient at a later date.

19. An adult patient started on cool mist therapy after extubation begins to develop stridor. Which
of the following actions should you recommend?
A) changing from cool mist to heated aerosol
B) administering a racemic epinephrine treatment
C) reintubating the patient immediately
D) drawing and analyzing an arterial blood gas
Ans: B
Response:
The development of stridor after extubation indicates glottic edema. When you hear stridor,
you should be wary of further problems, as the swelling can dramatically worsen. If stridor is
present, a racemic epinephrine treatment may be given to lessen the swelling. In children,
post-extubation edema is often subglottic and may require reinsertion of the airway.

Page 7
SAN PEDRO COLLEGE COMPETENCY APPRAISAL - OXYGENATION

20. An 8-hour-old, 28-week gestational age neonate is being maintained in an oxygen hood with
an FIO2 of 0.65. The neonatologist believes that the patient has infant respiratory distress
syndrome (IRDS). Based on the following results, what should you recommend?

pH 7.36
PaCO2 44 torr
HCO3 24 mEq/L
BE 0 mEq/L
PaO2 52 torr
A) increasing the O2 hood concentration to 100%
B) beginning inhaled nitric oxide (INO) therapy
C) administering pulmonary surfactant with Beractant (Survanta)
D) starting high-frequency ventilation
Ans: C
Response:
The acid-base balance for this infant is within normal range; hence mechanical ventilation is
not warranted. However, the patient has refractory hypoxemia (PaO2  60 with FIO2  0.60),
probably due to shunting from the IRDS. Surfactant administration is indicated for premature
babies (25–34 weeks) for the prevention and treatment of RDS since their ability to produce
their own surfactant is not fully developed. Lack of mature surfactant in premature babies is a
major contributor to IRDS. Increasing the FIO2 to 100% is not warranted at this time, and it
may increase the risk of retinopathy of prematurity (ROP). INO therapy is indicated only for
persistent pulmonary hypotension of newborn (PPHN).

21. A physician asks your recommendation regarding sedation for a mechanically ventilated
patient in the ICU. You would consider recommending all of the following to calm this patient
EXCEPT:
A) pentobarbital (Nembutal)
B) propofol (Diprivan)
C) lorazepam (Ativan)
D) cisatracurium (Nimbex)
Ans: D
Response:
Common sedatives used to calm patients in the ICU include benzodiazepines like lorazepam
(Ativan), barbiturates like pentobarbital (Nembutal), and propofol (Diprivan). Cisatracurium
(Nimbex) is a neuromuscular blocking agent, not a sedative.

22. A ventilator patient is admitted with atelectasis and a lower than predicted functional residual
capacity (FRC). This has caused hypoxemia. What should you recommend to correct the
atelectasis and improve the FRC?
A) adding PEEP
B) increasing the FIO2
C) suctioning the patient more frequently
D) starting pressure support ventilation
Ans: A
Response:
In mechanically ventilated patients with atelectasis and hypoxemia, PEEP is indicated. PEEP
increases the FRC by recruiting collapsed alveoli, which improves oxygenation by decreasing
shunting.

Page 8
SAN PEDRO COLLEGE COMPETENCY APPRAISAL - OXYGENATION

23. A 25-year-old asthma patient has continual symptoms that limit her physical activity. Along
with frequent exacerbations of her condition, her FEV1/FVC is less than 60% of predicted.
Which of the following drugs should you recommend to help control her condition over the
long term?

I. albuterol (Proventil) MDI 2 puffs 3 times a day (tid)


II. fluticasone (Flovent) MDI 2 puffs 4 times a day
III. salmeterol (Serevent) 2 puffs twice a day (bid)
A) II only
B) III only
C) II and III only
D) I and II only
Ans: C
Response:
Based on the symptoms and PFT results, the patient has severe persistent asthma. Long-term
control of severe persistent asthma is best achieved by combining both an inhaled
corticosteroid (like fluticasone) and a long-acting adrenergic bronchodilator (such as
salmeterol). This combination is available as the Advair discus, but that is not one of the
choices. Short-acting inhaled beta2-agonists like albuterol are also prescribed but are indicated
for quick relief of exacerbations, not long-term control.

24. Which of the following five adult patients receiving ventilatory support is the best candidate
for weaning?

Patient VC VE Spon Rate MVV MIP %Shunt


(L) (L/min) breaths/min (L/min) (cm H2O)
A 0.5 4.1 14 6.3 –21 16%
B 1.5 4.6 15 9.7 –33 17%
C 0.9 12.1 40 14.3 –28 12%
D 1.3 6.3 28 16.7 –42 28%
A) A
B) B
C) C
D) D
Ans: B
Response:
Assuming all else is equal, patient B (with a VC > 15 mL/kg, a resting VT of > 300 mL, an
NIF > –20 cm H2O, and a shunt fraction of 17%) is the most viable candidate for weaning.

25. While reviewing the lab chemistry of a patient in metabolic acidosis due to renal failure, you
would expect the following electrolyte to be abnormally high:
A) glucose
B) bicarbonate
C) chloride
D) potassium
Ans: D
Response:
Metabolic acidosis increases H+ levels outside the cells. In an effort to buffer this acidosis,
intracellular K+ ions are exchanged with extracellular H+, producing hyperkalemia. Correcting
the metabolic acidosis usually corrects the hyperkalemia.

Page 9
SAN PEDRO COLLEGE COMPETENCY APPRAISAL - OXYGENATION

26. A patient who is being mechanically ventilated in the SIMV mode has a machine rate of
10/min and a spontaneous rate of 28/min. The patient is using accessory muscles to breathe.
Which of the following would most likely help the patient?
A) adding pressure support
B) changing to control ventilation
C) turning off the sensitivity
D) instituting inverse ratio ventilation
Ans: A
Response:
The use of accessory muscles suggests that the work of breathing is too high for the patient.
Therefore, the therapist should help decrease the work of breathing by adding pressure support
to overcome the resistance in the ventilator tubing and augment spontaneous breaths. Another
alternative, increasing the SIMV machine rate, but that is not a choice.

27. A patient is receiving aerosol treatments with albuterol q6h following an uneventful
cholecystectomy. Patient assessment reveals clear breath sounds bilaterally. The patient has
no history of smoking and takes deep breaths with strong coughs voluntarily throughout the
day. You should recommend
A) discontinuing the aerosol treatment
B) continuing the treatment as ordered
C) postural drainage and percussion
D) a metered dose inhaler with albuterol
Ans: A
Response:
Bronchodilator treatments are used to reverse bronchospasm. There is no indication that this
patient is experiencing a bronchospasm. This patient has normal airways and does not need a
bronchodilator. It should be discontinued. Moreover, the strong cough indicates that the
patient has good airway clearance and probably only needs encouragement to continue
directed coughing, deep breathing and ambulation to prevent post-op problems.

28. A 60-year-old, 70 kg (154 lb) emphysema patient is being mechanically ventilated. Relevant
ventilator settings and arterial blood gas data are below.

Ventilator Settings Blood Gases


Mode SIMV pH 7.36
VT 600 mL PaCO2 71 mm Hg
Set Rate 9 breaths/min HCO3 39 mEq/L
Spont Rate 0 PaO2 105 mm Hg
FIO2 0.45 SaO2 98%

To initiate weaning the patient from the ventilator, which of the following should you
recommend be adjusted FIRST?
A) mode
B) tidal volume
C) rate
D) FIO2
Ans: D
Response:
The high PCO2 and HCO3 seen in the blood gas results suggest that this patient is a CO2
retainer, whose main stimulus to breath is low PO2. This is known as a hypoxic drive and with
such patients; the target PO2 range is about 50-70 mm Hg so as not to suppress their breathing.
In this instance, the FIO2 of 0.45 is resulting in a PO2 of 105 mm Hg which may well be
suppressing their stimulus to breathe. Consequently, the FIO2 should be lowered until the
patient begins spontaneous breathing (probably at a PaO2 of 50-70 mm Hg.).

Page 10
SAN PEDRO COLLEGE COMPETENCY APPRAISAL - OXYGENATION

29. Immediately after endotracheal tube extubation, an adult patient exhibit a high-pitched
inspiratory noise, heard without a stethoscope. Which of the following actions would you
recommend?
A) a STAT heated aerosol treatment with saline
B) careful observation of the patient for 6 hours
C) immediate reintubation via the nasal route
D) a STAT racemic epinephrine aerosol treatment
Ans: D
Response:
Stridor is a high pitched noise heard during inspiration. In adults, stridor indicates that the
airway is reduced to 5 mm or less in diameter. Stridor is more serious than hoarseness,
indicating increased airway resistance and work of breathing. Stridor is often treated with
racemic epinephrine (2.25% Vaponephrine) via aerosol. This reduces glottic edema by
mucosal vasoconstriction. Steroids may also be added to the aerosol to further reduce the
inflammation.

30. Which of the following five adult patients receiving ventilatory support is the best candidate
for weaning?

Patient VC VE Spon Rate MVV MIP


(L) (L/min) breaths/min (L/min) (cm H2O)
A 0.6 4.1 27 9.3 -17
B 0.9 5.6 15 6.7 -21
C 1.2 12.1 40 14.3 -28
D 1.7 5.8 17 14.7 -42
A) A
B) B
C) C
D) D
Ans: D
Response:
Assuming all other clinical finding are equal, patient 'D' (with a VC > 15 ml/kg, a resting VE
300 mL and a NIF > -20 cm H2O is the best candidate.

31. Which of the following drugs you would NOT recommend for the long-term management of
mild persistent asthma in an adult patient?
A) levalbuterol (Xopenex)
B) beclomethasone (Vanceril)
C) zafirlukast (Accolate)
D) cromolyn sodium (Intal)
Ans: A
Response:
Mild persistent asthma normally is treated with daily dosing of an anti-inflammatory drug.
Options include a low-dose inhaled corticosteroid (like beclomethasone), or cromolyn sodium
(Intal). Children usually begin with a trial of cromolyn or nedocromil. The leukotriene
modifier (Montelukast-Singulair) may also be considered.

Page 11
SAN PEDRO COLLEGE COMPETENCY APPRAISAL - OXYGENATION

32. A 56-year-old male is brought to the emergency room by ambulance complaining of tightness
in his chest with radiating left shoulder pain. You should recommend:
A) obtaining an arterial blood gas sample
B) administering supplemental oxygen
C) administering an albuterol treatment
D) obtaining a stat chest X-ray
Ans: B
Response:
The patient's presenting symptoms suggest a myocardial infarction (MI). In order to maximize
myocardial oxygenation and decrease myocardial workload, any patient with a suspected MI
should be given supplemental oxygen, usually in high concentrations (e.g., via a
nonrebreathing mask).

33. A 52 kg (114 lb) female with an acute closed head injury is on volume-cycled assist-control
ventilation with a tidal volume of 550 mL and a set rate of 18 at an FIO2 of 0.35. Blood gas
data are as follows:

pH 7.53
PaCO2 26 mm Hg
PaO2 110 mm Hg
HCO3 21 mEq/L
BE -2 mEq/L

Which of the following would you recommend?


A) wean the patient
B) maintain the present settings
C) decrease the set rate
D) decrease the tidal volume
Ans: B
Response:
Though somewhat controversial, patients with acute closed head injuries are often
hyperventilated for the first day or two after initial trauma in order to promote cerebral
vasoconstriction (low PaCO2) and decrease the intracranial pressure. Typically, the targeted
PaCO2 is in 25 to 30 mm Hg range. So in this case, the best action is to maintain the present
settings.

34. Which of the following medications would NOT be appropriate for the initial management of
a child during an acute asthmatic attack?
A) metaproterenol (Alupent)
B) cromolyn sodium (Intal)
C) albuterol (Proventil)
D) epinephrine
Ans: B
Response:
Intal is a prophylactic (preventative) medication designed to prevent asthma attacks in the
long-term. It will do nothing for an acute attack and can even make things worse if given
alone without a bronchodilator.

Page 12
SAN PEDRO COLLEGE COMPETENCY APPRAISAL - OXYGENATION

35. A physician's order for an aerosolized bronchodilator indicates three times the normal dose.
Which of the following actions is appropriate?
A) prolong the administration time
B) have the nurse give the treatment
C) postpone the therapy until contacting the physician
D) give the treatment as ordered
Ans: C
Response:
When an apparent incorrect dose of a respiratory medication is prescribed or the order is
incomplete, the prescribing physician should be contacted to clarify the order and ensure it is
correct.

36. The hospital pharmacy stocks Mucomyst only in 20% vials (10 mL). The doctor orders 4 mL
of 10% Mucomyst with albuterol for a patient with chronic bronchitis. What volume of 20%
Mucomyst and saline would you mix together to provide the desired dosage?
A) 4 mL Mucomyst/4 mL saline
B) 2 mL Mucomyst/2 mL saline
C) 3 mL Mucomyst/1 mL saline
D) 1 mL Mucomyst/3 mL saline
Ans: B
Response:
This is a classic proportion problem. Using the Volume 1 x Concentration 1 = Volume 2 x
Concentration 2 formula. By substituting the values in the problem, you get 4 mL x 10% = X
x 20%. By solving this, you get 2 mL of 20% solution are needed.

37. A patient has a serum potassium level of 3.4 mEq/L. Which of the following diuretic would
you recommend for this patient?
A) amiloride (Midamor)
B) chlorothiazide (Diuril)
C) quinethazone (Hydromox)
D) Acetazolamide (Diamox)
Ans: A
Response:
This patient's serum potassium is marginally low and many diuretics can cause further
potassium loss. For this reason, a potassium-sparing diuretic such as amiloride should be
recommended.

38. A mechanically ventilated patient is restless and fighting the ventilator. IV sedation has not
been successful in calming the patient. Which of the following drugs would you
recommended to assure patient-ventilator synchrony?
A) disopyramide phosphate (Norpace)
B) pancuronium (Pavulon)
C) succinylcholine (Anectine)
D) lorazepam (Ativan)
Ans: B
Response:
Pancuronium (Pavulon) is a nondepolarizing paralytic agent which can be used to facilitate
mechanical ventilation in patients fighting the ventilator who are not responsive to sedatives
alone. Because such agents do not affect sensation or the anxiety associated with being
paralyzed, patients receiving paralytic agent also should continue to receive sedation (usually
with a benzodiazepines like lorazepam). Succinylcholine (Anectine) is a short-acting
depolarizing paralytic agent used for special procedures like intubation. Norpace is an
antiarrhythmic drug indicated for the treatment of ventricular arrhythmias.

Page 13
SAN PEDRO COLLEGE COMPETENCY APPRAISAL - OXYGENATION

39. You note that the I:E ratio warning light is flashing and that the I:E ratio is 1:0.7 for a
mechanically ventilated patient. Which of the following would you recommend in this
situation?
A) obtaining an arterial blood gas
B) increasing the inspiratory flow
C) activating the I:E limit system
D) increasing the preset breathing rate
Ans: B
Response:
For most adults, an I:E ratio of 1:2 or 1:3 is desirable. A high I:E ratio (> 1:1.5) indicates that
inspiration is too long relative to exhalation. In this case, one should check and adjust the
inspiratory flow provided by the ventilator (increasing the flow decreases the inspiratory
time).

40. You are assisting in the treatment and evaluation of a heroin overdose patient in the ER who is
hypoventilating. The physician asks you to recommend a drug to help reverse the effects of
the narcotic and enable the patient's breathing to return to normal. Which of the following
drugs would you recommend?
A) indomethacin (Indocin)
B) naloxone (Narcan)
C) vecuronium (Norcuron)
D) fentanyl (Sublimaze)
Ans: B
Response:
Naloxone (Narcan) is an opioid antagonist that can reverse narcotic depression, including
respiratory depression, induced by natural and synthetic opioid narcotics such as heroin and
morphine.

41. The ICU nurse calls you to assess a patient that became agitated and complains of difficulty
breathing. The patient presents with decreased right chest excursion, absent breath sounds in
the right upper lobe and slight tracheal deviation to the left. Which of the following
procedures would you recommend to be done FIRST?
A) stat chest X-ray
B) lab chemistry
C) right side thoracentesis
D) a bronchoscopy
Ans: A
Response:
The signs and symptoms of this patient indicates the probability of a tension pneumothorax. A
chest X-ray will rule out the presence of a right-sided pneumothorax and can be done fairly
quickly. Other signs of a pneumothorax are hyperresonant percussion note in the affected side,
absent lung markings and radiolucency on the chest X-ray and sudden increased in peak
pressures during mechanical ventilation. The other procedures listed will not help in the
diagnosis and treatment of a pneumothorax.

Page 14
SAN PEDRO COLLEGE COMPETENCY APPRAISAL - OXYGENATION

42. A patient with airway obstruction due to laryngeal trauma has just arrived at the ER. Based on
the physician's evaluation, the patient will need long-term ventilatory support. Which of the
following would you recommend to establish a patent tracheal airway for this patient?
A) surgical tracheotomy
B) nasotracheal intubation
C) cricothyrotomy
D) orotracheal intubation
Ans: A
Response:
Because it enters the trachea directly, a tracheostomy tube is considered to be the most
efficient artificial airway, and is the device of choice for upper airway obstruction or trauma.

43. You are asked to transport a mechanically ventilated a patient to the MRI. What type of
ventilator would you recommend for this procedure?
A) a pneumatically powered and controlled ventilator
B) a high frequency oscillator
C) a micro processor controlled state-of-the-art ventilator
D) a pressure ventilator such as a BiPAP®.
Ans: A
Response:
Ventilators used in the MRI must be free of iron-based metals found in many ventilators. As a
result, pneumatically powered and controlled ventilators are best suited for this purpose.

44. You are providing bag-valve-mask support to a patient during a “code blue”. Despite proper
technique, you cannot achieve adequate ventilation and you are concerned about the potential
for aspiration. Protocol requires that only an anesthesiologist or nurse-anesthetist perform
tracheal intubation, but none is available. Which of the following alternative airways would
you recommend?

I. laryngeal mask airway


II. esophageal-tracheal Combitube
III. percutaneous tracheostomy
A) I only
B) III only
C) I or II only
D) I, II or III
Ans: C
Response:
Alternatives to endotracheal intubation include the laryngeal mask airway (LMA) and
esophageal-tracheal Combitube (ETC). When used by adequately trained healthcare
providers, the LMA and the ETC provide superior ventilation compared with face masks in
patients in cardiac arrest.

Page 15
SAN PEDRO COLLEGE COMPETENCY APPRAISAL - OXYGENATION

45. While assessing a mechanically ventilated patient, you notice that the peak pressure has
increased 10 cm H2O over the last hour. The plateau pressure has not changed. You also
notice that the patient has significant wheezing in both lungs that was not present earlier. You
would recommend:
A) nebulizing a bronchodilator
B) nebulizing a corticosteroid
C) suctioning the patient
D) increasing the VT by 100 mL
Ans: A
Response:
The difference between peak and plateau pressure is the pressure due to airway resistance.
Therefore, when peak pressure increases without an increase in plateau pressure, airway
resistance has increased. The patient's wheezing suggests bronchospasm, which would cause
airway resistance to rise. Based on these observations, this patient should be administered a
quick acting, sympathomimetic bronchodilator such as albuterol.

46. In a patient suspected of having a tension pneumothorax you would recommend all of the
following EXCEPT:
A) 100% oxygen via non-rebreather mask
B) stat chest X-ray
C) stat arterial blood gas
D) needle decompression on the affected side
Ans: C
Response:
Although an ABG is recommended for changes in patient status and increased WOB in this
case it is not the priority and it will not help to treat the tension pneumothorax.

47. You are administering an aerosolized albuterol (Proventil) treatment to a patient when you
note he complains of dizziness, heart palpitations and feeling flushed. You next action should
be to do which of the following?
A) continue the therapy but recommend reducing its frequency
B) stop the therapy, go to the nurse's station and review the chart
C) continue the therapy, but add saline to further dilute the albuterol
D) stop the therapy, monitor the patient and inform the patient's doctor
Ans: D
Response:
When patients experience an apparent adverse response to therapy, the best course of action
normally is to stop the treatment, monitor their condition and advise the nurse and doctor.

48. A patient with history of CHF is admitted in the ICU. On physical assessment you note severe
pedal edema, hepatomegaly and jugular venous distension. The ICU resident wants your
opinion on the management of his condition. You would recommend all of the following
EXCEPT:
A) use of inotropic agents
B) insert a central venous pressure catheter
C) record and document intake/outputs q1h
D) avoid the use of diuretics
Ans: D
Response:
In a patient with history of CHF signs of fluid balance alteration (fluid retention in this case)
are quite common. The use of inotropic medications to increase heart contractions and
diuretics to decrease the fluid overload is very important in the management of CHF patients.

Page 16
SAN PEDRO COLLEGE COMPETENCY APPRAISAL - OXYGENATION

49. A patient that has been in the ICU for 10 days shows signs of confusion and delirium. Which
of the following pharmacologic agents would you recommend to treat her symptoms?
A) succinylcholine (Anectine)
B) haloperidol (Haldol)
C) morphine sulfate
D) diazepam (Valium)
Ans: B
Response:
Due to her length of stay in the ICU this patient is likely to be suffering from ICU psychosis.
Antipsychotic drugs such as Haldol are the agents of choice to treat ICU psychosis.
Succinylcholine (Anectine) is a short-acting depolarizing agent use to facilitate intubation
procedures, morphine is analgesic agent and diazepam (Valium) is a benzodiazepine use for
moderate sedation to treat anxiety.

50. Which type of ventilator would you recommend for a patient with an artificial airway who
may be, or become a candidate for weaning?
A) a volume ventilator with correction for decreased atmosphere pressure
B) a non-invasive positive pressure (BiPAP) ventilator
C) a vent capable of pressure support ventilation
D) a pneumatically powered and control ventilator
Ans: C
Response:
In general, the more critically ill and unstable a patient is, the more support options/modes
should be available in the ventilator, including a variety of weaning tools such as pressure
support and monitoring alarms.

51. A patient in Intensive Care Unit with congestive heart failure receiving assist/control
ventilation with a set volume of 650 mL exhibits the following data on three consecutive
patient-ventilator checks:

Time Peak Plateau PEEP


Pressure Pressure
9:00 AM 40 25 8
10:00 AM 50 35 8
11:00 AM 60 45 8

The patient also exhibits diffuse crackles at the bases and some wheezing. Which of the
following would you recommend for this patient?
A) a diuretic
B) a bronchodilator
C) a mucolytic
D) a steroid
Ans: A
Response:
This patient is exhibiting a progressive decrease in compliance (rising plateau – PEEP
pressure difference). In patients with congestive heart failure, the most common cause for a
progressive decrease in compliance is the development of pulmonary edema. Among the
available choices, the correct course of action in treating acute pulmonary edema would be the
administration of a diuretic like Lasix.

Page 17
SAN PEDRO COLLEGE COMPETENCY APPRAISAL - OXYGENATION

52. A 60 kg. (132 lb.) COPD patient is receiving SIMV with a VT of 500 mL at 9/min with an
FIO2 of 0.35. Blood gases are as follows: pH = 7.36; PCO2 = 61 torr; HCO3 = 36 mEq/L;
PaO2 = 64 torr. Which of the following changes would you recommend at this time?
A) increase the IMV rate
B) increase the FIO2
C) maintain settings
D) increase the VT
Ans: C
Response:
The blood gas indicates a fully compensated respiratory acidosis. Since the pH is normal, you
don't want to recommend changing any ventilatory parameters. In terms of oxygenation, a
PaO2 of 64 torr (indicating an Hb saturation of at least 90%) is considered acceptable for a
COPD patient. In fact, increasing the PaO2 in such patients above this level can cause
hypoventilation. In summary, you should leave well enough alone and maintain these settings.

53. A 90 kg (198 lb) patient is being ventilated in the postanesthesia care unit (PACU) following
upper abdominal surgery. Ventilator settings and arterial blood gas data are below:

Ventilator Settings Blood Gases


Mode SIMV pH 7.53
VT 800 mL PaCO2 29 torr
Set Rate 14/min HCO3 24 mEq/L
Total Rate 14/min BE +1
FIO2 0.40 PaO2 105 torr
PEEP 5 cm H2O SaO2 99%

You should recommend which of the following?


A) increase the FIO2
B) decrease the rate
C) decrease the tidal volume
D) discontinue the PEEP
Ans: B
Response:
The patient's oxygenation is acceptable, so no change in either the FIO2 or PEEP is warranted.
The acid-base status however is abnormal, indicating an acute respiratory acidosis. To lower
the pH, you will need to raise the PaCO2. To raise the PaCO2, you must lower the minute
volume, either by reducing the tidal volume or the set rate. If the tidal volume is appropriate
(as here), the best way to lower the minute volume would be to reduce the rate.

Page 18
SAN PEDRO COLLEGE COMPETENCY APPRAISAL - OXYGENATION

54. A 45-year-old 70 kg male with a diagnosis of bilateral pneumonia is receiving volume-


controlled ventilation in the SIMV. Ventilator settings and arterial blood gas data are below.
Which of the following should be recommended?

Ventilator Settings Blood Gases


Mode SIMV pH 7.35
VT 600 mL PaCO2 45 torr
Set Rate 6/min HCO3 23 mEq/L
Total Rate 10/min BE -1
FIO2 0.65 PaO2 55 torr
PEEP 5 cm H2O SaO2 83%

You should recommend which of the following?


A) increase PEEP
B) increase the rate
C) increase the FIO2
D) add an inspiratory plateau
Ans: A
Response:
The patient's acid-base status is borderline normal, indicating adequate ventilation. Given that
the tidal volume is about 8-9 mL/kg, no change in rate or tidal volume is indicated. On the
other hand, the patient is hypoxemic (PaO2 = 55 torr). Given that the FIO2 > 0.60 and the PaO2
< 60 torr, the cause of the hypoxemia is shunting, which is best managed by increasing the
level of PEEP.

55. While ventilating an intubated patient with a bag-valve device during CPR, you observe a
doctor insert a catheter in the right subclavian vein. Soon thereafter, you note that it is
becoming increasingly difficult to ventilate the patient, and that there are no chest excursions
on the right side. Which of the following is the most likely cause of this problem?
A) right main stem intubation
B) gastric distention
C) broken ribs with flail chest
D) right-sided pneumothorax
Ans: D
Response:
One of the more common complications of inserting a central line into the subclavian vein is
pneumothorax. The findings (increasingly difficult to ventilate; no chest excursions on the
right) are consistent with this complication. The patient will need to have a chest tube inserted
immediately.

56. A patient complains of left-sided chest pain while receiving mechanical ventilation.
Inspection, palpation and auscultation reveal tachypnea, a weak and thready pulse, tracheal
deviation to the right, and decreased breath sounds and hyperresonance on the left. Which of
the following would you recommend?
A) bronchoscopy
B) chest tube insertion
C) thoracentesis
D) suctioning
Ans: B
Response:
All of the physical assessment signs detected here are consistent with a tension pneumothorax.
Patients with a tension pneumothorax typically show tracheal deviation AWAY from the
affected side, with decreased breath sounds and hyperresonance on the affected side. These
patients also tend to deteriorate rapidly, showing signs of cardiovascular collapse (including
decreased cardiac output and often severe hypotension). This patient likely requires immediate
insertion of a chest tube on the affected side.

Page 19
SAN PEDRO COLLEGE COMPETENCY APPRAISAL - OXYGENATION

57. What treatment would you recommend for a patient who has a 50% spontaneous
pneumothorax?
A) have the patient perform a Valsalva maneuver
B) insert a chest tube and apply negative pressure
C) have the patient perform PEP therapy
D) intubate and provide mechanical ventilation
Ans: B
Response:
Small spontaneous pneumothoraces (< 15%) that are not associated with major symptoms
often resolve without treatment. Increasing the FIO2 in these patients can speed resolution of
the problem. However, a 50% pneumothorax normally requires treatment via insertion of a
chest tube.

58. A CHF patient on a ventilator receiving a thiazide diuretic has the following blood chemistry
report:

Na 120 mEq/L
K 2.2 mEq/L
Cl 80 mEq/L
HCO3 40 mEq/L
pH 7.5

Which of the following would you recommend for this patient?


A) administer bicarbonate
B) increase the diuretic dose
C) administer IV KCl
D) add mechanical deadspace
Ans: C
Response:
The patient is suffering from hypochloremic metabolic alkalosis and hyponatremia, common
complications of thiazide diuretics. Treatment for this condition normally consists of IV
administration off potassium chloride (KCl) solution via IV. Consideration should also be
given to changing the diuretic.

59. You are monitoring a patient receiving volume control ventilation and note an increased peak
pressure and bilateral wheezes. Which of the following would you recommend?
A) a stat chest X-ray
B) fiberoptic bronchoscopy
C) bronchodilator administration
D) a decrease in tidal volume
Ans: C
Response:
Bronchoconstriction often results in wheezing and an increase in peak pressure in ventilator
patients. As a result, this patient should be administered a bronchodilator.

Page 20
SAN PEDRO COLLEGE COMPETENCY APPRAISAL - OXYGENATION

60. Which of the following airway clearance techniques would you recommend for a 15 month
old infant with cystic fibrosis?
A) postural drainage, percussion, and vibration
B) positive expiratory pressure (PEP) therapy
C) mechanical insufflation-exsufflation
D) intrapulmonary percussive ventilation
Ans: A
Response:
The recommended airway clearance technique for a 15 month old infant with cystic fibrosis is
postural drainage, percussion, and vibration. All other options require either cooperation and
understanding that cannot be expected from a 15-month old or equipment designed for adults
only.

61. A trauma patient has been receiving volume-targeted SIMV via an oral endotracheal tube for a
week. Based on the patient's current condition, there is little likelihood that weaning will be
considered for at least another 7-10 days. Which of the following actions would you
recommend?
A) switch from ET intubation to tracheostomy
B) switch to noninvasive positive pressure ventilation
C) switch to pressure control ventilation (PSV)
D) begin a trial of inspiratory resistance breathing
Ans: A
Response:
For a patient who has had an endotracheal tube in place for a week with no likelihood of
removal in the short-term future, tracheostomy is indicated.

62. A patient is admitted to the emergency department with severe trauma to the upper airway,
including the hypopharynx, larynx and esophagus. Which type of airway would you
recommend for this patient?
A) nasotracheal intubation
B) tracheostomy
C) laryngeal mask airway
D) esophageal-tracheal combitube
Ans: B
Response:
Tracheotomy is the preferred, primary route for overcoming upper airway obstruction or
trauma and for long-term care of patients with neuromuscular disease who have poor airway
control.

Page 21
SAN PEDRO COLLEGE COMPETENCY APPRAISAL - OXYGENATION

63. A patient with COPD is receiving assist-control volume-oriented ventilatory support. The
patient has a respiratory rate of 20/min and an inspiratory time of 1.5 seconds. You note that
the expiratory flow does not return to zero between breaths. Which of the following ventilator
changes would you recommend?
A) increase the set rate
B) increase the inspiratory flow
C) increase the tidal volume
D) decrease the inspiratory flow
Ans: B
Response:
With a rate of 20/min and an I-time of 1.5 sec, this patient has only about a 1.5 second
expiratory time. Because the E-time is inadequate, air trapping (auto-PEEP) is occurring, as
confirmed by the expiratory flow not returning to zero between breaths. To increase E-time,
you can either (a) decrease I-time (increase the inspiratory flow or decrease the tidal volume)
or (b) decrease the rate of machine breaths. Since decreasing the rate of machine breaths is
difficult in the assist-control mode (without sedating the patient or changing to SIMV), the
best option here is to decrease the I-time by increasing the inspiratory flow.

64. A patient with a restrictive disorder develops hypercapnic respiratory failure. Prior to
intubation, her breathing rate is 28/min. After being placed on volume-control ventilation in
the assist/control mode, her blood gas indicate respiratory alkalosis (pH=7.54). Which of the
following would you recommend?
A) increase the preset ventilator rate
B) switch the patient to control mode
C) switch the patient to the IMV mode
D) decrease the preset ventilator rate
Ans: C
Response:
When assist/control ventilation is used with a restrictive disease patient in hypercapnic
respiratory failure, the patient's typically high rate of breathing can cause a rapid swing to
respiratory alkalosis. Control mode is not much better, since these patients commonly 'fight'
this type of support. The IMV mode will allow the patient to breath spontaneously at her
higher rate and at the same time help avoid respiratory alkalosis, since not every breath will be
a machine breath.

65. Which of the following drugs would you recommend to paralyze a hemodynamically unstable
patient requiring long-term controlled mechanical ventilation?
A) cisatracurium (Nimbex)
B) pancuronium (Pavulon)
C) succinylcholine (Anectine)
D) vecuronium (Norcuron)
Ans: A
Response:
Nondepolarizing agents such as pancuronium (Pavulon), vecuronium (Norcuron), and
cisatracurium (Nimbex) produce prolonged paralysis, as may be needed to support long-term
controlled mechanical ventilation. Of the three, cisatracurium is the first choice if the patient
is hemodynamically unstable, has cardiac disease, or is at risk of histamine release.

Page 22
SAN PEDRO COLLEGE COMPETENCY APPRAISAL - OXYGENATION

66. All of the following are drugs that would be considered for control of mild persistent asthma
EXCEPT:
A) cromolyn sodium (Intal) by inhalation
B) beclomethasone (Vanceril) by inhalation
C) albuterol (Proventil)
D) zileuton (Zyflo) tablets by mouth
Ans: C
Response:
For mild persistent asthma daily administration of an anti-inflammatory drug is indicated.
This may involve inhalation of a low dose corticosteroid or a mast cell stabilizer such as
cromolyn or nedocromil. The leukotriene modifiers montelukast (Singulair), zafirlukast
(Accolate), or zileuton (Zyflo) may also be considered. Sustained-release theophylline
(titrated to a serum concentration of 5-15 mcg/mL) is an alternative, but not preferred,
therapy. Long-acting beta2-agonists like salmeterol generally are indicated only for moderate
to severe persistent asthma.

67. A patient has been given pancuronium (Pavulon) to induce paralysis. Once the procedure is
completed, the physician wishes to reverse the effects of this drug. Which of the following
drugs would you recommend to reverse the paralysis?
A) enalapril (Vasotec)
B) vecuronium bromide (Norcuron)
C) neostigmine (Prostigmin/Mestinon)
D) atracurium besylate (Tracrium)
Ans: C
Response:
The effects of nondepolarizing paralytic agents such as pancuronium (Pavulon) can be
reversed with neostigmine. Vecuronium and atracurium are nondepolarizing paralytic agents
and enalapril is an angiotensin-converting enzyme (ACE) inhibitor used to treat hypertension
and CHF.

68. A patient admitted to the emergency department with a history of congestive heart failure
(CHF) is on a nonrebreathing mask at 12 L/min, has a respiratory rate of 31/min and diffuse
bilateral infiltrates on the chest X-ray. You obtain the following arterial blood gas:

pH 7.52
PaCO2 29 mm Hg
PaO2 55 mm Hg
HCO3 22 mEq/L
BE -2 mEq/L

Which of the following should you recommend?


A) initiate mask CPAP or BiPAP™
B) intubate and provide mechanical ventilation
C) administer volume-oriented IPPB therapy
D) maintain the present therapy
Ans: A
Response:
Acutely ill patient with CHF who has diffuse bilateral infiltrates on chest X-ray and refractory
hypoxemia (FIO2 > 0.60; PaO2 < 0.60) likely has pulmonary edema. You can best reverse the
patient's refractory hypoxemia by switching to mask CPAP or BiPAP at a high FIO2. The
added pressure that CPAP/BiPAP provides will decrease the patient's venous return and
pulmonary capillary pressures, thereby aiding in clearance of excess alveoli fluid and
improving oxygenation. Intubation and mechanical ventilation are not needed at this time
because (1) patient has adequate spontaneous ventilation and (2) the problem is likely short-
term in nature.

Page 23
CHAPTER 16
Name: __________________________ Date: _____________

1. A patient who is breathing room air and in a coma as a result of acute carbon monoxide
poisoning has a PaO2 of 95 torr and PaCO2 of 30 torr. Which of the following changes in the
treatment plan should you recommend?
A) administer high concentrations of oxygen
B) initiate mechanical ventilation
C) initiate chest percussion and postural drainage
D) administer mask CPAP
Ans: A
Response:
In this case, the PaO2 is misleading because it represents only dissolved oxygen (CO alters
oxyhemoglobin saturations but not PaO2). The immediate history and finding of coma is a
sufficient basis to recommend administration of high concentrations of oxygen to eliminate
the carbon monoxide. In addition, notice that a ventilator is not needed, since the patient is
able to breathe well enough to have a PaCO2 of 30.

2. IPPB treatments are sometimes given to patients with emphysema to


A) lower the diaphragm
B) abolish the cough reflex
C) decrease the work of breathing
D) increase FRC
Ans: C
Response:
IPPB is used on patients who can not or will not take a deep breath. Its primary function is to
assist the patient by taking over most of work needed to increase alveolar ventilation and take
deep breaths.

3. Which of the following conditions is an indication for the use of CPAP?


A) tension pneumothorax
B) pulmonary embolism
C) pulmonary edema
D) asthma
Ans: C
Response:
CPAP can help overcome the shunting and hypoxemia common in both cardiogenic and
noncardiogenic pulmonary edema. In cardiogenic pulmonary edema, CPAP can help decrease
venous return and thus decrease pulmonary blood flows and pressures. In noncardiogenic
pulmonary edema (e.g., ARDS), CPAP (or PEEP) opens collapsed alveoli and improves the
V/Q ratio.

4. The primary aim in treating cardiogenic pulmonary edema is to:


A) increase venous return to the heart
B) decrease right heart and systemic venous pressures
C) decrease left heart and pulmonary vascular pressures
D) increase pulmonary fluid and blood volume
Ans: C
Response:
The primary aim in treating cardiogenic pulmonary edema is to decrease left heart and
pulmonary vascular pressures.

Page 1
SAN PEDRO COLLEGE COMPETENCY APPRAISAL - OXYGENATION

5. Despite an intensive regimen of incentive spirometry, chest percussion, and nasotracheal


suctioning, a postoperative patient continues to exhibit clinical manifestations of atelectasis
due to large airway obstruction. The best treatment approach in this case is:
A) intubation and mechanical ventilation
B) bedside therapeutic bronchoscopy
C) transtracheal aspiration
D) aerosol therapy with acetylcystine
Ans: B
Response:
Should conservative measures fail in treating atelectasis due to large airway obstruction,
therapeutic bronchoscopy is indicated. This procedure can usually be performed at the bedside
with conscious sedation. Note that data are collected and analyzed in the item, and you are
expected to pick the plan that best matches the data and analysis provided.

6. You should initiate O2 therapy in all of the following cases EXCEPT:


A) treating carbon monoxide poisoning
B) decreasing myocardial work
C) treating absorption atelectasis
D) treating arterial hypoxemia
Ans: C
Response:
Indications for O2 therapy include: documented hypoxemia, acute care situations in which
hypoxemia is common (e.g., shock, trauma, CO poisoning), acute myocardial infarction (to
decrease myocardial workload), and short-term therapy for patients likely to develop
hypoxemia (e.g., during post-anesthesia recovery). Absorption atelectasis is a potential hazard
of supplemental O2 therapy.

7. A patient who just suffered a severe closed head injury is being supported in the CMV mode
of ventilatory support. Which of the following goals should you recommend for the initial 24–
48 hours of ventilatory support?
A) allow as much spontaneous breathing as possible (IMV)
B) maintain a high mean pressure using PEEP at 10–15 cm H2O
C) maintain a PaCO2 of 50–60 torr (deliberate hypoventilation)
D) maintain a PaCO2 of 25–30 torr (deliberate hyperventilation)
Ans: D
Response:
In patients with recent closed head trauma and increased intracranial pressure (ICP),
deliberate 25–30 torr during the first 24–48 hours of care is indicated to help reduce ICP.
Knowing that a low PaCO2 causes cerebral vasoconstriction is required to select the best
answer.

8. You are reviewing a postoperative patient's care plan. The physician has changed the patient's
therapy from incentive spirometry to IPPB. The most likely goal for this change is to:
A) deliver aerosolized bronchodilators
B) improve inadequate alveolar oxygenation
C) prevent lower lobe atelectasis
D) treat progressive respiratory muscle weakness
Ans: D
Response:
IPPB is used instead of incentive spirometry when a patient can not or will not take a deep
breath. The only scenario that fits this indication for IPPB is progressive respiratory muscle
weakness, which would lead to a patient being unable to take a deep breath.

Page 2
SAN PEDRO COLLEGE COMPETENCY APPRAISAL - OXYGENATION

9. Which of the following would you recommend for a patient who has emphysema with chronic
PaCO2 retention and hypoxemia at rest?
A) CPAP
B) incentive spirometry
C) oxygen via nonrebreathing mask
D) low-flow oxygen
Ans: D
Response:
Patients with emphysema may hypoventilate when given moderate to high concentrations of
oxygen (due to elimination of their hypoxic drive). It is therefore recommended that
hypoxemic COPD patients receive no more oxygen than needed to raise their PaO2 to the 55–
60 torr range. Normally, this can be achieved using low-flow (1–2 L/min) oxygen or an air-
entrainment mask that will control the FIO2.

10. Which of the following would be considered objective data to use in the assessment of a
patient in order to formulate a respiratory care plan?
A) dyspnea
B) accessory muscle use
C) airway obstruction
D) bronchodilator thearpy
Ans: B
Response:
Dyspnea is the patient's subjective sensation of difficulty breathing. Accessory muscle use is
an objective observation you make of the patient. Airway obstruction is your assessment of
the problem, and bronchodilator therapy is a component of your recommended plan.

11. Which of the following is the first step in respiratory care protocol application?
A) observe universal precautions at all times
B) review medical records
C) check physician order for respiratory care protocol
D) perform initial patient evaluation
Ans: C
Response:
As with any prescribed therapy, the first step in respiratory care protocol application is always
to check and review the physician's order.

12. All of the following are potential sources of data for identifying patient care-related problems
in a comprehensive quality improvement program EXCEPT:
A) analysis of department budgeting
B) audit of patient medical records
C) examination of incident reports
D) review of patient ratings and complaints
Ans: A
Response:
Audits of patient records, review of incident reports, and assessment of patient ratings and
complaints can all provide date relevant to a quality improvement program. A department's
budget would not normally be a key consideration when investigating the causes of patient
care-related issues.

Page 3
SAN PEDRO COLLEGE COMPETENCY APPRAISAL - OXYGENATION

13. Which of the following could be used as quality assurance outcome criteria to assess the
effectiveness of bronchial hygiene therapy?

I. change in sputum production


II. change in chest X-ray
III. patient subjective response to therapy
IV. change in ABG values or oxygen saturation
A) I and II only
B) II and IV only
C) I, III, and IV only
D) I, II, III, and IV
Ans: D
Response:
All of these data could be used as quality assurance outcome criteria. Remember, outcome
criteria for quality assurance can include both objective and subjective data. If objective data
support improvement, but the patient does not feel any better, that is valuable information that
must be considered in the analysis and plan.

14. As part of your department's quality improvement program, you identify an infection control
problem. Which of the following should be investigated as potential causes of this problem?

I. lack of proper training


II. equipment or supply deficiencies
III. failure to follow set procedures
A) II only
B) III only
C) I and III only
D) I, II, and III
Ans: D
Response:
Lack of proper training, equipment or supply deficiencies, and failure of staff to follow set
procedures should be investigated as potential causes of an infection control problem.

15. Which of the following mechanisms is the best way to objectively assess the effectiveness of
therapy in a quality assurance program?
A) assess the post-treatment status of patients on one or more pre-defined criteria
B) survey a sample of patients who received the specified therapy to determine their
satisfaction
C) compare pre- and post-treatment patient status on one or more pre-defined criteria
D) analyze morbidity and mortality statistics for patients receiving the specified therapy
Ans: C
Response:
Although all these measures provide some indication of therapy effectiveness, comparison of
patients before and after treatment on pre-defined criteria is the best and most objective
measure of individual impact..

Page 4
SAN PEDRO COLLEGE COMPETENCY APPRAISAL - OXYGENATION

16. Which of the following is true regarding patients in the early stages of an asthmatic attack?
A) they all exhibit respiratory alkalosis
B) they always have moderate hypoxemia
C) they have decreased expiratory flows
D) they never respond to beta adrenergics
Ans: C
Response:
Patients in the early stages of an asthmatic attack characteristically exhibit decreased
expiratory flows. SOME (not all) will also exhibit respiratory alkalosis and have moderate
hypoxemia. Most will respond to beta adrenergics.

17. A comatose drug overdose patient breathing room air has a PaO2 of 65 torr and PaCO2 of 70
torr. Which of the following treatment plans would be best for this patient?
A) administer a high concentration of oxygen
B) intubate and initiate mechanical ventilation
C) administer mask CPAP
D) IV administration of a stimulant
Ans: B
Response:
The patient is suffering from hypercapnic respiratory failure with the hypoxemia due to
hypoventilation. Therefore, mechanical ventilation is needed to increase the patient's minute
ventilation and return his PaCO2 to normal. In addition, data indicates the patient is suffering
from a drug overdose, which further reinforces the need for airway protection (intubation) and
ventilatory support.

18. The therapeutic objective in the management of a patient who has a head trauma and is
receiving ventilatory support is to:
A) lower the minute ventilation
B) increase intrathoracic pressure
C) reduce intracranial pressure
D) release air from the pleural space
Ans: C
Response:
Because high intracranial pressure (ICP) can cause permanent brain damage, the therapeutic
objective is to reduce the ICP. This can be accomplished by deliberate hyperventilation.
Reducing the PaCO2 causes cerebral vasoconstriction, which in turn can minimize edema and
swelling.

19. Which of the following conditions is an NOT an indication for the use of PEEP?
A) atelectasis
B) pulmonary embolism
C) pulmonary edema
D) pulmonary shunting
Ans: B
Response:
PEEP (positive end-expiratory pressure) can help overcome the shunting and hypoxemia
common in with atelectasis. In cardiogenic pulmonary edema, PEEP can help decrease
venous return and thus decreases pulmonary blood flows and pressures. In noncardiogenic
pulmonary edema (e.g., ARDS), PEEP opens collapsed alveoli and improves the V/Q ratio.
PEEP is not indicated in the management of pulmonary embolism.

Page 5
SAN PEDRO COLLEGE COMPETENCY APPRAISAL - OXYGENATION

20. Which of the following patients can benefit most from intermittent use of noninvasive positive
pressure ventilation (NPPV)?
A) a patient with muscular dystrophy
B) a patient with pulmonary edema
C) a high cervical spine injury patient
D) a patient with a drug overdose
Ans: A
Response:
Patients who can benefit from NPPV generally fall into one of two categories or types. Type 1
patients have conditions in which cessation of ventilation could lead to imminent death. This
category includes both acutely ill patients (asthma, acute exacerbation of COPD, pulmonary
edema) and those requiring long-term 24-hour support (some quadriplegics, idiopathic
hypoventilation syndrome). Type 2 patients generally require only intermittent support.
Examples of patients in this category include those with chronic neuromuscular and chest wall
diseases, such as muscular dystrophy and kyphoscoliosis.

21. Relative contraindications against using noninvasive positive pressure ventilation (NPPV)
include all of the following EXCEPT:
A) severe upper airway dysfunction
B) copious secretions that require suctioning
C) FIO2 requirements exceeding 40%
D) muscular weakness or fatigue
Ans: D
Response:
Relative contraindications to NPPV include severe upper airway dysfunction, copious
secretions that cannot be cleared by spontaneous or assisted cough, or oxygen concentration
requirements exceeding 40%.

22. Absolute contraindications against using noninvasive positive pressure ventilation (NPPV)
include all of the following EXCEPT:
A) facial burns or trauma
B) need for airway protection
C) copious secretions
D) active upper GI bleeding
Ans: C
Response:
Absolute contraindications against using NPPV include: need for immediate intubation,
hemodynamic instability, active cardiac arrhythmias or ischemia, active upper gastrointestinal
bleeding, uncooperative patient, facial burns or trauma, and the need for airway protection.

23. Despite an intensive regimen of incentive spirometry, chest percussion, and nasotracheal
suctioning, a 180 lb male postoperative patient continues to exhibit signs of atelectasis. His
best and most recent bedside vital capacity measurement was 600 mL. The best treatment
approach in this case would be:
A) intubation and mechanical ventilation
B) intermittent positive pressure breathing
C) transtracheal aspiration
D) aerosol therapy with acetylcystine (Mucomyst)
Ans: B
Response:
Should conservative measures fail in treating atelectasis, intermittent positive pressure
breathing (IPPB) may be indicated before more invasive techniques such as bronchoscopy are
tried.

Page 6
SAN PEDRO COLLEGE COMPETENCY APPRAISAL - OXYGENATION

24. Which of the following modes of ventilatory support is indicated when a precise I:E ratio
must be maintained?
A) assist-control ventilation
B) intermittent mandatory ventilation
C) control mode ventilation
D) pressure support ventilation
Ans: C
Response:
Controlled ventilation is the preferred mode of ventilatory support when one needs to
maintain a precise I:E ratio (as during inverse ratio pressure control ventilation). This mode is
also sometimes used in patients with an unstable or changing ventilatory drive.

25. Indications for O2 therapy includes all of the following EXCEPT:


A) treating carbon monoxide poisoning
B) decreasing myocardial work
C) treating hypoxemia caused by hypoventilation
D) treating arterial hypoxemia
Ans: C
Response:
Indications for O2 therapy include: documented hypoxemia, acute care situations in which
hypoxemia is common (e.g., shock, trauma, CO poisoning), acute myocardial infarction (to
decrease myocardial workload) and short-term therapy for patients likely to develop
hypoxemia (e.g., post-anesthesia recovery). Hypoxemia caused by hypoventilation should be
treated by increasing alveolar ventilation, not increasing the FIO2.

26. Which of the following drug categories would you select if the goal of therapy was to achieve
bronchodilation with minimal cardiac side effects?
A) alpha-adrenergic
B) beta-2 adrenergic
C) cholinergic
D) beta-1 adrenergic
Ans: B
Response:
Beta-2 stimulation causes bronchodilation and vasodilation, mainly by relaxation of smooth
muscle. Drugs that stimulate beta-2 receptors are preferred for bronchodilation because fewer
cardiac side effects accompany their use.

27. You would recommend PEEP to achieve which of the following therapeutic goals?
A) decreased functional residual capacity
B) decreased physiological shunting
C) increased inspiratory capacity
D) increased pulmonary blood flow
Ans: B
Response:
Physiologic shunting occurs when blood travels past nonventilated alveoli, which results in
hypoxemia that does not respond well to O2 therapy (refractory hypoxemia). By recruiting or
opening up these nonventilated alveoli, PEEP tends to decrease physiologic shunting and
overcome refractory hypoxemia.

Page 7
SAN PEDRO COLLEGE COMPETENCY APPRAISAL - OXYGENATION

28. After confirming the physician's order, your next step in implementing a respiratory care
treatment protocol is to:
A) formulate a treatment plan
B) set physician notification criteria
C) review the relevant medical literature
D) conduct an initial patient evaluation
Ans: D
Response:
After confirming the physician's order, your next step in implementing a respiratory care
treatment protocol is to perform an initial patient evaluation. This includes: (1) reviewing the
medical record, (2) initiating patient contact (using universal precautions) and (3) assessing
the patient (e.g., vital signs; chest inspection, palpation, percussion, auscultation; bedside
spirometry; pulse oximetry; and other diagnostic evaluations as needed, such as arterial blood
gases.

29. A respiratory therapist formulates a protocol-based management plan for a patient that
includes the following: indications, therapy, precautions/contraindications, frequency,
monitoring and assessment (including adverse effects), and planned modifications to achieve
optimal efficacy. What essential elements are missing from this management plan?

I. protocol integration into overall patient management


II. specific therapeutic goals and objectives
III. explicit criteria for discontinuing the therapy
A) II and III only
B) I and III only
C) I and II only
D) I, II and III
Ans: D
Response:
An appropriate protocol-based management plan should include the following critical
elements: indications, therapeutic objectives, therapy (including where appropriate delivery
method, frequency, medication dose), precautions/ contraindications, patient/family education
(where appropriate), means to monitor therapy (including adverse effects), means to assess
efficacy of therapy (patient outcomes), planned modifications to achieve optimal efficacy and
outcome, and criteria to discontinue therapy. In addition the plan should identify the physician
and other appropriate health care personnel involved and be properly integrated into the
patient's overall management.

30. change in chest x-ray


III. patient subjective response to therapy
IV. change in ABG values or oxygen saturation
A) I and II only
B) II and IV only
C) I, III and IV only
D) I, II, III and IV
Ans: D
Response:
Quality assurance outcome criteria to assess the effectiveness of bronchial hygiene therapy
could include any of the following: change in sputum production; change in breath sounds of
the affected lung fields; changes in the chest x-ray; changes in arterial blood gas values or
oxygen saturation; and the patient's subjective response to therapy.

Page 8
SAN PEDRO COLLEGE COMPETENCY APPRAISAL - OXYGENATION

31. Aspects of respiratory care that are typically addressed in a comprehensive quality
improvement program include which of the following?

I. effectiveness of care
II. appropriateness of care
III. cost-efficiency of care
A) II only
B) III only
C) I and II only
D) I, II and III
Ans: D
Response:
A comprehensive quality improvement program should be designed to (1) provide ongoing
monitoring of both the quality and appropriateness of respiratory care; (2) assure that
respiratory care methods and procedures are cost-efficient; (3) assure that respiratory care
methods and procedures are effective; and (4) identify, rank, and resolve patient care-related
problems.

32. You are responsible for developing quality assurance criteria for bronchial hygiene therapy.
Which of the following components should be included in these criteria?

I. therapeutic goals
II. appropriateness of care
III. therapy effectiveness
A) I and II only
B) II and III only
C) I and III only
D) I, II and III
Ans: D
Response:
Successful implementation of the quality improvement plan demands that the respiratory care
service develop criteria addressing the therapeutic goals, appropriateness, and means of
evaluating the effectiveness of each specific high-utilization and high-risk procedure.

33. The primary indication for aerosolized ribavirin (Virazole) is in the treatment of:
A) viral hepatitis
B) herpes zoster
C) respiratory syncytial virus
D) influenza A and B
Ans: C
Response:
Ribavirin (Virazole) is a broad spectrum antiviral agent that can be used to treat respiratory
syncytial virus (RSV). Influenza A and B are also susceptible to ribavirin.

Page 9
SAN PEDRO COLLEGE COMPETENCY APPRAISAL - OXYGENATION

34. Clinical indications for cool bland water aerosols include:

I. post extubation
II. post bronchoscopy
III. pneumonia
IV. croup
A) II, III and IV only
B) I, II, III and IV
C) I, II and IV only
D) I and III only
Ans: C
Response:
Cool water aerosols are used for their soothing effect on inflammed tissues of the upper
airway of patients following removal of an artificial airway (extubation), following fiberoptic
bronchoscopy, and in the management of the inflammatory obstruction seen in laryngitis or
croup. The lower temperature of these aerosols may help decrease the swelling and tissue
edema common in these situations.

35. If a physician's goal were to facilitate the mobilization of respiratory tract secretions in a
hospitalized patient with an ineffective cough, which of the following airway clearance
techniques would you recommend?
A) relaxation positioning
B) inspiratory resistive breathing
C) pursed-lip breathing
D) postural drainage and percussion
Ans: D
Response:
The airway clearance technique best suited to help mobilize secretions in a patient with an
ineffective cough is postural drainage (perhaps combined with percussion). Postural drainage
facilitates the mobilization of secretions by patient positioning, using gravity to aid their
removal. This is accomplished by simply placing the segmental bronchus to be drained in a
vertical position relative to gravity.

36. In implementing a respiratory care protocol, you must notify the physician
A) prior to beginning each step in the protocol
B) whenever a notification criterion is triggered
C) after completing each step in the protocol
D) whenever planned modifications are needed
Ans: B
Response:
A medically acceptable respiratory care protocol must include a description of decision-points
and end-points where the physician must be notified. Only when these points (called
notification criteria) are triggered is it necessary to contact the physician. In a well-designed
protocol, one need not contact the physician either before or after each protocol step, nor
when making PLANNED modifications to the therapy.

Page 10
SAN PEDRO COLLEGE COMPETENCY APPRAISAL - OXYGENATION

37. A patient with severe COPD who is receiving oxygen must be observed closely because
which of the following may occur?
A) hypoventilation
B) pneumothorax
C) bronchospasm
D) pulmonary edema
Ans: A
Response:
Many patients with COPD have chronically elevated PCO2 levels. Consequently, their
primary respiratory drive originates not from excessive PCO2 levels, as normally occurs, but
rather because of a low PO2. This is known as the hypoxic drive. FIO2s in excess of 30 to 35%
in such patients can increase PO2 levels to the point where their hypoxic drive and overall
breathing is depressed resulting in hypoventilation.

38. Which of the following would be the most important data utilized to assess a patient's ability
to perform metered dose inhaler (MDI) aerosol therapy?
A) overall general appearance
B) stable vital signs
C) exercise tolerance
D) capacity to follow instructions
Ans: D
Response:
To perform complex tasks such as using an MDI correctly, a patient must be fully oriented
and able to follow instructions.

39. All of the following would be acceptable quality assurance criteria for assessing the
appropriateness of care for protocol-based bronchodilator therapy EXCEPT:
A) the patient must be diagnosed as having reversible airway obstruction
B) the patient must have exhibited signs of bronchospasm not associated with heart failure
C) the patient must show a significant improvement in FEV1% (> 15%) after treatment
D) drug dosing and deliver mode should follow the criteria specified in the protocol
Ans: C
Response:
Requiring the patient to show improvement is a therapeutic effectiveness criterion, not an
appropriateness criterion. All other criteria are suitable for assessing the appropriateness of
care for protocol-based bronchodilator therapy, i.e., appropriate diagnosis, appropriate signs
or symptoms and appropriate procedure or protocol.

40. For which of the following purposes are breathing exercises NOT indicated?
A) to improve the efficiency of alveolar ventilation
B) to improve cardiopulmonary exercise tolerance
C) to improve the delivery of therapeutic aerosols
D) to reverse the progression of chronic disease
Ans: D
Response:
Purposes of breathing exercises include: 1) increasing awareness and promoting better use of
the diaphragm; 2) diminishing/eliminating inefficient accessory muscles use; 3) preventing or
minimizing episodes of dyspnea; 4) improving the efficiency of alveolar ventilation; 5)
improving cough effectiveness; 6) improving delivery of aerosols; 7) improving the
strength/endurance of the respiratory muscles; and 8) improving fitness and cardiopulmonary
exercise tolerance.

Page 11
SAN PEDRO COLLEGE COMPETENCY APPRAISAL - OXYGENATION

41. Two days after a cholecystectomy, a 56-year-old patient who weighs 64 kg (140 lb) is
diagnosed with atelectasis. Treatment has included hourly deep breathing with a volume-
oriented incentive spirometer. Which of the following would indicate a poor response to the
prescribed therapy?
A) a decreased P(A-a)O2
B) vesicular breath sounds
C) a decreased respiratory rate
D) a reduced inspiratory capacity
Ans: D
Response:
Incentive spirometry is used to help increase the patient's inspiratory capacity and treat or
prevent atelectasis. Therefore, a reduced inspiratory capacity in a patient receiving incentive
spirometry indicates that the goal of the therapy is not being met. All other findings are
positive indicators for incentive spirometry.

42. Your patient is receiving aerosolized bronchodilators to treat her asthma. What is the best way
to determine whether this treatment is achieving the desired goal?
A) Measure the patient's MIP before-and-after treatment
B) Measure the patient's inspiratory capacity before-and-after treatment
C) Measure the patient's FEV1% before-and-after treatment
D) Calculate the patient's minute alveolar ventilation
Ans: C
Response:
The best way to determine the effectiveness of bronchodilator therapy at the bedside is to
measure the patient's forced expiratory flows before and after treatment. Either the FEV1% or
the peak expiratory flow rate (PEFR) can be used, although the FEV1% is a more reliable and
valid measure. In general, an improvement of at least 15% between the pre- and post-test
values is needed to indicate reversibility of the obstruction with the bronchodilator.

43. All of the following would indicate a successful outcome for postural drainage therapy
EXCEPT:
A) decreased sputum production
B) normalization in ABGs
C) improved breath sounds
D) improvement in chest X-ray
Ans: A
Response:
Criteria indicating a positive response to postural drainage therapy include 1) change in
sputum production, 2) improvement in breath sounds of lung fields being drained, 3)
improvement in vital signs, 4) improvement in chest x-ray, 5) improvement in arterial blood
gas values or oxygen saturation, 6) increased compliance/lower PIP (on ventilator), and 7)
patient's subjective improvement.

Page 12
SAN PEDRO COLLEGE COMPETENCY APPRAISAL - OXYGENATION

44. On reviewing the results of the attending physician's physical examination of a postoperative
patient's chest, you note 'dull percussion note and bronchial breath sounds - left and right LL.'
Which of the following should be considered for this patient's respiratory care plan?

I. lung expansion therapy


II. bronchial hygiene therapy
III. oxygen therapy
A) I only
B) II only
C) I and III only
D) I, II and III
Ans: C
Response:
The patient likely has either pulmonary infiltrates, atelectasis or consolidation of the affected
areas. Options to consider in formulating a respiratory care plan for this patient should include
lung expansion and oxygen therapy.

45. Which of the following therapeutic strategies can help minimize the likelihood of a patient
developing postoperative atelectasis?

I. early mobilization
II. frequent positional changes
III. deep breathing/coughing
A) II and III only
B) I, II and III
C) I and II only
D) II only
Ans: B
Response:
The best treatment for postoperative atelectasis is prevention. The likelihood of postoperative
atelectasis can be minimized by early mobilization of the patient, frequent positional changes,
and a vigorous regimen of deep breathing and coughing.

Page 13
CHAPTER 17

Name: __________________________ Date: _____________

1. During a “code blue” or other medical emergency, a physician is having trouble starting an
intravenous line. Which of the following drugs can be placed down an endotracheal tube
during emergency life support?

I. naloxone
II. lidocaine
III. atropine
IV. epinephrine
A) II and IV only
B) I, II, and III only
C) III and IV only
D) I, II, III, and IV
Ans: D
Response:
Naloxone, lidocaine, atropine, and epinephrine all can be administered via the endotracheal
tube during emergency life support.

2. Which of the following techniques can be used to determine effectiveness of ventilation?

I. looking for the rise and fall of the victim’s chest


II. feeling for air exchange near the victim’s mouth and nose
III. listening for audible breathing efforts from the victim
A) I and II only
B) II and III only
C) I and III only
D) I, II, and III
Ans: D
Response:
Once an airway has been opened and ventilation started, you should assess its effectiveness by
looking for the rise and fall of the victim's chest and listening for breathing efforts, which
should be audible. You can also feel for air exchange by placing a check near the victim's
mouth and nose.

3. At the onset of adult mouth-to-mouth or mouth-to-mask ventilation, you should:


A) provide two normal breaths, then assess the pulse
B) provide four fast, shallow breaths, then assess the pulse
C) provide four slow, deep breaths, then immediately start compressions
D) provide one very slow breath, then immediately start compressions
Ans: A
Response:
According to the American Heart Association guidelines, at the onset of adult mouth-to-
mouth or mouth-to-mask ventilation, once the airway is open, you should give two normal
breaths, each lasting about 1 second, then assess the pulse for up to 10 seconds.

Page 1
SAN PEDRO COLLEGE COMPETENCY APPRAISAL - OXYGENATION

4. After two attempts of ventilating an infant in respiratory arrest, you still cannot deliver
breaths. At this point you should:
A) apply back blows, followed by chest thrusts
B) try to ventilate again with smaller puffs
C) apply six to ten strong abdominal thrusts
D) go on to provide external cardiac compressions
Ans: A
Response:
After two failed attempts to ventilate an infant in respiratory arrest, there is most likely an
obstruction. For older patients, an obstructed airway is addressed via abdominal thrusts (the
Heimlich maneuver). However, for infants back blows are combined with chest thrusts. Chest
thrusts may also be used for pregnant women and in markedly obese persons. Both procedures
are normally followed by checking the airway and removal of any obstruction.

5. The ideal ratio of chest compressions to rescue breaths that should be given by a single
rescuer during a cardiopulmonary resuscitation (CPR) attempt on an adult is:
A) 5:1
B) 15:2
C) 2:15
D) 30:2
Ans: D
Response:
In accordance with AHA guidelines, in one- and two-rescuer CPR for adults, the ratio for
chest compressions to breaths is 30 compressions to every two breaths, with a reassessment
after five cycles.

6. When transporting critically ill patients who are receiving supplemental oxygen in
unpressurized aircraft, it is often necessary to make which adjustment in FIO2 in order to
maintain adequate oxygenation?
A) increase the FIO2
B) decrease the FIO2
C) increase ventilation
D) no change to ventilation or FIO2 is needed
Ans: A
Response:
In accordance with Dalton's law, the partial pressure of oxygen at sea level is 160 torr, versus
only 134 torr at the increased altitude. As a result, in order to maintain acceptable
oxygenation, it is often necessary to increase the FIO2 during air transport. For example, a
patient on room air would need an FIO2 of approximately 31% to maintain the same PaO2 at
an altitude of 10,000 feet.

7. The proper rate and depth of external chest compressions for an adult is:
A) rate of 80–100/min with a depth of one inch
B) rate of 70–80/min with a depth of 1 to 1½ inches
C) rate of 100/min with a depth of 1½ to 2 inches
D) rate of 60–80/min with a depth of 1½ to 2 inches
Ans: C
Response:
For adult resuscitation, a chest compression rate of approximately 100/min with a depth of 1½
to 2 inches should be achieved.

Page 2
SAN PEDRO COLLEGE COMPETENCY APPRAISAL - OXYGENATION

8. The initial energy level for defibrillation for ventricular fibrillation is:
A) 100 joules
B) 200 joules
C) 300 joules
D) 360 joules
Ans: D
Response:
The initial energy level for defibrillation (via typical monophasic device) is 360 joules, not
200 joules per the old AHA ACLS guidelines. Individual subsequent shocks are given after
each five cycles, also at 360 joules.

9. All of the following monitoring equipment is mandatory when transporting a critically ill
patient within or outside of the hospital except:
A) end-tidal CO2 monitor
B) an oxygen source/delivery device
C) blood pressure monitor/cuff
D) cardiac monitor/defibrillator
Ans: A
Response:
A blood pressure monitor (or standard blood pressure cuff), an oxygen source and delivery
device, as well as a cardiac monitor/defibrillator should accompany every critically ill patient
on transport.

10. If the number of ventilators needed to support patients in respiratory failure after a chemical
disaster is insufficient to meet the need, your initial response should be to:
A) contact other local facilities and arrange for patient transfers
B) call and order additional backup ventilators from the vendor
C) enlist non–respiratory personnel to perform manual ventilation
D) assign patients without needed equipment to triage priority Black
Ans: C
Response:
Preparedness planning for the respiratory care department includes estimating the number of
patients who may need ventilatory support, determining number of staff members required to
meet patient needs, and having in place a staff emergency call-back procedure. Should the
available equipment and personnel not be able to meet patient needs, the first step would be to
enlist non–respiratory personnel to perform manual ventilation using disposable BVMs. Once
adequate immediate patient support is ensured, you can attempt to get additional backup
equipment and/or make arrangements to transfer patients to other facilities.

11. One of the best ways to determine whether mouth-to-mouth breathing is effective during a
CPR attempt is to watch the patient's:
A) Chest rise when a breath is given
B) Skin color for cyanosis
C) Heart rate for a return to normal
D) Pupils for reaction to light
Ans: A
Response:
According to AHA guidelines, the best way to determine whether mouth-to-mouth breathing
is effective during CPR is to watch the patient's chest rise as the breath is administered.

Page 3
SAN PEDRO COLLEGE COMPETENCY APPRAISAL - OXYGENATION

12. To open an obstructed airway of a patient who has a possible cervical fracture, you should
A) perform a cricothyroid puncture with a 13 gauge needle
B) flex the patient's head after a cervical collar is in place
C) hyperextend the patient's head after a cervical collar is in place
D) perform the jaw thrust maneuver
Ans: D
Response:
To open an obstructed airway of a patient who has a possible cervical fracture, you should
perform the jaw thrust maneuver WITHOUT the head tilt.

13. Which of the following arrhythmias are considered potentially lethal?

I. ventricular fibrillation
II. ventricular tachycardia
III. 3rd degree heart block
IV. supraventricular tachycardia
A) I, II and III only
B) I and II only
C) II, III and IV only
D) I, II, III and IV
Ans: A
Response:
3rd degree heart block (atrial-venticular conduction disassociation), ventricular tachycardia,
and ventricular fibrillation are all potentially lethal arrhythmias.

14. After attaching the automatic external defibrillator (AED) sensor/electrodes on an arrest
victim in a sub-acute facility, you press the analyze button, and the AED gives you a "shock
indicated" message, and charges to the preset energy level. What do you do next?
A) press the SHOCK button
B) check for signs of circulation
C) shout "clear!" and make sure no one is touching the victim
D) begin chest compressions immediately
Ans: C
Response:
Before administering the shock via any defibrillator, rescuers must make sure that you and
any other caregivers or bystanders are clear of the patient.

Page 4
SAN PEDRO COLLEGE COMPETENCY APPRAISAL - OXYGENATION

15. A 70-year-old patient was found unresponsive by EMS. Upon arrival in the emergency
department, the patient is cyanotic, apneic, and pulseless. Manual ventilation is initiated by
the respiratory therapist. The cardiac rhythm below is noted in the monitor. Which of the
following should the respiratory therapist recommend next?

A) administer chest compressions


B) perform cardioversion
C) perform defibrillation
D) administer epinephrine
Ans: C
Response:
The patient has ventricular fibrillation and should be defibrillated without delay.

16. When assisting a physician with needle thoracostomy for a patient with a confirmed
pneumothorax, you should advise the physician that the needle should be inserted in what
anatomic location?
A) over the fourth rib, mid-sternal line
B) over the fourth rib, mid-clavicular line
C) over the second rib, mid-clavicular line
D) under the fourth rib, laterally
Ans: C
Response:
When assisting a physician with an emergency needle decompression of a tension
pneumothorax, you should recommend that it be inserted over the second rib, in the mid-
clavicular line.

17. For single-rescuer resuscitation of a child (1-8 years old), the proper rate, depth of external
chest compressions and compression to breath ratio is:
A) Rate of 100/min, depth of one inch and ratio of 30:2
B) Rate of 80/min, depth of 1 to 1½ inches and ratio of 15:2.
C) Rate of 100/min, depth of 1 to 1½ inches and ratio of 30:2
D) Rate of 80/min, a depth of 1½ to 2 inches and ratio of 30:2.
Ans: C
Response:
For resuscitation a child (1-8 years old), a chest compression rate of approximately 100/min,
depth of 1 to 1½ and compression to breath ratio of 30:2 should be achieved.

Page 5
SAN PEDRO COLLEGE COMPETENCY APPRAISAL - OXYGENATION

18. A patient in ventricular fibrillation should receive IV epinephrine at what dose and frequency?
A) 10 mg every 1-2 min
B) 1 mg every 3-5 min
C) 1 mg every 6-8 min
D) 0.10mg every 9-10 min
Ans: B
Response:
A patient in ventricular fibrillation should receive 1 mg epinephrine by IV push every 3-5
min.

19. During an in-hospital transport of an orally intubated patient, the respiratory therapist notes a
drop in SpO2 to 85%, gurgling sounds coming out of the patient's mouth and mild respiratory
distress is observed. Which of the following actions would be appropriate at this time?

I. stop the transport immediately


II. call the attending physician
III. ask the nurse to sedate the patient
IV. assess the chest for bilateral breath sounds
A) I and III only
B) I and II
C) II and IV only
D) I and IV only
Ans: D
Response:
Respiratory compromise and gurgling sounds are signs of a potential endotracheal tube cuff
leak or dislodgment. The transport should be immediately stopped and bilateral breath sounds
and epigastric sounds assessed for proper tube position. The tube cuff should be re-inflated if
needed. In cases of unplanned extubations, the patient should be immediately ventilated with a
manually resuscitator bag.

20. One of the best ways to ensure that hospital healthcare workers are prepared to deal with
disaster emergencies is to:
A) educate the community on disaster management
B) conduct regular emergency response training and disaster drills
C) run in-service classes on disaster management
D) provide self-teaching materials on dealing with disasters
Ans: B
Response:
In accordance with several authoritative sources, regular training and disaster response drills
is the best way to prepare hospital healthcare workers to deal with disaster emergencies.

Page 6
SAN PEDRO COLLEGE COMPETENCY APPRAISAL - OXYGENATION

21. During cardiopulmonary resuscitation, manual ventilation of a patient with a self-inflating


bag-valve-mask device fails to adequately inflate the patient's chest. You should do which of
the following?
A) intubate and mechanically ventilate the patient
B) switch to gas-powered resuscitator with mask
C) reposition the patient's head, neck and mask
D) insert a laryngeal mask airway (LMA)
Ans: C
Response:
The most common difficulty with bag-valve-mask is upper airway obstruction due to
improper positioning of the head/neck or a mask leak. To improve ventilation, you should first
reposition the patient's head, neck (both the head tilt-chin lift and jaw thrust can should
improve airway patency and optimize BVM ventilation). If an inadequate mask seal is causing
leakage, first try to reposition it. If that fails, you should have someone else tightly apply the
mask for you.

22. During a short pause from resuscitation of a child in the emergency department, you cannot
palpate a carotid pulse but observe the following rhythm on the ECG monitor:

Which of the following actions should you take at this time?


A) resume cardiac compressions and ventilation
B) discontinue compressions and monitor the patient
C) recommend cardioversion at 100 Joules
D) recommend epinephrine administration
Ans: A
Response:
The ECG indicates a normal sinus rhythm. However, one often can be fooled by a monitor
display. In this case the presence of normal electrical activity in combination with a lack of a
palpable pulse indicates pulseless electrical activity (PEA). Because PEA is associated with
minimal cardiac output, you must continue cardiac compressions (and ventilation). Always
treat the patient, not the monitor!

23. The ideal ratio of chest compressions to breaths that should be given when two health care
professionals provide cardiopulmonary resuscitation (CPR) to a child (1-8 years old) is:
A) 5:1
B) 15:2
C) 3:1
D) 30:2
Ans: B
Response:
In accordance with AHA guidelines, in two-person CPR for children 1-8 years old, the ratio
for chest compressions to breaths is 15 compressions to every two breaths.

Page 7
SAN PEDRO COLLEGE COMPETENCY APPRAISAL - OXYGENATION

24. The ideal ratio of chest compressions to breaths that should be given when two health care
professionals provide resuscitation to a newborn infant/neonate is:
A) 5:1
B) 15:2
C) 3:1
D) 30:2
Ans: C
Response:
In accordance with AHA guidelines, for neonatal resuscitation there should be a 3:1 ratio of
compressions to ventilations, with 90 compressions and 30 breaths to achieve approximately
120 events per minute. Respirations, heart rate, and color should be reassessed about every 30
seconds, and coordinated chest compressions and ventilations should continue until the
spontaneous heart rate is  60/min and improving.

25. When two health professionals are resuscitating an infant (< 1 year old), the proper rate, depth
of external chest compressions and compression to breath ratio is:
A) Rate of 100/min, depth of ½ to 1 inch, ratio of 15:2
B) Rate of 120/min, depth of 1 to 1½ inches, ratio of 15:2
C) Rate of 100/min, depth of 1 to 1½ inches, ratio of 30:2
D) Rate of 120/min, depth of ½ to 1 inches, ratio of 30:2
Ans: A
Response:
When two health professionals are resuscitating an infant (< 1 year old), a chest compression
rate of approximately 100/min should be maintained at a depth of 1/3 to 1/2 the depth of chest
or about 1/2 to 1 inch, with a compression to breath ratio of 15:2.

26. Which of the following ACLS drugs is used to treat most bradycardias, PEA or asystole?
A) vasopressin
B) atropine
C) amiodarone
D) lidocaine
Ans: B
Response:
During ACLS, atropine is the most commonly used drug to treat bradycardias, PEA or
asystole. Epinephrine is a cardiac stimulant and vasoconstrictor. Vasopressin (ADH) is also a
vasoconstrictor. Lidocaine is an anti-arrhythmic commonly used to treat ventricular
fibrillation and ventricular tachycardia.

27. You are transporting a patient in an unpressurized airplane at a cruising altitude of 10000 ft
(PB = 523 mm Hg). The patient was receiving 40% oxygen at sea level. What FIO2 should be
provided to this patient at this cruising altitude?
A) 0.30
B) 0.50
C) 0.60
D) 0.70
Ans: B
Response:
To compute the FIO2 needs of a patient at altitude compared to sea level, multiply the FIO2 at
sea level times [760/PB altitude]. In this case, FIO2 needed at altitude = 0.40 x [760/523] = .58
or about 60% O2.

Page 8
CHAPTER 18

Name: __________________________ Date: _____________

1. You are assisting a physician performing a bronchoscopy on a spontaneously breathing patient


who is alert, awake, and anxious. Which of the following medications should you recommend
for the patient's anxiety prior to the procedure?
A) lidocaine HCI (Xylocaine)
B) vecuronium bromide (Norcuron)
C) epinephrine 1:10,000 solution
D) midazolam HCI (Versed)
Ans: D
Response:
Benzodiazepines like midazolam (Versed) are usually recommended for the treatment of
anxiety prior to and during conscious (moderate) sedation procedures. The effects of
medication such as Versed are also reversible, if the need arises.

2. While performing a fiberoptic bronchoscopy, a patient's SpO2 drops from 91% to 84%. Which
of the following actions would be appropriate?

I. applying suction through the scope’s open channel


II. giving O2 through the scope’s open channel
III. increasing the cannula or mask O2 flow
A) I and II
B) II and III
C) I and III
D) I, II, and III
Ans: B
Response:
During bronchoscopy, the patient's oxygenation should be monitored continuously via pulse
oximeter. If desaturation occurs, the FIO2 should be increased during the procedure, or the
procedure can be halted and O2 can be given through the bronchoscope's open channel.

3. What initial energy level would you recommend to a physician preparing to perform
cardioversion on a patient in atrial flutter?
A) 50 joules
B) 150 joules
C) 200 joules
D) 360 joules
Ans: A
Response:
In accordance with AHA guidelines, the initial energy level for performing cardioversion for a
patient in atrial flutter should be 50 joules. Energy levels may be increased to 100, 200, or
more joules if initial attempts are unsuccessful.

4. In what position should you place an patient in order to facilitate a thoracentesis procedure?
A) lying flat, supine, head on pillow
B) lying flat, prone, feet raised 12 inches
C) sitting up and leaning slightly forward and supported in front
D) semi-Fowler's, with knees raised
Ans: C
Response:
When helping position a patient for a thoracentesis, you should sit the patient up and leaning
slightly forward with adequate support in front.

Page 1
SAN PEDRO COLLEGE COMPETENCY APPRAISAL - OXYGENATION

5. You are assisting a physician performing oral intubation of a 45-kg (98-lb.) adult female
patient. Which of the following endotracheal tube sizes should you select for this patient?
A) 6.0 mm
B) 7.0 mm
C) 8.0 mm
D) 9.0 mm
Ans: B
Response:
For adult females, an ET tube between 7.0 and 8.0 mm is usually selected. Since a 45-kg (98-
lb.) woman is small, a 7.0 would be appropriate.

6. You are caring for a mechanically ventilated patient who has been orally intubated for 2
weeks and has failed multiple weaning attempts over the past 2 days. What should you
recommend at this time?
A) immediate extubation
B) placing a smaller oral ET tube
C) performing a tracheostomy
D) placing a nasopharyngeal airway
Ans: C
Response:
A tracheostomy should be performed because recent weaning has been unsuccessful and it is
likely that the patient will remain mechanically ventilated for some time. In addition,
trachestomy tubes may actually facilitate weaning because they are much shorter than ET
tubes and therefore impose less resistance on the patient during weaning.

7. Which of the following statements regarding cardioversion are false?


A) it is indicated for atrial fibrillation and atrial flutter
B) synchronized shock helps prevent the procedure from causing an arrhythmia
C) it is the primary electrical therapy for ventricular fibrillation
D) direct current discharges of 50–100 joules are applied to adults
Ans: C
Response:
Cardioversion is the application of a QRS-synchronized shock to the myocardium.
Synchronization helps avoid causing a cardiac arrhythmia. Cardioversion is the primary
electrical therapy when the arrhythmia features distinct QRS complexes, as in all
supraventricular arrhythmias and ventricular tachycardia. Generally, discharges of 50–100
joules are sufficient to restore a normal cardiac rhythm in adults. Larger energy levels
generally of 360 joules are often indicated for ventricular fibrillation.

8. The physician asks for your input regarding chest tube placement for a patient with a
pneumothorax. In what anatomic location should you recommend that the tube be inserted?
A) the second intercostal space at the midclavicular line
B) the second or third intercostal space at the anterior axillary line
C) the fourth or fifth intercostal space at the midclavicular line
D) the fourth or fifth intercostal space at the anterior axillary line
Ans: D
Response:
In most instances, a chest tube is generally inserted through the fourth or fifth intercostal
space at the anterior axillary line.

Page 2
SAN PEDRO COLLEGE COMPETENCY APPRAISAL - OXYGENATION

9. A physician is about to perform cardioversion on a patient with unstable atrial flutter who is
receiving oxygen via a nonrebreather mask. After the initial shock has been delivered, the
patient's SpO2 drops to 85%, respirations become slow and shallow, and heart rate is now 82
beats/minute with normal sinus rhythm. The next immediate action should be:
A) open the airway, assist with a manual ventilator bag at 100% oxygen
B) administer 2 mg of naloxone (Narcan)
C) deliver another synchronized shock
D) intubate and place on mechanical ventilation
Ans: A
Response:
The next immediate action should be to open the patient's airway and provide for proper
oxygenation and ventilation. The patient's airway might have obstructed during the procedure
due to oversedation or the patient's own anatomy.

10. You are assisting a physician with an elective intubation. In addition to auscultation, what
methods should you recommend to confirm proper tube placement?

I. capnometry
II. esophageal detection device
III. pulse oximetry
IV. CO2 colorimetry
A) I, II, and IV only
B) II, III, and IV only
C) I, II, and III only
D) I, III, and IV only
Ans: A
Response:
Besides auscultation of chest and stomach, other methods like observation of chest movement,
tube length (cm to teeth), esophageal detection devices, capnometry, colorimetry, and
fiberoptic laryngoscopy can be used to confirm ET tube position at bedside. In addition, a
chest X-ray should also be ordered, which can help determine proper tube placement.

11. All of the following are potential hazards of thoracentesis, except:


A) bleeding
B) barotrauma
C) infection
D) liver laceration
Ans: B
Response:
Potential hazards of thoracentesis include bleeding, infection and liver puncture. In assessing
such patients during and after this procedure, you should be mindful of them.

12. What initial energy level would you recommend to a physician preparing to perform
cardioversion on a patient in atrial fibrillation?
A) 50 joules
B) 100 joules
C) 200 joules
D) 360 joules
Ans: B
Response:
In accordance with AHA guidelines, it is recommended that an initial energy level for
performing cardioversion for a patient in atrial fibrillation should be 100 joules. Energy levels
may be increased to 200 or more joules if initial attempts are unsuccessful.

Page 3
SAN PEDRO COLLEGE COMPETENCY APPRAISAL - OXYGENATION

13. A patient exhibits mild hypoxemia (SPO2 92%) after a fiberoptic bronchoscopy procedure.
Which of the following actions would you recommend?
A) continue O2 therapy and reassess in 4 hours
B) administer a benzodiazepine (e.g., Valium or Versed)
C) administer a racemic epinephrine aerosol treatment
D) have the patient refrain from eating or drinking
Ans: A
Response:
Hypoxemia that occurs during bronchoscopy may persist after completion. Oxygen therapy
should be maintained for up to 4 hours. Adequate oxygenation via pulse oximetry should be
confirmed before therapy is discontinued.

14. A first year resident asks you to assist him during an elective intubation of an adult patient.
Although the patient's head and neck are properly positioned, the resident is having difficulty
visualizing the larynx/cords with the laryngoscope. Which of the following procedures could
you perform to help the resident intubate the patient?
A) apply downward pressure on the cricoid cartilage
B) rotate the patient's head 45 degrees to the right
C) place the patient in the lateral decubitus position
D) using a gauze pad, grab the tongue and pull on it
Ans: A
Response:
Applying downward pressure on the cricoid cartilage during intubation (the Selleck
maneuver) can help provide better visualization of vocal cords. The same maneuver also is
used to help prevent gastric distention during bag-valve-mask ventilation and to prevent
aspiration of gastric contents during rapid sequence induction prior to intubation.

15. An anesthesiologist is planning a rapid sequence intubation of a spontaneously breathing adult


patient in the surgical ICU and asks that you pre-oxygenate the patient. Which of the
following would be the best approach to assure that this patient is properly pre-oxygenated?
A) bag the patient using a bag-valve-mask at 15 L/min for 30 seconds
B) place the patient on a nonrebreathing mask at 5 L/min for 5 minutes
C) bag the patient using a bag-valve-mask at 15 L/min for 5 minutes
D) provide 100% O2 at 12 L/min via jet nebulizer + aerosol mask for 5 minutes
Ans: C
Response:
Prior to rapid sequence induction for endotracheal intubation, patients should breath 100%
oxygen for at least 5 minutes. Of the options provided only a well-fitted bag-valve-mask (with
reservoir) at 15 L/min can provide 100% oxygen. In addition, at least 5 minutes are needed to
ensure complete nitrogen washout.

16. You are assisting a physician performing oral intubation of a 70 kg adult male patient. Which
of the following endotracheal tube sizes would you select for this patient?
A) 7.5 mm
B) 8.5 mm
C) 9.5 mm
D) 10.0 mm
Ans: B
Response:
For an average-sized adult male patient requiring endotracheal intubation, an 8.0 to 9.0 mm
ID tube is satisfactory. In general, you should always recommend selecting a tube at the larger
end of this range first.

Page 4
SAN PEDRO COLLEGE COMPETENCY APPRAISAL - OXYGENATION

17. You are assisting a physician performing a chest tube insertion for a spontaneously breathing
patient. The patient is alert, awake, and anxious. Which of the following medications should
you recommend be administered prior to the procedure?
A) lidocaine HCI (Xylocaine)
B) midazolam HCI (Versed)
C) ketamine hydrochloride
D) necuronium bromide (Norcuron)
Ans: B
Response:
Benzodiazepines like Midazolam (Versed) are usually recommended for the treatment of
anxiety prior and during procedures requiring moderate sedation, such as chest tube insertion,
bronchoscopy and cardioversion.

18. You are asked by a physician to assist in monitoring a patient during a chest tube insertion.
Which of the following would you recommend to monitor this patient for this procedure?

I. vital signs (pulse, respiratory rate)


II. heart rhythm via electrocardiogram (ECG)
III. forced expiratory flow rates (e.g., FEV1)
IV. arterial oxygen saturations via oximetry
A) I, II and III only
B) I, II and IV only
C) II, III and IV only
D) III and IV only
Ans: B
Response:
Patient monitoring before, during and immediately following many special procedures
including chest tube insertion and fiberoptic bronchoscopy procedures is often done by the
respiratory therapist. This assessment should include ongoing assessment of the pulse,
respiratory rate, and ECG. In addition, pulse oximetry can be used to monitor changes in
arterial oxygen saturations.

19. Which of the following are appropriate orders before an elective fiberoptic bronchoscopy
procedure scheduled for the next morning?

I. take nothing by mouth (NPO) after midnight


II. check coagulation studies (PT, PTT, INR)
III. establish a heparin lock and premedicate with diazepam (Valium)
A) II and III
B) I and II
C) I, II, and III
D) I and III
Ans: C
Response:
To reduce the risk of aspiration of stomach contents, patients scheduled for elective
bronchoscopy should refrain from food or drink for at least 8 hours prior to the procedure.
Vascular access should be obtained before the procedure (via an IV or heparin lock) and the
patient should be premedicated one to two hours in advance generally with a benzodiazepine
tranquilizer (such as Versed). The patient should be moderately sedated but alert enough to
follow commands, such as taking a deep breath. Finally, blood coagulation results should be
obtained and reviewed for prolonged clotting times, particularly if biopsy is likely.

Page 5
SAN PEDRO COLLEGE COMPETENCY APPRAISAL - OXYGENATION

20. If the first attempt at cardioversion is unsuccessful, what action should take place to avoid the
delivery of an unsynchronized shock during subsequent attempts?
A) immediately increase the energy level to 360 joules
B) ensure the “unsynchronized” mode is activated
C) ensure the “synch” mode is activated
D) immediately begin CPR
Ans: C
Response:
Most defibrillators default back to unsynchronized mode after delivery of a synchronized
shock. In order to avoid the delivery of an unsynchronized shock on subsequent cardioversion
attempts, ensure the “synch” mode is activated.

21. A surgical resident has asked that you assist her in an elective tracheotomy procedure on an
orally intubated patient. Which of the following would be an appropriate action?
A) remove the oral tube before the tracheotomy is performed
B) pull the oral tube only after the tracheostomy tube is placed
C) withdraw the oral tube 2-3 inches while the incision is made
D) remove the oral tube just prior to tracheostomy tube insertion
Ans: D
Response:
If an endotracheal tube is in place during a tracheotomy, it should not be removed until just
prior to the insertion of the tracheostomy tube, as this ensures a patent airway and provides
additional stability to the trachea during the procedure. Once the stoma is created and the
tracheostomy tube is selected and prepared for insertion, the endotracheal tube may be
removed.

22. You are asked to position a patient for orotracheal intubation. You should place the patient's
head
A) in the sniffing position
B) straight with the torso, with the neck hyperextended
C) tilted forward toward the chest
D) turned to the right, with the neck hyperextended
Ans: A
Response:
When preparing to intubate a patient, you should place the head in a sniffing position. i.e.,
elevating the patient's head with pads/towels under the skull with extension of the head at the
atlanto-occipital joint. This serves to align the oral, pharyngeal, and laryngeal axis, so that the
passage from the mouth the glottic opening is nearly a straight line.

Page 6
SAN PEDRO COLLEGE COMPETENCY APPRAISAL - OXYGENATION

23. A physician asks you to assist during a bronchoscopy of an intubated 8-year-old child who has
right upper lobe atelectasis. Which of the following monitors should the respiratory therapist
recommend for this procedure?

I. cardiac output
II. transcutaneous O2/CO2
III. pulse oximeter
IV. ECG monitor
A) I and II only
B) I and III only
C) III and IV only
D) I, III, and IV only
Ans: C
Response:
During conscious (moderate) sedation the patient's oxygenation status and cardiac rhythm
must be constantly monitored and assessed due to possible airway and cardiac complications
during the procedure.

24. You are assisting a resident perform orotracheal intubation of a semiconscious adult patient in
the emergency room. Despite three failed attempts by the resident to place the tube in the
trachea, you still are able to provide the patient with adequate ventilation and oxygenation
using a bag-valve-mask system. Which of the following would you recommend to the resident
as the next course of action?
A) suspend intubation efforts, continue bagging the patient, consider alternatives
B) try at least one more oral intubation attempt before abandoning the effort
C) perform an emergency cricothyrotomy and position an ET tube in the stoma
D) switch to the nasal route and perform a blind nasotracheal intubation
Ans: A
Response:
In a "can't intubate, but can ventilate" scenario, it is best to continue bag-valve-mask
ventilation with oxygen and defer intubation until either a more experienced laryngoscopist
arrives, or to consider an alternative technique, e.g., placement of a laryngeal mask airway or
fiberoptic intubation.

25. You are assisting a resident perform orotracheal intubation of an semiconscious adult patient
in the emergency room. After three failed attempts by the resident to place the tube in the
trachea, your efforts to ventilate the patient using a bag-valve-mask system fail. Which of the
following would you recommend to the resident as the next course of action?
A) carry out at least one more intubation attempt
B) perform a percutaneous dilational tracheotomy
C) suspend intubation efforts and continue bagging
D) insert a laryngeal mask airway
Ans: D
Response:
In a "can't intubate, can't ventilate" scenario, it its best to follow the emergency pathway of the
ASA difficult airway algorithm, which includes an initial attempt to restore ventilation via a
laryngeal mask airway.

Page 7
CHAPTER 19
Name: __________________________ Date: _____________

1. The essential goals of pulmonary rehabilitation include all of the following except:
A) improve exercise tolerance
B) reduce perceived dyspnea
C) improve health related quality of life
D) reverse lung damage
Ans: D
Response:
The goals of pulmonary rehabilitation include improving exercise tolerance, enhancing health-
related quality of life, and reducing perceived dyspnea in participants. The goals do not
include reversing lung damage.

2. In order to meet the goals of pulmonary rehabilitation and return the patient to the highest
level of functional capacity, all of the following approaches may be used except:
A) multidisciplinary approach
B) education and related counseling
C) mandatory participation for all COPD patients
D) flexible approaches to meet varied patients' needs
Ans: C
Response:
The goals of pulmonary rehabilitation, which include returning the patient to the highest level
of functional capacity, may be achieved by using a multidisciplinary approach, education and
related counseling, as well as flexible approaches to meet varied patients' needs, but not
through mandatory participation.

3. Pulmonary rehabilitation programs should include which of the following components:

I. patient education
II. breathing techniques and exercises
III. physical reconditioning
IV. smoking cessation and related counseling
A) III and IV
B) I, II, and III
C) II, III, and IV
D) I, II, III, and IV
Ans: D
Response:
Pulmonary rehabilitation programs will include patient education, breathing retraining,
physical reconditioning, and smoking cessation and counseling. All of these areas are essential
to the scope of practice as delineated for pulmonary rehabilitation.

4. Patients in pulmonary rehabilitation can perform physical conditioning exercises safely if they
exercise to what percent of their target heart rate?
A) 50%
B) 75%
C) 90%
D) 100%
Ans: B
Response:
In order to achieve cardiovascular benefit and improvement during physical reconditioning,
patients should safely exercise to 75% of their target heart rate. Working at levels less than
this do not result in any significant improvement, and working at levels greater than this
places the patient at risk for cardiovascular consequences.

Page 1
SAN PEDRO COLLEGE COMPETENCY APPRAISAL - OXYGENATION

5. What is the primary purpose of patient documentation in pulmonary rehabilitation?


A) demonstrate patient involvement and note outcomes
B) insurance reimbursement
C) investigative research
D) patient documentation is not required in pulmonary rehabilitation
Ans: A
Response:
The primary purpose of patient documentation in pulmonary rehabilitation is to demonstrate
patient involvement and note outcomes. The other responses are valid but not the primary
reason for documenting patient activity.

6. Emergency situations that home mechanical ventilation caregivers must be trained to


recognize and manage include all of the following except:
A) ventilator or power failure
B) tension pneumothorax
C) artificial airway obstruction
D) ventilator circuit problems
Ans: B
Response:
Emergency situations that caregivers must be trained to recognize and properly deal with
include ventilator or power failure, ventilator circuit problems, airway emergencies, and
cardiac arrest.

7. You have been asked to organize a patient/family education program as part of a discharge
plan for a patient requiring home ventilatory support. Which of the following methods would
be best for training the family in operation of the ventilator chosen?
A) put the patient on the selected device while still hospitalized
B) set up and review the ventilator after the patient gets home
C) show the family the ventilator in a full-day session
D) give the family the operating manual for the ventilator
Ans: A
Response:
Ideally, the patient should be placed on the actual ventilator that will be used in the home
setting before discharge.

8. Which of the following are acceptable indicators of hypoxemia for purposes of justifying
home oxygen therapy?

I. A resting PaO2 of 55 torr (room air)


II. A nocturnal fall in SaO2 from 95% to 91%
III. A resting arterial Hb sat of 84% (room air)
A) I and II only
B) II and III only
C) I and III only
D) I, II, and III
Ans: C
Response:
The threshold for qualifying for oxygen therapy under Medicare is a resting PaO2 of 55 torr or
less or an SaO2 of 88% or less for a single pulmonary diagnosis (COPD), or a PaO2 between
56 and 59 torr or an SaO2 of 89% with a secondary diagnosis (COPD and cor pulmonale).

Page 2
SAN PEDRO COLLEGE COMPETENCY APPRAISAL - OXYGENATION

9. Basic principles of infection control in the home care setting include which of the following?

I. avoiding visits by friends with respiratory infections


II. having caregivers follow proper handwashing technique
III. incinerating all disposable equipment and supplies
A) I and II only
B) II and III only
C) I and III only
D) I, II, and III
Ans: A
Response:
Basic principles of infection control in the home care setting include: (1) proper handwashing
technique by all caregivers; (2) discouraging visits to the patient from friends or relatives with
respiratory infections; (3) proper cleaning and disinfection of all permanent equipment; and
(4) NOT recycling disposable or single-patient use equipment on multiple patients.

10. In addition to counseling, all of the following prescription medications may be effective as
smoking cessation aids, EXCEPT:
A) buproprion SR
B) varenicline (Chantix)
C) inhaled steroids
D) nicotine replacement medications
Ans: C
Response:
Varenicline (Chantix), clonidine (Catapres), and buproprion SR are all medications that may
be of help to those attempting to quit smoking. Inhaled steroids have no effect on smoking
cessation efforts.

11. The scope of practice for pulmonary rehabilitation includes which of the following
components:

I. emphasizes the unique role of respiratory therapy


II. involves accurate diagnosis and therapy
III. cannot stabilize or reverse the pathology of chronic lung disease
IV. attempts to return the patient to the highest level of functional capacity
A) I and III
B) II and IV
C) I, III and IV
D) I, II, III and IV
Ans: B
Response:
The scope of practice for pulmonary rehabilitation is multi-disciplinary and individually
tailors rehabilitation efforts for patients based on accurate diagnosis, therapy, emotional
support and education thus attempting to stabilize or reverse the pathology associated with
chronic pulmonary disease and return the patient to the highest level of functional capacity
allowed by the pulmonary impairment and overall life situation.

Page 3
SAN PEDRO COLLEGE COMPETENCY APPRAISAL - OXYGENATION

12. Cleaning and disinfection in the home setting should employ all of the following strategies,
EXCEPT:
A) Where practical, disposable equipment should be used
B) Non-disposable equipment should be scrubbed, washed, rinsed, then air-dried.
C) All equipment should be sterilzed
D) Thorough handwashing should be done before and after patient care
Ans: C
Response:
Cleaning and disinfection in the home setting should involve using disposable equipment,
scrubbing washing and rinsing non-disposable equipment and handwashing. However,
sterilization is generally not needed in the home setting.

13. The cardiopulmonary exercise test is indispensable when evaluating a patient for pulmonary
rehabilitation because of which of the following reasons:

I. helps to differentiate between pulmonary and cardiac causes of dyspnea


II. determines the degree of oxygen desaturation resulting from physical activity
III. determines patient’s target heart rate
IV. can exclude patients from pulmonary rehabilitation
A) I and II
B) III and IV
C) II, III and IV
D) I, II, III and IV
Ans: D
Response:
Cardiopulmonary exercise testing is used for all of the reasons listed plus it helps establish a
patient's baseline and is also used to monitor a patient's progress and outcome after pulmonary
rehabilitation.

14. The primary benefit to participants in a pulmonary rehabilitation program is:


A) increased ability to perform activities of daily living (ADLs)
B) reversal of disease process
C) improved arterial blood gas results
D) improved cardiac function
Ans: A
Response:
One of the primary benefits to participants in pulmonary rehabilitation programs is an
increased ability to perform activities of daily living (ADLs), including daily hygiene,
cooking, possibly selected activities.

15. Which of the following is the most common indication for home CPAP therapy?
A) sleep apnea-hypopnea syndrome (SAHS)
B) bronchospasm associated with asthma
C) chronic hypoxemia
D) neuromuscular disorders
Ans: A
Response:
The most common indication for home CPAP therapy is sleep apnea-hypopnea syndrome
(SAHS).

Page 4
SAN PEDRO COLLEGE COMPETENCY APPRAISAL - OXYGENATION

16. The physical reconditioning component of a pulmonary rehabilitation program usually


includes which of the following?

I. aerobic exercises for the extremities


II. timed walking exercise
III. ventilatory muscle training
A) I, II and III
B) II and III only
C) I and III only
D) III only
Ans: A
Response:
The physical reconditioning component of the pulmonary rehabilitation program consists
primarily of an exercise prescription with target heart rate based on the results of the patient's
initial exercise evaluation. Typically, the exercise prescription includes four related
components: (1) lower extremity (leg) aerobic exercises; (2) timed walking; (3) upper
extremity (arm) aerobic exercises; and (4) ventilatory muscle training.

17. Smoking cessation programs generally involve the following components:

I. initial patient assessment and monitoring


II. ongoing support counseling
III. medications such as nicotine replacement
IV. encouragement, even in the face of a relapse
A) I and II
B) II and III
C) I, III and IV
D) I, II, III and IV
Ans: D
Response:
Smoking cessation programs generally involve initial patient assessment, ongoing support
counseling, medications such as nicotine replacement and encouragement, even in the face of
a relapse.

18. Advantages of liquid oxygen systems include all of the following EXCEPT:
A) no electricity, noise or heat
B) easily refill portables
C) delivers the highest FIO2
D) most economical
Ans: D
Response:
Advantages of liquid oxygen systems include no electricity, noise or heat, they can easily
refill portables and they deliver the highest FIO2. Such set-ups are not the most economical.

19. Which of the following oxygen systems is most suitable for a homecare patient with restricted
activity?
A) concentrator with cylinder back-up
B) liquid system with conserving device
C) hyperbaric oxygen deliver system
D) large gaseous cylinders
Ans: A
Response:
The most practical oxygen set-up for patients with restricted activity is an oxygen
concentrator with a gas cylinder back-up.

Page 5
SAN PEDRO COLLEGE COMPETENCY APPRAISAL - OXYGENATION

20. Prerequisites to be considered in discharging a ventilator dependent patient to the homecare


setting include all of the following EXCEPT:
A) Patient is clinically stable for at least two weeks
B) Continuous ventilation for at least 30 days with unsuccessful weaning
C) Patient and caregiver desire to go home
D) Whether the patient's health insurance coverage has run out
Ans: D
Response:
Prerequisites to be considered in discharging a ventilator dependent patient to the home
setting include whether the patient is clinically stable, on continuous ventilation for at least 30
days with unsuccessful weaning and if the patient desires to go home. It should not include
whether the patient's health insurance coverage has run out.

21. Which of the following topics should be covered in a rehabilitation education session covering
respiratory home care?

I. self-administration of therapy
II. care of home equipment. e.g., cleaning
III. safe use of home care equipment
A) I and II only
B) II and III only
C) I and III only
D) I, II and III
Ans: D
Response:
Topics that should be covered in a rehabilitation education session covering respiratory home
care include (1) the care and use of home care equipment and (2) self-administration of
therapy. A respiratory therapist with home care experience should provide this session.

22. You have been asked to organize a patient/family education program as part of a discharge
plan for a patient requiring home ventilatory support. Which of the following areas would you
be sure to cover?

I. equipment operation/disinfection
II. patient assessment and monitoring
III. airway management and clearance
IV. emergency procedures
A) I, II and III only
B) II and IV only
C) I, II, III and IV
D) III and IV only
Ans: C
Response:
To properly prepare patients, family, and other caregivers for home discharge, a
comprehensive educational program must be undertaken and completed. Essential skills that
must be taught include: (1)simple patient assessment; (2) airway management; (3) airway
clearance/ bronchial hygiene techniques; (4) drug administration (oral/aerosol); (5) patient
movement and ambulation; (6) equipment operation, maintenance, troubleshooting and
cleaning/disinfection; and (7) emergency procedures.

Page 6
SAN PEDRO COLLEGE COMPETENCY APPRAISAL - OXYGENATION

23. In order to achieve FIO2s greater than 0.21, the typical home ventilator:
A) 'bleeds' in O2 via a flow metering device
B) uses a venturi or air entrainment system
C) attaches directly to a 50 psi O2 source
D) attaches directly to a liquid O2 reservoir
Ans: A
Response:
In order to achieve FIO2s > 0.21, the typical home ventilator bleeds O2 into the system via a
flow metering device. Because these devices are electrically powered, a 50 psi O2 source is
not needed. Instead, the flow metering device may be attached to either a concentrator or LOX
system.

24. An ambulatory home oxygen therapy patient complains that her portable liquid system (at 2
L/min) doesn't last long enough for her to visit with her grandchildren. What might you
recommend to overcome this limitation?
A) decrease the flow to 1 L/min
B) use an oxygen-conserving device
C) put a couple E cylinders in her car
D) put the large liquid unit in her car
Ans: B
Response:
Most portable liquid oxygen units provide 5-8 hours of O2 at flow of 2 L/min. This in-use time
can be extended (doubled or even tripled) when the patient uses an oxygen-conserving device.

25. You know that an O2 concentrator is working properly if:


A) it provides at least 85% O2 at 5 L/min
B) the flow indicator reads 6 L/min at max
C) it provides at least 95% O2 at 5 L/min
D) the inlet filter is clean and dry
Ans: A
Response:
Oxygen concentrator FIO2s should be checked and confirmed as part of a routine monthly
maintenance visit. Routine maintenance of these devices should include cleaning and
replacing filters, checking the alarm system, and confirming the FIO2s using either the unit's
oxygen sensor or a separate calibrated O2 analyzer. In general, if the concentrator cannot
supply at least 85% oxygen at 5 L/min, the pellet canisters are probably exhausted and should
be replaced.

26. Which of the following home O2 delivery systems would you recommend for a stable COPD
patient with restricted activity?
A) aerosol mask mixing O2 from air compressor and O2 cylinder
B) O2 conserving device, liquid O2 reservoir + portable supply
C) nasal cannula with concentrator and backup O2 cylinder
D) air-entrainment mask with large-volume O2 cylinder (H or K)
Ans: C
Response:
The standard for providing low-flow oxygen at home to a patient with restricted activity is a
concentrator with a cylinder back-up in case of a power outage.

Page 7
CHAPTER 20
Name: __________________________ Date: _____________

Format: Multiple Choice


1. The essential goals of pulmonary rehabilitation include all of the following except:
A) improve exercise tolerance
B) reduce perceived dyspnea
C) improve health related quality of life
D) reverse lung damage
Ans: D
Response:
The goals of pulmonary rehabilitation include improving exercise tolerance, enhancing health-
related quality of life, and reducing perceived dyspnea in participants. The goals do not
include reversing lung damage.

Format: Multiple Choice


2. In order to meet the goals of pulmonary rehabilitation and return the patient to the highest
level of functional capacity, all of the following approaches may be used except:
A) multidisciplinary approach
B) education and related counseling
C) mandatory participation for all COPD patients
D) flexible approaches to meet varied patients' needs
Ans: C
Response:
The goals of pulmonary rehabilitation, which include returning the patient to the highest level
of functional capacity, may be achieved by using a multidisciplinary approach, education and
related counseling, as well as flexible approaches to meet varied patients' needs, but not
through mandatory participation.

Format: Multiple Choice


3. Pulmonary rehabilitation programs should include which of the following components:

I. patient education
II. breathing techniques and exercises
III. physical reconditioning
IV. smoking cessation and related counseling
A) III and IV
B) I, II, and III
C) II, III, and IV
D) I, II, III, and IV
Ans: D
Response:
Pulmonary rehabilitation programs will include patient education, breathing retraining,
physical reconditioning, and smoking cessation and counseling. All of these areas are essential
to the scope of practice as delineated for pulmonary rehabilitation.

Page 1
Format: Multiple Choice
4. Patients in pulmonary rehabilitation can perform physical conditioning exercises safely if they
exercise to what percent of their target heart rate?
A) 50%
B) 75%
C) 90%
D) 100%
Ans: B
Response:
In order to achieve cardiovascular benefit and improvement during physical reconditioning,
patients should safely exercise to 75% of their target heart rate. Working at levels less than
this do not result in any significant improvement, and working at levels greater than this
places the patient at risk for cardiovascular consequences.

Format: Multiple Choice


5. What is the primary purpose of patient documentation in pulmonary rehabilitation?
A) demonstrate patient involvement and note outcomes
B) insurance reimbursement
C) investigative research
D) patient documentation is not required in pulmonary rehabilitation
Ans: A
Response:
The primary purpose of patient documentation in pulmonary rehabilitation is to demonstrate
patient involvement and note outcomes. The other responses are valid but not the primary
reason for documenting patient activity.

Format: Multiple Choice


6. Emergency situations that home mechanical ventilation caregivers must be trained to
recognize and manage include all of the following except:
A) ventilator or power failure
B) tension pneumothorax
C) artificial airway obstruction
D) ventilator circuit problems
Ans: B
Response:
Emergency situations that caregivers must be trained to recognize and properly deal with
include ventilator or power failure, ventilator circuit problems, airway emergencies, and
cardiac arrest.

Format: Multiple Choice


7. You have been asked to organize a patient/family education program as part of a discharge
plan for a patient requiring home ventilatory support. Which of the following methods would
be best for training the family in operation of the ventilator chosen?
A) put the patient on the selected device while still hospitalized
B) set up and review the ventilator after the patient gets home
C) show the family the ventilator in a full-day session
D) give the family the operating manual for the ventilator
Ans: A
Response:
Ideally, the patient should be placed on the actual ventilator that will be used in the home
setting before discharge.

Page 2
Format: Multiple Choice
8. Which of the following are acceptable indicators of hypoxemia for purposes of justifying
home oxygen therapy?

I. A resting PaO2 of 55 torr (room air)


II. A nocturnal fall in SaO2 from 95% to 91%
III. A resting arterial Hb sat of 84% (room air)
A) I and II only
B) II and III only
C) I and III only
D) I, II, and III
Ans: C
Response:
The threshold for qualifying for oxygen therapy under Medicare is a resting PaO2 of 55 torr or
less or an SaO2 of 88% or less for a single pulmonary diagnosis (COPD), or a PaO2 between
56 and 59 torr or an SaO2 of 89% with a secondary diagnosis (COPD and cor pulmonale).

Format: Multiple Choice


9. Basic principles of infection control in the home care setting include which of the following?

I. avoiding visits by friends with respiratory infections


II. having caregivers follow proper handwashing technique
III. incinerating all disposable equipment and supplies
A) I and II only
B) II and III only
C) I and III only
D) I, II, and III
Ans: A
Response:
Basic principles of infection control in the home care setting include: (1) proper handwashing
technique by all caregivers; (2) discouraging visits to the patient from friends or relatives with
respiratory infections; (3) proper cleaning and disinfection of all permanent equipment; and
(4) NOT recycling disposable or single-patient use equipment on multiple patients.

Format: Multiple Choice


10. In addition to counseling, all of the following prescription medications may be effective as
smoking cessation aids, EXCEPT:
A) buproprion SR
B) varenicline (Chantix)
C) inhaled steroids
D) nicotine replacement medications
Ans: C
Response:
Varenicline (Chantix), clonidine (Catapres), and buproprion SR are all medications that may
be of help to those attempting to quit smoking. Inhaled steroids have no effect on smoking
cessation efforts.

Page 3
Format: Multiple Choice
11. The scope of practice for pulmonary rehabilitation includes which of the following
components:

I. emphasizes the unique role of respiratory therapy


II. involves accurate diagnosis and therapy
III. cannot stabilize or reverse the pathology of chronic lung disease
IV. attempts to return the patient to the highest level of functional capacity
A) I and III
B) II and IV
C) I, III and IV
D) I, II, III and IV
Ans: B
Response:
The scope of practice for pulmonary rehabilitation is multi-disciplinary and individually
tailors rehabilitation efforts for patients based on accurate diagnosis, therapy, emotional
support and education thus attempting to stabilize or reverse the pathology associated with
chronic pulmonary disease and return the patient to the highest level of functional capacity
allowed by the pulmonary impairment and overall life situation.

Format: Multiple Choice


12. Cleaning and disinfection in the home setting should employ all of the following strategies,
EXCEPT:
A) Where practical, disposable equipment should be used
B) Non-disposable equipment should be scrubbed, washed, rinsed, then air-dried.
C) All equipment should be sterilzed
D) Thorough handwashing should be done before and after patient care
Ans: C
Response:
Cleaning and disinfection in the home setting should involve using disposable equipment,
scrubbing washing and rinsing non-disposable equipment and handwashing. However,
sterilization is generally not needed in the home setting.

Format: Multiple Choice


13. The cardiopulmonary exercise test is indispensable when evaluating a patient for pulmonary
rehabilitation because of which of the following reasons:

I. helps to differentiate between pulmonary and cardiac causes of dyspnea


II. determines the degree of oxygen desaturation resulting from physical activity
III. determines patient’s target heart rate
IV. can exclude patients from pulmonary rehabilitation
A) I and II
B) III and IV
C) II, III and IV
D) I, II, III and IV
Ans: D
Response:
Cardiopulmonary exercise testing is used for all of the reasons listed plus it helps establish a
patient's baseline and is also used to monitor a patient's progress and outcome after pulmonary
rehabilitation.

Page 4
Format: Multiple Choice
14. The primary benefit to participants in a pulmonary rehabilitation program is:
A) increased ability to perform activities of daily living (ADLs)
B) reversal of disease process
C) improved arterial blood gas results
D) improved cardiac function
Ans: A
Response:
One of the primary benefits to participants in pulmonary rehabilitation programs is an
increased ability to perform activities of daily living (ADLs), including daily hygiene,
cooking, possibly selected activities.

Format: Multiple Choice


15. Which of the following is the most common indication for home CPAP therapy?
A) sleep apnea-hypopnea syndrome (SAHS)
B) bronchospasm associated with asthma
C) chronic hypoxemia
D) neuromuscular disorders
Ans: A
Response:
The most common indication for home CPAP therapy is sleep apnea-hypopnea syndrome
(SAHS).

Format: Multiple Choice


16. The physical reconditioning component of a pulmonary rehabilitation program usually
includes which of the following?

I. aerobic exercises for the extremities


II. timed walking exercise
III. ventilatory muscle training
A) I, II and III
B) II and III only
C) I and III only
D) III only
Ans: A
Response:
The physical reconditioning component of the pulmonary rehabilitation program consists
primarily of an exercise prescription with target heart rate based on the results of the patient's
initial exercise evaluation. Typically, the exercise prescription includes four related
components: (1) lower extremity (leg) aerobic exercises; (2) timed walking; (3) upper
extremity (arm) aerobic exercises; and (4) ventilatory muscle training.

Page 5
Format: Multiple Choice
17. Smoking cessation programs generally involve the following components:

I. initial patient assessment and monitoring


II. ongoing support counseling
III. medications such as nicotine replacement
IV. encouragement, even in the face of a relapse
A) I and II
B) II and III
C) I, III and IV
D) I, II, III and IV
Ans: D
Response:
Smoking cessation programs generally involve initial patient assessment, ongoing support
counseling, medications such as nicotine replacement and encouragement, even in the face of
a relapse.

Format: Multiple Choice


18. Advantages of liquid oxygen systems include all of the following EXCEPT:
A) no electricity, noise or heat
B) easily refill portables
C) delivers the highest FIO2
D) most economical
Ans: D
Response:
Advantages of liquid oxygen systems include no electricity, noise or heat, they can easily
refill portables and they deliver the highest FIO2. Such set-ups are not the most economical.

Format: Multiple Choice


19. Which of the following oxygen systems is most suitable for a homecare patient with restricted
activity?
A) concentrator with cylinder back-up
B) liquid system with conserving device
C) hyperbaric oxygen deliver system
D) large gaseous cylinders
Ans: A
Response:
The most practical oxygen set-up for patients with restricted activity is an oxygen
concentrator with a gas cylinder back-up.

Format: Multiple Choice


20. Prerequisites to be considered in discharging a ventilator dependent patient to the homecare
setting include all of the following EXCEPT:
A) Patient is clinically stable for at least two weeks
B) Continuous ventilation for at least 30 days with unsuccessful weaning
C) Patient and caregiver desire to go home
D) Whether the patient's health insurance coverage has run out
Ans: D
Response:
Prerequisites to be considered in discharging a ventilator dependent patient to the home
setting include whether the patient is clinically stable, on continuous ventilation for at least 30
days with unsuccessful weaning and if the patient desires to go home. It should not include
whether the patient's health insurance coverage has run out.

Page 6
Format: Multiple Choice
21. Which of the following topics should be covered in a rehabilitation education session covering
respiratory home care?

I. self-administration of therapy
II. care of home equipment. e.g., cleaning
III. safe use of home care equipment
A) I and II only
B) II and III only
C) I and III only
D) I, II and III
Ans: D
Response:
Topics that should be covered in a rehabilitation education session covering respiratory home
care include (1) the care and use of home care equipment and (2) self-administration of
therapy. A respiratory therapist with home care experience should provide this session.

Format: Multiple Choice


22. You have been asked to organize a patient/family education program as part of a discharge
plan for a patient requiring home ventilatory support. Which of the following areas would you
be sure to cover?

I. equipment operation/disinfection
II. patient assessment and monitoring
III. airway management and clearance
IV. emergency procedures
A) I, II and III only
B) II and IV only
C) I, II, III and IV
D) III and IV only
Ans: C
Response:
To properly prepare patients, family, and other caregivers for home discharge, a
comprehensive educational program must be undertaken and completed. Essential skills that
must be taught include: (1)simple patient assessment; (2) airway management; (3) airway
clearance/ bronchial hygiene techniques; (4) drug administration (oral/aerosol); (5) patient
movement and ambulation; (6) equipment operation, maintenance, troubleshooting and
cleaning/disinfection; and (7) emergency procedures.

Format: Multiple Choice


23. In order to achieve FIO2s greater than 0.21, the typical home ventilator:
A) 'bleeds' in O2 via a flow metering device
B) uses a venturi or air entrainment system
C) attaches directly to a 50 psi O2 source
D) attaches directly to a liquid O2 reservoir
Ans: A
Response:
In order to achieve FIO2s > 0.21, the typical home ventilator bleeds O2 into the system via a
flow metering device. Because these devices are electrically powered, a 50 psi O2 source is
not needed. Instead, the flow metering device may be attached to either a concentrator or LOX
system.

Page 7
Format: Multiple Choice
24. An ambulatory home oxygen therapy patient complains that her portable liquid system (at 2
L/min) doesn't last long enough for her to visit with her grandchildren. What might you
recommend to overcome this limitation?
A) decrease the flow to 1 L/min
B) use an oxygen-conserving device
C) put a couple E cylinders in her car
D) put the large liquid unit in her car
Ans: B
Response:
Most portable liquid oxygen units provide 5-8 hours of O2 at flow of 2 L/min. This in-use time
can be extended (doubled or even tripled) when the patient uses an oxygen-conserving device.

Format: Multiple Choice


25. You know that an O2 concentrator is working properly if:
A) it provides at least 85% O2 at 5 L/min
B) the flow indicator reads 6 L/min at max
C) it provides at least 95% O2 at 5 L/min
D) the inlet filter is clean and dry
Ans: A
Response:
Oxygen concentrator FIO2s should be checked and confirmed as part of a routine monthly
maintenance visit. Routine maintenance of these devices should include cleaning and
replacing filters, checking the alarm system, and confirming the FIO2s using either the unit's
oxygen sensor or a separate calibrated O2 analyzer. In general, if the concentrator cannot
supply at least 85% oxygen at 5 L/min, the pellet canisters are probably exhausted and should
be replaced.

Format: Multiple Choice


26. Which of the following home O2 delivery systems would you recommend for a stable COPD
patient with restricted activity?
A) aerosol mask mixing O2 from air compressor and O2 cylinder
B) O2 conserving device, liquid O2 reservoir + portable supply
C) nasal cannula with concentrator and backup O2 cylinder
D) air-entrainment mask with large-volume O2 cylinder (H or K)
Ans: C
Response:
The standard for providing low-flow oxygen at home to a patient with restricted activity is a
concentrator with a cylinder back-up in case of a power outage.

Page 8

You might also like